CBSE Notes For Class 6 History Chapter 2 From Hunting Gathering To Growing Food

CBSE Notes For Class 6 History Social Science Chapter 2 From Hunting Gathering To Growing Food

The Earliest People

  • The people who lived in the sub-continent two million years ago are described today as called hunter-gatherers.
  • The hunter-gatherers got their food by hunting wild animals, catching fish and birds, and gathering fruits, roots, nuts, seeds, leaves, stalks, and eggs.

Movement of Hunter-Gatherers

  • The hunter-gatherers moved from place to place. The reasons for their movement were as follows
  • Staying in one place for a long period might have depleted all the available plant and animal resources, so they traveled from place to place.
  • The animals moved from one place to another in search of their own food, which led the people that hunted them, to move from one place to another.
  • Plants and trees bear fruits in different seasons, so people might have moved from season to season in search of different plants.
  • People, plants, and animals need water to survive. So, people living on banks of seasonal lakes and rivers would have had to move to another place in search of water during the dry seasons (summer and winter).

Sources of Information about the Earliest People

Tools used by the Hunter-Gatherers

  • Tools of stone, wood, and bone have been found by archaeologists. These tools were used to cut meat and bone, scrape bark from trees and hides (animal skins), and chop fruit and roots.
  • Some tools may have been attached to handles of bone or wood in order to make spears and arrows for hunting.
  • Other tools were used to chop wood for the firewood.
  • Apart from firewood, wood was also used for making huts and tools.

Places to Live:  Archaeologists have found evidence of hunter-gatherers from the sites of Bhimbetka, Hunsgi, and Kurnool caves.

  • Apart from the above three sites, the evidence of hunter-gatherers also have been found in other places which were mostly located near sources of water like rivers and lakes.
  • People tried to find places where good quality stones were found.

Finding out about Fire: From the site of Kurnool caves, traces of ash have been found, which suggests that the use of fire was known in the past.

Fire could have been used for many purposes like as a source of light, to cook meat, and to scare away wild animals.

Rock Paintings: Many caves in which the early people lived have paintings on the walls. For example, caves in Madhya Pradesh and Southern Uttar Pradesh. These paintings show wild animals that were drawn with great accuracy and skill.

Names and Dates: Archaeologists divided the early period of human history into the following

  • Paleolithic Period The word ‘piano’ means old and ‘Uthos’ means stone. This period extended from 2 million years ago to about 12,000 years ago.
  • It is divided into the Lower, Middle, and Upper Palaeolithic. This long span of history covers over 99% of human history.
  • Mesolithic (Middle Stone) Period This period extended from 12,000 years ago to about 10,000 years ago. Stone tools found during this period were small, so they were called microliths, For Example. sickles.
  • Neolithic Period This period started about 10,000 years ago. Advanced stone tools were made in this period.

Class 6 History Social Science Chapter 2 A Changing Environment

  • There was a change in the climate of the world, around 12,000 years ago, with a shift to relatively warm conditions. This led to the development of grasslands in many areas.
  • The increase in grasslands led to an increase in animals that survived on grass like deer, antelope, goats, sheep, and cattle.
  • The people who hunted these animals started learning about their food habits and breeding seasons.
  • This helped people to start herding and rearing animals. Fishing also became an important activity.

Beginning of Farming and Herding: In different parts of the subcontinent, several grain-bearing types of grass, including wheat, barley, and rice started to grow naturally.

  • These grains were probably collected by men, women, and children, and they also learned about the place where they grew and when they were ripened. In this way, people became farmers.
  • People could have begun to tame animals by leaving food for them near their shelters (homes). The first animal to be tamed was the wild ancestor dog.
  • Later, people encouraged other animals such as sheep, goats, cattle, and pigs to come near the camps where they Lived.
  • These animals lived in herds and most of them ate grass. People also protected these animals from attacks by other wild animals. In this way, people became herders.
  • It is the process in which people grow plants and look after animals. The plants and animals that are domesticated by the pump are different from wild plants and animals.
  • The people select those plants and animals for domestication that are not prone to disease.
  • They select plants that yield large-size grain and have strong stalks, capable of bearing the weight of the ripe grain. They select animals that are relatively gently selected
  • Domestication was a gradual process that took place in many parts of the world. It began about 12,000 years ago.
  • Some of the earliest plants to be domesticated were wheat and barley. The earliest domesticated animals include sheep and goats.

Class 6 History Social Science Chapter 2 A New Way of Life

  • When people began growing plants, it meant that they had to stay in the same place for a long time till the grain ripened.
  • This process involved looking after the plants, watering them, weeding, and protecting them from animals and birds.
  • In many areas, people started making large clay pots, and baskets and dug pits into the ground in order to store grains for food and seeds.

First Farmers and Herders:  The evidences of early farmers and herders have been found from the sites of Burzahom, Chirand, Daojali Hading, Koldihwa, Mahagara, Hallur, Paiyampalli and Mehrgarh (now in Pakistan).

  • The remains of burnt grains have been recovered from these sites. This suggests that a number of crops were grown in different parts of the subcontinent.
  • Apart from these, the bones of different animals also have been found.

Towards a Settled Life: In Burzahom (in present-day Kashmir), people built pit houses, which were dug into the ground, with steps leading into them. These may have provided shelter in cold weather.

  • Cooking hearths have been found both inside and outside the huts. This suggests that people could cook food either indoors or outdoors, depending on the weather.
  • Stone tools have been found on many sites which were different from the earlier Palaeolithic tools and they are called Neolithic tools.
  • These included tools that were polished to give a fine cutting edge. Mortars” and pestles were used for grinding grain and other plant produce. Some of the tools were made of bone.
  • Various kinds of earthen pots have also been found, which were sometimes decorated, and were used for storing things.
    People started using pots for cooking food, especially grains like rice, wheat, and lentils.
  • People also started weaving cloth, using different kinds of materials, For Example. cotton.
  • However, in many areas, men and women continued to hunt and gather food, and elsewhere people adopted farming and herding slowly, over several thousands of years.

Animals as a Store of Food

  • Breeding of animals is a natural process. If they are looked after carefully, they are an important source of food and meat, whenever required.
  • In other words, animals that are reared can be used as a ‘store’ of food.

Class 6 History Social Science Chapter 2 A Closer Look: Site of Mehrgarh

  • Mehrgarh site is located in a fertile plain, near the Bolan Pass, which is one of the most important routes into Iran.
  • Mehrgarh was probably one of the places where women and men learned to grow barley and wheat, and rear sheep and goats for the first time.

The sources that have been found on this site are

  • Bones of many animals such as deer, pig, sheep, and goats have been found in Mehrgarh.
  • The remains of square and rectangular houses have been discovered from this site. Each house had four or more compartments, some of which may have been used for storage.
  • Several burial sites have also been from Mehrgarh.
  • The dead person was buried with goats, which were probably meant to serve as food in the next world. There was a belief that there was some form of life after death.

CBSE Notes For Class 6 History Chapter 3 In The Earliest Cities

CBSE Notes For Class 6 History Social Science Chapter 3 In The Earliest Cities

The Story Of Harappa

  • About 150 years ago, when railway lines were being laid down for the first time in the Punjab, engineers encountered the site of Harappa (in present-day Pakistan).
  • About 80 years ago, archaeologists found the site of Harappa and realized that it was one of the oldest cities in the sub-continent.
  • Harappa was the first city to be discovered, thus all the other sites, where similar buildings (and other things) were found, were described as Harappan. These sites developed about 4700 years ago.

Harappan Cities

Many of the Harappan cities were divided into two or more parts as follows

Citadel It was the Western part of the city which was smaller but higher.

Lower Town It was the Eastern part of the city which was larger but lower.

  • The strong walls of well-baked bricks were built around each part. The bricks were laid down in an interlocking pattern.
  • In some cities, special buildings were constructed. For example, Great Bath in Mohenjodaro which was a special tank.
  • It was built with bricks, coated with plaster, and made water-tight with a layer of natural tar. It had steps leading down to it from two sides and there were rooms on all sides.
  • Water was probably brought in from a well, and drained out after use. On special occasions, important people of the city took a dip in the tank.
  • The cities such as Kalibangan and Lothal had fire altars where sacrifices may have been performed. Some cities like Mohenjodaro, Harappa, and Lothal had elaborate storehouses.

Houses, Drains, and Streets:  In the Harappan cities, the houses, drains, and streets were probably planned and built at the same time.

  • The houses were either one or two storeys high with rooms built around a courtyard. Most of the houses had a separate bathing area and some had wells to supply water.
  • The drains were covered in many cities and the inspection holes were provided at intervals to clean them.
  • The drains were laid out in straight lines and each drain had a gentle slope so that water could flow through it.
  • Mostly, the drains in houses were connected to the drains on the streets, and smaller drains led into bigger ones.

Class 6 History Social Science Chapter 3 Life in the City

CBSE Notes For Class 6 History Social Science Chapter 3 In The Earliest Cities Harappan Seal

  • The people who constructed special buildings in the city were probably the rulers.
  • The rulers probably sent people to distant lands to get metal, precious stones, and other valuable objects.
  • They may have kept the most valuable objects, such as ornaments of gold and silver, or beautiful beads for themselves.
  • The craftspersons (men and women) made all kinds of things, either in their homes or in special workshops, For Example. terracotta1 toys.

New Crafts in the City

  • Most of the things found in Harappan cities by archaeologists were made of stones, shells, and metals including copper, bronze, gold, and silver.
  • Copper and bronze were used to make tools, weapons, ornaments, and vessels. Gold and silver were used to make ornaments and vessels.
  • Apart from these things, beads, weights, and blades are the most important things found by the archaeologists.
  • Seals were also made with stone by the Harappans, which are generally rectangular and have an animal carved on them. They also made pots with beautiful black designs.

CBSE Notes For Class 6 History Social Science Chapter 3 In The Earliest Cities Beads Many of these were made out of Carnelian

Cotton and Cloth

  • Cotton was probably grown at Mehrgarh about 7000 years ago. From the site of Mohenjodaro, actual pieces of cloth were found attached to the lid of a silver vase and some copper objects.
  • Archaeologists have also found spindle whorles, made of terracotta and faience, which were used to spin thread.
  • Many of these things were made by the specialists. A specialist is a person (man or woman) who is trained to do only one kind of work. For example, cutting stones, polishing beads, or carving seals.

Raw Materials: Raw materials are those substances that are processed to produce finished goods. They are either found naturally (such as wood, or ores of metals) or produced by farmers or herders.

  • Some of the raw materials used by the Harappans were locally available but many items such as copper, tin, gold, silver, and precious stones had to be brought from distant places.
  • The Harappans probably got copper from present-day Rajasthan and Oman in West Asia. Tin which was mixed with copper to produce bronze, which may have been brought from present-day Afghanistan and Iran.
  • Gold could have been brought from present-day Karnataka, and precious stones from present-day Gujarat, Iran, and Afghanistan.

Class 6 History Social Science Chapter 3 Food for People in the Cities

  • The people living in the countryside grew crops and reared animals. These farmers and herders supplied food to craftspersons, scribes, and rulers in the cities.
  • It was evident from the remains of plants that the Harappans grew wheat, barley, pulses, peas, rice, sesame, linseed, and mustard.
  • A new tool, the plough³, was used to dig the Earth for turning the soil and planting seeds. Toy models of plows have been found in the cities.
  • Some form of irrigation may have been used as these cities did not receive heavy rainfall. Water was stored and supplied to the fields when the plants were growing.
  • Water and pastures’ were available around the settlements as the Harappans reared cattle, sheep, goats, and buffalo.
  • During the summer months, large herds of animals were probably taken to greater distances in search of grass and Hnntppans collected fruits like her, caught fish, and hunted wild animals like the antelope.

Class 6 History Social Science Chapter 3 Harappan Towns in Gujarat

The Harappan towns of Dholavira and Lothal have been discovered in the state of Gujarat.

Dholavira: The city of Dholavira was located on Khadir Beyt in the Rann of Kutch, where there was fresh water and fertile soil.

  • Dholavira was divided into three parts, and each part was surrounded by massive stone walls, with entrances through gateways. There was also a large open area in the settlement where public ceremonies could be held.
  • In Dholavira large letters of Harappan script were found which were carved out of white stone and inlaid in wood.
  • It is a unique find because Harappan writing has been generally found on small objects such as seals.

Lothal: The city of Lothal was located on the bank of a tributary of the Sabarmati river in Gujarat, close to the Gulf of Khambat.

  • This site was an important center for making objects from stone, shell, and metal. Raw materials such as semi-precious stones were easily available here.
  • There was also a storehouse in this city where many seals and sealings (the impression of seals on clay) were found.
  • The pieces of stone, half-made beads, tools for bead making, and finished beads have been found here which suggests that there was a workshop for making beads in Lotha

Class 6 History Social Science Chapter 3 End of the Harappan Cities

  • Around 3900 years ago, people stopped living in many of the cities, and the writing, seals, and weights were no longer used.
  • The buying of raw materials from long distances was reduced. In Mohenjodaro garbage piled up, on the streets, the drainage system broke down and new houses were built, even over the streets.

The reasons suggested by some scholars for these changes were

  1. The rivers dried up and there was deforestation because a large quantity of fuel was required for baking bricks and for smelting5 copper ores. ;
  2. Grazing by large herds of cattle, sheep, and goats may have destroyed the green cover, while there were floods in some areas.
  3. However, these reasons do not explain the end of all the cities, because the flooding or drying of a river would have had an effect only in some areas. It appears that the rulers had lost control over the cities.
  4. The sites in Sind and West Punjab (present-day Pakistan) were abandoned, while many people moved into newer, smaller settlements to the East and the South. New cities emerged about 1400 years later

CBSE Notes For Class 6 History Chapter 1 Introduction What Where How And When

CBSE Notes For Class 6 History Social Science Chapter 1 Introduction What Where How And When

There are several sources of findings about the past. Historians use the information obtained from these sources to know about the lives of ancient people, including the food they ate and the different houses in which they lived. In this chapter, we will learn about where and how did people live, and about the various sources of knowing the past.

Finding The Past

There are various things that can be found about the past. For example, the food that people ate, the different kinds of clothes they wore, and the different kinds of houses in which they lived.

Information about the lives of hunters, herders, farmers, rulers, merchants, priests, craftsmen, artists, musicians, and scientists can also be found.

Class 6 History Social Science Chapter 1 Habitat Of People

  • The people lived close to the banks of the Narmada River several hundred thousand years ago.
  • Some of the earliest people were skilled gatherers i.e. the people who gathered food for themselves. They knew about the vast variety of plants and collected roots, fruits, and other forest produce for their food. They also hunted animals.
  • In some areas of the Sulaiman and Kirthar hills to the northwest, women and men first began to grow crops such as wheat and barley about 8000 years ago.
  • People also began rearing animals like sheep, goats, and cattle and lived in villages. Agriculture developed in the areas like the Garo hills to the North-East and the Vindhyas in central India. Rice was first grown in the area to the North of the Vindhyas.

Development of Cities: The Indus River and its tributaries were responsible for the development of the earliest cities about 4700 years ago.

  • In the later period, nearly 2500 years ago, the cities were developed near the banks of the Ganga river and its tributaries, like the Son, Chambal, and Yamuna rivers, and along the sea coasts.
  • The powerful Mahajanapada named Magadha (Now in Bihar) was located to the South of Ganga and its rulers were very powerful and set up a large kingdom. Kingdoms were also set up in other parts of the country.

The Purpose Of Travelling: Throughout the past, people traveled from one part of the subcontinent to another for various reasons as follows

  • People traveled from one place to another in search of their livelihood, to escape from natural calamities (floods or droughts), and to conquer other lands.
  • The merchants traveled from place to place with caravans or ships, carrying valuable goods.
  • Religious teachers moved from one place to another to deliver their religious understanding to others.
  • Some people also traveled with the spirit of adventure to find out new places.

All these led to the sharing of ideas between peoples and enriched the cultural traditions. For example, people have shared new ways of carving stones, composing music, and cooking food.

Hills, mountains, and seas form the natural frontiers of the sub-continent which sometimes makes the journeys of people difficult.

Names Of The Land: There are two words that are often used for our country-India and Bharata.

  • The origin of the word ‘India’ comes from the name of the Indus River, which is known as Sindhu in Sanskrit.
  • The Iranians and the Greeks, who came through the North-West about 2500 years ago, called the Indus the Hindos or the Indos. The land to the East of the Indos River was called India.
  • The name Bharata was used for a group of people who lived in the North-West, and who are mentioned in the Rigveda, which was composed in Sanskrit about 3500 years ago. Later the word ‘Bharata’ was used for India.

Class 6 History Social Science Chapter 1 Finding Out About The Past

There are several sources for finding out about the past. Historians i.e. scholars who study the past, often use the word source to refer to the information found in manuscripts, inscriptions, and archaeology. Historians use all these sources to find out about the past.

Manuscripts: Manuscripts are books that were written long ago. The word manuscript comes from the Latin word ‘manu which means hand.

  • Manuscripts were usually written on palm leaves or on specially prepared bark of a tree known as the birch, which grows in the Himalayas
  • Many manuscripts are preserved in temples and monasteries² while some were destroyed.
  • Manuscripts dealt with all kinds of subjects like religious beliefs and practices, the lives of kings, medicine and science, epics, poems, and plays.
  • These books were written in Sanskrit, Prakrit (the language used by ordinary people), and Tamil.

CBSE Notes For Class 6 History Social Science Chapter 1 Introduction What Where How And When A Page From A Palm Leaf Manuscript

 

Inscriptions:  This inscription dates to about 2250 years ago and was found in Kandahar, present-day Afghanistan. It was inscribed on the orders of a ruler named Ashoka.

This inscription was inscribed in two different scripts and languages, Greek and Aramaic, which were used in this area.

Archaeological Sources: Many objects were made and used in the past. Those who study these objects are called archaeologists.

CBSE Notes For Class 6 History Social Science Chapter 1 Introduction What Where How And When An Archaeological Source

 

  • They study the remains of buildings made of stone and brick, paintings, and sculptures.
  • Archaeologists explore and excavate (dig under the surface of the Earth) to find different objects like weapons, tools, pots, pans, ornaments, and coins.
  • Some of these objects may be made of stones, bones, baked clay, or metal. Objects that are made of hard substances usually survive for a long time.
  • Archaeological sources like bones of animals, birds, and fish give information regarding what people ate in the past.
  • Plant remains are less likely to survive. If seeds of grain or pieces of wood have been burnt, they survive in a charred form.

Class 6 History Social Science Chapter 1 Different Pasts for Different People

  • The past was different for different groups of people. For example, the lives of herders or farmers were different from those of kings and queens, the lives of merchants were different from those of crafts persons, etc.
  • Even today, people follow different practices and customs in different parts of the country.
  • For example, the people of the Andaman Islands get their food by fishing, hunting, and collecting forest produce while it is different for the people living in cities, who depend on others for supplies of food.
  • Generally, the kings kept their records of victories, but the ordinary people such as hunters, fishing folk, and gatherers.
  • Farmers or herders did not keep records of what they did. The archaeologists try to find out about life of the ordinary people through the objects found in the excavator

Meaning of Dates: All dates before the birth of Christ (The founder of Christianity) are counted backward and usually denoted with the letters BC which stands for Before Christ.

  • Sometimes, AD is prefixed before dates which stands for Anno Domini. It means after the birth of Jesus Christ.
  • Sometimes CE is used instead of AD which stands for Common Era and BCE instead of BC which stands for Before Common Era. Sometimes, the letters BP meaning Before Present are also used.

Determinants Class 12 Maths Important Questions Chapter 4

Determinants Exercise 4.1

Question 1. \(\left|\begin{array}{cc}
2 & 4 \\
-5 & -1
\end{array}\right|\)

Solution:

Let A=\(\left|\begin{array}{cc}2 & 4 \\ -5 & -1\end{array}\right| \Rightarrow|A|=2 \times(-1)-(-5) \times\) 4=-2+20=18

Question 2. 1.\(\left|\begin{array}{cc}\cos \theta & -\sin \theta \\ \sin \theta & \cos \theta\end{array}\right|\)

2. \(\left|\begin{array}{cc}x^2-x+1 & x-1 \\ x+1 & x+1\end{array}\right|\)

Solution:

⇒ \(\left|\begin{array}{cc}
\cos \theta & -\sin \theta \\
\sin \theta & \cos \theta
\end{array}\right|\)

=\((\cos \theta)(\cos \theta)-(\sin \theta)(-\sin \theta) \)

=\(\cos ^2 \theta+\sin ^2 \theta=1\left (\sin ^2 \theta+\cos ^2 \theta=1\right)\)

⇒ \(\left|\begin{array}{cc}
x^2-x+1 & x-1 \\
x+1 & x+1
\end{array}\right|=\left(x^2-x+1\right)(x+1)-(x+1)(x-1)\)

= \(x^3-x^2+x+x^2-x+1-\left(x^2-1\right)=x^3+1-\left(x^2-1\right)=x^3-x^2+2\)

Question 3. If A= \(\left[\begin{array}{ll}
1 & 2 \\
4 & 2
\end{array}\right] \)
, then show that |2 A|=4|A|

Solution:

Given, A=\(\left[\begin{array}{ll}
1 & 2 \\
4 & 2
\end{array}\right]\)

2 A=2\(\left[\begin{array}{ll}
1 & 2 \\
4 & 2
\end{array}\right]=\left[\begin{array}{ll}
2 \times 1 & 2 \times 2 \\
2 \times 4 & 2 \times 2
\end{array}\right]=\left[\begin{array}{ll}
2 & 4 \\
8 & 4
\end{array}\right] \)

LHS =|2 A|=\(\left[\begin{array}{ll}
2 & 4 \\
8 & 4
\end{array}\right]=(2 \times 4)-(8 \times 4)\)=-24

Read and Learn More Class 12 Maths Chapter Wise with Solutions

Now, |A|=\(\left|\begin{array}{ll}
1 & 2 \\
4 & 2
\end{array}\right|=(1 \times 2)-(4 \times 2)\)=-6

RHS =4|A|=4 \(\times(-6)\)=-24

LHS = RHS

Hence; |2A|=4|A|

CBSE Class 12 Maths Chapter 4 Determinants Important Question And Answers

Question 4. If A=\(\left[\begin{array}{lll}
1 & 0 & 1 \\
0 & 1 & 2 \\
0 & 0 & 4
\end{array}\right]\)
 , then show that |3 A|=27|A|

Solution:

Given, A=\(\left[\begin{array}{lll}
1 & 0 & 1 \\
0 & 1 & 2 \\
0 & 0 & 4
\end{array}\right]\)

then 3 A=\(\left[\begin{array}{ccc}
3 & 0 & 3 \\
0 & 3 & 6 \\
0 & 0 & 12
\end{array}\right]\)

Now; \(|\mathrm{A}|=\left|\begin{array}{lll}
1 & 0 & 1 \\
0 & 1 & 2 \\
0 & 0 & 4
\end{array}\right|\)=1(4-0)-0(0)+1(0)=4

⇒ \(|3 \mathrm{~A}|=\left|\begin{array}{lll}
3 & 0 & 3 \\
0 & 3 & 6 \\
0 & 0 & 12
\end{array}\right|\)=3(36-0)-0(0)+3(0)=108

⇒ \(|3 \mathrm{~A}|=108=27 \times 4=27|\mathrm{~A}|\),Hence proved.

Question 5. Evaluate the determinants.

  1. \(\left|\begin{array}{ccc}3 & -1 & -2 \\ 0 & 0 & -1 \\ 3 & -5 & 0\end{array}\right|\)
  2. \(\left|\begin{array}{ccc}3 & -4 & 5 \\ 1 & 1 & -2 \\ 2 & 3 & 1\end{array}\right|\)
  3. \(\left|\begin{array}{ccc}0 & 1 & 2 \\ -1 & 0 & -3 \\ -2 & 3 & 0\end{array}\right|\)
  4. \(\left|\begin{array}{ccc}2 & -1 & -2 \\ 0 & 2 & -1 \\ 3 & -5 & 0\end{array}\right|\)

Solution:

1. Let |A|=\(\left|\begin{array}{ccc}3 & -1 & -2 \\ 0 & 0 & -1 \\ 3 & -5 & 0\end{array}\right|\)

By expanding along \(R_2\)

⇒ \(|\mathrm{A}|=-0\left|\begin{array}{cc}
-1 & -2 \\
-5 & 0
\end{array}\right|+0\left|\begin{array}{cc}
3 & -2 \\
3 & 0
\end{array}\right|-(-1)\left|\begin{array}{cc}
3 & -1 \\
3 & -5
\end{array}\right|=\{3 \times(-5)-3 \times(-1)\}\)=-15+3=-12

2. Let |A|=\(\left|\begin{array}{ccc}3 & -4 & 5 \\ 1 & 1 & -2 \\ 2 & 3 & 1\end{array}\right|\)

By expanding along \(\mathrm{R}_1\) (first row), we get

⇒ \(|A| =3\left|\begin{array}{cc}
1 & -2 \\
3 & 1
\end{array}\right|-(-4)\left|\begin{array}{cc}
1 & -2 \\
2 & 1
\end{array}\right|+5\left|\begin{array}{ll}
1 & 1 \\
2 & 3
\end{array}\right|\)

=3(1+6)+4(1+4)+5(3-2)=3(7)+4(5)+5(1)=21+20+5=46

3. Let |A|=\(\left|\begin{array}{ccc}0 & 1 & 2 \\ -1 & 0 & -3 \\ -2 & 3 & 0\end{array}\right|\)

By expanding along \(R_1\) (first row), we get

⇒ \(|A|=0\left|\begin{array}{cc}
0 & -3 \\
3 & 0
\end{array}\right|-1\left|\begin{array}{cc}
-1 & -3 \\
-2 & 0
\end{array}\right|+2\left|\begin{array}{ll}
-1 & 0 \\
-2 & 3
\end{array}\right|\)

=-1(0-6)+2(-3-0)=-1(-6)+2(-3)=6-6=0

4. Let \(|A|=\left|\begin{array}{ccc}2 & -1 & -2 \\ 0 & 2 & -1 \\ 3 & -5 & 0\end{array}\right|\)

By expanding along \(R_2\) (second row), we get

|A|=\(-0\left|\begin{array}{cc}
-1 & -2 \\
-5 & 0
\end{array}\right|+2\left|\begin{array}{cc}
2 & -2 \\
3 & 0
\end{array}\right|-(-1)\left|\begin{array}{cc}
2 & -1 \\
3 & -5
\end{array}\right|\)

=0+2(0+6)+(-10+3)=12-7=5

Question 6. If A=\(\left[\begin{array}{lll}
1 & 1 & -2 \\
2 & 1 & -3 \\
5 & 4 & -9
\end{array}\right]\)
, find|A| ?

Solution:

Given, A=\(\left[\begin{array}{lll}1 & 1 & -2 \\ 2 & 1 & -3 \\ 5 & 4 & -9\end{array}\right]\)

By expanding along R_1 (first row), we get

⇒ \(|A|=1\left|\begin{array}{ll}
1 & -3 \\
4 & -9
\end{array}\right|-1\left|\begin{array}{ll}
2 & -3 \\
5 & -9
\end{array}\right|+(-2)\left|\begin{array}{ll}
2 & 1 \\
5 & 4
\end{array}\right|\)

=1(-9+12)-1(-18+15)-2(8-5)

=1(3)-1(-3)-2(3)=3+3-6=0

Question 7. Find the values of x, if:

1. \(\left|\begin{array}{cc}2 & 4 \\ 5 & 1\end{array}\right|=\left|\begin{array}{cc}2 x & 4 \\ 6 & x\end{array}\right|\)

2. \(\left|\begin{array}{ll}2 & 3 \\ 4 & 5\end{array}\right|=\left|\begin{array}{cc}x & 3 \\ 2 x & 5\end{array}\right|\)

Solution:

1. Given, \(\left|\begin{array}{cc}2 & 4 \\ 5 & 1\end{array}\right|=\left|\begin{array}{cc}2 \mathrm{x} & 4 \\ 6 & \mathrm{x}\end{array}\right|\)

On expanding both determinants, we get

⇒ \((2 \times 1)-(5 \times 4)=(2 x \times x)-(6 \times 4) \Rightarrow 2-20=2 x^2-24\)

⇒ \(2 x^2=-18+24 \Rightarrow x^2=\frac{6}{2}=3 \Rightarrow x= \pm \sqrt{3}\)

2. Given, \(\left|\begin{array}{ll}2 & 3 \\ 4 & 5\end{array}\right|=\left|\begin{array}{cc}x & 3 \\ 2 x & 5\end{array}\right|\)

On expanding both determinants, we get

⇒ \((2 \times 5)-(4 \times 3)=(5 \times x)-(3 \times 2 x)\)

10-12=5 x-6 x \(\Rightarrow-2=-x \Rightarrow\) x=2

Question 8. If \(\left|\begin{array}{cc}
x & 2 \\
18 & x
\end{array}\right|=\left|\begin{array}{cc}
6 & 2 \\
18 & 6
\end{array}\right|\), then x is equal to ?

  1. 6
  2. \(\pm\) 6
  3. -6
  4. 0

Solution: 2. \(\pm\) 6

On expanding both determinants, we get

⇒ \((\mathrm{x} \times \mathrm{x})-(18 \times 2)=(6 \times 6)-(18 \times 2) \Rightarrow \mathrm{x}^2-36=36-36\)

\(\mathrm{x}^2-36=0 \Rightarrow \mathrm{x}^2=36 \Rightarrow \mathrm{x}= \pm 6\)

So, (B) is the correct option.

Determinants Exercise 4.2

Question 1. Find the area of the triangle with vertices at the points in each of the following:

(1,0), (6,0), (4,3)

(2,7). (1,1), (10,8)

(-2,-3), (3,2), (-1,-8)

Solution:

Area of triangle =\(\frac{1}{2}\left|\begin{array}{lll}
x_1 & y_1 & 1 \\
x_2 & y_2 & 1 \\
x_3 & y_3 & 1
\end{array}\right|\)

1. Required area =\(\frac{1}{2}\left|\begin{array}{lll}
1 & 0 & 1 \\
6 & 0 & 1 \\
4 & 3 & 1
\end{array}\right|=\frac{1}{2}|1(0-3)-0(6-4)+1(18-0)|\)

⇒ \(\frac{1}{2}|{-3+18}|=\frac{15}{2}\)..sq. units

2. Required area =\(\frac{1}{2}\left|\begin{array}{lll}
2 & 7 & 1 \\
1 & 1 & 1 \\
10 & 8 & 1
\end{array}\right|\)

= \(\frac{1}{2}|2(1-8)-7(1-10)+1(8-10)|\)

= \(\frac{1}{2}|2(-7)-7(-9)+1(-2)|=\frac{1}{2}|-14+63-2|=\frac{47}{2}\) sq. units

3. Required area =\(\frac{1}{2}\left|\begin{array}{ccc}
-2 & -3 & 1 \\
3 & 2 & 1 \\
-1 & -8 & 1
\end{array}\right|\)

= \(\frac{1}{2}|-2(2+8)+3(3+1)+1(-24+2)|\)

= \(\frac{1}{2}|-20+12-22|=\frac{1}{2}|-30|\)=15 sq. units

(Since the area of the triangle is always positive)

Question 2. Show that the points A(a,b + c), B(b,c +a), C(e, a + b) are collinear

Solution:

Area of \(\triangle A B C =\frac{1}{2}\left|\begin{array}{lll}
a & b+c & 1 \\
b & c+a & 1 \\
c & a+b & 1
\end{array}\right|\)

=\(\frac{1}{2}|a\{(c+a) \times 1-(a+b) \times 1\}-(b+c)\{b \times 1-1 \times c\}+1\{b \times(a+b)-(c+a) \times c\}|\)

=\(\frac{1}{2}\left|a(c+a-a-b)-(b+c)(b-c)+1\left(a b+b^2-c^2-a c\right)\right|\)

=\(\frac{1}{2}\left|a c-a b-b^2+c^2+a b+b^2-c^2-a c\right|=\frac{1}{2} \times \)0=0

Since, the area of AABC = 0.

Hence, points A(a, b + c), … C(c, a + b) are collinear,

Question 3. Find the value of k, the area of a triangle is 4 sq. units and the vertices are:

1. (k,0), (4,0),(0,2)

2. (-2,0),(0,4),(0, k)

Solution:

Area of triangle =\(\frac{1}{2}\left|\begin{array}{lll}x_1 & y_1 & 1 \\ x_2 & y_2 & 1 \\ x_3 & y_3 & 1\end{array}\right|\)

1. Given, \(\frac{1}{2}\left|\begin{array}{lll}
\mathrm{k} & 0 & 1 \\
4 & 0 & 1 \\
0 & 2 & 1
\end{array}\right|=4\)

⇒ \(|\mathrm{k}(0-2)+1(8-0)|=8 \Rightarrow \mathrm{k}(0-2)+1(8-0)= \pm 8\)

On taking positive sign;

-2k + 8 = 8 ⇒ -2k = 0 ⇒ k = 0

On taking negative sign;

-2k + 8 = 8 ⇒-2k = -16⇒ k = 8

k = 0,8

2. Given, \(\frac{1}{2}\left|\begin{array}{ccc}-2 & 0 & 1 \\ 0 & 4 & 1 \\ 0 & \mathrm{k} & 1\end{array}\right|=4 \Rightarrow|-2(4-\mathrm{k})+1(0-0)|\)=8

⇒ –\(2(4-\mathrm{k})+1(0-0)= \pm 8 \Rightarrow(-8+2 \mathrm{k})= \pm\) 8

On taking positive sign, 2 k-8=8 \(\Rightarrow\) 2 \(\mathrm{k}=16 \Rightarrow \)k=8

On taking negative sign, 2 k-8=-8 \(\Rightarrow 2 \mathrm{k}\)=0 \(\Rightarrow\) k=0

k =0,8

Question 4. 1. Find the equation of the line joining (1, 2) and (3, 6) using determinants.

2. Find the equation of the line joining (3, 1) and (9, 3) using determinants.

Solution.

1. Let P(x,y) beany point on the line joining A(1,2)andB(3,6) .

⇒ Area of triangle =\(\frac{1}{2}\left|\begin{array}{lll}x_1 & y_1 & 1 \\ x_2 & y_2 & 1 \\ x_3 & y_3 & 1\end{array}\right|\)=0

⇒ \(\frac{1}{2}\left|\begin{array}{lll}
1 & 2 & 1 \\
3 & 6 & 1 \\
x & y & 1
\end{array}\right|=0 \Rightarrow \frac{1}{2}[1(6-y)-2(3-x)+1(3 y-6 x)]\)=0

6-y-6+2x+3y-6x = 0 ⇒ 2y-4x = 0 ⇒y = 2x

Fluence, the equation of the line joining the given points is y = 2x.

2. Let P(x, y) be any point on the line joining A(3,i)and B(9.3)

Area of triangle =\(\frac{1}{2}\left|\begin{array}{lll}
x_1 & y_1 & 1 \\
x_2 & y_2 & 1 \\
x_3 & y_3 & 1
\end{array}\right|\)=0

⇒ \(\frac{1}{2}\left|\begin{array}{lll}
3 & 1 & 1 \\
9 & 3 & 1 \\
x & y & 1
\end{array}\right|\)=0

⇒ \(\frac{1}{2}|3(3-y)-1(9-x)+1(9 y-3 x)\)|=0

9-3 y-9+x+9 y-3 x=0 \(\Rightarrow 6 y-2 x=0 \Rightarrow\) x-3 y=0

Hence, the equation of the line joining the given points is x-3 y=0.

Question 5. If the area of a triangle is 35 sq. units with vertices (2,-6),(5,4), and (k,4), then k is.

  1. 12
  2. -2
  3. -1 2, -2
  4. 12,-2

Solution: 4. 12,-2

Given, \(\frac{1}{2}\left|\begin{array}{ccc}
2 & -6 & 1 \\
5 & 4 & 1 \\
k & 4 & 1
\end{array}\right|\)=35

|2(4-4)+6(5-k)+1(20-4 k)|=70

2(4-4)+6(5-k)+1(20-4 k)= \(\pm\) 70

30-6 k+20-4 k= \(\pm 70\)

On taking positive sign, -10 k +50=70 \(\Rightarrow\)-10 k =20 \(\Rightarrow\) k=-2

On taking negative sign, -10 k +50=-70 \(\Rightarrow\)-10 \(\mathrm{k}=-120 \Rightarrow\) k =12

k=12,-2 Hence, the correct option is 4.

Determinants Exercise 4.3

Question 1. \(\left|\begin{array}{cc}2 & -4 \\ 0 & 3\end{array}\right|\)

2. \(\left|\begin{array}{ll}a & c \\ b & d\end{array}\right|\)

Solution:

Here, \(\left|\begin{array}{cc}2 & -4 \\ 0 & 3\end{array}\right|\)is given

Minors, \(M_{11}=3, M_{12}=0, M_{21}=-4 and M_{22}\)=2

Also cofactors, \(A_{11}=(-1)^{1+1} \mathrm{M}_{11}=1 \times 3=3\)

⇒ \(A_{12}=(-1)^{1+2} M_{12}=(-1) \times\) 0=0

⇒ \(A_{21}=(-1)^{2+1} M_{21}=(-1) \times(-4)\)=4

and \(\mathrm{A}_{22}=(-1)^{2+2} \mathrm{M}_{22}=1 \times \)2=2

2. Here, \(\left|\begin{array}{ll}\mathrm{a} & \mathrm{c} \\ \mathrm{b} & \mathrm{d}\end{array}\right|\) is given

Minors, \(\mathrm{M}_{11}=\mathrm{d}, \mathrm{M}_{12}=\mathrm{b}, \mathrm{M}_{21}=\mathrm{c}\) and \(\mathrm{M}_{22}=\mathrm{a}\)

Also cofactors, \(A_{11}=(-1)^{1+1} M_{11}=1 \times\) d=d

⇒ \(A_{12}=(-1)^{1+2} M_{12}=(-1) \times \)b=-b

⇒ \(A_{21}=(-1)^{2+1} M_{21}=(-1) \times c\)=-c

and \(A_{22}=(-1)^{2+2} \mathrm{M}_{22}=1 \times \mathrm{a}=\mathrm{a}\)

Question 2. \(\left|\begin{array}{lll}1 & 0 & 0 \\ 0 & 1 & 0 \\ 0 & 0 & 1\end{array}\right|\)

\(\left|\begin{array}{ccc}1 & 0 & 4 \\ 3 & 5 & -1 \\ 0 & 1 & 2\end{array}\right|\)

Solution:

1. Here, \(\left|\begin{array}{lll}
1 & 0 & 0 \\
0 & 1 & 0 \\
0 & 0 & 1
\end{array}\right|\) is given

Minors of elements of the first row are:

⇒ \(M_{11}=\left|\begin{array}{ll}
1 & 0 \\
0 & 1
\end{array}\right|=1-0=1, M_{12}=\left|\begin{array}{ll}
0 & 0 \\
0 & 1
\end{array}\right|\)=0-0=0

and \(M_{13}=\left|\begin{array}{ll}
0 & 1 \\
0 & 0
\end{array}\right|\)=0-0=0

Minors of elements of the second row are:

\(M_{21}=\left|\begin{array}{ll}
0 & 0 \\
0 & 1
\end{array}\right|=0-0=0, M_{22}=\left|\begin{array}{ll}
1 & 0 \\
0 & 1
\end{array}\right|\)=1-0=1

and  \(M_{23}=\left|\begin{array}{ll}
1 & 0 \\
0 & 0
\end{array}\right|\)=0-0=0

Minors of elements of the third row are:

⇒ \(M_{31}=\left|\begin{array}{ll}
0 & 0 \\
1 & 0
\end{array}\right|=0-0=0, M_{32}=\left|\begin{array}{ll}
1 & 0 \\
0 & 0
\end{array}\right|\)=0-0=0

and \(M_{33}=\left|\begin{array}{ll}
1 & 0 \\
0 & 1
\end{array}\right|\)=1-0=1

Hence, the cofactors of elements of the first row are:

⇒ \(A_{11}=(-1)^{1+1} M_{11}=1 \times 1=1, A_{12}\)

=\((-1)^{162} M_{12}=(-1) \times 0=0, A_{13}=(-1)^{1+1} M_{13}=1 \times\) 0=0

The cofactors of elements of the second row are:

⇒ \(A_{21}=(-1)^{2+1} M_{21}=(-1) \times 0=0, A_{22}=(-1)^{2+2} M_{21}=1 \times \)1=1,

⇒ \(A_{23}=(-1)^{2 \times 3} M_3=(-1) \times \)0=0

The cofactors of elements of the third row are:

⇒ \(A_{31}=(-1)^{3+1} M_{34}=1 \times 0=0, A_{32}=(-1)^{3.2} M_{12}=(-1) \times\) 0=0

⇒ \(A_{33}=(-1)^{3,3} M_{33}=1 \times \)1=1

2. Here, \(\left|\begin{array}{ccc}
1 & 0 & 4 \\
3 & 5 & -1 \\
0 & 1 & 2
\end{array}\right|\) is given

Minors of elements of the first row are:

⇒ \(\mathrm{M}_{11}=\left|\begin{array}{cc}
5 & -1 \\
1 & 2
\end{array}\right|=10+1=11, \mathrm{M}_{13}=\left|\begin{array}{cc}
3 & -1 \\
0 & 2
\end{array}\right|=6-0=6\) and

⇒ \(\mathrm{M}_{13}=\left|\begin{array}{ll}
3 & 5 \\
⇒ 0 & 1
\end{array}\right|\)=3-0=3

Minors of elements of the second row are:

⇒ \(M_{31}=\left|\begin{array}{ll}
0 & 4 \\
1 & 2
\end{array}\right|\)=0-4=-4,

⇒ \(M_{23}=\left|\begin{array}{ll}
1 & 4 \\
0 & 2
\end{array}\right|\)=2-0=2 and

\(M_{23}=\left|\begin{array}{ll}
1 & 0 \\
0 & 1
\end{array}\right|\)=1-0=1

Minors of elements of the third row are:

⇒ \(M_{31}=\left|\begin{array}{cc}
0 & 4 \\
5 & -1
\end{array}\right|\)=0-20=-20,

⇒ \(M_{32}=\left|\begin{array}{cc}
1 & 4 \\
3 & -1
\end{array}\right|\)=-1-12=-13 and

⇒ \(M_{33}=\left|\begin{array}{ll}
1 & 0 \\
3 & 5
\end{array}\right|\)=5-0=5

Hence, the cofactors of elements of the first row are:

⇒ \(A_{11}=(-1)^{1+1} M_{14}=1 \times 11=11, A_{12}=(-1)^{1+2} M_{12}=(-1) \times 6=-6, A_{13}\)

=\((-1)^{1+3} M_{13}=1 \times\) 3=3

The cofactors of elements of the second row are:

\(A_{21}=(-1)^{2+1} M_{21}=(-1) \times(-4)=4, A_{22}=(-1)^{2+2} M_{22}=1 \times \)2=2

⇒ \(A_{21}=(-1)^{2+3} M_{21}=(-1) \times\) 1=-1

The cofactors of elements of the third row are:

⇒ \(A_{11}=(-1)^{3+1} M_{31}=1 \times(-20)=-20, A_{12}=(-1)^{3+2} M_{12}=(-1) \times(-13)=13\)

⇒ \(A_{31}=(-1)^{3-3} M_{33}=1 \times\) 5=5

Question 3. Using cofactors of the elements of the second row, evaluate \(\Delta=\left|\begin{array}{lll}
5 & 3 & 8 \\
2 & 0 & 1 \\
1 & 2 & 3
\end{array}\right|\)

Solution:

Given, \(\Delta=\left|\begin{array}{lll}5 & 3 & 8 \\ 2 & 0 & 1 \\ 1 & 2 & 3\end{array}\right|\)

The cofactors of the elements of the second row are:

⇒ \(\mathrm{A}_{21}=(-1)^{2+1}\left|\begin{array}{ll}
3 & 8 \\
2 & 3
\end{array}\right|\)=-(9-16)=7,

⇒ \([\mathrm{A}_4=(-1)^{* / 1} \mathrm{M}_4]\)

⇒ \(\mathrm{A}_{22}=(-1)^{2+2}\left|\begin{array}{ll}
5 & 8 \\
1 & 3
\end{array}\right|\)=(15-8)=7

and \(A_y=(-1)^{2+3}\left|\begin{array}{ll}5 & 3 \\ 1 & 2\end{array}\right|=-(10-3)=-7\)

Now, expansion of \(\Delta\) using cofactors of elements of second row is given by

⇒ \(\Delta=a_{y 1} A_{y t}+a_{y 2} A_{y z}+a_y A_{y y}=(2 \times 7)+(0 \times 7)+(1) \times(-7)=14-7=7\)

Question 4. Using cofactors of elements of the third column, evaluate \(\Delta=\left|\begin{array}{lll}
1 & x & y z \\
1 & y & z x \\
1 & z & x y
\end{array}\right|\)

Solution:

Given, \(\Delta=\left|\begin{array}{lll}1 & x & y z \\ 1 & y & x \\ 1 & z & x y\end{array}\right|\)

The cofactors of the elements of the third column are:

⇒ \(A_{13}=(-1)^{1+3}\left|\begin{array}{ll}
1 & y \\
1 & z
\end{array}\right|=1(z-y)\)=z-y,

⇒ \(A_{23}=(-1)^{2+3}\left|\begin{array}{ll}
1 & x \\
1 & z
\end{array}\right|\)=-1(z-x)=x-z,

⇒ \(A_{x 3}=(-1)^{3+3}\left|\begin{array}{ll}
1 & x \\
1 & y
\end{array}\right|\)=1(y-x)=y-x

Now, the expansion of A using cofactors of elements of the third column is given by

\(\Delta =a_{13} A_{13}+a_{39} A_{23}+a_{33} A_{31}=y z(z-y)+z x(x-z)+x y(y-x)\)

=\(y z^2-y^2 z+z x^2-z^2 x+x y^2-x^2 \)

y=\(x^2(z-y)+x\left(y^2-z^2\right)+y z(z-y)\)

= \((z-y)\left\{x^2-x(y+z)+y z\right\}=(z-y)\left\{x^2-x y-x z+y z\right\}\)

= (z-y)[x(x-y)-z(x-y)]=(y-z)(x-y)(z-x)=(x-y)(y-z)(z-x)

Question 5. If \(\Delta=\left|\begin{array}{lll}
a_{11} & a_{12} & a_{13} \\
a_{21} & a_{22} & a_{23} \\
a_{31} & a_{32} & a_{33}
\end{array}\right|\)
 and \(A_1\) is cofactor of \(a_{i j}\), then value of \(\Delta\) is given by ?

  1. \(a_{11} A_{31}+a_{12} A_{32}+a_{19} A_{30}\)
  2. \(a_{11} A_{11}+a_{12} A_{21}+a_{10} A_n\)
  3. \(a_{31} A_{11}+a_{23} A_{12}+a_{2 y} A_{13}\)
  4. \(a_{11} \mathbf{A}_{11}+a_{2 /} A_{24}+a_n A_3\)

Solution: 4. \(a_{11} \mathbf{A}_{11}+a_{2 /} A_{24}+a_n A_3\)

⇒ \(\Delta\) is equal to the sum of the products of the elements of a row (or a column) with their corresponding cofactors.

⇒ \(\Delta=a_{11} A_{11}+a_{12} A_{12}+a_{13} A_{13} \text { or } a_{21} A_{21}+a_{22} A_{22}+a_{23} A_{23}\)

or \(a_{12} A_{12}+a_{22} A_{22}+a_{12} A_{32}\) or \(a_{13} A_{13}+a_{23} A_{23}+a_{33} A_{33}\)

Hence, the sum of the products of the elements of the first column with their corresponding cofactors is \(\Delta=a_{11} A_{11}+a_{21} A_{21}+a_{31} A_{31}\)

Hence, the correct option is 4.

Determinants Exercise 4.4

Question 1. \(\left[\begin{array}{ll}1 & 2 \\ 3 & 4\end{array}\right]\)

Solution :

Let A=\(\left[\begin{array}{ll}
1 & 2 \\
3 & 4
\end{array}\right]\)

⇒  \(A_{11}=4, A_{12}=-3, A_{21}\)=-2 and \(A_{22}\)=1

⇒ \({adj} A=\left[A_4\right]=\left[\begin{array}{ll}
A_{11} & A_{22} \\
A_{21} & A_{22}
\end{array}\right]^{\prime}\)

= \(\left[\begin{array}{cc}
4 & -3 \\
-2 & 1
\end{array}\right]=\left[\begin{array}{cc}
4 & -2 \\
-3 & 1
\end{array}\right]\)

Question 2. \(\left[\begin{array}{ccc}
1 & -1 & 2 \\
2 & 3 & 5 \\
-2 & 0 & 1
\end{array}\right]\)

Solution:

Let A=\(\left[\begin{array}{ccc}
1 & -1 & 2 \\
2 & 3 & 5 \\
-2 & 0 & 1
\end{array}\right]\)

The cofactors of elements of the first row are:

\(A_{11}=\left|\begin{array}{ll}
3 & 5 \\
0 & 1
\end{array}\right|\)=3-0=3,

⇒  \(A_{12}=-\left|\begin{array}{cc}
2 & 5 \\
-2 & 1
\end{array}\right|=-(2+10)=-12 \text { and } A_{13}=\left|\begin{array}{cc}
2 & 3 \\
-2 & 0
\end{array}\right|\)=0-(-6)=6

The cofactors of elements of the second row are:

⇒ \(A_{21}=-\left|\begin{array}{cc}
-1 & 2 \\
0 & 1
\end{array}\right|=-(-1-0)=1, A_2=\left|\begin{array}{cc}
1 & 2 \\
-2 & 1
\end{array}\right|\)

= \((1+4)=5 and A_{23}=-\left|\begin{array}{cc}
1 & -1 \\
-2 & 0
\end{array}\right|\)=-(0-2)=2

The cofactors of elements of the third row are:

⇒  \(A_{11}=\left|\begin{array}{cc}
-1 & 2 \\
3 & 5
\end{array}\right|=(-5-6)=-11,\)

⇒ \(A_{93}=-\left|\begin{array}{ll}
1 & 2 \\
2 & 5
\end{array}\right|\)=-(5-4)=-1

and \(A_{33}=\left|\begin{array}{cc}
1 & -1 \\
2 & 3
\end{array}\right|\)=3+2=5

Hence,\({adj}(A)=\left[A_0\right]^{\prime}=\left[\begin{array}{ccc}3 & -12 & 6 \\ 1 & 5 & 2 \\ -11 & -1 & 5\end{array}\right]^{\prime}=\left[\begin{array}{ccc}3 & 1 & -11 \\ -12 & 5 & -1 \\ 6 & 2 & 5\end{array}\right]\)

Question 3. \(\left[\begin{array}{cc}
2 & 3 \\
-4 & -6
\end{array}\right]\)

Solution:

Let A=\(\left[\begin{array}{cc}2 & 3 \\ -4 & -6\end{array}\right],|A|=\left|\begin{array}{cc}2 & 3 \\ -4 & -6\end{array}\right|=-12-(-12)\)=-12+12=0

⇒ \(|\mathrm{A}| \mathrm{I}=0\left[\begin{array}{ll}
1 & 0 \\
0 & 1
\end{array}\right]=\left[\begin{array}{ll}
0 & 0 \\
0 & 0
\end{array}\right]=\mathrm{O}\)

Cofactors of A are \(A_{11}=-6, A_{12}=4, A_{21}=-3, A_2=2\)

⇒ \({adj}(A)=\left[A_{i 1}\right]=\left[\begin{array}{ll}
-6 & 4 \\
-3 & 2
\end{array}\right]=\left[\begin{array}{cc}
-6 & -3 \\
4 & 2
\end{array}\right]\)

Now, \((adj A) \mathrm{A}=\left[\begin{array}{cc}-6 & -3 \\ 4 & 2\end{array}\right]\left[\begin{array}{cc}2 & 3 \\ -4 & -6\end{array}\right]=\left[\begin{array}{cc}-12+12 & -18+18 \\ 8-8 & 12-12\end{array}\right]=\left[\begin{array}{ll}0 & 0 \\ 0 & 0\end{array}\right]=\mathrm{0}\)

Also, \(A(adj A)=\left[\begin{array}{cc}2 & 3 \\ -4 & -6\end{array}\right]\left[\begin{array}{cc}-6 & -3 \\ 4 & 2\end{array}\right]=\left[\begin{array}{cc}-12+12 & -6+6 \\ 24-24 & 12-12\end{array}\right]=\left[\begin{array}{ll}0 & 0 \\ 0 & 0\end{array}\right]\)=0

Hence, \(\mathrm{A}({adj} \mathrm{A})=({adj} \mathrm{A}) \mathrm{A}=|\mathrm{A}| \mathrm{I}_2\).

Question 4. \(\left[\begin{array}{ccc}
1 & -1 & 2 \\
3 & 0 & -2 \\
1 & 0 & 3
\end{array}\right]\)

Solution:

Let A=\(\left[\begin{array}{ccc}1 & -1 & 2 \\ 3 & 0 & -2 \\ 1 & 0 & 3\end{array}\right]\)

Now, \(|A|=\left|\begin{array}{ccc}1 & -1 & 2 \\ 3 & 0 & -2 \\ 1 & 0 & 3\end{array}\right|\)=1(0-0)-(-1)(9+2)+2(0-0)=0+11+0=11

⇒ \(|A| I=11\left[\begin{array}{lll}
1 & 0 & 0 \\
0 & 1 & 0 \\
0 & 0 & 1
\end{array}\right]\)

= \(\left[\begin{array}{ccc}
11 & 0 & 0 \\
0 & 11 & 0 \\
0 & 0 & 11
\end{array}\right]\)

Cofactors of A are:

⇒ \(A_{11}=0, A_{12}=-(9+2)=-11, A_{13}=0, A_{21}=-(-3-0)=3, A_{22}=3-2=1, A_{33}\)

=-(0+1)=-1,

⇒ \(A_{31}=2-0=2, A_{32}=-(-2-6)=8, A_{39}=0+3=3 \)

⇒ \({adj}(A)=\left[A_7\right]^{\prime}=\left[\begin{array}{ccc}
0 & -11 & 0 \\
3 & 1 & -1 \\
2 & 8 & 3
\end{array}\right]=\left[\begin{array}{ccc}
0 & 3 & 2 \\
-11 & 1 & 8 \\
0 & -1 & 3
\end{array}\right]\)

Now, \(A({adj} A)=\left[\begin{array}{ccc}
1 & -1 & 2 \\
3 & 0 & -2 \\
1 & 0 & 3
\end{array}\right]\left[\begin{array}{ccc}
0 & 3 & 2 \\
-11 & 1 & 8 \\
0 & -1 & 3
\end{array}\right]\)

= \(\left[\begin{array}{lll}
0+11+0 & 3-1-2 & 2-8+6 \\
0+0+0 & 9+0+2 & 6+0-6 \\
0+0+0 & 3+0-3 & 2+0+9
\end{array}\right]\)

= \(\left[\begin{array}{ccc}
11 & 0 & 0 \\
0 & 11 & 0 \\
0 & 0 & 11
\end{array}\right]\)

Also, \(({adj} A) A=\left[\begin{array}{ccc}0 & 3 & 2 \\ -11 & 1 & 8 \\ 0 & -1 & 3\end{array}\right]\left[\begin{array}{ccc}1 & -1 & 2 \\ 3 & 0 & -2 \\ 1 & 0 & 3\end{array}\right]\)

= \(\left[\begin{array}{ccc}
0+9+2 & 0+0+0 & 0-6+6 \\
-11+3+8 & 11+0+0 & -22-2+24 \\
0-3+3 & 0+0+0 & 0+2+9
\end{array}\right]\)

= \(\left[\begin{array}{ccc}
11 & 0 & 0 \\
0 & 11 & 0 \\
0 & 0 & 11
\end{array}\right]\)

=\(11\left[\begin{array}{lll}
1 & 0 & 0 \\
0 & 1 & 0 \\
0 & 0 & 1
\end{array}\right]\)

Hence, \(\mathrm{A}({adj} \mathrm{A})=({adj} \mathrm{A}) \mathrm{A}=|\mathrm{A}| \mathrm{I}\)

Question 5. \(\left[\begin{array}{cc}
2 & -2 \\
4 & 3
\end{array}\right]\)

Solution:

Let A=\(\left[\begin{array}{cc}2 & -2 \\ 4 & 3\end{array}\right].\)

We have, \(|\mathrm{A}|=\left|\begin{array}{cc}2 & -2 \\ 4 & 3\end{array}\right|=6-(-8)=14 \neq 0 \mathrm{A}^{-1}\) exists

Cofactors of \(\mathrm{A}_{11} are \mathrm{A}_{11}\)=3,

⇒ \(\mathrm{~A}_{12}=-4, \mathrm{~A}_{21}=2, \mathrm{~A}_{22}=2\)

⇒ \({adj}(\mathrm{A})=\left[\mathrm{A}_{i j}\right]^{\prime}=\left[\begin{array}{cc}
3 & -4 \\
2 & 2
\end{array}\right]^{\prime}=\left[\begin{array}{cc}
3 & 2 \\
-4 & 2
\end{array}\right]\)

Now, \(A^{-1}=\frac{1}{|A|}({adj} A)=\frac{1}{14}\left[\begin{array}{cc}
3 & 2 \\
-4 & 2
\end{array}\right]\)

= \(\left[\begin{array}{cc}
\frac{3}{14} & \frac{2}{14} \\
-\frac{4}{14} & \frac{2}{14}
\end{array}\right]\)

= \(\left[\begin{array}{cc}
\frac{3}{14} & \frac{1}{7} \\
-\frac{2}{7} & \frac{1}{7}
\end{array}\right]\)

Question 6. \(\left[\begin{array}{ll}
-1 & 5 \\
-3 & 2
\end{array}\right]\)

Solution:

Let A=\(\left[\begin{array}{ll}-1 & 5 \\ -3 & 2\end{array}\right]\).

We have, \(|A|==-2-(-15)=13 \neq\) 0

\(\mathrm{A}^{-1}\) exists

Now, cofactors of A are \(A_{11}=2, A_{12}=3, A_{21}=-5, A_{21}\)=-1

⇒ \({adj}(A)=\left[A_0\right]^{\prime}=\left[\begin{array}{cc}
2 & 3 \\
-5 & -1
\end{array}\right]^{\prime}=\left[\begin{array}{ll}
2 & -5 \\
3 & -1
\end{array}\right]\)

Now, \(A^{-1}=\frac{1}{|A|}({adj} A)=\frac{1}{13}\left[\begin{array}{cc}2 & -5 \\ 3 & -1\end{array}\right]\)

= \(\left[\begin{array}{cc}\frac{2}{13} & -\frac{5}{13} \\ \frac{3}{13} & -\frac{1}{13}\end{array}\right]\)

Question 7. \(\left[\begin{array}{lll}
1 & 2 & 3 \\
0 & 2 & 4 \\
0 & 0 & 5
\end{array}\right]\)

Solution:

Let A=\(\left[\begin{array}{lll}1 & 2 & 3 \\ 0 & 2 & 4 \\ 0 & 0 & 5\end{array}\right]\)

We have, |\(\mathrm{A}|=1(10-0)-2(0-0)+3(0-0)=10 \neq 0\)

⇒ \(\mathrm{A}^{-1}\) exists

Now, cofactors of A are

⇒ \(A_{11}\)=10-0=10, \(A_{12}\)=-(0-0)=0, \(A_{13}\)=0-0=0,

⇒ \(A_{21}\)=-(10-0)=-10, \(A_{22}\)=5-0=5, \(A_{23}\)=-(0-0)=0 ,

⇒ \(\mathrm{A}_{31}=8-6=2, \mathrm{~A}_{32}=-(4-0)=-4, \quad \mathrm{~A}_{35}=2-0=2 \)

⇒ \({adj}(\mathrm{A})=\left[\mathrm{A}_{i j}\right]^{\prime}=\left[\begin{array}{ccc}
10 & 0 & 0 \\
-10 & 5 & 0 \\
2 & -4 & 2
\end{array}\right]^{\prime}\)

= \(\left[\begin{array}{ccc}
10 & -10 & 2 \\
0 & 5 & -4 \\
0 & 0 & 2
\end{array}\right] \)

Now, \(A^{-1}=\frac{1}{|A|}({adj} A)=\frac{1}{10}\left[\begin{array}{ccc}
10 & -10 & 2 \\
0 & 5 & -4 \\
0 & 0 & 2
\end{array}\right]\)

= \(\left[\begin{array}{ccc}
1 & -1 & \frac{1}{5} \\
0 & \frac{1}{2} & -\frac{2}{5} \\
0 & 0 & \frac{1}{5}
\end{array}\right]\).

Question 8. \(\left[\begin{array}{ccc}
1 & 0 & 0 \\
3 & 3 & 0 \\
5 & 2 & -1
\end{array}\right]\)

Solution:

Let A=\(\left[\begin{array}{ccc}1 & 0 & 0 \\ 3 & 3 & 0 \\ 5 & 2 & -1\end{array}\right]\)

⇒ \(|A|=1(-3-0)-0+0=-3 \neq\) 0

⇒ \(\mathrm{A}^{-1}\) exists

Cofactors of A are:

⇒ \(A_{11}=-3-0=-3, A_{12}=-(-3-0)=3, A_{13}=6-15=-9\),

⇒ \(A_{21}=-(0-0)=0, A_{22}=-1-0=-1, A_{23}=-(2-0)=-2\),

⇒ \(A_{31}=0-0=0, A_{12}=-(0-0)=0, A_{33}\)=3-0=3

⇒ \({adj}(A)=\left[A_{i 1}\right]^{\prime}=\left[\begin{array}{ccc}
-3 & 3 & -9 \\
0 & -1 & -2 \\
0 & 0 & 3
\end{array}\right]^{\prime}\)

= \(\left[\begin{array}{ccc}
-3 & 0 & 0 \\
3 & -1 & 0 \\
-9 & -2 & 3
\end{array}\right]\)

Now, \(A^{-1}=\frac{1}{|A|}({adj} A)=\frac{1}{-3}\left[\begin{array}{ccc}
-3 & 0 & 0 \\
3 & -1 & 0 \\
-9 & -2 & 3
\end{array}\right]\)

= \(\left[\begin{array}{ccc}
1 & 0 & 0 \\
-1 & \frac{1}{3} & 0 \\
3 & \frac{2}{3} & -1
\end{array}\right]\)

Question 9. \([\left[\begin{array}{ccc}
2 & 1 & 3 \\
4 & -1 & 0 \\
-7 & 2 & 1
\end{array}\right]\)

Solution:

Let A=\(\left[\begin{array}{ccc}2 & 1 & 3 \\ 4 & -1 & 0 \\ -7 & 2 & 1\end{array}\right]\)

|A|=2(-1-0)-1(4-0)+3(8-7)=-2-4+3=-3 \(\neq\) 0

⇒ \(\mathrm{A}^{-1}\) exists

Cofactors of A are:

⇒ \(A_{11}=-1-0=-1, A_{12}=-(4-0)=-4, A_{13}=8-7=1\)

⇒ \(A_{21}=-(1-6)=5, A_{22}=2+21=23, \quad A_{33}\)=-(4+7)=-11,

⇒ \(A_{31}=0+3=3, A_{32}=-(0-12), A_{33}\)=-2-4=-6

⇒ \({adj}(A)=\left[A_{61}\right]^{\prime}=\left[\begin{array}{ccc}
-1 & -4 & 1 \\
5 & 23 & -11 \\
3 & 12 & -6
\end{array}\right]\)

=\(\left[\begin{array}{ccc}
-1 & 5 & 3 \\
-4 & 23 & 12 \\
1 & -11 & -6
\end{array}\right]\)

Now, \(A^{-1}=\frac{1}{|A|}({adj}A)\)

= \(\frac{1}{-3}\left[\begin{array}{ccc}
-1 & 5 & 3 \\
-4 & 23 & 12 \\
1 & -11 & -6
\end{array}\right]\)

= \(\left[\begin{array}{ccc}
\frac{1}{3} & -\frac{5}{3} & -1 \\
\frac{4}{3} & -\frac{23}{3} & -4 \\
-\frac{1}{3} & \frac{11}{3} & 2
\end{array}\right]\)

Question 10. \(\left[\begin{array}{ccc}
1 & -1 & 2 \\
0 & 2 & -3 \\
3 & -2 & 4
\end{array}\right]\)

Solution:

Let A=\(\left[\begin{array}{ccc}1 & -1 & 2 \\ 0 & 2 & -3 \\ 3 & -2 & 4\end{array}\right]\)

We have, \(|A|=\left|\begin{array}{ccc}1 & -1 & 2 \\ 0 & 2 & -3 \\ 3 & -2 & 4\end{array}\right|\)

=1(8-6)-(-1)(0+9)+2(0-6)=2+9-12=-1 \(\neq\) 0

⇒ \(\mathrm{A}^{-1}\) exists

Cofactors of A are:

⇒ \(A_{11}=8-6=2, A_{12}=-(0+9)=-9, A_{12}=0-6=-6\),

⇒ \(A_{21}=-(-4+4)=0, A_{22}=4-6=-2, A_{23}=-(-2+3)=-1\)

⇒ \(A_{31}=3-4=-1, A_{22}=-(-3-0)=3, A_{33}=2-0=2\)

⇒ \({adj}(A)=\left[A_4\right]^{+}=\left[\begin{array}{ccc}
2 & -9 & -6 \\
0 & -2 & -1 \\
-1 & 3 & 2
\end{array}\right]^{\prime}=\left[\begin{array}{ccc}
2 & 0 & -1 \\
-9 & -2 & 3 \\
-6 & -1 & 2
\end{array}\right]\)

Now; \(A^{-1}=\frac{1}{|A|}({adj} A)=\frac{1}{-1}\left[\begin{array}{ccc}
2 & 0 & -1 \\
-9 & -2 & 3 \\
-6 & -1 & 2
\end{array}\right]\)

= \(\left[\begin{array}{ccc}
-2 & 0 & 1 \\
9 & 2 & -3 \\
6 & 1 & -2
\end{array}\right]\)

Question 11. \(\left[\begin{array}{ccc}
1 & 0 & 0 \\
0 & \cos \alpha & \sin \alpha \\
0 & \sin \alpha & -\cos \alpha
\end{array}\right]\)

Solution:

Let A=\(\left[\begin{array}{ccc}1 & 0 & 0 \\ 0 & \cos \alpha & \sin \alpha \\ 0 & \sin \alpha & -\cos \alpha\end{array}\right] \Rightarrow|A|\)

= \(\left|\begin{array}{ccc}1 & 0 & 0 \\ 0 & \cos \alpha & \sin \alpha \\ 0 & \sin \alpha & -\cos \alpha\end{array}\right|=1\left(-\cos ^2 \alpha-\sin ^2 \alpha\right)\)

=-\(\left(\cos ^2 \alpha+\sin ^2 \alpha\right)=-1 \neq 0 \left[ \cos ^2 \theta+\sin ^2 \theta=1\right]\)

⇒ \(\mathrm{A}^{-1}\) exists

Cofactors of A are:

⇒ \(\mathrm{A}_{11}=-\cos ^2 \alpha-\sin ^2 \alpha\)=-1,

⇒ \(\mathrm{~A}_{12}=-(0-0)=0, \mathrm{~A}_{13}\)=0-0=0

⇒ \(\mathrm{~A}_{21}=-(0-0)=0, \mathrm{~A}_{22}=-\cos \alpha-0=-\cos \alpha, \mathrm{A}_{23}=-(\sin \alpha-0)=-\sin \alpha,\)

⇒ \(\mathrm{A}_{21}=0-0=0, \mathrm{~A}_{32}=-(\sin \alpha-0)=-\sin \alpha, \mathrm{A}_{35}=\cos \alpha-0=\cos \alpha \)

⇒ \({adj}(\mathrm{A})=\left[\mathrm{A}_{1 j}\right]^{\prime}=\left[\begin{array}{ccc}
-1 & 0 & 0 \\
0 & -\cos \alpha & -\sin \alpha \\
0 & -\sin \alpha & \cos \alpha
\end{array}\right]^{\prime}\)

= \(\left[\begin{array}{ccc}
-1 & 0 & 0 \\
0 & -\cos \alpha & -\sin \alpha \\
0 & -\sin \alpha & \cos \alpha
\end{array}\right]\)

Now, \(\mathrm{A}^{-1}=\frac{1}{|\mathrm{~A}|}({adj} \mathrm{A})=\frac{1}{-1}\left[\begin{array}{ccc}
-1 & 0 & 0 \\
0 & -\cos \alpha & -\sin \alpha \\
0 & -\sin \alpha & \cos \alpha
\end{array}\right]\)

= \(\left[\begin{array}{ccc}
1 & 0 & 0 \\
0 & \cos \alpha & \sin \alpha \\
0 & \sin \alpha & -\cos \alpha
\end{array}\right]\)

Question 12. Let A=\(\left[\begin{array}{ll}
3 & 7 \\
2 & 5
\end{array}\right]\) and\( \mathrm{B}=\left[\begin{array}{ll}
6 & 8 \\
7 & 9
\end{array}\right]\). Verify that \((A B)^{-1}=B^{-1} A^{-1}\).

Solution:

Given, A=\(\left[\begin{array}{ll}3 & 7 \\ 2 & 5\end{array}\right] ;|A|=\left|\begin{array}{ll}3 & 7 \\ 2 & 5\end{array}\right|=15-14=1 \neq 0\)

⇒ \(\mathrm{A}^{-1}\) exists

Cofactors of A are \(\mathrm{A}_{11}\)=5,

⇒ \(\mathrm{~A}_{12}=-2, \mathrm{~A}_{21}=-7, \mathrm{~A}_{27}\)=3

⇒ \({adj}(A)=\left[A_{i j}\right]^{\prime}=\left[\begin{array}{cc}5 & -2 \\ -7 & 3\end{array}\right]^{\prime}=\left[\begin{array}{cc}5 & -7 \\ -2 & 3\end{array}\right]\)

Now, \(A^{-1}=\frac{1}{|A|}({adj} A)=\frac{1}{1}\left[\begin{array}{cc}5 & -7 \\ -2 & 3\end{array}\right]=\left[\begin{array}{cc}5 & -7 \\ -2 & 3\end{array}\right] \)

Here, B=\(\left[\begin{array}{ll}6 & 8 \\ 7 & 9\end{array}\right] |B|=\left|\begin{array}{ll}6 & 8 \\ 7 & 9\end{array}\right|=54-56=-2 \neq \)

⇒ \(\mathrm{B}^{-1}\) exists

Cofactors of B are \(\mathrm{B}_{11}=9, \mathrm{~B}_{22}=-7, \mathrm{~B}_{21}=-8, \mathrm{~B}_{22}\)=6

⇒ \({adj}(B)=\left[B_{i j}\right]^{\prime}=\left[\begin{array}{cc}
9 & -7 \\
-8 & 6
\end{array}\right]^{\prime}=\left[\begin{array}{cc}
9 & -8 \\
-7 & 6
\end{array}\right]\)

⇒ \(B^{-1}=\frac{1}{|B|}(\text { adj } B)=\frac{1}{-2}\left[\begin{array}{cc}
9 & -8 \\
-7 & 6
\end{array}\right]\)

Now,\(B^{-1} A^{-1} =\frac{1}{-2}\left[\begin{array}{cc}
9 & -8 \\
-7 & 6
\end{array}\right]\left[\begin{array}{cc}
5 & -7 \\
-2 & 3
\end{array}\right]\)

=\(\frac{1}{-2}\left[\begin{array}{cc}
45+16 & -63-24 \\
-35-12 & 49+18
\end{array}\right] \)

=\(\frac{1}{-2}\left[\begin{array}{cc}
61 & -87 \\
-47 & 67
\end{array}\right]\)

=\(\left[\begin{array}{cc}
-\frac{61}{2} & \frac{87}{2} \\
\frac{47}{2} & -\frac{67}{2}
\end{array}\right]\) Equation 1

Now, let \(\mathrm{C}=\mathrm{AB}=\left[\begin{array}{ll}3 & 7 \\ 2 & 5\end{array}\right]\left[\begin{array}{ll}6 & 8 \\ 7 & 9\end{array}\right]\)

=\(\left[\begin{array}{ll}18+49 & 24+63 \\ 12+35 & 16+45\end{array}\right]\)

=\(\left[\begin{array}{ll}67 & 87 \\ 47 & 61\end{array}\right]\)

⇒ \(|\mathrm{AB}|=\left|\begin{array}{ll}
67 & 87 \\
47 & 61
\end{array}\right|\)

=\((67 \times 61)-(47 \times 87)=4087-4089=-2 \neq 0\)

∴ \(\mathrm{C}^{-1} exists\),

Cofactors of C are \(C_{11}=61, C_{12}=-47, C_{21}=-87, C_{22}=67\)

⇒ \({adj}(\mathrm{AB})=\left[\mathrm{C}_{\mathrm{ii}}\right]^{+}=\left[\begin{array}{cc}
61 & -47 \\
-87 & 67
\end{array}\right]^{\prime}=\left[\begin{array}{cc}
61 & -87 \\
-47 & 67
\end{array}\right]\)

⇒ \((\mathrm{AB})^{-1}=\frac{1}{|\mathrm{AB}|}(\mathrm{adj} \mathrm{AB})=\frac{1}{-2}\left[\begin{array}{cc}
61 & -87 \\
-47 & 67
\end{array}\right]\)

= \(\left[\begin{array}{cc}
-\frac{61}{2} & \frac{87}{2} \\
\frac{47}{2} & -\frac{67}{2}
\end{array}\right]\)  → Equation 2

From Eq. (1) and (2), we get \((A B)^{-1}=B^{-1} A^{-1}\)

Hence, the given result is proved.

Question 13. If A=\(\left[\begin{array}{cc}
3 & 1 \\
-1 & 2
\end{array}\right]\), show that \(A^2-5 A+7 I=0\). Hence, find \(A^{-1}\).

Solution:

Given, A=\(\left[\begin{array}{cc}3 & 1 \\ -1 & 2\end{array}\right]\)

⇒ \(A^2=A \cdot A=\left[\begin{array}{cc}
3 & 1 \\
-1 & 2
\end{array}\right]\left[\begin{array}{cc}
3 & 1 \\
-1 & 2
\end{array}\right]\)

= \(\left[\begin{array}{cc}
9-1 & 3+2 \\
-3-2 & -1+4
\end{array}\right]=\left[\begin{array}{cc}
8 & 5 \\
-5 & 3
\end{array}\right]\)

Now,\( A^2-5 A+7 I\)

= \(\left[\begin{array}{cc}
8 & 5 \\
-5 & 3
\end{array}\right]-5\left[\begin{array}{cc}
3 & 1 \\
-1 & 2
\end{array}\right]+7\left[\begin{array}{ll}
1 & 0 \\
0 & 1
\end{array}\right]\)

= \(\left[\begin{array}{cc}
8 & 5 \\
-5 & 3
\end{array}\right]-\left[\begin{array}{cc}
15 & 5 \\
-5 & 10
\end{array}\right]+\left[\begin{array}{ll}
7 & 0 \\
0 & 7
\end{array}\right]\)

= \(\left[\begin{array}{cc}
8-15+7 & 5-5+0 \\
-5+5+0 & 3-10+7
\end{array}\right]\)

=\(\left[\begin{array}{ll}
0 & 0 \\
0 & 0
\end{array}\right]\)=0

\(A^2-5 A+7 I\)=0

⇒ \(|A|=\left|\begin{array}{cc}
3 & 1 \\
-1 & 2
\end{array}\right|=6+1=7 \neq \)0, \(A^{-1}\) exists.

Now, A \(\cdot\) A-5 A=-71

Post multiplying by \(\mathrm{A}^{-1}\) on both sides, we get

A \(\cdot A\left(A^{-1}\right)-5 A^{-1}=-7 A^{-1} \Rightarrow A I-5 I=-7 A^{-1}\)

Using \(A^{-1}=I\) and \(I^{-1}=A^{-1}\)

⇒ \(A^{-1}=-\frac{1}{7}(A-5 I) \Rightarrow A^{-1}=\frac{1}{7}(5 I-A)=\frac{1}{7}\left(\left[\begin{array}{ll}
5 & 0 \\
0 & 5
\end{array}\right]-\left[\begin{array}{cc}
3 & -1 \\
-1 & 2
\end{array}\right]\right)\)

=\(\frac{1}{7}\left[\begin{array}{cc}
2 & -1 \\
1 & 3
\end{array}\right] \)

⇒  \(A^{-1}=\frac{1}{7}\left[\begin{array}{cc}
2 & -1 \\
1 & 3
\end{array}\right]\)

Question 14. For the matrix A=\(\left[\begin{array}{ll}
3 & 2 \\
1 & 1
\end{array}\right]\), find the numbers ‘ a ‘ and ‘ b ‘ such that \(A^2+a A+b I\)=0.

Solution:

Given, A=\(\left[\begin{array}{ll}3 & 2 \\ 1 & 1\end{array}\right]\)

⇒ \(A^2=A \cdot A=\left[\begin{array}{ll}
3 & 2 \\
1 & 1
\end{array}\right]\left[\begin{array}{ll}
3 & 2 \\
1 & 1
\end{array}\right]\)

=\(\left[\begin{array}{ll}
9+2 & 6+2 \\
3+1 & 2+1
\end{array}\right]=\left[\begin{array}{cc}
11 & 8 \\
4 & 3
\end{array}\right]\)

Given, \(\mathrm{A}^2+\mathrm{aA}+\mathrm{bI}\)=0

On putting the values of \(\mathrm{A}^2, \mathrm{~A}\) and 1 ; we get

⇒ \(\left[\begin{array}{ll}
11 & 8 \\
4 & 3
\end{array}\right]+\mathrm{a}\left[\begin{array}{ll}
3 & 2 \\
1 & 1
\end{array}\right]+\mathrm{b}\left[\begin{array}{ll}
1 & 0 \\
0 & 1
\end{array}\right]\)=0

⇒ \(\left[\begin{array}{cc}
11+3 a+b & 8+2 a+0 \\
4+a+0 & 3+a+b
\end{array}\right]=0 \)

⇒ \({\left[\begin{array}{cc}
11+3 a+b & 8+2 a \\
4+a & 3+a+b
\end{array}\right]=\left[\begin{array}{ll}
0 & 0 \\
0 & 0
\end{array}\right] }\)

If two matrices are equal, then their corresponding elements are also equal.

11+3a+b=0  → Equation 1

8+2 a=0  → Equation 2

4+a=0 → Equation 3

and 3+a+b=0  → Equation 4

Solving Eq. (3) and (4), we get 4+\(\mathrm{a}=0 \Rightarrow \mathrm{a}\)=-4

And 3+a+b=0 \(\Rightarrow 3-4+b=0 \Rightarrow\) b=1

Thus, a=-4 and b=1

Question 15. For the matrix A=\(\left[\begin{array}{ccc}
1 & 1 & 1 \\
1 & 2 & -3 \\
2 & -1 & 3
\end{array}\right]\), show that \(A^3-6 A^2+5 A+111\)=0. Hence, find \(A^{-1}\),

Solution:

Given, A=\(\left[\begin{array}{ccc}1 & 1 & 1 \\ 1 & 2 & -3 \\ 2 & -1 & 3\end{array}\right] ;|A|=1(6-3)-1(3+6)+1(-1-4)-3-9-5=-11 \neq 0\)

⇒ \(\mathrm{A}^{-1}\) exists

Now,\(\mathbf{A}^2=\mathbf{A} \cdot \mathrm{A}  =\left[\begin{array}{ccc}
1 & 1 & 1 \\
1 & 2 & -3 \\
2 & -1 & 3
\end{array}\right]\left[\begin{array}{ccc}
1 & 1 & 1 \\
1 & 2 & -3 \\
2 & -1 & 3
\end{array}\right]\)

=\(\left[\begin{array}{ccc}
1+1+2 & 1+2-1 & 1-3+3 \\
1+2-6 & 1+4+3 & 1-6-9 \\
2-1+6 & 2-2-3 & 2+3+9
\end{array}\right]=\left[\begin{array}{ccc}
4 & 2 & -1 \\
-3 & 8 & -14 \\
7 & -3 & 14
\end{array}\right]\)

And \(A^3=A^2 \cdot A =\left[\begin{array}{ccc}
4 & 2 & 1 \\
-3 & 8 & -14 \\
7 & -3 & 14
\end{array}\right]\left[\begin{array}{ccc}
1 & 1 & 1 \\
1 & 2 & -3 \\
2 & -1 & 3
\end{array}\right]\)

=\(\left[\begin{array}{ccc}
4+2+2 & 4+4-1 & 4-6+3 \\
-3+8-28 & -3+16+14 & -3-24-42 \\
7-3+28 & 7-6-14 & 7+9+42
\end{array}\right]\)

=\(\left[\begin{array}{ccc}
8 & 7 & 1 \\
-23 & 27 & -69 \\
32 & -13 & 58
\end{array}\right]\)

⇒ \(A^3-6 A^2+  \text { A + } 111 \\
=\left[\begin{array}{ccc}
8 & 7 & 1 \\
-23 & 27 & -69 \\
32 & -13 & 58
\end{array}\right]-6\left[\begin{array}{ccc}
4 & 2 & 1 \\
-3 & 8 & -14 \\
7 & -3 & 14
\end{array}\right]+5\left[\begin{array}{ccc}
1 & 1 & 1 \\
1 & 2 & -3 \\
2 & -1 & 3
\end{array}\right]+11\left[\begin{array}{ccc}
1 & 0 & 0 \\
0 & 1 & 0 \\
0 & 0 & 1
\end{array}\right]\)

=\(\left[\begin{array}{ccc}
8 & 7 & 1 \\
-23 & 27 & -69 \\
32 & -13 & 58
\end{array}\right]-\left[\begin{array}{ccc}
24 & 12 & 6 \\
-18 & 48 & -84 \\
42 & -18 & 84
\end{array}\right]+\left[\begin{array}{ccc}
5 & 5 & 5 \\
5 & 10 & -15 \\
10 & -5 & 15
\end{array}\right]+\left[\begin{array}{ccc}
11 & 0 & 0 \\
0 & 11 & 0 \\
0 & 0 & 11
\end{array}\right]\)

=\(\left[\begin{array}{ccc}
8-24+5+11 & 7-12+5+0 & 1-6+5+0 \\
-23+18+5+0 & 27-48+10+11 & -69+84-15+0 \\
32-42+10+0 & -13+18-5+0 & 58-84+15+11
\end{array}\right]=\left[\begin{array}{lll}
0 & 0 & 0 \\
0 & 0 & 0 \\
0 & 0 & 0
\end{array}\right]\)=0

Now, \(A^3-6 A^2+5 A+111\)=0

⇒ \((A A A) A^{-1}-6(A A) A^{-1}+5 A A^{-1}+111 A^{-1}\)=0 (Post-multiplying by }\( A^{-t} as |A| \neq 0)\)

⇒ \(A A\left(A A^{-1}\right)-6 A\left(A^{-1}\right)+5\left(A^{-1}\right)+11\left(\mathrm{IA}^{-1}\right)\)=0

⇒ \(\mathrm{AAI}-6 \mathrm{AI}+5 \mathrm{I}+11 \mathrm{~A}^{-1}=0 (Using \mathrm{AA}^{-1}=\mathrm{I} and\mathrm{IA}^{-1}=\mathrm{A}^{-1}\)

⇒ \(\mathrm{A}^2-6 \mathrm{~A}+5 \mathrm{I}=-11 \mathrm{~A}^{-1} \)

⇒ \((Using \mathrm{AAI}=\mathrm{A}^2 and \mathrm{AI}=\mathrm{A}\)

⇒ \(A^{-1}=-\frac{1}{11}\left(A^2-6 A+51\right) \Rightarrow A^{-1}=\frac{1}{11}\left(-A^2+6 A-5 I\right)\)

=\(\frac{1}{11}\left\{-\left[\begin{array}{ccc}
4 & 2 & 1 \\
-3 & 8 & -14 \\
7 & -3 & 14
\end{array}\right]+6\left[\begin{array}{ccc}
1 & 1 & 1 \\
1 & 2 & -3 \\
2 & -1 & 3
\end{array}\right]-5\left[\begin{array}{lll}
1 & 0 & 0 \\
0 & 1 & 0 \\
0 & 0 & 1
\end{array}\right]\right\} \)

= \(\frac{1}{11}\left\{\left[\begin{array}{ccc}
-4 & -2 & -1 \\
3 & -8 & 14 \\
-7 & 3 & -14
\end{array}\right]+\left[\begin{array}{ccc}
6 & 6 & 6 \\
6 & 12 & -18 \\
12 & -6 & 18
\end{array}\right]-\left[\begin{array}{ccc}
5 & 0 & 0 \\
0 & 5 & 0 \\
0 & 0 & 5
\end{array}\right]\right\}\)

=\(\frac{1}{11}\left[\begin{array}{ccc}
-4+6-5 & -2+6-0 & -1+6-0 \\
3+6-0 & -8+12-5 & 14-18-0 \\
-7+12-0 & 3-6-0 & -14+18-5
\end{array}\right]\)

=\(\frac{1}{11}\left[\begin{array}{ccc}
-3 & 4 & 5 \\
9 & -1 & -4 \\
5 & -3 & -1
\end{array}\right]\)

Question 16. If A=\(\left[\begin{array}{ccc}
2 & -1 & 1 \\
-1 & 2 & -1 \\
1 & -1 & 2
\end{array}\right]\)
 , verify that \(A^3-6 A^2+9 A-4\) I=O and hence, find \(A^{-1}\)

Solution:

Given, A=\(\left[\begin{array}{ccc}
2 & -1 & 1 \\
-1 & 2 & -1 \\
1 & -1 & 2
\end{array}\right] \)

\(A^2=A \cdot A=\left[\begin{array}{ccc}
2 & -1 & 1 \\
-1 & 2 & -1 \\
1 & -1 & 2
\end{array}\right]\left[\begin{array}{ccc}
2 & -1 & 1 \\
-1 & 2 & -1 \\
1 & -1 & 2
\end{array}\right]\)

= \(\left[\begin{array}{ccc}
4+1+1 & -2-2-1 & 2+1+2 \\
-2-2-1 & 1+4+1 & -1-2-2 \\
2+1+2 & -1-2-2 & 1+1+4
\end{array}\right]\)

= \(\left[\begin{array}{ccc}
6 & -5 & 5 \\
-5 & 6 & -5 \\
5 & -5 & 6
\end{array}\right]\)

and \(A^3=A^2 \cdot A =\left[\begin{array}{ccc}
6 & -5 & 5 \\
-5 & 6 & -5 \\
5 & -5 & 6
\end{array}\right]\left[\begin{array}{ccc}
2 & -1 & 1 \\
-1 & 2 & -1 \\
1 & -1 & 2
\end{array}\right]\)

= \(\left[\begin{array}{ccc}
12+5+5 & -6-10-5 & 6+5+10 \\
-10-6-5 & 5+12+5 & -5-6-10 \\
10+5+6 & -5-10-6 & 5+5+12
\end{array}\right]\)

=\(\left[\begin{array}{ccc}
22 & -21 & 21 \\
-21 & 22 & -21 \\
21 & -21 & 22
\end{array}\right]\)

⇒ \(A^3-6 A^2 +9 A-41\)

= \(\left[\begin{array}{ccc}
22 & -21 & 21 \\
-21 & 22 & -21 \\
21 & -21 & 22
\end{array}\right]-6\left[\begin{array}{ccc}
6 & -5 & 5 \\
-5 & 6 & -5 \\
5 & -5 & 6
\end{array}\right]+9\left[\begin{array}{ccc}
2 & -1 & 1 \\
-1 & 2 & -1 \\
1 & -1 & 2
\end{array}\right]-4\left[\begin{array}{ccc}
1 & 0 & 0 \\
0 & 1 & 0 \\
0 & 0 & 1
\end{array}\right]\)

= \(\left[\begin{array}{ccc}
22 & -21 & 21 \\
-21 & 22 & -21 \\
21 & -21 & 22
\end{array}\right]-\left[\begin{array}{ccc}
36 & -30 & 30 \\
-30 & 36 & -30 \\
30 & -30 & 36
\end{array}\right]+\left[\begin{array}{ccc}
18 & -9 & 9 \\
-9 & 18 & -9 \\
9 & -9 & 18
\end{array}\right]-\left[\begin{array}{ccc}
4 & 0 & 0 \\
0 & 4 & 0 \\
0 & 0 & 4
\end{array}\right]\)

=\(\left[\begin{array}{ccc}
22-36+18-4 & -21+30-9-0 & 21-30+9-0 \\
-21+30-9-0 & 22-36+18-4 & -21+30-9-0 \\
21-30+9-0 & -21+30-9-0 & 22-36+18-4
\end{array}\right]\)

= \(\left[\begin{array}{lll}
0 & 0 & 0 \\
0 & 0 & 0 \\
0 & 0 & 0
\end{array}\right]\)=0

⇒ \(A^3-6 A^2+9 A-4 I=0 \Rightarrow(A A A) A^{-1}-6(A A) A^{-1}+9 A^{-1}-4 I A^{-1}\)=0

(Post-multiplying by } \(\mathrm{A}^{-1} \text { as }|\mathrm{A}| \neq 0 \)

⇒ \([|A|=|\begin{array}{ccc}
2 & -1 & 1 \\
-1 & 2 & -1 \\
1 & -1 & 2
\end{array}|\)=2(4-1)+1(-2+1)+1(1-2)=6-1-1=4 \(\neq 0\)

⇒ \(\mathrm{AA}\left(\mathrm{AA}^{-1}\right)-6 \mathrm{~A}\left(\mathrm{AA}^{-1}\right)+9\left(\mathrm{AA}^{-1}\right)-4\left(\mathrm{IA}^{-1}\right)\)=0

⇒ \(\mathrm{AAI}-6 \mathrm{AI}+9 \mathrm{I}-4 \mathrm{~A}^{-1}=0\)

(Using }\(\mathrm{AA}^{-1}= I\) and  \(1 \mathrm{~A}^{-1}=\mathrm{A}^{-1}\)

⇒ \(A^2-6 A+9 I=4 A^{-1}\)

(Using \(A^2 T=A^2\) and A\(\mathrm{~A}^{-1}\)=A )

⇒ \(A^{-1}=\frac{1}{4}\left(A^2-6 A+91\right)\)

⇒ \(\frac{1}{4}\left\{\left[\begin{array}{ccc}
6 & -5 & 5 \\
-5 & 6 & -5 \\
5 & -5 & 6
\end{array}\right]-6\left[\begin{array}{ccc}
2 & -1 & 1 \\
-1 & 2 & -1 \\
1 & -1 & 2
\end{array}\right]+9\left[\begin{array}{ccc}
1 & 0 & 0 \\
0 & 1 & 0 \\
0 & 0 & 1
\end{array}\right]\right\}\)

⇒ \(\frac{1}{4}\left\{\left[\begin{array}{ccc}
6 & -5 & 5 \\
-5 & 6 & -5 \\
5 & -5 & 6
\end{array}\right]-\left[\begin{array}{ccc}
12 & -6 & 6 \\
-6 & 12 & -6 \\
6 & -6 & 12
\end{array}\right]+\left[\begin{array}{ccc}
9 & 0 & 0 \\
0 & 9 & 0 \\
0 & 0 & 9
\end{array}\right]\right\}\)

=\(\frac{1}{4}\left[\begin{array}{ccc}
6-12+9 & -5+6+0 & 5-6+0 \\
-5+6+0 & 6-12+9 & -5+6+0 \\
5-6+0 & -5+6+0 & 6-12+9
\end{array}\right]\)

=\(\frac{1}{4}\left[\begin{array}{ccc}
3 & 1 & -1 \\
1 & 3 & 1 \\
-1 & 1 & 3
\end{array}\right]\)

Question 17. Let A be anon-singular square matrix of order 3×3, then |adj A| equal to?

  1. \(|\mathrm{A}|\)
  2. \(|\mathrm{A}|^2\)
  3. \(|\mathrm{A}|\)
  4. 3\(|\mathrm{~A}|\)

Solution: 2. \(|\mathrm{A}|^2\)

We know that \(({adj} \mathrm{A}) \mathrm{A}=|\mathrm{A}| \mathrm{I}\)

= \(|A|\left[\begin{array}{ccc}
1 & 0 & 0 \\
0 & 1 & 0 \\
0 & 0 & 1
\end{array}\right]=\left[\begin{array}{ccc}
|A| & 0 & 0 \\
0 & |A| & 0 \\
0 & 0 & |A|
\end{array}\right]\)

⇒ \(|({adj} A) A|=\left|\begin{array}{ccc}
A & 0 & 0 \\
0 & |A| & 0 \\
0 & 0 & |A|
\end{array}\right|\)

=\(|A|^3\left|\begin{array}{lll}
1 & 0 & 0 \\
0 & 1 & 0 \\
0 & 0 & 1
\end{array}\right|=|A|^3|\mathrm{I}|\)

⇒ \(|{adj} A| A|=| A|^3 (|I|=1)\)

⇒ \(|{adj} A|=|A|^2\)

Hence, the correct option is 2

Question 18. If A is an invertible matrix of order 2, then det \(\left(\mathrm{A}^{-1}\right)\) is equal to?

  1. \({det}(\mathrm{A})\)
  2. \(\frac{1}{det}(\mathrm{A}\)
  3. 1
  4. zero

Solution:

We know that \(\mathrm{AA}^{-1}=\mathrm{I}\)

⇒ \(|\mathrm{AA}^{-1}|=|\mathrm{I}| \Rightarrow|\mathrm{A}|\mathrm{A}^{-1}|\)=1

⇒ {Using\(|\mathrm{AA}^{-1}|=|\mathrm{A}||\mathrm{A}^{-1}|\) and \(|\mathrm{I}|=1)\)

⇒ \(|\mathrm{A}^{-1}|=\frac{1}{|\mathrm{~A}|}=\frac{1}{{det}(\mathrm{A})}\) .

Hence, the correct option is (B).

Determinants Exercise 4.5

Question 1. x+2 y=2,2 x+3 y=3.

Solution:

The given system can be written as AX= B, where

⇒ \(\mathrm{A}=\left[\begin{array}{ll}
1 & 2 \\
2 & 3
\end{array}\right], \mathrm{X}=\left[\begin{array}{l}
\mathrm{x} \\
\mathrm{y}
\end{array}\right] \)

and \(\mathrm{B}=\left[\begin{array}{l}
2 \\
3
\end{array}\right]\)

Here, \(|\mathrm{A}|=\left|\begin{array}{ll}1 & 2 \\ 2 & 3\end{array}\right|=1(3)-2(2)=3-4=-1 \neq\) 0

A is non-singular. Therefore, \(\mathrm{A}^{-1}\) exists.

Hence, the given system of equations is consistent.

Question 2. 2 x-y=5, x+y=4

Solution:

The given system can be written as A X=B, where

A=\(\left[\begin{array}{cc}
2 & -1 \\
1 & 1
\end{array}\right], X=\left[\begin{array}{l}
x \\
y
\end{array}\right] \text { and } B=\left[\begin{array}{l}
5 \\
4
\end{array}\right]\)

Here, \(|A|=\left|\begin{array}{cc}2 & -1 \\ 1 & 1\end{array}\right|=2(1)-(-1)(1)=2+1=3 \neq \)

A is non-singular. Therefore, \(\mathrm{A}^{-1}\) exists.

Hence, the given system of equations is consistent.

Question 3. x+3 y=5,2 x+6 y=8

Solution:

The given system can be written as AX = B, where

A=\(\left[\begin{array}{ll}
1 & 3 \\
2 & 6
\end{array}\right], X=\left[\begin{array}{l}
x \\
y
\end{array}\right] \text { and } B=\left[\begin{array}{l}
5 \\
8
\end{array}\right]\)

Here, \(|A|=\left|\begin{array}{ll}1 & 3 \\ 2 & 6\end{array}\right|=1(6)-3(2)=6-6=0\)

A is a singular matrix.

Nothing can be said about consistency as yet. We compute

⇒ \(({adj} \mathrm{A}) \mathrm{B} =\left[\begin{array}{cc}
6 & -2 \\
-3 & 1
\end{array}\right]\left[\begin{array}{l}
5 \\
8
\end{array}\right]\)

=\(\left[\begin{array}{cc}
6 & -3 \\
-2 & 1
\end{array}\right]\left[\begin{array}{l}
5 \\
8
\end{array}\right]=\left[\begin{array}{c}
30-24 \\
-10+8
\end{array}\right]\)

= \(\left[\begin{array}{c}
6 \\
-2
\end{array}\right] \neq 0\)

Thus, the solution of the given system of equations does not exist. Hence, the system of equations is inconsistent.

Question 4. x+y+z=1,2 x+3 y+2 z=2, a x+a y+2 a z=4

Solution:

The given system can be written as AX – B, where

A=\(\left[\begin{array}{ccc}
1 & 1 & 1 \\
2 & 3 & 2 \\
a & a & 2 a
\end{array}\right], X=\left[\begin{array}{l}
x \\
y \\
z
\end{array}\right]\) and

B=\(\left[\begin{array}{l}
1 \\
2 \\
4
\end{array}\right]\)

Here,|A| =\(\left|\begin{array}{lll}
1 & 1 & 1 \\
2 & 3 & 2 \\
a & a & 2 a
\end{array}\right|=1(6 a-2 a)-1(4 a-2 a)+1(2 a-3 a)\)

=4 a-2 a-a=4 a-3 a=a \(\neq 0\)

A is non-singular. Therefore, \(\mathrm{A}^{-1}\) exists.

Hence, the given system of equations is consistent.

Question 5. 3 x-y-2 z=2,2 y-z=-1,3 x-5 y=3

Solution:

The given system is 3 x-y-2 z=2,0 x+2 y-z=-1 and 3 x-5 y+0 z=3 which can be written as AX =B, where

A=\(\left[\begin{array}{ccc}
3 & -1 & -2 \\
0 & 2 & -1 \\
3 & -5 & 0
\end{array}\right], X=\left[\begin{array}{l}
x \\
y \\
z
\end{array}\right]\) and

B=\(\left[\begin{array}{c}
2 \\
-1 \\
3
\end{array}\right]\)

Here, \(|A|=\left|\begin{array}{ccc}3 & -1 & -2 \\ 0 & 2 & -1 \\ 3 & -5 & 0\end{array}\right|=3(0-5)+1(0+3)-2(0-6)\)=-15+3+12=0

A is a singular matrix.

Therefore, nothing can be said about consistency as yet. So, we compute (adj A)B.

Cofactors of A are:

⇒ \(A_{11}=-5, A_{12}=-3, A_{13}\)=-6,

⇒ \(A_{21}=10, A_{22}=6, A_{23}=12\),

⇒ \(A_{91}=5, A_{32}=3, A_{33}\)=6

⇒ \({adj}(A)=\left[A_4\right]^{\prime}=\left[\begin{array}{ccc}
-5 & -3 & -6 \\
10 & 6 & 12 \\
5 & 3 & 6
\end{array}\right]=\left[\begin{array}{ccc}
-5 & 10 & 5 \\
-3 & 6 & 3 \\
-6 & 12 & 6
\end{array}\right]\)

⇒ \(({adj} A) B=\left[\begin{array}{ccc}
-5 & 10 & 5 \\
-3 & 6 & 3 \\
-6 & 12 & 6
\end{array}\right] \cdot\left[\begin{array}{c}
2 \\
-1 \\
3
\end{array}\right]\)

= \(\left[\begin{array}{c}
-10-10+15 \\
-6-6+9 \\
-12-12+18
\end{array}\right]=\left[\begin{array}{c}
-5 \\
-3 \\
-6
\end{array}\right] \neq 0\)

Thus, the solution of the given system of equations does not exist.

Hence, the system of equations is inconsistent.

Question 6. 5 x-y+4 z=5,2 x+3 y+5 z=2,5 x-2 y+6 z=-1

Solution:

The given system can be written as A X=B, where

⇒ \(\mathbf{A}=\left[\begin{array}{ccc}
5 & -1 & 4 \\
2 & 3 & 5 \\
5 & -2 & 6
\end{array}\right]\),

X=\(\left[\begin{array}{l}
x \\
y \\
z
\end{array}\right]\)

and \(B=\left[\begin{array}{c}
5 \\
2 \\
-1
\end{array}\right]\)

Here, \(|A|=\left|\begin{array}{ccc}5 & -1 & 4 \\ 2 & 3 & 5 \\ 5 & -2 & 6\end{array}\right|\)

=5(18+10)-(-1)(12-25)+4(-4-15)=140-13-76=51 \(\neq 0\)

A is non-singular.

Therefore, \(\mathrm{A}^{-1}\) exists.

Hence, the given system of equations is consistent.

Question 7. 5 x+2 y=4,7 x+3 y=5

Solution:

The given system can be written as AX = B, where

⇒ \(\mathrm{A}=\left[\begin{array}{ll}5 & 2 \\ 7 & 3\end{array}\right], X=\left[\begin{array}{l}\mathrm{x} \\ \mathrm{y}\end{array}\right] \)

and \(\mathrm{B}=\left[\begin{array}{l}4 \\ 5\end{array}\right]\)

Here, \(|\mathrm{A}|=\left|\begin{array}{ll}5 & 2 \\ 7 & 3\end{array}\right|=15-14=1 \neq 0\)

Thus, A is non-singular.

Therefore, its \(\mathrm{A}^{-1}\) exists.

Therefore, the given system is consistent and has a unique solution given by

⇒ \(\mathrm{A}^{-1}(\mathrm{AX})=\mathrm{A}^{-1} \mathrm{~B} \Rightarrow \mathrm{X}=\mathrm{A}^{-1} \mathrm{~B}\)

Cofactors of A are, \(A_{11}=3, A_{12}=-7, A_{21}=-2, A_{22}=5\)

⇒ \({adj}(A)=\left[A_{i j}\right]^{\prime}=\left[\begin{array}{cc}
3 & -7 \\
-2 & 3
\end{array}\right]^{\prime}=\left[\begin{array}{cc}
3 & -2 \\
-7 & 5
\end{array}\right]\)

Now, \(A^{-1}=\frac{1}{|A|}({adj} A)=\frac{1}{1}\left[\begin{array}{cc}3 & -2 \\ -7 & 5\end{array}\right]=\left[\begin{array}{cc}3 & -2 \\ -7 & 5\end{array}\right]\)

⇒ \(\mathrm{X}=\mathrm{A}^{-1} \mathrm{~B}=\left[\begin{array}{cc}
3 & -2 \\
-7 & 5
\end{array}\right]\left[\begin{array}{l}
4 \\
5
\end{array}\right]=\left[\begin{array}{l}
3 \times 4+(-2) \times 5 \\
(-7) \times 4+5 \times 5
\end{array}\right]\)

= \(\left[\begin{array}{c}
2 \\
-3
\end{array}\right] \Rightarrow\left[\begin{array}{l}
x \\
y
\end{array}\right]=\left[\begin{array}{c}
2 \\
-3
\end{array}\right]\)

Hence, x=2 and y=-3.

Question 8. 2 x-y=-2,3 x+4 y=3

Solution:

The given system can be written as AX = B, where

A=\(\left[\begin{array}{cc}
2 & -1 \\
3 & 4
\end{array}\right], X=\left[\begin{array}{l}
x \\
y
\end{array}\right]\)

and \(B=\left[\begin{array}{c}
-2 \\
3
\end{array}\right]\)

Here, |A|=\(\left|\begin{array}{cc}2 & -1 \\ 3 & 4\end{array}\right|=2 \times 4-(-3)=11 \neq 0\)

Thus, A is non-singular.

Therefore, its \(\mathrm{A}^{-1}\) exists.

Therefore, the given system is consistent and has a unique solution given by

⇒ \(\mathrm{X}=\mathrm{A}^{-1} \mathrm{~B}\)

Cofactors of A are, \(A_{11}=4, A_{12}=-3, A_{21}=1, A_{21}=2\)

⇒ \({adj}(A)=\left[A_i\right]^{\prime}=\left[\begin{array}{cc}
4 & -3 \\
1 & 2
\end{array}\right]^{\prime}-\left[\begin{array}{cc}
4 & 1 \\
-3 & 2
\end{array}\right]\)

⇒ \(A^{-1}=\frac{1}{|A|}({adj} A)=\frac{1}{11}\left[\begin{array}{cc}
4 & 1 \\
-3 & 2
\end{array}\right]\)

Now, \(X=A^{-1} B=\frac{1}{11}\left[\begin{array}{cc}4 & 1 \\ -3 & 2\end{array}\right]\left[\begin{array}{c}-2 \\ 3\end{array}\right]=\frac{1}{11}\left[\begin{array}{c}-8+3 \\ 6+6\end{array}\right]\)

= \(\frac{1}{11}\left[\begin{array}{c}-5 \\ 12\end{array}\right]=\left[\begin{array}{c}-\frac{5}{11} \\ \frac{12}{11}\end{array}\right]\)

⇒ \(\left[\begin{array}{l}x \\ y\end{array}\right]=\left[\begin{array}{c}\frac{-5}{11} \\ \frac{12}{11}\end{array}\right]\)

Hence, x=\(\frac{-5}{11}\) and y=\(\frac{12}{11}\).

Question 9. 4 x-3 y=3,3 x-5 y=7

Solution:

The given system can be written as AX = B, where

A=\(\left[\begin{array}{ll}
4 & -3 \\
3 & -5
\end{array}\right]\),

X=\(\left[\begin{array}{l}
x \\
y
\end{array}\right] \)and B=\(\left[\begin{array}{l}
3 \\
7
\end{array}\right]\)

Here, \(|A|=\left|\begin{array}{ll}4 & -3 \\ 3 & -5\end{array}\right|=4(-5)-3(-3)=-20+9=-11 \neq 0\)

Thus, A is non-singular.

Therefore, its \(\mathrm{A}^{-1}\) exists,

Therefore, the given system is consistent and has a unique solution given by

⇒ \(\mathrm{X}=\mathrm{A}^{-1} \mathrm{~B}\)

Cofactors of A are, \(A_{11}=-5, A_{12}=-3, A_{21}=3, A_{22}=4\)

⇒ \({adj}(A)=\left[A_{i j}\right]^{\prime}=\left[\begin{array}{cc}
-5 & -3 \\
3 & 4
\end{array}\right]^{\prime}=\left[\begin{array}{ll}
-5 & 3 \\
-3 & 4
\end{array}\right]\)

⇒ \(A^{-1}=\frac{1}{|A|}({adj} A)=\frac{1}{-11}\left[\begin{array}{ll}
-5 & 3 \\
-3 & 4
\end{array}\right] \)

Now, \(X=A^{-1} B=\frac{1}{-11}\left[\begin{array}{ll}
-5 & 3 \\
-3 & 4
\end{array}\right]\left[\begin{array}{l}
3 \\
7
\end{array}\right]\)

=\(\frac{1}{-11}\left[\begin{array}{c}
-15+21 \\
-9+28
\end{array}\right]=\frac{1}{-11}\left[\begin{array}{c}
6 \\
19
\end{array}\right] \)

⇒ \(\left[\begin{array}{l}
x \\
y
\end{array}\right]=\left[\begin{array}{c}
\frac{-6}{11} \\
\frac{-19}{11}
\end{array}\right]\)

Hence, x=-\(\frac{6}{11}\) and y=-\(\frac{19}{11}\)

Question 10. 5 x+2 y=3,3 x+2 y=5

Solution:

The given system can be written as AX = B, where

⇒ \(\mathrm{A}=\left[\begin{array}{ll}5 & 2 \\ 3 & 2\end{array}\right], \mathrm{X}=\left[\begin{array}{l}\mathrm{x} \\ \mathrm{y}\end{array}\right] and \mathrm{B}=\left[\begin{array}{l}3 \\ 5\end{array}\right]\)

Here, |A|=\(\left|\begin{array}{ll}5 & 2 \\ 3 & 2\end{array}\right|=10-6=4 \neq 0\)

Thus, A is non-singular.

Therefore, its \(\mathrm{A}^{-1}\) exists.

Therefore, the given system is consistent and has a unique solution given by \(X=A^{-1} B\)

Cofactors of A are, \(A_{11}=2, \mathrm{~A}_{12}=-3, \mathrm{~A}_{21}=-2, \mathrm{~A}_{22}\)=5

adj \((A)=\left[A_4\right]^{\prime}=\left[\begin{array}{cc}2 & -3 \\ -2 & 5\end{array}\right]^*=\left[\begin{array}{cc}2 & -2 \\ -3 & 5\end{array}\right]\)

⇒ \(\mathrm{A}^{-1}=\frac{1}{\mid \mathrm{A}}({adj} \mathrm{A})=\frac{1}{4}\left[\begin{array}{cc}2 & -2 \\ -3 & 5\end{array}\right]\)

Now, X=\(\mathrm{A}^{-1}
\mathrm{~B}=\frac{1}{4}\left[\begin{array}{cc}2 & -2 \\ -3 & 5\end{array}\right]\left[\begin{array}{l}3 \\ 5\end{array}\right]\)

=\(\frac{1}{4}\left[\begin{array}{c}6-10 \\ -9+25\end{array}\right]=\frac{1}{4}\left[\begin{array}{c}-4 \\ 16\end{array}\right]\)

=\(\left[\begin{array}{c}-1 \\ 4\end{array}\right] \Rightarrow\left[\begin{array}{l}x \\ y\end{array}\right]=\left[\begin{array}{c}-1 \\ 4\end{array}\right]\)

Hence, x=-1 and y=4

Question 11. 2 x+y+z=1, x-2 y-z=\(\frac{3}{2}\), 3 y-5 z=9

Solution:

The given system can be written as AX=B where

A=\(\left[\begin{array}{ccc}
2 & 1 & 1 \\
2 & -4 & -2 \\
0 & 3 & -5
\end{array}\right], X=\left[\begin{array}{l}
x \\
y \\
z
\end{array}\right]\)

and B=\(\left[\begin{array}{l}
1 \\
3 \\
9
\end{array}\right]\)

Here, \(|A|=\left|\begin{array}{ccc}2 & 1 & 1 \\ 2 & -4 & -2 \\ 0 & 3 & -5\end{array}\right|=2(20+6)-1(-10-0)+1(6-0)=52+10+6=68 \neq 0\)

Thus, A is non-singular.

Therefore, its \(\mathrm{A}^{-1}\) exists.

Therefore, the given system is consistent and has a unique solution given by X=\(\mathrm{A}^{-1} \mathrm{~B}\)

Cofactors of A are,

⇒ \(A_{11}=20+6=26, A_{12}=-(-10-0)=10, A_{13}\)=6-0=6

⇒ \(A_{21}=-(-5-3)=8, A_{22}=-10-0=-10, A_{23}=-(6-0)\)=-6

⇒ \(A_{31}=(-2+4)=2, A_{32}=-(-4-2)=6, A_{33}\)=-8-2=-10

⇒ \({adj}(A)=\left[A_1\right]^{\prime}=\left[\begin{array}{ccc}
26 & 10 & 6 \\
8 & -10 & -6 \\
2 & 6 & -10
\end{array}\right]=\left[\begin{array}{ccc}
26 & 8 & 2 \\
10 & -10 & 6 \\
6 & -6 & -10
\end{array}\right]\)

⇒ \(A^{-1}=\frac{1}{|A|}({adj} A)=\frac{1}{68}\left[\begin{array}{ccc}
26 & 8 & 2 \\
10 & -10 & 6 \\
6 & -6 & -10
\end{array}\right]\)

Now, \(X=A^{-1} B \Rightarrow\left[\begin{array}{l}x \\ y \\ z\end{array}\right]=\frac{1}{68}\left[\begin{array}{ccc}26 & 8 & 2 \\ 10 & -10 & 6 \\ 6 & -6 & -10\end{array}\right]\left[\begin{array}{l}1 \\ 3 \\ 9\end{array}\right]\)

⇒ \(\left[\begin{array}{l}
x \\
y \\
z
\end{array}\right]\)

=\(\frac{1}{68}\left[\begin{array}{c}
26+24+18 \\
10-30+54 \\
6-18-90
\end{array}\right]=\frac{1}{68}\left[\begin{array}{c}
68 \\
34 \\
-102
\end{array}\right]\)

⇒ \(\Rightarrow\left[\begin{array}{l}
x \\
y \\
z
\end{array}\right]=\left[\begin{array}{c}
1 \\
\frac{1}{2} \\
\frac{-3}{2}
\end{array}\right]\)

Hence; x=1, y=\(\frac{1}{2}\) and z=\(\frac{-3}{2}\)

Question 12. x-y+z=4, 2 x+y-3 z=0, x+y+z=2

Solution:

The given system can be written as AX=B, where

A=\(\left[\begin{array}{ccc}
1 & -1 & 1 \\
2 & 1 & -3 \\
1 & 1 & 1
\end{array}\right], X=\left[\begin{array}{l}
x \\
y \\
z
\end{array}\right]\)

and B=\(\left[\begin{array}{l}
4 \\
0 \\
2
\end{array}\right]\)

Here, \(|\mathrm{A}|=\left|\begin{array}{ccc}1 & -1 & 1 \\ 2 & 1 & -3 \\ 1 & 1 & 1\end{array}\right|=1(1+3)-(-1)(2+3)+1(2-1)=4+5+1=10 \neq 0\)

Thus, A is non-singular.

Therefore, its \(\mathrm{A}^{-1}\) exists.

Therefore, the given system is consistent and has a unique solution given by \(\mathrm{X}=\mathrm{A}^{-1} \mathrm{~B}\)

Cofactors of A are.

⇒ \(A_{11}=1+3=4, A_{13}=-(2+3)=-5, A_{13}=2-1=1 \)

⇒ \(A_{21}=-(-1-1)=2, A_{22}=1-1=0, A_{25}=-(1+1)=-2\)

⇒ \(A_{71}=3-1=2, A_{22}=-(-3-2)=5, A_{23}=1+2=3\)

⇒ \({adj}(A)=\left[A_{i j}\right]^{\prime}=\left[\begin{array}{ccc}
4 & -5 & 1 \\
2 & 0 & -2 \\
2 & 5 & 3
\end{array}\right]=\left[\begin{array}{ccc}
4 & 2 & 2 \\
-5 & 0 & 5 \\
1 & -2 & 3
\end{array}\right]\)

⇒ \(A^{-1}=\frac{1}{|A|}(\text { adj } A)=\frac{1}{10}\left[\begin{array}{ccc}
4 & 2 & 2 \\
-5 & 0 & 5 \\
1 & -2 & 3
\end{array}\right]\)

Now, \(X=A^{-1} B \Rightarrow\left[\begin{array}{l}x \\ y \\ z\end{array}\right]=\frac{1}{10}\left[\begin{array}{ccc}4 & 2 & 2 \\ -5 & 0 & 5 \\ 1 & -2 & 3\end{array}\right]\left[\begin{array}{l}4 \\ 0 \\ 2\end{array}\right]\)

⇒ \(\left[\begin{array}{l}
x \\
y \\
z
\end{array}\right]=\frac{1}{10}\left[\begin{array}{c}
16+0+4 \\
-20+0+10 \\
4+0+6
\end{array}\right] \Rightarrow\left[\begin{array}{l}
x \\
y \\
z
\end{array}\right]\)

=\(\frac{1}{10}\left[\begin{array}{c}
20 \\
-10 \\
10
\end{array}\right]=\left[\begin{array}{c}
2 \\
-1 \\
1
\end{array}\right]\)

Hence; x=2, y=-1 and z=1

Question 13. 2 x+3 y+3 z=5, x-2 y+z=-4, 3 x-y-2 z=3

Solution:

The given system can be written as AX = B, where

A=\(\left[\begin{array}{ccc}
2 & 3 & 3 \\
1 & -2 & 1 \\
3 & -1 & -2
\end{array}\right], X=\left[\begin{array}{l}
x \\
y \\
z
\end{array}\right]\)

and B=\(\left[\begin{array}{c}
5 \\
-4 \\
3
\end{array}\right]\)

Here, \(|\mathrm{A}|=\left|\begin{array}{ccc}2 & 3 & 3 \\ 1 & -2 & 1 \\ 3 & -1 & -2\end{array}\right|=2(4+1)-3(-2-3)+3(-1+6)=10+15+15=40 \neq 0\)

Thus, A is non-singular.

Therefore, its \(\mathrm{A}^{-4}\) exists.

Therefore, the given system is consistent and has a unique solution given by X =\(\mathrm{A}^{-1}\) B Cofictors of A are,

⇒ \(A_{11}=4+1=5, A_{12}=-(-2-3)=5, A_{13}=(-1+6)=5\)

⇒ \(A_{21}=-(-6+3)=3, A_{27}=(-4-9)=-13, A_{33}=-(-2-9)=11\)

⇒ \(A_{31}=3+6=9, A_{21}=-(2-3)=1, A_{32}=-4-3=-7\)

⇒ \({adj}(\mathrm{A})=\left[\mathrm{A}_1\right]^{\prime}=\left[\begin{array}{ccc}
5 & 5 & 5 \\
3 & -13 & 11 \\
9 & 1 & -7
\end{array}\right]^{\prime}=\left[\begin{array}{ccc}
5 & 3 & 9 \\
5 & -13 & 1 \\
5 & 11 & -7
\end{array}\right]\)

⇒ \(\mathrm{A}^{-1}=\frac{1}{|\mathrm{~A}|}({adj} \mathrm{A})=\frac{1}{40}\left[\begin{array}{ccc}
5 & 3 & 9 \\
5 & -13 & 1 \\
5 & 11 & -7
\end{array}\right]\)

⇒ \(\mathrm{X}=\mathrm{A}^{-1} \mathrm{~B} \Rightarrow\left[\begin{array}{l}
\mathrm{x} \\
\mathrm{y} \\
\mathrm{z}
\end{array}\right]=\frac{1}{40}\left[\begin{array}{ccc}
5 & 3 & 9 \\
5 & -13 & 1 \\
5 & 11 & -7
\end{array}\right] \cdot\left[\begin{array}{c}
5 \\
-4 \\
3
\end{array}\right]\)

= \(\frac{1}{40}\left[\begin{array}{c}
25-12+27 \\
25+52+3 \\
25-44-21
\end{array}\right]=\frac{1}{40}\left[\begin{array}{c}
40 \\
80 \\
-40
\end{array}\right]\)

= \(\left[\begin{array}{c}
1 \\
2 \\
-1
\end{array}\right]\)

Hence, x=1, y=2 and z=-1

Question 14. x-y+2 z=7,3 x+4 y-5 z=-5,2 x-y+3 z=12

Solution:

The given system can be written as A X=B, where

⇒ \(\mathrm{A}=\left[\begin{array}{ccc}
1 & -1 & 2 \\
3 & 4 & -5 \\
2 & -1 & 3
\end{array}\right]\),

X=\(\left[\begin{array}{l}
x \\
y \\
z
\end{array}\right],\)

and B=\(\left[\begin{array}{c}
7 \\
-5 \\
12
\end{array}\right]\)

Here, \(|A|=\left|\begin{array}{ccc}1 & -1 & 2 \\ 3 & 4 & -5 \\ 2 & -1 & 3\end{array}\right|=1(12-5)-(-1)(9+10)+2(-3-8)=7+19-22=4 \neq 0\)

Thus, A is non-singular.

Therefore, its \(\mathrm{A}^{-1}\) exists.

Therefore, the given system is consistent and has a unique solution given by

⇒ \(\mathrm{X}=\mathrm{A}^{-1} \mathrm{~B}\)

Cofactors of A are,

⇒ \(A_{11}=12-5=7, A_{22}=-(9+10)=-19, A_{13}\)=-3-8=-11

⇒ \(A_{21}=-(-3+2)=1, A_{22}=3-4=-1, A_{23}=-(-1+2)\)=-1

⇒ \(A_{31}=5-8=-3, A_{32}=-(-5-6)=11, A_{39}\)=4+3=7

⇒ \({adj}(\mathrm{A})=\left[\mathrm{A}_{\mathrm{ij}}\right]^{\prime}=\left[\begin{array}{ccc}
7 & -19 & -11 \\
1 & -1 & -1 \\
-3 & 11 & 7
\end{array}\right]^{\prime}=\left[\begin{array}{ccc}
7 & 1 & -3 \\
-19 & -1 & 11 \\
-11 & -1 & 7
\end{array}\right] \)

⇒ \(\mathrm{A}^{-1}=\frac{1}{|\mathrm{~A}|}({adj} \mathrm{A})=\frac{1}{4}\left[\begin{array}{ccc}
7 & 1 & -3 \\
-19 & -1 & 11 \\
-11 & -1 & 7
\end{array}\right]\)

⇒ \(\mathrm{X}=\mathrm{A}^{-1} \mathrm{~B} \Rightarrow\left[\begin{array}{l}
\mathrm{x} \\
\mathrm{y} \\
\mathrm{z}
\end{array}\right]\)

=\(\frac{1}{4}\left[\begin{array}{ccc}
7 & 1 & -3 \\
-19 & -1 & 11 \\
-11 & -1 & 7
\end{array}\right] \cdot\left[\begin{array}{c}
7 \\
-5 \\
12
\end{array}\right]\)

= \(\frac{1}{4}\left[\begin{array}{c}
49-5-36 \\
-133+5+132 \\
-77+5+84
\end{array}\right] \Rightarrow\left[\begin{array}{l}
x \\
y \\
z
\end{array}\right]\)

= \(\frac{1}{4}\left[\begin{array}{l}
8 \\
4 \\
12
\end{array}\right]=\left[\begin{array}{l}
2 \\
1 \\
3
\end{array}\right]\)

Hence, x=2, y=1 and z=3

 Question 15. If A=\(\left[\begin{array}{ccc}2 & -3 & 5 \\ 3 & 2 & -4 \\ 1 & 1 & -2\end{array}\right]\), find \(A^{-1}\). Using \(A^{-1}\), solve the system of equations 2 x-3 y+5 z=11, 3 x+2 y-4 z=-5, x+y-2 z=-3.

Solution:

The given system can written as AX = R,where A = =\(\left[\begin{array}{ccc}
2 & -3 & 5 \\
3 & 2 & -4 \\
1 & 1 & -2
\end{array}\right]\),

X=\(\left[\begin{array}{l}
x \\
y \\
z
\end{array}\right] and B=\left[\begin{array}{c}
11 \\
-5 \\
-3
\end{array}\right]\)

Here, \(|A|=\left|\begin{array}{ccc}
2 & -3 & 5 \\
3 & 2 & -4 \\
1 & 1 & -2
\end{array}\right|\)

=2(-4+4)-(-3)(-6+4)+5(3-2)=0-6+5=-1 \(\neq 0\)

Thus, A is non-singular.

Therefore, its \(\mathrm{A}^{-1}\) exists.

Therefore, the given system is consistent and has a unique solution given by X=\(A^{-1}\) B Cofactors of A are,

⇒ \(A_{11}=-4+4=0, A_{12}=-(-6+4)=2 \cdot A_{13}=3-2=1\)

⇒ \(A_{21}=-(6-5)=-1, A_{22}=-4-5=-9 \cdot A_{33}=-(2+3)=-5\)

⇒ \(A_{13}=(12-10)=2, A_{12}=-(-8-15)=23, A_{73}=4+9=13\)

⇒ \({adj}(\mathrm{A})=\left[\mathrm{A}_{\mathrm{ii}}\right]^{\prime}=\left[\begin{array}{ccc}
0 & 2 & 1 \\
-1 & -9 & -5 \\
2 & 23 & 13
\end{array}\right]=\left[\begin{array}{ccc}
0 & -1 & 2 \\
2 & -9 & 23 \\
1 & -5 & 13
\end{array}\right]\)

⇒ \(A^{-1}=\frac{1}{|A|}({adj} \mathrm{A})=\frac{1}{-1}\left[\begin{array}{ccc}
0 & -1 & 2 \\
2 & -9 & 23 \\
1 & -5 & 13
\end{array}\right]=\left[\begin{array}{ccc}
0 & 1 & -2 \\
-2 & 9 & -23 \\
-1 & 5 & -13
\end{array}\right]\)

Now, \(X=A^{-1} \mathrm{~B} \Rightarrow\left[\begin{array}{l}
\mathrm{x} \\
\mathrm{y} \\
\mathrm{z}
\end{array}\right]=\left[\begin{array}{ccc}
0 & 1 & -2 \\
-2 & 9 & -23 \\
-1 & 5 & -13
\end{array}\right]\left[\begin{array}{c}
11 \\
-5 \\
-3
\end{array}\right]\)

⇒ \(\left[\begin{array}{l}
x \\
y \\
z
\end{array}\right]=\left[\begin{array}{c}
0-5+6 \\
-22-45+69 \\
-11-25+39
\end{array}\right]=\left[\begin{array}{l}
1 \\
2 \\
3
\end{array}\right]\)

Hence, x=1, y=2 and z=3

Question 16. The cost of 4 kg onion, 3 kg wheat, and 2kg rice is ₹ 60. The cost of 2 kg} onion, 4 kg wheat, and 6kg rice is ₹ 90. The cost of 6kg onion, 2 kg wheat, and 3 kg rice is ₹ 70. Find the cost of each item per kg by matrix method.

Solution:

Let the prices (per kg ) of onion, wheat, and rice be ₹ x,₹ y, and ₹ z respectively. Then 4 x+3 y+2 z=60,2 x+4 y+6 z=90,6 x+2 y+3 z=70

The given system can be written as AX = B, where A =\(\left[\begin{array}{lll}4 & 3 & 2 \\ 2 & 4 & 6 \\ 6 & 2 & 3\end{array}\right], \mathrm{X}=\left[\begin{array}{l}\mathrm{x} \\ \mathrm{y} \\ \mathrm{z}\end{array}\right]\)

and B =\(\left[\begin{array}{l}60 \\ 90 \\ 70\end{array}\right]\)

Here, \(|A|=\left|\begin{array}{lll}4 & 3 & 2 \\ 2 & 4 & 6 \\ 6 & 2 & 3\end{array}\right|=4(12-12)-3(6-36)+2(4-24)=0+90-40=50 \neq 0\)

Thus, A is non-singular.

Therefore, its \(A^{-1}\) exists.

Therefore, the given system is consistent and has a unique solution given by X=\(\mathrm{A}^{-1} B\)

Cofactors of A are,

⇒ \(A_{11}=12-12=0, A_{12}=-(6-36)=30, A_{13}=4-24=-20,\)

⇒ \(A_{21}=-(9-4)=-5, A_{22}=12-12=0, A_{23}=-(8-18)=10\),

⇒ \(A_{31}=(18-8)=10, A_{92}=-(24-4)=-20, A_{31}=16-6\)=10

⇒ \({adj}(A)=\left[A_{11}\right]^{\prime}=\left[\begin{array}{ccc}
0 & 30 & -20 \\
-5 & 0 & 10 \\
10 & -20 & 10
\end{array}\right]=\left[\begin{array}{ccc}
0 & -5 & 10 \\
30 & 0 & -20 \\
-20 & 10 & 10
\end{array}\right]\)

⇒ \(A^{-1}=\frac{1}{|A|}(\text { adj } A)=\frac{1}{50}\left[\begin{array}{ccc}
0 & -5 & 10 \\
30 & 0 & -20 \\
-20 & 10 & 10
\end{array}\right]\)

Now, \(X=A^{-1} B \Rightarrow\left[\begin{array}{l}
x \\
y \\
z
\end{array}\right]=\frac{1}{50}\left[\begin{array}{ccc}
0 & -5 & 10 \\
30 & 0 & -20 \\
-20 & 10 & 10
\end{array}\right]\)

=\(\left[\begin{array}{l}
60 \\
90 \\
70
\end{array}\right]=\frac{1}{50}\left[\begin{array}{c}
\sigma-450+700 \\
1800+0-1400 \\
-1200+900+700
\end{array}\right]\)

⇒ \(\left[\begin{array}{l}
x \\
y \\
z
\end{array}\right]=\frac{1}{50}\left[\begin{array}{l}
250 \\
400 \\
400
\end{array}\right]\)

= \(\left[\begin{array}{l}
5 \\
8 \\
8
\end{array}\right] \Rightarrow\left[\begin{array}{l}
x \\
y \\
z
\end{array}\right]=\left[\begin{array}{l}
5 \\
8 \\
8
\end{array}\right]\)

x=5, y=8 and z=8 .

Hence, the price of onion per kg is ₹ 5, the price of wheat per kg is ₹ 8 and that of rice per kg is ₹ 8.

Determinants Miscellaneous Exercise

Question 1. Prove that the determinant \(\left|\begin{array}{ccc}x & \sin \theta & \cos \theta \\ -\sin \theta & -x & 1 \\ \cos \theta & 1 & x\end{array}\right|\) is independent of \theta.

Solution:

Let \(|A|=\left|\begin{array}{ccc}x & \sin \theta & \cos \theta \\ -\sin \theta & -x & 1 \\ \cos \theta & 1 & x\end{array}\right|\)

Expanding to the corresponding first row, we get

⇒ \(|A| =x\left|\begin{array}{cc}
-x & 1 \\
1 & x
\end{array}\right|-\sin \theta\left|\begin{array}{cc}
-\sin \theta & 1 \\
\cos \theta & x
\end{array}\right|+\cos \theta\left|\begin{array}{cc}
-\sin \theta & -x \\
\cos \theta & 1
\end{array}\right|\)

= x\(\left(-x^2-1\right)-\sin \theta(-x \sin \theta-\cos \theta)+\cos \theta(-\sin \theta+x \cos \theta)\)

=-\(x^3-x+x \sin ^2 \theta+\sin \theta \cos \theta-\sin \theta \cos \theta+x \cos ^2 \theta \quad\left(\sin ^2 \theta+\cos ^2 \theta=1\right)\)

=-\(x^3-x+x\left(\sin ^2 \theta+\cos ^2 \theta\right)=-x^3-x+x\)

= \(-x^3\)

Hence, A is independent of \(\theta\).

Question 2. Evaluate \(\left|\begin{array}{ccc}
\cos \alpha \cos \beta & \cos \alpha \sin \beta & -\sin \alpha \\
-\sin \beta & \cos \beta & 0 \\
\sin \alpha \cos \beta & \sin \alpha \sin \beta & \cos \alpha
\end{array}\right|\).

Solution:

Given, \(|A|=\left|\begin{array}{ccc}\cos \alpha \cos \beta & \cos \alpha \sin \beta & -\sin \alpha \\ -\sin \beta & \cos \beta & 0 \\ \sin \alpha \cos \beta & \sin \alpha \sin \beta & \cos \alpha\end{array}\right|\)

Expanding corresponding to \(R_1\), we get

|A| =\(\cos \alpha \cos \beta(\cos \alpha \cos \beta-0)-\cos \alpha \sin \beta(-\cos \alpha \sin \beta-0)\)

–\(\sin \alpha\left(-\sin ^2 \beta \sin \alpha-\cos ^2 \beta \sin \alpha\right)\)

= \(\cos ^2 \alpha(\cos ^2 \beta+\sin ^2 \beta)+\sin ^2 \alpha(\sin ^2 \beta+\cos ^2 \beta) [\sin ^2 \theta+\cos ^2 \theta=1]\)

= \(\cos ^2 \alpha(1)+\sin ^2 \alpha(1)=\cos ^2 \alpha+\sin ^2 \alpha=1\) .

Question 3. If \(A^{-1}=\left[\begin{array}{ccc}3 & -1 & 1 \\ -15 & 6 & -5 \\ 5 & -2 & 2\end{array}\right]\) and

B=\(\left[\begin{array}{ccc}1 & 2 & -2 \\ -1 & 3 & 0 \\ 0 & -2 & 1\end{array}\right]\), find \((A B)^{-1}\).

Solution:

We know that \((\mathrm{AB})^{-1}=\mathrm{B}^{-1} \mathrm{~A}^{-1}\) and \(\mathrm{A}^{-1}\) is known, therefore we proceed to find \(\mathrm{B}^{-1}\).

Here, \(|B|=\left|\begin{array}{ccc}1 & 2 & -2 \\ -1 & 3 & 0 \\ 0 & -2 & 1\end{array}\right|=1(3-0)-2(-1-0)-2(2-0)=3+2-4=1 \neq 0\)

⇒ \(\mathrm{B}^{-1}\) exists. Cofactors of B are:

⇒ \(\mathrm{B}_{11}=(3-0)=3, \mathrm{~B}_{12}=-(-1-0)=1\),

⇒ \(\mathrm{~B}_{13}\)=(2-0)=2,

⇒ \(\mathrm{~B}_{11}=-(2-4)=2, \mathrm{~B}_{21}=(1-0)\)=1,

⇒ \(\mathrm{~B}_{23}\)=-(-2-0)=2,

⇒ \(\mathrm{~B}_{31}\)=(0+6)=6,

⇒ \(\mathrm{~B}_{32}\)=-(0-2)=2,

⇒ \(\mathrm{~B}_{33}=(3+2)\)=5,

⇒ \({adj}(\mathrm{B})=\left[\mathrm{B}_6\right]^{+}=\left[\begin{array}{lll}
3 & 1 & 2 \\
2 & 1 & 2 \\
6 & 2 & 5
\end{array}\right]=\left[\begin{array}{lll}
3 & 2 & 6 \\
1 & 1 & 2 \\
2 & 2 & 5
\end{array}\right]\)

⇒ \(B^{-1}=\frac{1}{|B|} {adj}(B)=\frac{1}{1}\left[\begin{array}{lll}
3 & 2 & 6 \\
1 & 1 & 2 \\
2 & 2 & 5
\end{array}\right]\)

= \(\left[\begin{array}{lll}
3 & 2 & 6 \\
1 & 1 & 2 \\
2 & 2 & 5
\end{array}\right]\)

Now,\((A B)^{-1}=B^{-1} A^{-1}=\left[\begin{array}{lll}
3 & 2 & 6 \\
1 & 1 & 2 \\
2 & 2 & 5
\end{array}\right]\left[\begin{array}{ccc}
3 & -1 & 1 \\
-15 & 6 & -5 \\
5 & -2 & 2
\end{array}\right]\)

= \(\left[\begin{array}{ccc}
9-30+30 & -3+12-12 & 3-10+12 \\
3-15+10 & -1+6-4 & 1-5+4 \\
6-30+25 & -2+12-10 & 2-10+10
\end{array}\right]\)

= \(\left[\begin{array}{ccc}
9 & -3 & 5 \\
-2 & 1 & 0 \\
1 & 0 & 2
\end{array}\right]\)

Question 4. Evaluate \(\left|\begin{array}{ccc}x & y & x+y \\ y & x+y & x \\ x+y & x & y\end{array}\right|\)

Solution:

Let \(\Delta=\left|\begin{array}{ccc}x & y & x+y \\ y & x+y & x \\ x+y & x & y\end{array}\right|\)

Expand along \(R_1\)

⇒ \(\Delta=x\left(x y+y^2-x^2\right)-y\left(y^2-x^2-x y\right)+(x+y)\left(x y-x^2-2 x y-y^2\right)\)

⇒ \(\Delta=x^2 y+x y^2-x^3-y^3+x^2 y+x y^2+x^2 y-x^3-2 x^2 y-x y^2+x y^2-x^2 y-2 x y^2-y^2\)

⇒ \(\Delta=-2\left(x^3+y^3\right)\)

Question 5. Evaluate \(\left|\begin{array}{ccc}1 & x & y \\ 1 & x+y & y \\ 1 & x & x+y\end{array}\right|\),

Solution:

Let \(\Delta=\left|\begin{array}{ccc}1 & x & y \\ 1 & x+y & y \\ 1 & x & x+y\end{array}\right|\)

Expand along C

⇒ \(\Delta=1\left(x^2+2 x y+y^2-x y\right)-1\left(x^2+x y-x y\right)+1\left(x y-x y-y^2\right)\)

⇒ \(\Delta-x^2+x y+y^2-x^2-y^2\)=x y

Question 6. If x, y, z are non-zero real numbers, then the inverse of matrix A=\(\left[\begin{array}{lll}x & 0 & 0 \\ 0 & y & 0 \\ 0 & 0 & z\end{array}\right]\) is ?

  1. \(\left[\begin{array}{ccc}x^{-1} & 0 & 0 \\ 0 & y^{-1} & 0 \\ 0 & 0 & z^{-1}\end{array}\right]\)
  2. \(x y z\left[\begin{array}{ccc}x^{-1} & 0 & 0 \\ 0 & y^{-1} & 0 \\ 0 & 0 & z^{-1}\end{array}\right]\)
  3. \(\frac{1}{x y z}\left[\begin{array}{ccc}x & 0 & 0 \\ 0 & y & 0 \\ 0 & 0 & z\end{array}\right]\)
  4. \(\frac{1}{x y z}\left[\begin{array}{lll}1 & 0 & 0 \\ 0 & 1 & 0 \\ 0 & 0 & 1\end{array}\right]\)

Solution: 1. \(\left[\begin{array}{ccc}x^{-1} & 0 & 0 \\ 0 & y^{-1} & 0 \\ 0 & 0 & z^{-1}\end{array}\right]\)

Given, A=\(\left[\begin{array}{lll}x & 0 & 0 \\ 0 & y & 0 \\ 0 & 0 & z\end{array}\right]\)

⇒ \(|A|=\left[\begin{array}{lll}x & 0 & 0 \\ 0 & y & 0 \\ 0 & 0 & z\end{array}\right]=x(y z-0)=x y z \neq\) 0(because x, y and z are non-zero )

⇒\(\mathrm{A}^{-1}\) exists

Cofactors of A are:

⇒ \(A_{11}=(y z-0)=y z_1 A_{12}=-(0-0)=0, A_{13}=0-0=0\),

⇒ \(A_{21}=-(0-0)=0, A_{22}=x z-0=x z_{23}\)=-(0-0)=0,

⇒ \(A_{31}=0-0=0, A_{31}=-(0-0)=0, A_{13}=(x y-0)\)=x y

⇒ \({adj}(A)=\left[A_{i t}\right]^{\prime}=\left[\begin{array}{ccc}
y z & 0 & 0 \\
0 & x z & 0 \\
0 & 0 & x y
\end{array}\right]^{\prime}=\left[\begin{array}{ccc}
y z & 0 & 0 \\
0 & x z & 0 \\
0 & 0 & x y
\end{array}\right] \)

Now, \(A^{-1}=\frac{1}{|A|}({adj} A) \Rightarrow A^{-1}=\frac{1}{x y z}\left[\begin{array}{ccc}
y z & 0 & 0 \\
0 & x z & 0 \\
0 & 0 & x y
\end{array}\right]=\left[\begin{array}{ccc}
\frac{1}{x} & 0 & 0 \\
0 & \frac{1}{y} & 0 \\
0 & 0 & \frac{1}{z}
\end{array}\right]\)

= \(\left[\begin{array}{ccc}
x^{-1} & 0 & 0 \\
0 & y & 0 \\
0 & 0 & z^{-1}
\end{array}\right]\)

Hence, the correct option is 1.

Question 7. Let A=\(\left[\begin{array}{ccc}1 & \sin \theta & 1 \\ -\sin \theta & 1 & \sin \theta \\ -1 & -\sin \theta & 1\end{array}\right]\), where 0 \(\leq \theta \leq 2 \pi\), then:

  1. \({\det} \mathrm{A}\)=0
  2. \({\det} \mathrm{A} \in(2, \infty)\)
  3. \({\det} \mathrm{A} \in(2,4)\)
  4. \({\det} \mathrm{A} \in[2,4]\)

Solution: 4. \({det} \mathrm{A} \in[2,4]\)

Given, A=\(\left[\begin{array}{ccc}1 & \sin \theta & 1 \\ -\sin \theta & 1 & \sin \theta \\ -1 & -\sin \theta & 1\end{array}\right]\)

⇒ \(|A|=\left|\begin{array}{ccc}
1 & \sin \theta & 1 \\
-\sin \theta & 1 & \sin \theta \\
-1 & -\sin \theta & 1
\end{array}\right|\)

=1\(\left(1+\sin ^2 \theta\right)-\sin \theta(-\sin \theta+\sin \theta)+1\left(\sin ^2 \theta+1\right)\)

⇒ \(|A|=2+2 \sin ^2 \theta\)

For 0 \(\leq \theta \leq 2 \pi,-1 \leq \sin \theta \leq 1 \Rightarrow 0 \leq \sin ^2 \theta \leq 1\)

1 \(\leq 1+\sin ^2 \theta \leq 2 \Rightarrow 2 \leq 2\left(1+\sin ^2 \theta\right) \leq 4 {det}(A) \in[2,4]\)

Hence, the correct option is 4.

Application of Derivatives Class 12 Maths Important Questions Chapter 6

Application Of Derivatives Exercise – 6.1

Question 1. Find the rate of change of the area of a circle with respect to its radius r when

r = 3 cm

r = 4 cm

Solution:

The area of a circle (A) with radius (r) is given by, A =\(\pi r^2\)

Now, the rate of change of the area with respect to its radius is given by,
\(\frac{d \Lambda}{d r}=\frac{d}{d r}\left(\pi r^2\right)=2 \pi r\)

1. When r = 3 cm,\(\frac{\mathrm{dA}}{\mathrm{dr}}=2 \pi(3)\) = 6 \(\pi\)

Hence, the area of the circle is changing at the rate of 6\(\pi {cm}^2 /cm\) when its radius is 3 cm.

2. When r = 4cm,\(\frac{d A}{d r}=2 \pi(4)=8 \pi\)

Hence, the area of the circle is changing at the rate of 8\(\pi \mathrm{cm}^2 / \mathrm{cm}\) when its radius is 4 cm.

Question 2. The volume of a cube is increasing at the rate of 8 \({cm}^2\)s. How fast is the surface area increasing when the length of an edge is 12 cm?

Solution:

Let x be the length of the side, V be the volume and S be the surface area of the cube.

Then, V=\(x^3\) and S=\(6 x^2\)

It is given that \(\frac{\mathrm{dV}}{\mathrm{dt}}=8 \mathrm{~cm}^3 / \mathrm{s}\)

Then, by using the chain rule, we have:

8=\(\frac{d V}{d t}=\frac{d}{d t}\left(x^3\right)-3 x^2, \frac{d x}{d t} \Rightarrow \frac{d x}{d t}=\frac{8}{3 x^2}\)

Now; \(\frac{d S}{d t}=\frac{d}{d t}\left(6 x^2\right)=\frac{d}{d x}\left(6 x^2\right) \cdot \frac{d x}{d t}\) [By chain rule]

=12 x \(\cdot \frac{d x}{d t}=12 x \cdot\left(\frac{8}{3 x^2}\right)=\frac{32}{x}\) [From (1)]

Thus, when x=12\( \mathrm{~cm}, \frac{\mathrm{dS}}{\mathrm{dt}}=\frac{32}{12} \mathrm{~cm}^2 / \mathrm{s}=\frac{8}{3} \mathrm{~cm}^2 / \mathrm{s}\)

Hence, if the length of the edge of the cube is 12 cm, then the surface area is increasing at the rate of \(\frac{8}{3} \mathrm{~cm}^2 / \mathrm{s}\).

Read and Learn More Class 12 Maths Chapter Wise with Solutions

Question 3. The radius of a circle is increasing uniformly at the rate of 3 cm/s. Find the rate at which the area of the circle is increasing when the radius is 10 cm.

Solution:

The area of a circle (A) with radius (r) is given by, \(\mathrm{A}=\pi r^2\)

Now, the rate of change of area (A) with respect to time (t) is given by,
\(\frac{d \mathrm{~A}}{\mathrm{dt}}=\frac{\mathrm{d}}{\mathrm{dt}}\left(\pi r^2\right)=2 \pi r \frac{\mathrm{dr}}{\mathrm{dt}}\)

It is given that, \(\frac{\mathrm{dr}}{\mathrm{dt}}=3 \mathrm{~cm} / \mathrm{s}\)

⇒ \(\frac{d A}{d t}=2 \pi r(3)=6 \pi r\)

Thus, when \(\mathrm{r}=10 \mathrm{~cm}, \frac{\mathrm{dA}}{\mathrm{dt}}=6 \pi(10)=60 \pi \mathrm{cm}^2 / \mathrm{s}\)

Hence, the rate at which the area of the circle is increasing when the radius is 10 cm, is 60n \({~cm}^2/s\).

Question 4. An edge of a variable cube is increasing at the rate of 3 cm/s. How fast is the volume of the cube increasing when the edge is 10 cm long?

Solution:

Let x be the length of a side and V be the volume of the cube. Then, V = \(x^2\)

⇒ \(\frac{\mathrm{dV}}{\mathrm{dt}}=3 \mathrm{x}^2 \cdot \frac{\mathrm{dx}}{\mathrm{dt}}\)

It is given that, \(\frac{\mathrm{dx}}{\mathrm{dt}}=3 \mathrm{~cm} / \mathrm{s}\)

⇒ \(\frac{d V}{d t}=3 x^2(3)=9 x^2\)

Therefore, when x=10 \(\mathrm{~cm}, \frac{d V}{d t}=9(10)^2=900 \mathrm{~cm}^3 / \mathrm{s}\)

Hence, the volume of the cube is increasing at the rate of 900 \({~cm}^3\) when the edge is 10 cm long.

Question 5. A stone is dropped into a quiet lake and waves move in circles at the speed of 5 cm/s. At the instant when the radius of the circular wave is 8 cm, how fast is the enclosed area increasing?

Solution:

The area of a circle (A) with radius (r) is given by A = \(\pi r^2\).

Therefore, the rate of change of area (A) with respect to time (t) is given by,

⇒ \(\frac{\mathrm{d} A}{\mathrm{dt}}=\frac{\mathrm{d}}{\mathrm{dt}}\left(\pi \mathrm{r}^2\right)=\frac{\mathrm{d}}{\mathrm{dr}}\left(\pi \mathrm{r}^2\right) \frac{\mathrm{dr}}{\mathrm{dt}}\)

=2 \(\pi \mathrm{r} \frac{\mathrm{dr}}{\mathrm{dt}}\) [By chain rule]

It is given that \(\frac{\mathrm{dr}}{\mathrm{dt}}=5 \mathrm{~cm} / \mathrm{s}\)

Thus, when \(\mathrm{r}=8 \mathrm{~cm}, \frac{\mathrm{dA}}{\mathrm{dt}}=2 \pi(8)(5)=80 \pi\)

Hence, when the radius of the circular wave is 8 cm, the enclosed area is increasing at the rate of 80n \({~cm}^2\).

CBSE Class 12 Maths Chapter 6 Application Of Derivatives Important Question And Answers

Question 6. The radius of a circle is increasing at the rate of 0.7 cm/s. What is the rate of increase of its circumference?

Solution:

The circumference of a circle (C) with radius ( r ) is given by \(\mathrm{C}-2 \pi \mathrm{r}\).

Therefore, the rate of change of circumference (C) with respect to time (t) is given by,

⇒ \(\frac{\mathrm{dC}}{\mathrm{dt}} =\frac{\mathrm{dC}}{\mathrm{dr}} \cdot \frac{\mathrm{dr}}{\mathrm{dt}}\)

⇒ \(=\frac{\mathrm{d}}{\mathrm{dr}}(2 \pi r) \frac{\mathrm{dr}}{\mathrm{dt}}=2 \pi \cdot \frac{\mathrm{dr}}{\mathrm{dt}}
\) [By chain rule]

It is given that \(\frac{\mathrm{dr}}{\mathrm{dt}}=0.7 \mathrm{~cm} / \mathrm{s}\)
Hence, the rate of increase of the circumference is \(2 \pi(0.7)=1.4 \pi \mathrm{cm} / \mathrm{s}\).

Question 7. The length x of a rectangle is decreasing at the rate of 5 cm/minute and the width y is increasing at the rate of 4 cm/minute. When x = 8 cm and y = 6 cm., find the rates of change of (a) the perimeter, and (b) the area of the rectangle..

Solution:

Since the length (x) is decreasing at the rate of 5 cm/minute and the width (y) is increasing at the rate of 4 cm/minute, we have:

⇒ \(\frac{\mathrm{dx}}{\mathrm{dt}}=-5 \mathrm{~cm} / \mathrm{min} \) and \(\frac{\mathrm{dy}}{\mathrm{dt}}=4 \mathrm{~cm} / \mathrm{min}
\)

1. The perimeter (P) of a rectangle is given by, P=2(x+y)

⇒ \(\frac{\mathrm{dP}}{\mathrm{dt}}=2\left(\frac{\mathrm{dx}}{\mathrm{dt}}+\frac{\mathrm{dy}}{\mathrm{dt}}\right)=2(-5+4)=-2 \mathrm{~cm} / \mathrm{min}\)

Hence, the perimeter is decreasing at the rate of \(2 \mathrm{~cm} / \mathrm{min}\).

2. The area (A) of a rectangle is given by, A=\(x \times y\)

⇒ \(\frac{d A}{d t}=\frac{d x}{d t} \cdot y+x \cdot \frac{d y}{d t}=-5 y+4 x\)

When x=8 cm and y=6 cm, \(\frac{\mathrm{dA}}{\mathrm{dt}}=(-5 \times 6+4 \times 8) \mathrm{cm}^2 / \mathrm{min}=2 \mathrm{~cm}^2 / \mathrm{min}\)

Hence, the area of the rectangle is increasing at the rate of 2 \({~cm}^2\)min.

Question 8. A balloon, which always remains spherical on inflation, is being inflated by pumping in 900 cubic centimeters of gas per second. Find the rate at which the radius of the balloon increases when the radius is 15 cm.

Solution:

The volume of a sphere (V) with radius (r) is given by, V =\(\frac{4}{3} \pi \mathrm{r}^3\)

The rate of change of volume (V) with respect to time (t) is given by,

⇒ \(\frac{d V}{d t} =\frac{d V}{d r} \cdot \frac{d r}{d t}\) [Chain rule]

=\(\frac{d}{d r}\left(\frac{4}{3} \pi r^3\right) \cdot \frac{d r}{d t}=4 \pi r^2 \cdot \frac{d r}{d t}\)

It is given that \(\frac{\mathrm{dV}}{\mathrm{dt}}=900 \mathrm{~cm}^3 / \mathrm{s}\)

900=4 \(\pi \mathrm{r}^2 \cdot \frac{\mathrm{dr}}{\mathrm{dt}} \Rightarrow \frac{\mathrm{dr}}{\mathrm{dt}}=\frac{900}{4 \pi \mathrm{r}^2}=\frac{225}{\pi \mathrm{r}^2}\)

Therefore, when radius -15 \(\mathrm{~cm}, \frac{\mathrm{dr}}{\mathrm{dt}}=\frac{225}{\pi(15)^2}=\frac{1}{\pi}\)

Hence, the rate at which the radius of the balloon increases, when the radius is 15 cm, is \(\frac{1}{\pi} \mathrm{cm} / \mathrm{s}\).

Question 9. A balloon, which always remains spherical has a variable radius. Find the rate at which its volume is increasing with the radius when the latter is 10 cm.

Solution:

The volume of a sphere (V) with radius (r) is given by \(\mathrm{V}=\frac{4}{3} \pi \mathrm{r}^3\).

The rate of change of volume (V) with respect to its radius (r) is given by,

⇒ \(\frac{d V}{d r}=\frac{d}{d r}\left(\frac{4}{3} \pi r^3\right)=\frac{4}{3} \pi\left(3 r^2\right)=4 \pi r^2\)

Therefore when radius =10 \(\mathrm{~cm}, \frac{d V}{d r}=4 \pi(10)^2=400 \pi\)

Hence, the volume of the balloon is increasing at the rate of 400 \(\pi \mathrm{cm}^7 / \mathrm{cm}\).

Question 10. A ladder 5 m long is leaning against a wall. The bottom of the ladder is pulled along the ground, away from the wall, at the rate of 2 cm/s. How fast is its height on the wall decreasing when the foot of the ladder is 4 m away from the wall?

Solution:

Let y m be the height of the wall at which the ladder touches. Also, let the foot of the ladder be x m away from the wall.

Then, by Pythagoras theorem, we have: \(x^2+y^2\)=25 [Length of the ladder =5m]

y=\(\sqrt{25-x^2}\)

Then, the rate of change of height (y) with respect to time (t) is given by \(\frac{d y}{d t}=\frac{-x}{\sqrt{25-x^2}} \cdot \frac{d x}{d t}\)

It is given that \(\frac{d x}{d t}=0.02 \mathrm{~m} / \mathrm{s}  \frac{d y}{d t}=\frac{-x}{\sqrt{25-x^2}} \times \frac{2}{100} \mathrm{~m} / \mathrm{s}\)

Now when x=4 \(\mathrm{~m}, we have: \frac{d y}{d t}=\frac{-4 \times 2}{100 \times 3} \mathrm{~m} / \mathrm{s}\)

=-\(\frac{8}{300} \mathrm{~m} / \mathrm{s}=-\frac{8}{3} \mathrm{~cm} / \mathrm{s}\)

Hence, the height of the ladder on the wall is decreasing at the rate of \(\frac{8}{3} \mathrm{~cm} / \mathrm{s}\),

Question 11. A particle moves along the curve \(6 y=x^3+2\). Find the points on the curve at which the y-coordinate is changing 8 times as fast as the x-coordinate.

Solution:

The equation of the curve is given as:

6 y=\(x^3+2\)

The rate of change of the position of the particle with respect to time (t) is given by

6 \(\frac{d y}{d t}=3 x^2 \frac{d x}{d t}+0 \Rightarrow 2 \frac{d y}{d t}=x^2 \frac{d x}{d t}\)

When the y-coordinate of the particle changes 8 times as fast as the x-coordinate i.e., \(\left(\frac{\mathrm{dy}}{\mathrm{dt}}=8 \frac{\mathrm{dx}}{\mathrm{dt}}\right)\), we have:

2\(\left(8 \frac{d x}{d t}\right)=x^2 \frac{d x}{d t} \Rightarrow 16 \frac{d x}{d t}=x^2 \frac{d x}{d t}\)

⇒ \(\left(x^2-16\right) \frac{d x}{d t}=0 \Rightarrow x^2=16 \Rightarrow x= \pm 4\)

When x=4, y=\(\frac{4^3+2}{6}=\frac{66}{6}=11\)

When x=-4, y=\(\frac{(-4)^3+2}{6}=-\frac{62}{6}=-\frac{31}{3}\)

Hence, the points on the curve are (4,11) and \(\left(-4, \frac{-31}{3}\right)\)

Question 12. The radius of an air bubble is increasing at the rate of \(\frac{1}{2} \mathrm{~cm} / \mathrm{s}\). At what rate is the volume of the bubble increasing when the radius is 1 cm ?

Solution:

The air bubble is in the shape of a sphere.

Now, the volume of an air bubble (V) with radius ( r ) is given by, V=\(\frac{4}{3} \pi r^3\)

The rate of change of volume (V) with respect to time (t) is given by,

⇒ \(\frac{\mathrm{dV}}{\mathrm{dt}} =\frac{4}{3} \pi \frac{\mathrm{d}}{\mathrm{dr}}\left(\mathrm{r}^3\right) \cdot \frac{\mathrm{dr}}{\mathrm{dt}}\)

=\(\frac{4}{3} \pi\left(3 \mathrm{r}^2\right) \frac{\mathrm{dr}}{\mathrm{dt}}-4 \pi \mathrm{r}^2 \frac{\mathrm{dr}}{\mathrm{dt}}\) [By chain rule]

It is given that \(\frac{\mathrm{dr}}{\mathrm{dt}}=\frac{1}{2} \mathrm{~cm} / \mathrm{s}\)

Therefore, when \(\mathrm{r}=1 \mathrm{~cm}, \frac{\mathrm{dV}}{\mathrm{dt}}=4 \pi(1)^2\left(\frac{1}{2}\right)=2 \pi \mathrm{cm}^3 / \mathrm{s}\)

Hence, the rate at which the volume of the bubble increases is 2 \(\pi \mathrm{cm}^3 / \mathrm{s}\).

Question 13. A balloon, which always remains spherical, has a variable diameter \(\frac{3}{2}(2 x+1)\). Find the rate of change of its volume with respect to x.

Solution:

The volume of a sphere (V) with radius (r) is given by, V = \(\frac{4}{3} \pi r^3\)

It is given that:

Diameter =\(\frac{3}{2}(2 x+1) \Rightarrow r=\frac{3}{4}(2 x+1)\)

V=\(\frac{4}{3} \pi\left(\frac{3}{4}\right)^3(2 x+1)^3=\frac{9}{16} \pi(2 x+1)^3\)

Hence, the rate of change of volume with respect to x is:

⇒ \(\frac{\mathrm{dV}}{\mathrm{dx}}=\frac{9}{16} \pi \frac{\mathrm{d}}{\mathrm{dx}}(2 \mathrm{x}+1)^3\)

= \(\frac{9}{16} \pi \times 3(2 \mathrm{x}+1)^2 \cdot \times 2=\frac{27}{8} \pi(2 \mathrm{x}+1)^2\)

Question 14. Sand is poured from a pipe at the rate of 12 cm. The falling sand forms a cone on the ground in such a way that the height of the cone is always one-sixth of the radius of the base. How fast is the height of the sand cone increasing when the height is 4 cm?

Solution:

The volume of a cone (V) with radius (r) and height (h) is given by, \(\mathrm{V}=\frac{1}{3} \pi r^2 \mathrm{~h}\)

It is given that, h=\(\frac{1}{6} r \Rightarrow r=6 h\)

V=\(\frac{1}{3} \pi(6 h)^7 h=12 \pi h^3\)

The rate of change of volume with respect to time (t) is given by,

⇒ \(\frac{d V}{d t}=12 \pi \frac{d}{d h}\left(h^2\right) \cdot \frac{d h}{d t}\)

=12 \(\pi\left(3 h^2\right) \frac{d h}{d t}=36 \pi h^2 \frac{d h}{d t}\) [By Chain rule]

It is also given that \(\frac{d V}{d t}=12 \mathrm{~cm}^3 / \mathrm{s}\)

Therefore, when h=4 cm, we have:

12=36 \(\pi(4)^2 \frac{\mathrm{dh}}{\mathrm{dt}}\)

⇒ \(\frac{\mathrm{dh}}{\mathrm{dt}}=\frac{12}{36 \pi(16)}=\frac{1}{48 \pi}\)

Hence, when the height of the sand cone is 4 cm, its height is increasing at the rate of \(\frac{1}{48 \pi} \mathrm{cm} / \mathrm{s}\).

Question 15. The total cost C (x) in Rupees associated with the production of x units of an item is given by C(x)=\(0.007 x^3-0.003 x^2+15\) x+4000. Find the marginal cost when 17 units are produced.

Solution:

Marginal cost is the rate of change of total cost with respect to output.

Marginal cost (M C)=\(\frac{d C}{d x}=0.007\left(3 x^2\right)\)-0.003(2 x)+15=0.021 x^2-0.006 x+15

When x=17, MC =0.021\(\left(17^3\right)\)-0,006(17)+15

= 0.021(289) – 0.006(17) + 15 – 6.069 – 0.102 + 15 – 20.967

Hence, when 17 units are produced, the marginal cost is Rs, 20.967.

Question 16. The total revenue in Rupees received from the sale of x units of a product is given by R(x) = \(13 x^1+26 x+15\). Find the marginal revenue when x = 7.

Solution:

Marginal revenue is the rate of change of total revenue with respect to the number of units sold.

Marginal Revenue(M R)=\(\frac{d R}{d x}\)=13(2 x)+26=26 x+26

When x = 7, MR = 26(7) + 26 = 182 + 26 = 208.

Hence, the required marginal revenue is Rs 208.

Question 17. The rate of change of the area of a circle with respect to its radius r at r = 6 cm is?

  1. 10 \(\pi\)
  2. 12 \(\pi\)
  3. 8 \(\pi\)
  4. 11 \(\pi\)

Solution: 2. 12 \(\pi\)

The area of a circle (A) with radius (r) is given by, A=\(\pi \mathrm{r}^2\)

Therefore, the rate of change of the area with respect to its radius r is \(\frac{\mathrm{dA}}{\mathrm{dr}}=\frac{\mathrm{d}}{\mathrm{dr}}\left(\pi \mathrm{r}^2\right)=2 \pi r\).

When \(\mathrm{r}=6 \mathrm{~cm}, \frac{\mathrm{dA}}{\mathrm{dr}}=2 \pi \times 6=12 \pi \mathrm{cm}^2 / \mathrm{cm}\)

Hence, the required rate of change of the area of a circle is 12 \(\pi \mathrm{cm}^2 cm\)

The correct answer is 2.

Question 18. The total revenue in Rupees received from the sale of x units of a product is given by R(x) = \(3 x^2+36 x+5\). The marginal revenue, when x = 15, is?

  1. 116
  2. 96
  3. 90
  4. 126

Solution: 4. 126

Marginal revenue is the rate of change of total revenue with respect to the number of units sold.

Marginal Revenue (MR) = \(\frac{\mathrm{dR}}{\mathrm{dx}}=3(2 \mathrm{x})+36=6 x +36\)

When x = 15, MR = 6(15) + 36 = 90 + 36 = 126

Hence, the required marginal revenue is Rs 126.

The correct answer is 4.

Applications Of Derivatives Exercise 6.2

Question 1. Show that the function given by f(x)=3 x+17 is strictly increasing on R.

Solution:

Let \(x_2, x_2 \in\) R, such that \(x_1<x_1 \Rightarrow 3 x_1<3 x_1 \Rightarrow 3 x_1+17<3 x_2+17 \Rightarrow f\left(x_1\right)<f\left(x_2\right)\)

Hence, f is strictly increasing on R.

Alternate method:

Given \(f^{\prime}(x)=3 x+17\). On diff. w.r.t. x, we get \(f^{\prime}(x)=3>0\), in every interval of R.

Thus, the function is strictly increasing on R.

Question 2. Show that the function given by $f(x)=e^{2 n}$ is strictly increasing on R.

Solution:

Let \(x_1, x_2 \in\) R, such that \(x_1<x_1\)

⇒ \(2 x_1<2 x_1 \Rightarrow e^{2 x_1}<e^{2 x_1}\)

f\(\left(x_1\right)<f\left(x_2\right)\)

Hence, f is strictly increasing on R.

Question 3. Show that the function given by f(x)=\(\sin x\) is

  1. strictly increasing in \(\left(0, \frac{\pi}{2}\right)\)
  2. strictly decreasing in \(\left(\frac{\pi}{2}, \pi\right)\)
  3. neither increasing nor decreasing in (0, \(\pi\))

Solution:

The given function is f(x)-\(\sin x\).

⇒ \(f^{\prime}(x)=\cos \mathrm{x}\)

1. Since for each x \(\in\left(0, \frac{\pi}{2}\right), \cos x>0\) we have \(f^{\prime}(x)>0\),

Hence, f is strictly increasing in \(\left(0, \frac{\pi}{2}\right)\).

2. Since for each x \(\in\left(\frac{\pi}{2}, \pi\right), \cos x<0\), we have \(f^{\prime \prime}(x)<0\).

Hence, f is strictly decreasing in \(\left(\frac{\pi}{2}, \pi\right)\)

3. From the results obtained in (1) and (2) it is clear that f is strictly increasing in \(\left(0, \frac{\pi}{2}\right)[\) and strictly decreasing in \(\left(\frac{\pi}{2}, \pi\right)\)

So, f is neither increasing nor decreasing in (0, \(\pi\)).

Question 4. Find the intervals in which the function f given by f(x)=2 \(x^2-3\) x is

  1. strictly increasing
  2. strictly decreasing

Solution:

The given function is f(x)=2 \(x^2-3 x, f^{\prime}(x)\)=4 x-3

Putting f(x)=0 \(\Rightarrow 4 x-3=0 \Rightarrow x=\frac{3}{4}\)

Now, the point \(\frac{3}{4}\) divides the real line into two disjoint intervals i.e. \(\left(-\infty, \frac{3}{4}\right) and \left(\frac{3}{4}, \infty\right)\)

Applications Of Derivatives Two Disjoint Intervals

In interval \(\left(-\infty, \frac{3}{4}\right), \mathrm{f}^{\prime}(\mathrm{x})<0\)

Hence, the given function (f) is strictly decreasing in the interval \(\left(-\infty, \frac{3}{4}\right)\).

In interval \(\left(\frac{3}{4}, \infty\right), \mathrm{f}^{\prime}(\mathrm{x})>0\)

Hence, the given function (f) is strictly increasing in the interval \(\left(\frac{3}{4}, \infty\right)\).

Question 5. Find the intervals in which the function f given by f(x)=2 \(x^3-3 x^2-36 x+7\) is

  1. Strictly increasing
  2. Strictly decreasing

Solution:

The given function is f(x)=\(2 x^3-3 x^2-36 x+7\).

⇒ \(f(x)=6 x^2-6 x-36=6\left(x^2-x-6\right)=6(x+2)(x-3)\)

⇒ \(\mathrm{f}(\mathrm{x})=0 \Rightarrow \mathrm{x}=-2,3\)

The points x=-2 and x =3 divide the real line into three disjoint intervals i.e, \((-\infty,-2),(-2,3) and (3, \infty)\)

Applications Of Derivatives Three Disjoint Intervals

In intervals \((-\infty,-2) and (3, \infty), f^{\prime}(x)\) is positive while in interval \((-2,3), f^{\prime}(x)\) is negative.

Hence, the given function f(x) is strictiy increasing in intervals \((-\infty,-2) \cup(3, \infty)\), while function f(x) is strictly decreasing in interval (-2,3).

Question 6. Find the intervals in which the following functions are strictly increasing or decreasing:

  1. \(x^2+2 x-5\)
  2. 10-6 \(\mathrm{x}-2 \mathrm{x}^2\)
  3. \(-2 x^3-9 x^2-12 x+1\)
  4. \(6-9 x-x^2\)

Solution 1. \(x^2+2 x-5\)

⇒ \((x+1)^3(x-3)^2\)

1. We have, \(f(x)=x^7+2 x-5\)

⇒ \(f^{\prime}(x)=2 x+2\)

Now, \(\mathrm{f}^{\prime}(\mathrm{x})=0 \Rightarrow \mathrm{x}=-1\)

Point x=-1 divides the real line into two disjoint intervals i.e., \((-\infty,-1) and (-1, \infty)\)

Applications Of Derivatives The Real Line Into Two Disjoint Intervals

In interval \((-\infty,-1), f^{\prime}(x)<0\).

f is strictly decreasing in the interval (-\infty,-1)

Thus, f is strictly decreasing for x<-1.

In interval (-1, \(\infty), f^{\prime}(x)>0\)

f is strictly increasing in interval \((-1, \infty)\)

Thus, f is strictly increasing for x>-1

2. We have, f(x)=\(10-6 x-2 x^3\)

\(f^{\prime}(x)=-6-4 x\)

Now, \(\mathrm{r}^{\prime}(\mathrm{x})=0 \Rightarrow \mathrm{x}=-\frac{3}{2}\)

The point x=-\(\frac{3}{2}\) divides the real line into two disjoint intervals i.e., \(\left(-\infty,-\frac{3}{2}\right) and \left(-\frac{3}{2}, \infty\right)\)

Applications Of Derivatives The Point Divides The Real Line Into Two Disjoint Intervals

In interval \(\left(-\infty,-\frac{3}{2}\right)\) i.e., when \(x<-\frac{3}{2}, f^{\prime}(x)>0\).

f is strictly increasing for \(\mathrm{x}<-\frac{3}{2}\).

In interval \(\left(-\frac{3}{2}, \infty\right)\) i.e., when \(x>-\frac{3}{2}, f^{\prime}(x)<0\).

f is strictly decreasing for \(x>-\frac{3}{2}\)

3. We have, f(x)=\(-2 x^3-9 x^2-12 x+1\)

f(x)=\(-6 x^2-18 x-12=-6\left(x^2+3 x+2\right)=-6(x+1)(x+2)\)

Now; \(f^{\prime}(x)=0 \Rightarrow x=-1\) and x=-2

Points x=-1 and x=-2 divide the real line into three disjoint intervals
i.e., \((-\infty,-2),(-2,-1)\) and (-1, \(\infty)\)

Applications Of Derivatives The Real Line Into Three Disjoint Intervals

In intervals \((-\infty,-2) and (-1, \infty)\) i.e., when x<-2 and \(x>-1, f^{\prime}(x)<0\)

f is strictly decreasing for x<-2 \(\cup x>-1.\)

Now, in interval (-2,-1) i.e., when \(-2<x<-1, f^{\prime}(x)>0.\)

f is strietly increasing for \(-2<\mathrm{x}<-1\).

4. We have, f(x)=\(6-9 x-x^2\)

⇒ \(f^{\prime}(x)\)=-9-2 x

Now, \(f^{\prime}(x)\)=0 gives x=-\(\frac{9}{2}\)

The point x =\(-\frac{9}{2}\) divides the real line into two disjoint intervals ie. \(\left(-\infty,-\frac{9}{2}\right)\) and \(\left(-\frac{9}{2}, \infty\right)\)

Applications Of Derivatives The Point X Divides The Real Line Into Two Disjoint Intervals

In interval \(\left(-\infty,-\frac{9}{2}\right)\) i.e., for \(x<-\frac{9}{2}, f^{\prime}(x)>0\).

f is strictly increasing for \(\mathrm{x}<-\frac{9}{2}\)

In interval \(\left(-\frac{9}{2}, \infty\right)\) i.e., for \(x>-\frac{9}{2}, f^{\prime}(x)<0\).

f is strictly decreasing for \(\mathrm{x}>-\frac{9}{2}\).

5. We have, f(x)=\((x+1)^3(x-3)^5\)

⇒ \(f^{\prime}(x) =3(x+1)^2(x-3)^3+3(x-3)^2(x+1)^3=3(x+1)^2(x-3)^2[x-3+x+1]\)

=\(3(x+1)^2(x-3)^2(2 x-2)=6(x+1)^4(x-3)^2(x-1)\)

Now, \(f^{\prime}(x)=0 \Rightarrow x=-i, 3, i\)

The points x=-1, x 1, and x=3 divide the real line into four disjoint intervals i.e., \((-\infty,-1),(-1,1),(1,3)\) and \((3, \infty\)).

Applications Of Derivatives Four Disjoint Intervals

In intervals \((-\infty,-1)\) and \((-1,1), \mathrm{f}^{\prime}(\mathrm{x})<0\).

f is strictly decreasing in intervals \((-\infty,-1) \cup(-1,1)\).

In intervals (1,3) and (3, \(\infty), f^{\prime}(x)>0\).

f is strictly increasing in intervals (1,3) \(\cup(3, \infty)\)

Question 7. Show that y=\(\log (1+x)-\frac{2 x}{2+x}\), x>-1, is an increasing function of x throughout its domain.

Solution:

We have, y=\(\log (1+x)-\frac{2 x}{2+x}\)

⇒ \(\frac{d y}{d x}=\frac{1}{1+x}-\frac{(2+x)(2)-2 x(1)}{(2+x)^2}=\frac{1}{1+x}-\frac{4}{(2+x)^2}=\frac{x^2}{(1+x)(2+x)^2}\)

When x \(\in(-1, \infty), then \frac{x^2}{(2+x)^2}>0\) and (1+x)>0

⇒ \(\frac{d y}{d x}>0\) when x>-1.

y is an increasing function of x throughout its domain, i.e. (x>-1)

Question 8. Find the values of x for which y=\([x(x-2)]^2\) is an increasing function.

Solution:

We have, y=\([x(x-2)]^2-\left[x^2-2 x\right]^1\)

⇒ \(\frac{d y}{d x}=2\left(x^2-2 x\right)(2 x-2)=4 x(x-2)(x-1)\)

⇒ \(\frac{d y}{d x}=0 \Rightarrow\) x=0, x=2, x=1

The points x =0, x =1, and x=2 divide the real line into four disjoint intervals i.e., \((-\infty, 0), [0,1][1,2]\) and \([2, \infty)\)

Applications Of Derivatives The Real Line Into Four Disjoint Intervals

In intervals \((-\infty, 0]\) and \([1,2], \frac{\mathrm{dy}}{\mathrm{dx}} \leq 0\)

y is decreasing in intervals \((-\infty, 0] \cup[1,2]\).

However, in intervals [0,1] and [2, x), \(\frac{d y}{d x} \geq 0\)

y is increasing in intervals [0,1] \(\cup[2, \infty)\).

Question 9. Prove that y=\(\frac{4 \sin \theta}{(2+\cos \theta)}-\theta\) is an increasing function of \(\theta\) in \(\left[0, \frac{\pi}{2}\right]\).

Solution:

We have, y=\(\frac{4 \sin \theta}{(2+\cos \theta)}-\theta\)

⇒ \(\frac{d y}{d \theta} =\frac{(2+\cos \theta)(4 \cos \theta)-4 \sin \theta(-\sin \theta)}{(2+\cos \theta)^2}-1\)

=\(\frac{8 \cos \theta+4 \cos ^2 \theta+4 \sin ^2 \theta}{(2+\cos \theta)^2}-1=\frac{8 \cos \theta+4}{(2+\cos \theta)^2}-1\)

⇒ \(\frac{d y}{d \theta}=\frac{8 \cos \theta+4-\left(4+\cos ^2 \theta+4 \cos \theta\right)}{(2+\cos \theta)^2}-\frac{4 \cos \theta-\cos ^2 \theta}{(2+\cos \theta)^2}\)

=\(\frac{\cos \theta(4-\cos \theta)}{(2+\cos \theta)^2}\)

In interval \(\left[0, \frac{\pi}{2}\right]\), we have \(\cos \theta \geq 0\). Also, \(4>\cos \theta \Rightarrow 4-\cos \theta>0\)

⇒ \(\cos \theta(4-\cos \theta)>0\) and also \((2+\cos \theta)^2>0\)

⇒ \(\frac{\cos \theta(4-\cos \theta)}{(2+\cos \theta)^2} \geq 0 \Rightarrow \frac{d y}{d x} \geq 0\)

Therefore, y is increasing in the interval \(\left[0, \frac{\pi}{2}\right]\).

Question 10. Prove that the logarithmic function is strictly increasing on (0, \(\infty)\).

Solution:

Let f(x)=\(\log x\).

⇒ \(\mathrm{f}^{\prime}(\mathrm{x})=\frac{1}{\mathrm{x}}\)

It is clear that for x>0, \(f^{\prime}(x)=\frac{1}{x}>0\)

Hence, f(x)= log x is strictly increasing in the interval \((0, \infty)\).

Question 11. Prove that the function f given by f(x)=\(x^2-x+1\) is neither strictly increasing nor strictly decreasing on (-1,1).

Solution:

The given function is f(x)=\(x^2-x+1\).

⇒ \(f^{\prime}(x)=2 x-1\)

Now, \(f^{\prime}(x)=\theta \Rightarrow x=\frac{1}{2}\)

The point \(\frac{1}{2}\) divides the interval (-1,1) into two disjoint intervals i.e., \(\left(-1, \frac{1}{2}\right) and \left(\frac{1}{2}, 1\right)\)

Applications Of Derivatives The Point Divides The Interval Into Two Disjoint Intervals

Now, in interval \(\left(-1, \frac{1}{2}\right), \mathrm{f}^{\prime}(\mathrm{x})=2 \mathrm{x}-1<0\)

Therefore, f is strictly decreasing in the interval \(\left(-1, \frac{1}{2}\right)\).

However, in interval \(\left(\frac{1}{2}, 1\right), f^{\prime}(x)=2 x-1>0\)

Therefore, f is strictly increasing in the interval \(\left(\frac{1}{2}, 1\right)\).

Hence, f is neither strictly increasing nor strictly decreasing in interval (-1,1).

Question 12. Which of the following functions are strictly decreasing on \(\left(0, \frac{\pi}{2}\right)\)?

  1. \(\cos x\)
  2. \(\cos 2 x\)
  3. \(\cos 3 x\)
  4. \(\tan x\)

Solution:

1. Let \(f_2(x)=\cos x\)

⇒ \(f_1^{\prime}(x)=-\sin x\)

In interval \(\left(0, \frac{\pi}{2}\right), f_1^{\prime}(x)=-\sin x<0\)

⇒ \(\mathrm{f}_1(\mathrm{x})\) is strictly decreasing in interval \(\left(0, \frac{\pi}{2}\right)\).

2. Let \(f_2(x)=\cos 2 x\)

⇒ \(f_2(x)=-2 \sin 2 x\)

Now, \(0<x<\frac{\pi}{2} \Rightarrow 0<2 x<\pi \Rightarrow \sin 2 x>0 \Rightarrow-2 \sin 2 x<0\)

⇒ \(\mathrm{f}_2^{\prime}(\mathrm{x})=-2 \sin 2 \mathrm{x}<0 on \left(0, \frac{\pi}{2}\right)\)

⇒ \(\mathrm{f}_2(\mathrm{x})\) is strictly decreasing in interval \(\left(0, \frac{\pi}{2}\right)\).

3. Let \(f_1(x)=\cos 3 x\).

⇒ \(f_5^{\prime}(x)=-3 \sin 3 x\)

Now, \(f^{\prime}(x)=0\)

⇒ \(\sin 3 x=0 \Rightarrow 3 x=\pi, as x \in\left(0, \frac{\pi}{2}\right) \Rightarrow x=\frac{\pi}{3}\)

The point \(\mathrm{x}=\frac{\pi}{3}\) divides the interval \(\left(0, \frac{\pi}{2}\right)\) into two disjoint intervals i.e., \(\left(0, \frac{\pi}{3}\right)\) and \(\left(\frac{\pi}{3}, \frac{\pi}{2}\right)\)

Now in interval \(\left(0, \frac{\pi}{3}\right), f_3^{\prime}(x)=-3 \sin 3 x<0\) \(\left[\right. as \left.0<x<\frac{\pi}{3} \Rightarrow 0<3 x<\pi\right]\)

⇒ \(\mathrm{f}(\mathrm{x})\) is strictly decreasing in interval \(\left(0, \frac{\pi}{3}\right)\)

However, in interval \(\left(\frac{\pi}{3}, \frac{\pi}{2}\right), \mathrm{f}^{\prime}(\mathrm{x})\)

=\(-3 \sin 3 \mathrm{x}>0 \left[\right.as\left.\frac{\pi}{3}<\mathrm{x}<\frac{\pi}{2} \Rightarrow \pi<3 \mathrm{x}<\frac{3 \pi}{2}\right]\)

⇒ \(\mathrm{f}_3(\mathrm{x})\) is strictly increasing in interval \(\left(\frac{\pi}{3}, \frac{\pi}{2}\right)\).

Hence, \(f_1\) is neither strictly increasing nor strictly decreasing in the interval \(\left(0, \frac{\pi}{2}\right)\).

4. Let \(f_4(x)=\tan x\)

⇒ \(f_4^{\prime}(x)=\sec ^2 x\)

In interval \(\left(0, \frac{\pi}{2}\right), \mathrm{f}_4^{\prime}(\mathrm{x})=\sec ^2 \mathrm{x}>0\)

⇒ \(\mathrm{f}_4\) is strictly increasing in interval \(\left(0, \frac{\pi}{2}\right)\)

Therefore, functions \(\cos x\) and \(\cos 2 x\) are strictly decreasing in \(\left(0, \frac{\pi}{2}\right)\)

Hence, the correct answers are 1 and 2.

Question 13. On which of the following intervals is the function f given by f(x)=\(x^{100}+\sin x-1\) strictly decreasing?

Solution:

  1. (0,1)
  2. \(\left(\frac{\pi}{2}, \pi\right)\)
  3. \(\left(0, \frac{\pi}{2}\right)\)
  4. None of these

Solution: 4. None of these

We have, \(f(x)=x^{i n g}+\sin x-1\)

⇒ \(f^{\prime}(x)=100 x^{00}+\cos x\)

In interval (0,1),cos x>0 and 100 x^n>0

⇒ \(f^{\prime}(x)>0\)

Thus, function f is strictly increasing in interval (0,1).

In interval \(\left(\frac{\pi}{2}, \pi\right), \cos x<0 and 100 x^{1 / 4}>0. Also, 100 x^*>\cos x\)

⇒ \(f^{\prime}(x)>0 in \left(\frac{\pi}{2}, \pi\right)\).

Thus, the function is strictly increasing in the interval \(\left(\frac{\pi}{2}, \pi\right)\)

In interval \(\left(0, \frac{\pi}{2}\right), \cos x>0\) and \(100 x^{\infty}>0 100 x^n+\cos x>0\)

⇒ \(\mathrm{f}^{\prime}(\mathrm{x})>0 on \left(0, \frac{\pi}{2}\right)\)

f is strictly increasing in interval \(\left(0, \frac{\pi}{2}\right)\)

Hence function f is strictly decreasing in none of the intervals.

The correct answer is 4.

Question 14. Find the least value of a such that the function f given by f(x)=\(x^2+a x+1\) is strictly increasing on [1,2].

Solution:

We have, f(x)=\(x^2+a x+1\)

⇒ \(\mathrm{f}^{\prime}(\mathrm{x})=2 \mathrm{x}+\mathrm{a}\)

Now, \(1<\mathrm{x}<2 \Rightarrow 2+\mathrm{a}<2 \mathrm{x}+\mathrm{a}<4+\mathrm{a} \Rightarrow 2+\mathrm{a}<\mathrm{f}^{\prime}(\mathrm{x})<4+\mathrm{a}\)

Now, function f will be strictly increasing in [1,2], if \(f^{\prime}(x)>0 in [1,2]\).

⇒ \(\Rightarrow 2+a \geq 0 \Rightarrow a \geq-2\)

The least Value of a for f to be strictly increasing in [1,2] is a=-2,

Question 15. Let I be any interval disjoint from [-1,1]. Prove that the function f given by \(f(x)=x+\frac{1}{x}\) is strictly increasing on I.

Solution:

We have, \(f(x)=x+\frac{1}{x} f^{\prime}(x)=1-\frac{1}{x^2} \Rightarrow f^{\prime}(x)=\frac{x^2-1}{x^2}\)

Now, \(\mathrm{f}^{\prime}(\mathrm{x})-0 \Rightarrow \mathrm{x}= \pm 1\)

The points x =1 and x=-1 divide the real line in three disjoint intervals i.e., \((-\infty,-1),(-1,1)\) and \((1, \infty)\).

f is strictly increasing on \((-\infty,-1) \cup(1, \infty)\)

Hence, function f is strictly increasing in the interval I disjoint from [-1,1].

Hence, the given result is proved.

Question 16. Prove that the function f given by f(x)=\(\log \sin x\) is strictly increasing on \(\left(0, \frac{\pi}{2}\right)\) and strictly decreasing on \(\left(\frac{\pi}{2}, \pi\right)\).

Solution:

We have, f(x)=\(\log \sin x\)

⇒ \(f^{\prime}(x)=\frac{1}{\sin x}(\cos x)=\cot x\)

In interval \(\left(0, \frac{\pi}{2}\right), \mathrm{f}^{\prime}(\mathrm{x})=\cot \mathrm{x}>0 \mathrm{f}\) is strictly increasing on \(\left(0, \frac{\pi}{2}\right)\).

In interval \(\left(\frac{\pi}{2}, \pi\right), f^{\prime}(x)=\cot x<0\), f is strictly decreasing on \(\left(\frac{\pi}{2}, \pi\right)\).

Question 17. Prove that the function f given by f(x)=\(\log \cos x\) is strictly decreasing on \(\left(0, \frac{\pi}{2}\right)\) and strictly increasing on \(\left(\frac{\pi}{2}, \pi\right)\).

Solution:

We have, f(x)=\(\log \cos x\) \(f^{\prime}(x)=\frac{1}{\cos x}(-\sin x)=-\tan x\)

In interval \(\left(0, \frac{\pi}{2}\right), \tan x>0 \Rightarrow-\tan x<0\)

⇒ \(\mathrm{f}^{\prime}(\mathrm{x})<0 on \left(0, \frac{\pi}{2}\right) \mathrm{f}\) is strictly decreasing on \(\left(0, \frac{\pi}{2}\right)\).

In interval \(\left(\frac{\pi}{2}, \pi\right), \tan x<0 \Rightarrow-\tan x>0\)

⇒ \(f^{\prime}(x)>0 on \left(\frac{\pi}{2}, \pi\right)\) f is strictly increasing on \(\left(\frac{\pi}{2}, \pi\right)\)

Question 18. Prove that the function given by \(f(x)=x^3-3 x^2+3 x-100\) is increasing in R.

Solution:

We have, f(x)=\(x^3-3 x^2+3 x-100\)

⇒ \(f^{\prime}(x)=3 x^2-6 x+3-3\left(x^2-2 x+1\right)-3(x-1)^2\)

For any x \(\in R,(x-1)^2 \geq 0\).

3\((x-1)^2 \geq 0 \Rightarrow f^{\prime}(x) \geq 0 \forall x \in R\)

Hence, the given function f(x) is increasing in \(\mathbf{R}\).

Question 19. The interval in which \(y=x^2 e^{-3}\) is increasing is

  1. \((-\infty, \infty)\)
  2. (-2,0)
  3. \((2, \infty)\)
  4. (0,2)

Solution: 4. (0,2)

We have, y=\(x^2 e^{-7}\)

⇒ \(\frac{d y}{d x} =2 x e^{-x}-x^2 e^{-5}=x e^{-x}(2-x)\)

=-\(x e^{-x}(x-2)=0 \Rightarrow x=0,2 (\mathrm{e}^{-3}\) is always positive)

For increasing function, we have :

⇒ \(-x e^{-x}(x-2) \geq 0\)

⇒ \(x^{-4}(x-2) \leq 0\)

Hence, the required interval is (0,2).

Application Of Derivatives Exercise – 6.3

Question 1: Find the maximum and minimum values, if any, of the following functions given by:

  1. f(x)=\((2 x-1)^2+3\)
  2. f(x)=\(9 x^4+12 x+2\)
  3. f(x)=\(-(x-1)^2+10\)
  4. g(x)=\(x^{\prime}+1\)

Solution:

1. The given function is \(f(x)=(2 x-1)^2+3\).

It can be observed that \((2 x-1)^2 \geq 0\) for every x \(\in\) R.

Therefore, \(f(x)=(2 x-1)^2+3 \geq 3 \)for every x \(\in\) R.

Minimum value of f(x)=3

Minimum value of f(x) = 3

and function f(x) does not have a maximum value.

2. The given function is f(x)=\(9 x^2+12 x+2=(3 x+2)^2-2\).

It can be observed that \((3 \mathrm{x}+2)^2 \geq 0\) for every \(\mathrm{x} \in \mathbf{R}\).

Therefore, f(x)=\((3 x+2)^3-2 \geq-2\) for every \(x \in R\).

Minimum value of f(x)=-2

and function f does not have a maximum value.

3. The given function is f(x)=-\((x-1)^3+10\).

It can be observed that \((x-1)^2 \geq 0\) for every x \(\in\) R.

–\((x-1)^2 \leq 0\) for every x \(\in R\)

Therefore, f(x)=\(-(x-1)^2+10 \leq 10\) for every x \(\in \mathbf{R}\).

Maximum value of f(x)=10

and function f(x) does not have a minimum value.

4. The given function is \(g(x)=x^{\prime}+1\). and

At x \(\rightarrow \infty \quad g(x) \rightarrow \infty\)

At \(\mathrm{x} \rightarrow-\infty \quad \mathrm{g}(\mathrm{x}) \rightarrow-\infty\)

Hence, functioning (x neither has a maximum value nor a minimum value.

Question 2. Find the maximum and minimum values, if any, of the following functions given by 0

  1. f(x)=|x+2|-1
  2. g(x)=-|x+1|+3
  3. \(\mathrm{h}(\mathrm{x})=\sin (2 \mathrm{x})+5\)
  4. f(x)=\(|\sin 4 x+3|\)
  5. \(h(x)=x+1, x \in(-1,1)\)
  6. f(x)=|x+2|-1

Solution:

1. We know that |x+2| \(\geq 0\) for every x \(\in\) R.

Therefore, f(x)=|x+2|-1 \(\geq-1\) for every x \(\in\) R.

Minimum value of f(x)=-1

and function f(x) does not have a maximum value.

2. \(g(x)=x^3-3 x\)

⇒ \(g^{\prime}(x)=3 x^2-3\) and \(g^7(x)-6 x\)

Now for maxima or minima \(g^{\prime}(x)=0\)

⇒ \(3 x^2-3=0 \Rightarrow x- \pm 1 \)when x-1

when x=\(-1 g^{\prime \prime}(1)=6>0 \)

⇒  \(g'(-1)=-6<0\)

By the second derivative test, x=1 is a point of local minima and local minimum value of \(\mathrm{g}(\mathrm{x})\) at x=1 is \(g(1)=1^3-3=1-3=-2\).

However, x=-1 is a point of local maxima, and the local maximum value of g(x) at x=-1 is

⇒  \(g(-1)-(-1)^3-3(-1)–1+3=2\)

3. \(\mathrm{h}(\mathrm{x})-\sin \mathrm{x}+\cos \mathrm{x}, 0<\mathrm{x}<\frac{\pi}{2}\)

⇒  \(h^{\prime}(x)=\cos x-\sin x\)

for maxima or minima, \(h^{\prime}(x)=0 \Rightarrow \sin x=\cos x \Rightarrow \tan x=1\)

x=\(\frac{\pi}{4} \in\left(0, \frac{\pi}{2}\right)\)

⇒  \(h^{\prime \prime}(x)=-\sin x-\cos x=-(\sin x+\cos x)\)

⇒  \(h^{\prime
\prime}\left(\frac{\pi}{4}\right)=-\left(\frac{1}{\sqrt{2}}+\frac{1}{\sqrt{2}}\right)=-\frac{2}{\sqrt{2}}=-\sqrt{2}<0\)

Therefore, by the second derivative test, x=\(\frac{\pi}{4}\) is a point of local maxima and the local maximum value of h(x) at x=\(\frac{\pi}{4} is h\left(\frac{\pi}{4}\right)\)

=\(\sin \frac{\pi}{4}+\cos \frac{\pi}{4}=\frac{1}{\sqrt{2}}+\frac{1}{\sqrt{2}}=\sqrt{2}\)

4. \(f(x)=\sin x-\cos x, 0<x<2 \pi\)

⇒  \(f^{\prime}(x)=\cos x+\sin x\)

for maxima or minima

⇒  \(f^{\prime}(x)=0 \Rightarrow \cos x=-\sin x=\tan x=-1 \Rightarrow x=\frac{3 \pi}{4}, \frac{7 \pi}{4} \in(0,2 \pi)\)

⇒  \(f^{\prime \prime}(x)=-\sin x+\cos x\)

⇒  \(f^{\prime \prime}\left(\frac{3 \pi}{4}\right)=-\sin \frac{3 \pi}{4}+\cos \frac{3 \pi}{4}=-\frac{1}{\sqrt{2}}-  \frac{1}{\sqrt{2}}=-\sqrt{2}<0\)

⇒ \(f^{\prime \prime}\left(\frac{7 \pi}{4}\right)=-\sin \frac{7 \pi}{4}+\cos \frac{7 \pi}{4}=\frac{1}{\sqrt{2}}+\frac{1}{\sqrt{2}}=\sqrt{2}>0\)

Therefore, by second derivative test, x=\(\frac{3 \pi}{4}\) is a point of local maxima and the local maximum value of f(x) at \(x=\frac{3 \pi}{4} is f\left(\frac{3 \pi}{4}\right)\)

=\(\sin \frac{3 \pi}{4}-\cos \frac{3 \pi}{4}=\frac{1}{\sqrt{2}}+\frac{1}{\sqrt{2}}=\sqrt{2}\).

However, x=\(\frac{7 \pi}{4}\) is a point of local minima and the local minimum value of f(x) at x=\(\frac{7 \pi}{4} is f\left(\frac{7 \pi}{4}\right)\)

= \(\sin \frac{7 x}{4}-\cos \frac{7 \pi}{4}=-\frac{1}{\sqrt{2}}-\frac{1}{\sqrt{2}}=-\sqrt{2}\)

5. f(x)=\(x^3-6 x^2+9 x+15\) and \(f^{\prime}(x)=3 x^2-12 x+9\)

for maxima or minima

⇒  \(f^{\prime}(x)=0 \Rightarrow 3\left(x^2-4 x+3\right)=0 \Rightarrow 3(x-1)(x-3)-0 \Rightarrow x=1,3\)

Now,\(f^{\prime}(x)=6 x-12=6(x-2)\)

when x=1, \(f^{\prime}(1)=6(1-2)=-6<0\)

when x=3, \(f^{\prime}(3)=6(3-2)\)=6>0

Therefore, by the second derivative test, x =1 is a point of local maxima and the local maximum value of f(x) at x=1 is f(1)=1-6+9+15=19.

However, x=3 is a point of local minima, and the local minimum value of f(x) at x=3 is f(3)=27-54+27+15=15

6. \(g(x)=\frac{x}{2}+\frac{2}{x}, x>0\) and \(g^{\prime}(x)=\frac{1}{2}-\frac{2}{x^2}\)

for maxima or minimal

⇒  \(g^{\prime}(x)=0 \text { gives } \frac{2}{x^2}=\frac{1}{2} \Rightarrow x^2=4 \Rightarrow x= \pm 2\)

Since x>0, we take x=2.

Now, \(g^{\prime \prime}(x)=\frac{4}{x^3} \Rightarrow g^{\prime \prime}(2)=\cdot \frac{4}{2^{\prime}}=\frac{1}{2}>0\)

Therefore, by the second derivative test, x=2 is a point of local minima and the local minimum value of g(x) at x-2 is \(g(2)-\frac{2}{2}+\frac{2}{2}=1+1=2\)

7. g(x)=\(\frac{1}{x^2+2}\) and \(g^{\prime}(x)=\frac{-(2 x)}{\left(x^2+2\right)^2}\)

for maxima or minima, \(g^{\prime}(x)=0 \Rightarrow \frac{-2 x}{\left(x^2+2\right)^2}=0 \Rightarrow\) x=0

Now, for values close to x=0 and to the left of 0, \(g^{\prime}(x)>0\).

Also, for values close to x=0 and to the right of 0, \(\mathrm{~g}^{\prime}(\mathrm{x})<0\).

Therefore, by the first derivative test, x=0 is a point of local maximum and the local maximum value of g(0) is \(\frac{1}{0+2}=\frac{1}{2}\)

8. \(f(\mathrm{x})=\mathrm{x} \sqrt{1-\mathrm{x}}, 0<\mathrm{x}<1\)

⇒  \(f^{\prime}(x)=\sqrt{1-x}+x \cdot \frac{1}{2 \sqrt{1-x}}(-1)=\sqrt{1-x}-\frac{x}{2 \sqrt{1-x}}=\frac{2(1-x)-x}{2 \sqrt{1-x}}=\frac{2-3 x}{2 \sqrt{1-x}}\)

for maxima or minima

⇒  \(f^{\prime}(\mathrm{x})=0 \Rightarrow \frac{2-3 \mathrm{x}}{2 \sqrt{1-x}}=0 \Rightarrow 2-3 \mathrm{x}=0 \Rightarrow \mathrm{x}\)

=\(\frac{2}{3} \Rightarrow \mathrm{f}^{\prime}(\mathrm{x})=\frac{2-3 \mathrm{x}}{2 \sqrt{1-\mathrm{x}}}\)

⇒  \(f^{\prime \prime}(x)=\frac{1}{2}\left[\frac{\sqrt{1-x}(-3)-(2-3 x)\left(\frac{-1}{2 \sqrt{1-x}}\right)}{1-x}\right]\)

=\(\frac{\sqrt{1-x}(-3)+(2-3 x)\left(\frac{1}{2 \sqrt{1-x}}\right)}{2(1-x)} \)

=\(\frac{-6(1-x)+(2-3 x)}{4(1-x)^{\frac{2}{2}}}-\frac{3 x-4}{4(1-x)^{\frac{2}{2}}}\)

⇒  \(\mathrm{f} *\left(\frac{2}{3}\right)=\frac{3\left(\frac{2}{3}\right)^{-4}}{4\left(1-\frac{2}{3}\right)^{\frac{2}{3}}}\)

=\(\frac{2-4}{4\left(\frac{1}{3}\right)^3}=\frac{-1}{2\left(\frac{1}{3}\right)^{\frac{1}{3}}}<0 \)

Therefore, by the second derivative test, x=\(\frac{2}{3}\) is a point of local maxima and the local maximum value of f(x) at x=\(\frac{2}{3}\) is \( f\left(\frac{2}{3}\right)\)

=\(\frac{2}{3} \sqrt{1-\frac{2}{3}}=\frac{2}{3} \sqrt{\frac{1}{3}}=\frac{2}{3 \sqrt{3}}=\frac{2 \sqrt{3}}{9}\)

Question 4. Prove that the following functions do not have maxima or minima:

  1. \(f(x)=e^x\)
  2. \(g(x)=\log x\)
  3. \(h(x)=x^2+x^2+x+1\)

Solution:

1. We have, f(x)=\(e^”\)

⇒  \(f^{\prime}(x)=e^x\)

Now, if \(\mathrm{f}^{\prime}(\mathrm{x})\)=0, then \(\mathrm{e}^{\prime}\)=0, But, the exponential function can never assume 0 for any value of x.

Therefore, there does not exist x \(\in\) R such that \(f^{\prime}(x)\)=0

Hence, function f does not have maxima or minima.

2. We have, \(\mathrm{g}(\mathrm{x})=\log \mathrm{x}\)

⇒  \(g^{\prime}(\mathrm{x})=\frac{1}{\mathrm{x}}\)

Since log x is defined for a positive number x, \(\mathrm{g}^{\prime}(\mathrm{x})>0\) for any x.

Therefore, there does not exist x \(\in R\) such that \(g^{\prime}(x)\)=0.

Hence, function g does not have maxima or minima.

3. We have, \(\mathrm{h}(\mathrm{x})=\mathrm{x}^2+\mathrm{x}^2+\mathrm{x}+1\)

⇒  \(h^{\prime}(\mathrm{x})-3 \mathrm{x}^2+2 \mathrm{x}+1\)

Now, \(h^{\prime}(x)=0 \Rightarrow 3 x^2+2 x+1=0 \Rightarrow x=\frac{-2 \pm 2 \sqrt{2} i}{6}=\frac{-1 \pm \sqrt{2} i}{3} \notin R\)

Therefore, there does not exist x \(\in\) R such that \(h^{\prime}(x)\)=0.

Hence, function b does not have maxima or minima.

Question 5. Find the absolute maximum value and the absolute minimum value of the following functions in the given intervals:

  1. f(x)=\(x^3, x \in[-2,2]\)
  2. f(x)=\(\sin x+\cos x, x \in[0, \pi]\)
  3. \(f(\mathrm{x})=4 \mathrm{x}-\frac{1}{2} \mathrm{x}^2, \mathrm{x} \in\left[-2, \frac{9}{2}\right]\)
  4. f(x)=\((x-1)^2+3, x \in[-3,1]\)

Solution:

1. The given function is f(x)=x.

⇒  \(\mathrm{f}(\mathrm{x})=3 \mathrm{x}^2\)

Now, for maxima or minima

Put \(f^{\prime}(x)=0 \Rightarrow x=0\)

Then, we evaluate the value of f(x) at critical point x=0 and at endpoints of the interval [-2,2].

At x=0, f(0)=0

At x=-2, f(-2)=\((-2)^3\)=-8

At x=2, f(2)=\((2)^3\)=8

Hence, we can conclude that the absolute maximum value of f(x) on [-2,2] is 8 occurring at x = 2.

Also, the absolute minimum value of f(x) on [-2,2] is -8 occurring at x=-2.

2. The given function is f(x)=\(\sin x+\cos x\).

⇒  \(f^{\prime}(x)=\cos x-\sin x\)

Now, for maxima or minima \(f^{\prime}(x)=0 \Rightarrow \sin x=\cos x \Rightarrow \tan x=1 \Rightarrow x=\frac{\pi}{4}\)

Then, we evaluate the value of f(x) at critical point x=\(\frac{\pi}{4}\) and at the end points of the interval \([0, \pi]\).

⇒  \(f\left(\frac{\pi}{4}\right)=\sin \frac{\pi}{4}+\cos \frac{\pi}{4}=\frac{1}{\sqrt{2}}+\frac{1}{\sqrt{2}}=\frac{2}{\sqrt{2}}=\sqrt{2}\)

f(0)=\(\sin 0+\cos\) 0=0+1=1

f\((\pi)=\sin \pi+\cos \pi\)=0-1=-1

Hence, we can conclude that the absolute maximum value of f(x) on \([0, \pi]\) is \(\sqrt{2}\) occurring at x=\(\frac{\pi}{4}\) and the absolute minimum value of f(x) on \(\{0, \pi\}\) is -1 occurring at x=\(\pi\).

The given function is f(x)=\(4 x-\frac{1}{2} x^2\) and \(f^{\prime}(x)=4-\frac{1}{2}(2 x)=4-x\)

Now, for maxima or minima \(\mathrm{f}^{\prime}(\mathrm{x})=0 \Rightarrow \mathrm{x}-4\)

Then, we evaluate the value of f(x) at critical point x=4 and at the endpoints of the interval \(\left[-2, \frac{9}{2}\right]\).

⇒  \(f(4)=16-\frac{1}{2}(16)\)=16-8=8

⇒  \(f(-2)=-8-\frac{1}{2}(4)\)=-8-2=-10

⇒  \(f\left(\frac{9}{2}\right)=4\left(\frac{9}{2}\right)-\frac{1}{2}\left(\frac{9}{2}\right)^2=18-\frac{81}{8}=18-10.125=7.875\)

Hence, we can conclude that the absolute maximum value of f(x) on \(\left[-2, \frac{9}{2}\right]\) is 8 occurring at x=4 and the absolute minimum value of f(x) on \(\left[-2, \frac{9}{2}\right]\) is -10 occurring at x=-2.

4. The given function is f(x)=\((x-1)^2+3\) and \(f^{\prime}(x)=2(x-1)\)

Now, for maxima or minima

⇒  \(f^{\prime}(x)=0 \Rightarrow 2(x-1)=0 \Rightarrow x=1\)

Then, we evaluate the value of f(x) at critical point x=1 and at the endpoints of the interval [-3,1].

⇒ \(f(1)=(1-1)^2+3=0+3=3 \)

f(-3)=(-3-1)^2+3=16+3=19

Hence, we can conclude that the absolute maximum value of f(x) on [-3,1] is 19 occurring at x=-3, and the minimum value of f(x) on [-3,1] is 3 occurring at x=1.

Question 6. Find the maximum profit that a company can make, if the profit function is given by p(x)=41-72 x-18\( x^2\)

Solution:

The profit function is, p(x)=\(41-72 x-18 x^2\)

⇒  \(\Rightarrow p^{\prime \prime}(x)\)=-72-36 x and \(p^{\prime \prime}(x)\)=-36

for maxima or minima

⇒  \(p^{\prime}(x)\)=0

⇒ \(-72-36 x=0 \Rightarrow x=\frac{-72}{36} \Rightarrow x=-2 \)

⇒  \(p^{\prime \prime}(-2)=-36<0\)

Then, by the second derivative test, x=-2 is the point of local maxima of p(x).

So, the local maximum value is \(p(-2)=41-72(-2)-18(-2)^2=41+144-72=113\)

Therefore, the maximum profit that the company can make is 113 units.

Question 7. Find both the maximum value and minimum value of 3 \(x^{\prime}-8 x^{\prime}+12 x^{\prime}-48 x+25\) on the interval [0.3].

Solution:

Let f(x)=\(3 x^4-8 x^3+12 x^2-48 x+25\) on [0.3]

⇒  \(f^{\prime}(x)\)=12 x^3-24 x^2+24 x-48

for maxima or minima \(\mathrm{f}^{\prime}(\mathrm{x})\)=0

12 \(\mathrm{x}^2-24 \mathrm{x}^2+24 \mathrm{x}-48=0 \)

⇒  \(\mathrm{x}^3-2 \mathrm{x}^2+2 \mathrm{x}-4=0 \Rightarrow(\mathrm{x}-2)\left(\mathrm{x}^2+2\right)=0\)

x =2 \(\left(\mathrm{x}^2+2 \neq 0\right)\)

x=2 is the turning point.

Now we evaluate the value of f(x) at x=2 and at the endpoints of the interval [0,3]

f(2)=3(16)-8(8)+12(4)-48(2)+25=-39

f(0)=25

f(3)=3(81)-8(27)+12(9)-48(3)+25-16

Therefore. the absolute minimum value is -39 at x=2 and the absolute maximum value is 25 at x=0.

Question 8. At what points in the interval [0.2 \(\pi\)]? does the function sin 2 x attain its maximum value?

Solution:

Let f(x)=sin 2 x.

⇒  \(f^{\prime}(x)=2 \cos 2 x\)

Now, for maxima or minima

⇒  \(\mathrm{f}^{\prime}(\mathrm{x})=0 \Rightarrow \cos 2 \mathrm{x}\)=0

2 \(\mathrm{x}=\frac{\pi}{2} \cdot \frac{3 \pi}{2}, \frac{5 \pi}{2} \cdot \frac{7 \pi}{2}\)

⇒  \(\mathrm{x}=\frac{\pi}{4}, \frac{3 \pi}{4}, \frac{5 \pi}{4}, \frac{7 \pi}{4}\)

Then. we evaluate the values of f(x) at critical points x=\(\frac{\pi}{4}, \frac{3 \pi}{4}, \frac{5 \pi}{4}, \frac{7 \pi}{4}\), and at the end points of the interval [0.2\( \pi\)].

⇒  \(f\left(\frac{\pi}{4}\right)=\sin \frac{\pi}{2}\)=1,

⇒  \(f\left(\frac{3 \pi}{4}\right)=\sin \frac{3 \pi}{2}\)=-1,

⇒  \(f\left(\frac{5 \pi}{4}\right)=\sin \frac{5 \pi}{2}\)=1,

⇒  \(f\left(\frac{7 \pi}{4}\right)=\sin \frac{7 \pi}{2}\)=-1,

⇒  \(f(0)=\sin 0=0, f(2 \pi)=\sin 2 \pi\)=0

Hence, we can conclude that the absolute maximum value of f(x) on [0,2 \(\pi\)] is 1 occurring at

x=\(\frac{\pi}{4}\) and x=\(\frac{5 \pi}{4}\)

Question 9. What is the maximum value of the function sin x+cos x?

Solution:

Let f(x)=\(\sin x+\cos x\).

⇒  \(f^{\prime}(x)=\cos x-\sin x\)

for maxima or minima

⇒  \(f^{\prime}(x)=0 \Rightarrow \sin x=\cos x \Rightarrow \tan x=1 \Rightarrow x=\frac{\pi}{4}, \frac{5 \pi}{4} \ldots, n \)

⇒  \(f^{\prime \prime}(x)=-\sin x-\cos x=-(\sin x+\cos x)\)

Now, \(\mathrm{f}^{\prime}(\mathrm{x})\) will be negative when \((\sin \mathrm{x}+\cos \mathrm{x})\) is positive i.e., when sin x and cos x are both positive.

Also, we know that sin x and cos x both are positive in the first quadrant. Then, \(\mathrm{P}^{\prime}(x)\) will be negative when x \(\in\left(0, \frac{\pi}{2}\right)\)

Thus, we consider x =\(\frac{\pi}{4}\)

⇒  \(f^{\prime \prime}\left(\frac{\pi}{4}\right)=-\left(\sin \frac{\pi}{4}+\cos \frac{\pi}{4}\right)\)

=-\(\left(\frac{2}{\sqrt{2}}\right)=-\sqrt{2}<0\)

By second derivative test, \(\mathrm{f}(\mathrm{x})\) will be the maximum at x =\frac{\pi}{4} and the maximum value of \(\mathrm{f}(\mathrm{x})\) is

f\(\left(\frac{\pi}{4}\right)=\sin \frac{\pi}{4}+\cos \frac{\pi}{4}\)

=\(\frac{1}{\sqrt{2}}+\frac{1}{\sqrt{2}}=\frac{2}{\sqrt{2}}=\sqrt{2}\)

Question 10. Find the maximum value of \(2 x^3-24 x+107\) in the interval [1,3]. Find the maximum value of the same function in [-3,-1].

Solution:

Let \(f(x)-2 x^3-24 x+107\).

⇒ \(f^{\prime}(x)=6 x^2-24=6\left(x^2-4\right)\)

Now, for maxima or minima

⇒  \(f^{\prime}(x)=0 \Rightarrow 6\left(x^2-4\right)=0 \Rightarrow x^2=4 \Rightarrow x= \pm 2\)

We first consider the interval [1,3].

Then, we evaluate the value of f(x) at the critical point x-2 \(\in\)[1,3] and at the endpoints of the interval [1,3].

f(1)=2(1)-24(1)+107=2-24+107=85

f(2)=2(8)-24(2)+107=16-48+107=75

f(3)=2(27)-24(3)+107=54-72+107=89

Hence the absolute maximum value of f(x) in the interval [1,3] is 89 occurring at x=3.

Next, we consider the interval [-3,-1]

Now, we evaluate the value of f(x) at the critical point x = -2 \(\in\) [-3, -1] and at the endpoints of the interval [-3, -1 ].

f(-1) = 2(-1)- 24 (-1)+ 107 = -2 + 24 + 107= 129

f(~2) = 2(-8)- 24 (-2)+ 107 = -16 + 48 + 107 = 139

f(-3) = 2 (-27)- 24(-3) + 1 07 = -54 + 72 + 1 07 = 125

Hence, the absolute maximum value of (x) in the interval [-3, -1] is 139 occurring at x= -2.

Question 11. It is given that at x=1, the function \(x^4-62 x^2+a x+9\) attains its maximum value, on the interval [0,2].

Solution:

Find the value of a.

Let \(f(x)=x^4-62 x^2+a x+9\).

⇒  \(f^{\prime}(x)=4 x^2-124 x+a\)

It is given that function f attains its maximum value on the interval [0,2] at x=1.

⇒  \(\mathrm{f}^{\prime}(1)=0\)

4-124+a=0 \(\Rightarrow\) a=120. Hence, the value of a is 120

Question 12. Find the maximum and minimum values of x+sin 2 x on [0,2 \(\pi\)].

Solution:

Let f(x)=x+\(\sin 2 x and f^{\prime}(x)=1+2 \cos 2 x\)

Now, for maxima or minima

⇒  \(f(\mathrm{x})=0 \Rightarrow \cos 2 \mathrm{x}=-\frac{1}{2}=-\cos \frac{\pi}{3}=\cos \left(\pi-\frac{\pi}{3}\right)=\cos \frac{2 \pi}{3}\)

2\( \mathrm{x}=2 \pi \pm \frac{2 \pi}{3}, \mathrm{n} \in \mathrm{Z} \)

⇒ \(\mathrm{x}=\mathrm{n} \pi \pm \frac{\pi}{3}, \mathrm{n} \in \mathrm{Z} \Rightarrow \mathrm{x}=\frac{\pi}{3}, \frac{2 \pi}{3}, \frac{4 \pi}{3}, \frac{5 \pi}{3} \in[0,2 \pi]\)

Then, we evaluate the value of f(x) at critical points x=\(\frac{\pi}{3}, \frac{2 \pi}{3}, \frac{4 \pi}{3}\), \(\frac{5 \pi}{3}\) and at the endpoints of the interval [0,2 \(\pi\)].

⇒ \(f\left(\frac{\pi}{3}\right)=\frac{\pi}{3}+\sin \frac{2 \pi}{3}=\frac{\pi}{3}+\frac{\sqrt{3}}{2}\) ;

f\(\left(\frac{2 \pi}{3}\right)=\frac{2 \pi}{3}+\sin \frac{4 \pi}{3}=\frac{2 \pi}{3}-\frac{\sqrt{3}}{2}\)

⇒ \(f\left(\frac{4 \pi}{3}\right)=\frac{4 \pi}{3}+\sin \frac{8 \pi}{3}=\frac{4 \pi}{3}+\frac{\sqrt{3}}{2}\) ;

⇒ \(f\left(\frac{5 \pi}{3}\right)=\frac{5 \pi}{3}+\sin \frac{10 \pi}{3}=\frac{5 \pi}{3}-\frac{\sqrt{3}}{2}\)

f(0)=0+sin 0=0

⇒ \(f(2 \pi)=2 \pi+\sin 4 \pi=2 \pi+0=2 \pi+0=2 \pi\)

Hence, we can conclude that the absolute maximum value of f(x) in the interval [0,2 \(\pi\)] is 2 \(\pi\) occurring at x=2 \(\pi\) and the absolute minimum value of f(x) in the interval \([0,2 \pi]\) is \(\theta\) occurring at x=0.

Question 13. Find two numbers whose sum is 24 and whose product is as large as possible.

Solution:

Let the two numbers be x and y.

According to the question, x+y=24

y=24-x

And let z is the product of x and y.

⇒ \(\mathrm{z}=x \mathrm{y}\)

⇒ \(\mathrm{z}=\mathrm{x}(24-\mathrm{x})\)

⇒ \(\mathrm{z}=24 \mathrm{x}-\mathrm{x}^2 \Rightarrow \frac{\mathrm{dz}}{\mathrm{dx}}=24-2 \mathrm{x}\) and \(\frac{\mathrm{d}^2 \mathrm{z}}{\mathrm{dx}^2}=-2\) [From equation (1)]

Now to find turning point, \(\frac{\mathrm{dz}}{\mathrm{dx}}\)=0

24-2 x=0 \(\Rightarrow\) x=12

At x=12, \(\frac{d^2 z}{d^2}\)=-2<0

x =12 is a point of local maxima and z is maximum at x=12.

From equation (1), y=24-12=12

Therefore, the two required numbers are 12 and 12.

Question 14. Find two positive numbers x and y such that x+y=60 and \(x y^3\) is maximum.

Solution:

The two numbers are x and y such that x+y=60.

y=60-x

Let f(x)=\(x y^3\).

⇒ \(\mathrm{f}(\mathrm{x})=\mathrm{x}(60-\mathrm{x})^2\)

⇒ \(\mathrm{f}^{\prime}(\mathrm{x})=(60-\mathrm{x})^3-3 \mathrm{x}(60-\mathrm{x})^2-(60-\mathrm{x})^2[60-\mathrm{x}-3 \mathrm{x}]=(60-\mathrm{x})^2(60-4 \mathrm{x})\)

And \(f^{\prime \prime}(x)=-2(60-x)(60-4 x)-4(60-x)^2=-2(60-x)[60-4 x+2(60-x)]\)

=-2(60-x)(180-6 x)

=-12(60-x)(30-x)

for maxima or minima

Now \(\mathrm{f}^{\prime}(\mathrm{x})=0=\mathrm{x}=60 or \mathrm{x}=15\)

When x=\(60 f^{\prime}(x)\)=0

When x=15. \(f^{\prime \prime}(x)=-12(60-15)(30-15)=-12 \times 45 \times 15<0=-8100<0\)

By second order derivative test, x=15 is a point of local maxima of f(x).

Thus, function \(x y^3\) is maximum when x=15 and y=60-15=45.

Hence, the required numbers are 15 and 45.

Question 15. Find two positive numbers x and y such that their sum is 35 and the product \(x^2 y^3\) is a maximum.

Solution:

Let one number be x. Then, the other number is y =(35-x).

Let P(x)=\(x^2 y^2\), Then, we have

P(x)=\(x^2(35-x)^5\)

⇒ \(P^{\prime}(x) =2 x(35-x)^3-5 x^2(35-x)^4\)

=\(x(35-x)^4[2(35-x)-5 x]=x(35-x)^4(70-7 x)-7 x(35-x)^4(10-x)\)

And, \(\mathrm{P}^{\prime \prime}(\mathrm{x})\)

=7\((35-\mathrm{x})^4(10-\mathrm{x})+7 \mathrm{x}\left[-(35-\mathrm{x})^4-4(35-\mathrm{x})^3(10-\mathrm{x})\right]\)

=\(7(35-\mathrm{x})^4(10-\mathrm{x})-7 \mathrm{x}(35-\mathrm{x})^4-28 \mathrm{x}(35-\mathrm{x})^3(10-\mathrm{x})\)

⇒ \(-7(35-\mathrm{x})^3[(35-\mathrm{x})(10-\mathrm{x})-\mathrm{x}(35-\mathrm{x})-4 \mathrm{x}(10-\mathrm{x})]\)

=\(7(35-\mathrm{x})^3[(350-45 \mathrm{x}+\mathrm{x}^2-35 \mathrm{x}+\mathrm{x}^2-40 \mathrm{x}+4 \mathrm{x}^2]\).

=7\((35-\mathrm{x})^3\left(6 \mathrm{x}^2-120 \mathrm{x}+350\right)\)

for maxima or minima

Now, \(\mathrm{P}^{\prime}(\mathrm{x})=0 \Rightarrow \mathrm{x}-0,35,10\)

When x=35 or x=0. This will make the product \(x^3 y^3 \)equal to 0 .

x=0 and y =35 cannot be the possible values of x.

When x=10, we have

⇒ \(\mathrm{P}^{-}(\mathrm{x})=7(35-10)^3(6 \times 100-120 \times 10+350)=7(25)^3(-250)<0\)

By second order derivative test, P(x) will be the maximum when x=10 and y=35-10=25. Hence, the required numbers are 10 and 25.

Question 17. A square piece of tin side 18 cm is to be made into a box without a top, by cutting a square from each corner and folding up the flaps to form the box. What should be the side of the square to be cut off so that the volume of the box is the maximum possible?

Solution:

Let the side of the square to be cut off be x cm. Then, the length and the breadth of the box will be (1 8- 2x) cm each and the height of the box is x cm. Therefore, the volume V(x) of the box is given by

V(x) =x\((18-2 x)^2\)

⇒ \(V^{\prime}(x) =(18-2 x)^2-4 x(18-2 x)=(18-2 x)[18-2 x-4 x]\)

=(18-2 x)(18-6 x)=12(9-x)(3-x)

And \(V^{\prime \prime}(x) =12[-(9-x)-(3-x)]=-12(9-x+3-x)\)

Applications Of Derivatives A Square Piece Of Tin Is Made Into A Box

=-12(12-2 x)=-24(6-x)

for maxima or minima

⇒ \(\mathrm{V}^{\prime}(\mathrm{x})=0 \Rightarrow \mathrm{x}=9 or x=3\)

If x=9, then the length and the breadth will become 0.

x \(\neq 9 \Rightarrow\) x=3

Now, \(\mathrm{V}^*(3)=-24(6-3)=-72<0\)

By second order derivative test, x == 3 is the point of maxima of V,

Hence, if we remove a square of side 3 cm from each corner of the square tin and make a box.

Question 18. A rectangular sheet of tin 45 cm by 24 cm is to be made into a box without a top, by cutting off a square from each corner and folding up the flaps. What should be the side of the square to be cut off so that the volume of the box is the maximum possible?

Solution:

Let the side of the square to be cut off be x cm. Then, the height of the box is x, the length is 45- 2x, and the breadth is 24- 2x.

Therefore, the volume V(x) of the box is given by

⇒ \(V(x)=x(45-2 x)(24-2 x)=x\left(1080-90 x-48 x+4 x^2\right)=4 x^3-138 x^2+1080 x\)

⇒ \(V^{\prime \prime}(x)=12 x^2-276 x+1080-12\left(x^3-23 x+90\right)=12(x-18)(x-5)\)

⇒ \(V^{\prime \prime}(x)=24 x-276=12(2 x-23)\)

for maxima or minima

V’ (x) = 0 ⇒ x = 18 and x- 5

Applications Of Derivatives A Rectangle Sheet Of Tin Is Made Into A Box Without Top

It is not possible to cut off a square of side 18 cm from each corner of the rectangular sheet.

Thus, x cannot be equal to 18.

x \(\neq\) 18

x = 5

Now, V” (5) = 12 (10- 23) = 1 2 (- 1 3) = – 156 < 0

By second order derivative test, x = 5 is the point of maxima.

Question 19. Show that of all the rectangles inscribed in a given fixed circle, the square has the maximum area.

Solution:

Let a rectangle of length l and breadth b be inscribed in the given circle of radius a.

Then, the diagonal passes through the center and is of length 2 cm.

Now. by applying the Pythagoras theorem, we have:

⇒ \((2 a)^2=\ell^2+b^2\)

⇒ \(b^2=4 a^2-c^2 \Rightarrow b=\sqrt{4 a^2-\ell^2}\)

Applications Of Derivatives The Rectangle Inscribed In A Given Fixed Circle

Area of the rectangle, \(\Lambda=\ell \sqrt{4 \mathrm{a}^2-\ell^2} \Rightarrow A^2=\ell^2\left(4 \mathrm{a}^2-\ell^2\right)=\mathrm{B}(\text { let })\)

If A is the maximum B is also maximum

⇒ \(\frac{d B}{d \ell}=8 \mathrm{a}^2 \ell-4 \ell^3 \Rightarrow \frac{d^2 B}{d \ell^2}=8 a^2-12 \ell^2\)

for maxima or minima \(\frac{\mathrm{dB}}{\mathrm{d} \ell}=0 \Rightarrow 8 \mathrm{a}^2 \ell-4 \ell^3=0 \Rightarrow \ell=0\)

or \(\ell=\sqrt{2} \mathrm{a}\)

b=\(\sqrt{4 a^2-2 a^2}=\sqrt{2 a^2}=\sqrt{2 a}\)

Now, when \(\ell=\sqrt{2} \mathrm{a}\)

⇒ \(\frac{d^2 B}{d \ell^2}=8 a^2-12\left(2 a^2\right)=-16 a^2<0\)

By the second order derivative test, when \(\ell=\sqrt{2}\) a, then the area of the rectangle is the maximum.

Since \(\ell=b=\sqrt{2} \mathrm{a}\), the rectangle is a square.

Hence, it has been proved that of all the rectangles inscribed in the given fixed circle, the square has the maximum area.

Question 20. Show that the right circular cylinder of a given surface and maximum volume is such that its height is equal to the diameter of the base.

Solution:

Let r and h be the radius and height of the cylinder respectively.

Then, the surface area (S’) of the cylinder is given by,

⇒ \(\mathrm{S}=2 \pi \mathrm{r}^2+2 \pi \mathrm{h} \Rightarrow \mathrm{h}=\frac{\mathrm{S}-2 \pi \mathrm{r}^2}{2 \pi \mathrm{r}} \Rightarrow \mathrm{h}=\frac{\mathrm{S}}{2 \pi \mathrm{r}}-\mathrm{r}\) →  Equation 1

Let V be the volume of the cylinder, Then,

Applications Of Derivatives The Right Circular Cylinder Of A Given Surface

⇒ \(\mathrm{V}=\pi \mathrm{r}^2 \mathrm{~h}=\pi \mathrm{r}^2\left[\frac{\mathrm{S}}{2 \pi}-\mathrm{r}\right]=\frac{\mathrm{Sr}}{2}-\pi \mathrm{r}^3\)

Then, \(\frac{\mathrm{dV}}{\mathrm{dr}}=\frac{\mathrm{S}}{2}-3 \pi \mathrm{r}^2, \frac{\mathrm{d}^2 \mathrm{~V}}{\mathrm{dt}}=-6 \pi \mathrm{rr}\)

for maxima or minima

⇒ \(\frac{d V}{d r}=0 \Rightarrow \frac{S}{2}=3 \pi r^2 \)

⇒ \(r^2=\frac{S}{6 \pi}\)

When \(r^2=\frac{S}{6 \pi}, then \frac{d^2 V}{d r^2}=-6 \pi\left(\sqrt{\frac{S}{6 \pi}}\right)<0\)

By second order derivative test, the volume is the maximum when \(r^2=\frac{S}{6 \pi}\)

Now, when \(r^2=\frac{S}{6 \pi}, or S=6 \pi r^2\) then h=\(\frac{6 \pi r^2}{2 \pi r}-r=3 r-r=2 r\)

Hence, the volume is maximum when the height is twice the radius ie., when the height is equal to the diameter.

Question 21. Of all the closed cylindrical cans (right circular), of a given volume of 100 cubic centimeters. find the dimensions of the can which has the minimum surface area.

Solution:

Let r and h be the radius and height of the cylinder respectively, then the volume (V) of the cylinder is given by,

V =\(\pi r^2 \mathrm{~h}=100\) (given)

h =\(\frac{100}{\pi r^2}\)

The surface area (S) of the cylinder is given by, S =\(2 \pi \mathrm{r}^2+2 \pi \mathrm{h}=2 \pi \mathrm{r}^2+\frac{200}{\mathrm{r}}\)

⇒ \(\frac{\mathrm{dS}}{\mathrm{dr}}=4 \pi \mathrm{r}-\frac{200}{\mathrm{r}^2}, \frac{\mathrm{d}^2 \mathrm{~S}}{\mathrm{dr}^2}=4 \pi+\frac{400}{\mathrm{r}^3}\)

for maxima or minima

⇒ \(\frac{d \mathrm{~S}}{\mathrm{dr}} =0 \Rightarrow 4 \pi r=\frac{200}{\mathrm{r}^2}\)

⇒ \(\mathrm{r}^3=\frac{200}{4 \pi}=\frac{50}{\pi}\)

⇒ \(r^2=\frac{50}{\pi}\)

r=\(\left(\frac{50}{\pi}\right)^{\frac{1}{3}}\)

Now, it is observed that when \(\mathrm{r}=\left(\frac{50}{\pi}\right)^{\frac{1}{3}} \cdot \frac{\mathrm{d}^2 \mathrm{~S}}{\mathrm{dt}^2}>0\)

By second order derivative test, the surface area is the minimum when the radius of the cylinder is

⇒ \(\left(\frac{50}{\pi}\right)^{\frac{1}{3}} \mathrm{~cm}\)

Now, \(\frac{d^2 S}{d r^2}=\frac{400}{r^{\prime}}+4 \pi=8 \pi+4 \pi=12 \pi>0\)

When r=\(\left(\frac{50}{\pi}\right)^{\frac{1}{2}}, \mathrm{~h}=\frac{100}{\pi\left(\frac{50}{\pi}\right)^{\frac{1}{3}}}=\frac{2 \times 50}{(50)^2\left(\frac{1}{\pi}\right)^{\frac{2}{3}} \pi}=2\left(\frac{50}{\pi}\right)^{\frac{1}{3}} \mathrm{~cm}\).

Hence, the required dimensions of the can which has the minimum surface area is given by radius =\(\left(\frac{50}{\pi}\right)^{\frac{1}{3}} \mathrm{~cm}\)

and height =\(2\left(\frac{50}{\pi}\right)^{\frac{3}{3}} \mathrm{~cm}\).

Question 22. A wire of length 28 m is to be cut into two pieces. One of the pieces is to be made into a square and the other into a circle. What should be the length of the two pieces so that the combined area of the square and the circle is minimal?

Solution:

Let a piece of length l be cut from the given wire to make a square.

Then, the other piece of wire to be made into a circle is of length (28- i) m

Now, side of square =\(\frac{t}{4}\).

Let r be the radius of the circle. Then, 2 \(\pi r=28-\ell \Rightarrow r=\frac{1}{2 \pi}(28-\theta)\)

The combined areas of the square and the circle (A) are given by, A= (side of the square) \(^2+\pi \mathrm{se}^{\prime}\)

=\(\frac{\ell^2}{16}+\pi\left[\frac{1}{2 \pi}(28-\ell)\right]^2=\frac{\ell^2}{16}+\frac{1}{4 \pi}(28-)^2 \)

⇒ \(\frac{\mathrm{dA}}{\mathrm{d} \ell}=\frac{2 \ell}{16}+\frac{2}{4 \pi}(28-\ell)(-1)=\frac{\ell}{8}-\frac{1}{2 \pi}(28-\ell)\)

⇒ \(\frac{\mathrm{dA}}{\mathrm{d} \ell}=\frac{\ell}{8}-\frac{1}{2 \pi}(28-\ell) \Rightarrow \frac{\mathrm{d}^2 \mathrm{~A}}{\mathrm{~d} \ell^2}=\frac{1}{8}+\frac{1}{2 \pi}\)

for maxima or minima, \(\frac{\mathrm{dA}}{\mathrm{d} \ell}=0 \Rightarrow \frac{\ell}{8}-\frac{1}{2 \pi}(28-\ell)\)=0

⇒ \(\frac{\pi i-4(28-\theta)}{8 \pi}=0 \Rightarrow(\pi+4(-112=0 \Rightarrow t=\frac{112}{\pi+4}\).

Thus, when \(\ell=\frac{112}{\pi+4}, \frac{d^2 \mathrm{~A}}{\mathrm{~d} \ell^2}>0\)

By second order derivative test, the area (A) is the minimum when \(\ell=\frac{112}{\pi+4}\).

Hence, the combined area is minimum when the length of the wire in making the square is \(\frac{112}{\pi+4}\) m while the length of the wire in making the circle is \(28-\frac{112}{\pi+4}=\frac{28 \pi}{\pi+4} \mathrm{~m}\)

Question 23. Prove that the volume of the largest cone that can be inscribed in a sphere of radius R is \(\frac{8}{27}\) of the volume of the sphere.

Solution:

Let r and h be the radius and height of the cone respectively inscribed in a sphere of radius R.

Let V be the volume of the cone.

Then, V=\(\frac{1}{3} \pi r^2 h\)

The height of the cone is given by,

Applications Of Derivatives The Volume Of Largest Cone That Can Be Inscribed In A Sphere

⇒ \(\mathrm{h}-\mathrm{R}+\mathrm{AB}=\mathrm{R}+\sqrt{\mathrm{R}^2-\mathrm{r}^2}\)[ABC is a right triangle]

⇒ \(\mathrm{r}^2=2 \mathrm{hR}-\mathrm{h}^2\)

⇒ \(\mathrm{V}=\frac{1}{3} \pi \mathrm{h}\left(2 \mathrm{hR}-\mathrm{h}^2\right) \Rightarrow \mathrm{V}=\frac{1}{3} \pi\left(2 \mathrm{~h}^2 \mathrm{R}-\mathrm{h}^3\right)\)

⇒ \(\frac{\mathrm{dV}}{\mathrm{dh}}=\frac{1}{3} \pi\left(4 \mathrm{hR}-3 \mathrm{~h}^2\right) and \frac{\mathrm{d}^2 \mathrm{~V}}{\mathrm{dh}^2}=\frac{1}{3} \pi(4 \mathrm{R}-6 \mathrm{~h})\)

for maxima or minima

⇒ \(\frac{\mathrm{dV}}{\mathrm{dh}}=0 \Rightarrow 4 \mathrm{hR}-3 \mathrm{~h}^2=0 \Rightarrow \mathrm{h}=0 or \mathrm{h}=\frac{4 \mathrm{R}}{3}\)

rejecting h =0 , h=\(\frac{4 \mathrm{R}}{3}\)

When h=\(\frac{4 \mathrm{R}}{3}, then \frac{\mathrm{d}^2 \mathrm{~V}}{\mathrm{dh}^2}=\frac{1}{3} \pi(4 \mathrm{R}-8 \mathrm{R})=-\frac{4 \pi \mathrm{R}}{3}<0\)

from (1) \(r^2=2 h R-h^2\)

when h=\(\frac{4 \mathrm{R}}{3}, \mathrm{r}^2=\frac{8 \mathrm{R}^2}{9}\)

By second order derivative test, the volume of the cone is the maximum when \(r^2=\frac{8}{9} R^2\).

Therefore, the volume of a cone

⇒ \(\mathrm{V}=\frac{1}{3} \pi\left(\frac{8}{9} \mathrm{R}^2\right)\left(\frac{4}{3} \mathrm{R}\right)\)

=\(\frac{8}{27}\left(\frac{4}{3} \pi \mathrm{R}^3\right)=\frac{8}{27} \times\) (Volume of the sphere)

Hence, the volume of the largest cone that can be inscribed in the sphere is \(\frac{8}{27}\) the volume of the sphere.

Question 24. The point on the curve \(x^2=2\) y which is nearest to the point (0,5) is?

  1. \((2 \sqrt{2}, 4)\)
  2. \((2 \sqrt{2}, 0)\)
  3. (0,0)
  4. (2,2)

Solution: 1. \((2 \sqrt{2}, 4)\)

Given curve is \(x^2=2 y\)

Let the point (x, y) on the curve nearest to the point (0,5).

Now distance (D) between (x, y) and (0,5) is

D=\(\sqrt{x^2+(y-5)^2} \Rightarrow D=\sqrt{2 y+(y-5)^2}\)

⇒ \(D^2=2 y+y^2+25-10 y \Rightarrow D^2=y^2-8 y+25\)

Let \(\mathrm{D}^{\mathrm{t}}=\mathrm{z}\)

Then \(z=y^3-8 y+25\)

⇒ \(x^{\prime}=2 y-8\) and \(z^{\prime \prime}=2\)

for maxima or minima

Now, \(z^{\prime}=0 \Rightarrow y=4\)

At \(y=4, x^n=2>0\)

i.e. y=4 is the point of minima.

At y=4, x= \(\pm 2 \sqrt{2}\)

Hence, the required point is ( \(\pm 2 \sqrt{2}, 4\)).

The correct answer is 1.

Question 25. For all real values of x, the minimum value of \(\frac{1-x+x^2}{1+x+x^3}\) is?

  1. 0
  2. 1
  3. 3
  4. \(\frac{1}{3}\)

Solution: 4.

Let f(x)=\(\frac{1-x+x^2}{1+x+x^2}\)

⇒ \(f^{\prime}(x)=\frac{\left(1+x+x^2\right)(-1+2 x)-\left(1-x+x^2\right)(1+2 x)}{\left(1+x+x^2\right)^2}\)

=\(\frac{-1+2 x-x+2 x^2-x^2+2 x^3-1-2 x+x+2 x^2-x^2-2 x^3}{\left(1+x+x^2\right)^2}\)

=\(\frac{2 x^2-2}{\left(1+x+x^2\right)^2}=\frac{2\left(x^2-1\right)}{\left(1+x+x^2\right)^2}\)

for maxima or minima

f(x)=0 \(\Rightarrow x^2=1 \Rightarrow x= \pm 1\)

Now,f(x)=\(\frac{2\left[\left(1+x+x^2\right)^2(2 x)-\left(x^2-1\right)(2)\left(1+x+x^2\right)(1+2 x)\right]}{\left(1+x+x^2\right)^4}\)

=\(\frac{4\left(1+x+x^2\right)\left[\left(1+x+x^2\right) x-\left(x^2-1\right)(1+2 x)\right]}{\left(1+x+x^2\right)^4}\)

=\(\frac{4\left[x+x^2+x^3-x^3-2 x^3+1+2 x\right]}{\left(1+x+x^2\right)}=\frac{4\left[1+3 x-x^3\right]}{\left(1+x+x^2\right)}\)

And, \(f(1)=\frac{4[1+3-1]}{(1+1+1)^3}=\frac{4(3)}{(3 y}=\frac{4}{9}>0\)

Also, \(f^*(-1)=\frac{4[1-3+1]}{(1-1+1)^2}=4(-1)=-4<0\)

By second order derivative test, f is minimum at x=1 and the minimum value is given by \(f(1)=\frac{1-1+1}{1+1+1}=\frac{1}{3}\),

The correct answer is D.

Question 26. The maximum value of \([x(x-1)+1], 0 \leq x \leq 1\) is ?

  1. \(\left(\frac{1}{3}\right)^{\frac{1}{1}}\)
  2. \(\frac{1}{2}\)
  3. 1
  4. 0

Solution: 3. 1

Let f(x)=\([x(x-1)+1]^{\frac{1}{2}}\)

⇒ \((\mathrm{x})=\frac{2 \mathrm{x}-1}{3[\mathrm{x}(\mathrm{x}-1)+1]^{\frac{2}{3}}}\)

for maxima or minima

f(x)=0 \(\Rightarrow x=\frac{1}{2}\)

Then, we evaluate the value of f at critical point x=\(\frac{1}{2}\) and at the endpoints of the interval [0,1] {i.e., at x=0 and x=1}.

f(0)=\([0(0-1)+1]^{\frac{1}{j}}=1\)

f(1)=\([1(1-1)+1]^{\frac{1}{5}}=1\)

⇒ \(f\left(\frac{1}{2}\right)=\left[\frac{1}{2}\left(-\frac{1}{2}\right)+1\right]^{\frac{1}{3}}=\left(\frac{3}{4}\right)^{\frac{1}{3}}\)

Hence, we can conclude that the maximum value of f in the interval [0,1] is 1.

The correct answer is 3.

Application Of Derivatives Miscellaneous Exercise

Question 1. Show that the function given by f(x)=\(\frac{\log x}{x}\) has maximum at x=e.

Solution:

The given function is f(x)=\(\frac{\log x}{x}\),

⇒ \(f^{\prime}(x)=\frac{x\left(\frac{1}{x}\right)-\log x}{x^2}=\frac{1-\log x}{x^2}\)

for maxima or minima \(f^{\prime}(x)=0 \Rightarrow \frac{1-\log x}{x^2}=0 \Rightarrow \log x=1 \Rightarrow \log x=\log e \Rightarrow x=c\)

Now, \(f”(x)=\frac{x^2\left(-\frac{1}{x}\right)-(1-\log x)(2 x)}{x^4}=\frac{-x-2 x(1-\log x)}{x^4}\)

= \(\frac{-3+2 \log x}{x^3} at x=e, f^{\prime}(e)=\frac{-3+2 \log e}{e^3}=\frac{-3+2}{e^3}=\frac{-1}{e^3}<0\)

Therefore, by second order derivative test, f(x) is maximum at x=e.

Question 2. The two equal sides of an isosceles triangle with fixed base b are decreasing at the rate of 3 cm per second. How fast is the area decreasing when the two equal sides are equal to the base ?

Solution:

Let \(\triangle\) A B C be isosceles where BC is the base of fixed length b.

Let the length of the two equal sides of \triangle \(\mathrm{ABC}\) be a.

Draw AD \(\perp\) BC

Applications Of Derivatives Two Equal Sides Of An Isosceles Triangle With Fixed Base

Now, in \(\triangle\) A D C, by applying the Pythagoras theorem,

we have: A D=\(\sqrt{a^2-\frac{b^2}{4}}\)

Area of triangle A B C is A=\(\frac{1}{2} B C \cdot A D=\frac{b}{2} \sqrt{a^2-\frac{b^3}{4}}\)

The rate of change of the area with respect to time (t) is given by,

⇒ \(\frac{d A}{d t}=\frac{1}{2} b \cdot \frac{2 a}{2 \sqrt{a^2-\frac{b^2}{4}}} \frac{d a}{d t}=\frac{a b}{\sqrt{4 a^2-b^2}} \frac{d a}{d t}\)

It is given that the two equal sides of the triangle are decreasing at the rate of 3 cm per second.

⇒ \(\frac{\mathrm{da}}{\mathrm{dt}}=-3 \mathrm{~cm} / \mathrm{s}\)

⇒ \(\frac{\mathrm{dA}}{\mathrm{dt}}=\frac{-3 \mathrm{ab}}{\sqrt{4 \mathrm{a}^2-\mathrm{b}^2}}\)

Then, when a=b, we have: \(\frac{d A}{d t}=\frac{-3 b^2}{\sqrt{4 b^2-b^2}}=\frac{-3 b^2}{\sqrt{3 b^2}}=-\sqrt{3} \mathrm{~cm}^2 / \mathrm{s}\)

Hence, if the two equal sides are equal to the base, then the area of the triangle is decreasing at the rate of \(\sqrt{3} \mathrm{~b} \mathrm{~cm}^2 / \mathrm{s}\).

Question 3. Find the intervals in which the function f is given by \(f(\mathrm{x})=\frac{4 \sin \mathrm{x}-2 \mathrm{x}-\mathrm{x} \cos \mathrm{x}}{2+\cos \mathrm{x}}\) is

  1. increasing
  2. decreasing

Solution:

f(x) =\(\frac{4 \sin x-2 x-x \cos x}{2+\cos x}\)

⇒ \(f^{\prime}(x)=\frac{(2+\cos x)(4 \cos x-2-\cos x+x \sin x)-(4 \sin x-2 x-x \cos x)(-\sin x)}{(2+\cos x)^2}\)

=\(\frac{(2+\cos x)(3 \cos x-2+x \sin x)+\sin x(4 \sin x-2 x-x \cos x)}{(2+\cos x)^2}\)

=\(\frac{6 \cos x-4+2 x \sin x+3 \cos ^2 x-2 \cos x+x \sin x \cos x+4 \sin ^2 x-2 x \sin x-x \sin x \cos x}{(2+\cos x)^2}\)

=\(\frac{4 \cos x-4+3 \cos ^2 x+4 \sin ^2 x}{(2+\cos x)^2}=\frac{4 \cos x-4+3 \cos ^2 x+4-4 \cos ^2 x}{\left(2+\cos ^2 x\right)^2-}\)

=\(\frac{4 \cos x-\cos ^2 x}{(2+\cos x)^2}=\frac{\cos x(4-\cos x)}{(2+\cos x)^2}\)

Now, x=\(\frac{\pi}{2}\) and x=\(\frac{3 \pi}{2}\) divides [0,2 \(\pi\)] into three intervals i.e. \(\left[0, \frac{\pi}{2}\right],\left[\frac{\pi}{2}, \frac{3 \pi}{2}\right]\) and \(\left[\frac{3 \pi}{2}, 2 \pi\right]\).

In intervals \(\left[0, \frac{\pi}{2}\right]\) and \(\left[\frac{3 \pi}{2}, 2 \pi\right], \mathrm{f}^{\prime}(\mathrm{x})>0\)

Thus, f(x) is increasing for \(\left[0, \frac{\pi}{2}\right]\) and \(\left[\frac{3 \pi}{2}, 2 \pi\right]\)

In the interval \(\left[\frac{\pi}{2}, \frac{3 \pi}{2}\right], \mathrm{f}^{\prime}(\mathrm{x}) \leq 0\).

Thus, \(\mathrm{f}(\mathrm{x})\) is decreasing for \(\left[\frac{\pi}{2}, \frac{3 \pi}{2}\right]\).

Question 4. Find the intervals in which the function f given by \(f(\mathrm{x})=\mathrm{x}^3+\frac{1}{\mathrm{x}^3}, \mathrm{x} \neq\) 0 is

  1. increasing
  2. decreasing

Solution:

⇒ \(f(\mathrm{x})=\mathrm{x}^3+\frac{1}{\mathrm{x}^3}\)

⇒ \(f^{\prime}(\mathrm{x})=3 \mathrm{x}^2-\frac{3}{\mathrm{x}^4}=\frac{3 \mathrm{x}^6-3}{\mathrm{x}^4}\)

Then, \(f^{\prime}(x)=0 \Rightarrow 3 x^6-3=0 \Rightarrow x^6=1 \Rightarrow x= \pm 1\)

Now, the points x=1 and x=-1 divide the real line into three intervals i.e., \((-\infty,-1],[-1,1]\) and \([1, \infty)\).

In intervals \((-\infty,-1] and [1, \infty) f^{\prime}(x) \geq 0\)

⇒ f(x) is increasing in \((-\infty,-1] \cup[1, \infty)\)

In interval [-1,1] \(\mathrm{f}^{\prime}(\mathrm{x})<0\).

f(x) is decreasing in [-1,1]

Question 5. Find the maximum area of an isosceles triangle inscribed in the ellipse \(\frac{x^2}{a^2}+\frac{y^2}{b^2}\)=1 with its vertex at one end of the major axis.

Solution:

The given ellipse is \(\frac{x^2}{a^2}+\frac{y^2}{b^2}\)=1, Its paranctric form is x=a \(\cos \theta, y=b \sin \theta\)

Let the major axis be along the x-axis.

Applications Of Derivatives The Maximum Arear Of An Isosceles Triangle

Let ABC be the triangle inscribed in the ellipse where vertex C is at (a, 0)

Since the ellipse is symmetrical with respect to the x-axis and y-axis, we can assume the coordinates of A to be \((-\mathrm{a} \cos \theta, \mathrm{b} \sin \theta)\) and the coordinates of B to be \((-\mathrm{a} \cos \theta,-\mathrm{b} \sin \theta )\)

⇒ \(\mathrm{AB}=2 \mathrm{~b} \sin \theta\) and \(\mathrm{DC}=(\mathrm{a}+\mathrm{a} \cos \theta)\)

the area of triangle ABC is given by,

A=\(\frac{1}{2} A B \cdot D C \Rightarrow A=\frac{1}{2}(2 b \sin \theta) \cdot(a+a \cos \theta)\)

A=a b \(\sin \theta(1+\cos \theta)\)

⇒ \(\mathrm{AB}=2 \mathrm{bsin} \theta\) and \(\mathrm{DC}=(\mathrm{a}+\mathrm{a} \cos \theta)\)

the area of triangle A B C is given by,

A=\(\frac{1}{2} A B \cdot D C \Rightarrow A=\frac{1}{2}(2 b \sin \theta) \cdot(a+a \cos \theta)\)

A=a b \(\sin \theta(1+\cos \theta)\)

⇒ \(\frac{d A}{d \theta}=a b[\sin \theta(-\sin \theta)+(1+\cos \theta) \cos \theta] \Rightarrow \frac{d A}{d \theta}\)

=a b\(\left[\cos \theta+\cos ^2 \theta-\sin ^2 \theta\right]\)

and \(\frac{d^2 A}{d \theta^2}=a b[-\sin \theta-2 \sin \theta \cos \theta-2 \sin \theta \cos \theta] \Rightarrow \frac{d^2 A}{d \theta^2}=-a b[\sin \theta+4 \sin \theta \cos \theta]\)

for maxima or minima \(\frac{\mathrm{dA}}{\mathrm{d} \theta}\)=0

a b\(\left[\cos \theta+\cos ^2 \theta-\sin ^2 \theta\right]=0 \Rightarrow \cos \theta+\cos ^2 \theta-1+\cos ^2 \theta=0 \)

⇒ \(2 \cos ^2 \theta+\cos \theta-1=0 \Rightarrow \cos \theta=-1 or \cos \theta=\frac{1}{2}\)

⇒ ⇒ \(\theta=\pi or \theta=\frac{\pi}{3}\)

rejecting \(\theta=\pi\)

\(\theta=\frac{\pi}{3}\)

Now at \(\theta=\frac{\pi}{3}, \frac{d^2 A}{d \theta^2}=-a b\left[\sin \frac{\pi}{3}+4 \sin \frac{\pi}{3} \cos \frac{\pi}{3}\right]\)

= \(-a b\left[\frac{\sqrt{3}}{2}+4 \cdot \frac{\sqrt{3}}{2} \cdot \frac{1}{2}\right]<0\)

By the second derivative test, the Area (A) of an isosceles triangle is maximum when \(\theta=\frac{\pi}{3}\)

The maximum area of the triangle is given by,

⇒ \(\left.A=a b \sin \theta(1+\cos \theta)=a b \sin \frac{\pi}{3}\left(1+\cos \frac{\pi}{3}\right)-a b \cdot \frac{\sqrt{3}}{2} / 1+\frac{1}{2}\right)\)

=\(\frac{3 \sqrt{3}}{4} a b \cdot s q \cdot\) units

Question 6. A tank with a rectangular base and rectangular sides, open at the top is to be constructed so that its deÿtdis lava volume is 8 ark If the buildup of the tank costs Rs 70 per square meter for the base and Rs 45 per square meter for the sides, What is the cost of least expensive tank?

Solution:

Let l, b, and h represent the length, breadth, and height of the tank respectively.

Then, we have height (h) = 2 m

Volume of the tank. = 8m³ ; Volume of the tank =L x b x h

Now, the area of the base = lb = 4

Area of the 4 walls (A) = 2h(l + b)

A =4\(\left(\ell+\frac{4}{\ell}\right) \Rightarrow \frac{\mathrm{dA}}{\mathrm{d} \ell}=4\left(1-\frac{4}{\ell^2}\right)\)

for maxima or minima

⇒ \(\frac{\mathrm{dA}}{\mathrm{d} \ell}=0 \Rightarrow 1-\frac{4}{\ell^2}=0 \Rightarrow \ell^2=4 \Rightarrow \ell= \pm 2\)

However, the length cannot be negative. Therefore, we have \(\ell\)=2

b=\(\frac{4}{\ell}=\frac{4}{2}\)=2

Now, \(\frac{\mathrm{d}^2 \mathrm{~A}}{\mathrm{~d} \ell^2}=\frac{32}{\ell^3}\)

When \(\ell=2, \frac{\mathrm{d}^2 \mathrm{~A}}{\mathrm{~d} \ell^2}=\frac{32}{8}=4>0\).

Thus, by second order derivative test, the area is the minimum when \(\ell\)=2.

We have t=\(\mathrm{b}=\mathrm{h}\)=2.

Cost of building the base = Rs 70 \(\times(\ell b)\)= Rs 70(4)= Rs 280

Cost of building the walls = Rs 2 h\((\ell+\mathrm{b}) \times 45\)= Rs 90(2)(2+2)= Rs 720

Required total cost =Rs(280+720)= Rs 1000

Hence, the total cost of the tank will be Rs 1000.

Question 7. The sum of the perimeter of a circle and square is k, where k is some constant. Prove that the sum of their areas is least when the side of the square is double the radius of the circle.

Solution:

Let ‘r’ be the radius of the circle and ‘a’ be the side of the square.

Then, we have :

2 \(\pi r+4 a=k\) (where k is constant)

a=\(\frac{\mathrm{k}-2 \pi \mathrm{r}}{4}\)

The sum of the areas of the circle and the square (A) is given by,

A =\(\pi \mathrm{r}^2+\mathrm{a}^2=\pi\)

⇒ \(\mathrm{r}^2+\frac{(\mathrm{k}-2 \pi \mathrm{r})^2}{16}\)

⇒ \(\frac{\mathrm{dA}}{\mathrm{dr}}=2 \pi \mathrm{r}+\frac{2(\mathrm{k}-2 \pi \mathrm{r})(-2 \pi)}{16}=2 \pi \mathrm{r}-\frac{\pi(\mathrm{k}-2 \pi \mathrm{r})}{4}\)

for maxima or minima

⇒ \(\frac{\mathrm{dA}}{\mathrm{dr}}\) =0

2 \(\pi \mathrm{r} =\frac{\pi(\mathrm{k}-2 \pi \mathrm{r})}{4}\)

8 \(\mathrm{r}=\mathrm{k}-2 \pi \mathrm{r} \Rightarrow(8+2 \pi) \mathrm{r}=\mathrm{k} \Rightarrow \mathrm{r}=\frac{\mathrm{k}}{8+2 \pi}=\frac{\mathrm{k}}{2(4+\pi)}\)

Now, \(\frac{\mathrm{d}^2 \mathrm{~A}}{\mathrm{dr}^2}=2 \pi+\frac{\pi^2}{2}\)

When r=\(\frac{k}{2(4+\pi)}, \frac{d^2 A}{d r^2}>0\).

The sum of the areas is least when r =\(\frac{\mathrm{k}}{2(4+\pi)}\).

When r=\(\frac{k}{2(4+\pi)}, a=\frac{k-2 \pi\left[\frac{k}{2(4+\pi)}\right]}{4}=\frac{k-\frac{\pi k}{(4+\pi)}}{4}=\frac{4 k+\pi k-\pi k}{4(4+\pi)}=\frac{k}{4+\pi}=2 r\)

Hence, it has been proved that the sum of their areas is least when the side of the square is Double the radius of the circle.

Question 8. A window is in the form of a rectangle surmounted by a semicircular opening. The total perimeter of the window is 10 m. Find the dimensions of the window to admit maximum light through the whole opening.

Solution:

Let 2x and y be the length and breadth of the rectangular window respectively.

The radius of the semicircular opening = x

It is given that the perimeter of the window is 10 m.

⇒ \(2 x+2 y+\pi x=10 \Rightarrow y=5-x-\frac{\pi x}{2}\)

Now, the Area of the window is given by.

Applications Of Derivatives A Window Is In The Form Of Rectangle Surmounted By A Semicircular Opening

A =2 \(\mathrm{xy}+\frac{\pi \mathrm{x}^2}{2} \Rightarrow \mathrm{A}=2 \mathrm{x}\left(5-\mathrm{x}-\frac{\pi \mathrm{x}}{2}\right)+\frac{\pi \mathrm{x}^2}{2}\)

⇒ \(\mathrm{A}=10 \mathrm{x}-2 \mathrm{x}^2-\pi \mathrm{x}^2+\frac{\pi \mathrm{x}^2}{2} \Rightarrow \mathrm{A}=10 \mathrm{x}-2 \mathrm{x}^2-\frac{\pi \mathrm{x}^2}{2}\)

⇒ \(\frac{\mathrm{dA}}{\mathrm{dx}}=10-4 \mathrm{x}-\pi \mathrm{x} \frac{\mathrm{d}^2 \mathrm{~A}} {\mathrm{dx}^2}=-4-\pi\)

for maxima or minima

⇒ \(\frac{\mathrm{dA}}{\mathrm{dx}}=0 \Rightarrow 10-4 \mathrm{x}-\pi \mathrm{x}=0 \Rightarrow \mathrm{x}=\frac{10}{\pi+4}\)

Thus, when x=\(\frac{10}{\pi+4}, then \frac{d^2 A}{d x^2}<0\).

Therefore, by second-order derivative test, the area is the maximum when length 2 x=\(\frac{20}{\pi+4} m \)

Now,y=5-\(\frac{10}{\pi+4}-\frac{\pi}{2}\left(\frac{10}{\pi+4}\right) \Rightarrow y=\frac{10}{\pi+4}\)

Hence, the required dimensions of the window to admit maximum light is given by length =\(\frac{20}{\pi+4}\) m and breadth =\(\frac{10}{\pi+4}\) m.

Question 9. A point on the hypotenuse of a triangle is at distance ‘ a ‘ and ‘ b ‘ from the sides of the triangle.
Show that the minimum length of the hypotenuse is \(\left(a^{\frac{2}{3}}+b^{\frac{2}{3}}\right)^{\frac{2}{2}}\)

Solution:

Let \(\triangle\) A B C be right-angled at B. Let A B=x and B C=y.

Let P be a point on the hypotenuse of the triangle such that P is at a distance of a and b from the sides AB and BC respectively.

Let \(\angle \mathrm{C}=\theta\).

We have, A C=\(\sqrt{x^2+y^2}\)

Now, P C=b \({cosec} \theta\) and, \(A P=a \sec \theta\)

AC=A P+P C

Applications Of Derivatives A Point On The Hypotenuse Of A Triangle Is At A Distance

AC=b {cosec} \(\theta+a \sec \theta\)

⇒ \(\frac{d(A-C)}{d \theta}=-b {cosec} \theta \cot \theta+a \sec \theta \tan \theta\)

⇒ \(\frac{d(A-C)}{d \theta}=-b {cosec} \theta \cot \theta+a \sec \theta \tan \theta\)

and \(\frac{d^2(A-C)}{d \theta^2}=b\left({cosec} \theta \cot ^2 \theta+{cosec}^3 \theta\right)+a\left(\sec ^3 \theta+\sec \theta \tan ^2 \theta\right)\)

for maxima or minima

⇒ \(\frac{\mathrm{d}(\mathrm{AC})}{\mathrm{d} \theta}=0 \Rightarrow \mathrm{asec} \theta \tan \theta=b {cosec} \theta \cot \theta\)

⇒ \(\frac{\mathrm{a}}{\cos \theta} \cdot \frac{\sin \theta}{\cos \theta}=\frac{\mathrm{b}}{\sin \theta} \frac{\cos \theta}{\sin \theta}\)

⇒ \({asin}^3 \theta=b \cos ^3 \theta \Rightarrow(\mathrm{a})^{\frac{1}{3}} \sin \theta=(b)^{\frac{1}{3}} \cos \theta \Rightarrow \tan \theta=\left(\frac{b}{a}\right)^{\frac{1}{3}}\)

⇒ \(\sin \theta =\frac{(b)^{\frac{1}{3}}}{\sqrt{a^{\frac{2}{3}}+b^{\frac{2}{3}}}} \text { and } \cos \theta=\frac{(\mathrm{a})^{\frac{1}{3}}}{\sqrt{a^{\frac{2}{3}}+b^{\frac{2}{3}}}}\)

Since, \(0<\theta<\frac{\pi}{2}\), so trigonometric ratios are positive.

Also, a>0 and b>0 \(\frac{d^2(A C)}{d \theta^2}\) is positive.

It can be clearly shown that \(\frac{\mathrm{d}^2(\mathrm{AC})}{\mathrm{d} \theta^2}>0\) when\(\tan \theta=\left(\frac{\mathrm{b}}{\mathrm{a}}\right)^{\frac{1}{3}}\)

Therefore, by second order derivative test, the length of the hypotenuse is minimum when

⇒ \(\tan \theta=\left(\frac{b}{a}\right)^{\frac{1}{3}}\) .

Now, when \(\tan \theta=\left(\frac{b}{a}\right)^{\frac{1}{3}}\),

we have : A C=\(\frac{b \sqrt{a^{\frac{2}{3}}+b^{\frac{2}{3}}}}{b^{\frac{1}{3}}}+\frac{a \sqrt{a^{\frac{2}{3}}+b^{\frac{2}{3}}}}{a^{\frac{1}{3}}}\)

=\(\sqrt{a^{\frac{2}{3}}+b^{\frac{2}{3}}\left(b^{\frac{2}{3}}+a^{\frac{2}{3}}\right)=\left(a^{\frac{2}{3}}+b^{\frac{2}{3}}\right)^{\frac{1}{2}}} [Using (1) and (2)]\)

Hence, the minimum length of the hypotenuse is \(\left(a^{\frac{2}{3}}+b^{\frac{2}{3}}\right)^{\frac{3}{2}}.\)

Question 10. Find the points at which the function f given by \(f(x)=(x-2)^4(x+1)^3\) has

  1. local maxima
  2. local minima
  3. point of inflection

Solution:

The given function is f(x)=\((x-2)^4(x+1)^3\).

⇒ \(f^{\prime}(x) =4(x-2)^3(x+1)^3+3(x+1)^2(x-2)^4\)

=\((x-2)^5(x+1)^2[4(x+1)+3(x-2)]=(x-2)^3(x+1)^2(7 x-2)\)

For maxima or minima

⇒ \(\mathrm{f}(\mathrm{x})=0 \Rightarrow(\mathrm{x}-2)^3(\mathrm{x}+1)^2(7 \mathrm{x}-2)\)=0

⇒ \(f^{\prime}(x)=0 \Rightarrow x=-1 and x=\frac{2}{7} or x=2\)

Now, for values of x close to \(\frac{2}{7}\) and to the left of \(\frac{2}{7}, f^{\prime}(x)>0\).

Also, for values of x close to \(\frac{2}{7}\) and to the right of \(\frac{2}{7}, \mathrm{f}^{\prime}(\mathrm{x})<0\).

Thus, x=\(\frac{2}{7}\) is the point of local maxima.

Now, for values of x close to 2 and to the left of 2, \(f^{\prime}(x)<0\). Also, for values of x close to 2 and to the right of 2, \(f^{\prime}(\mathrm{x})>0\)

Thus, x=2 is the point of local minima.

Now, as the value of x varies through -1, \(f^{\prime}(x)\) does not change its sign.

Thus, x=-1 is the point of inflection.

Question 11. Find the absolute maximum and minimum values of the function f given by

Solution:

f(x)=\(\cos ^2 x+\sin x, x \in[0, \pi]\)

f(x)=\(\cos ^2 x+\sin x\)

⇒ \(f^{\prime}(x)=2 \cos x(-\sin x)+\cos x=-2 \sin x \cos x+\cos x\)

Now, \(\mathrm{f}^{\prime}(\mathrm{x})\)=0

2 \(\sin x \cos x=\cos x\)

⇒ \(\cos x(2 \sin x-1)=0 \Rightarrow \sin x=\frac{1}{2}\)

or \(\cos x=0 \Rightarrow x=\frac{\pi}{6}\) or \(\frac{\pi}{2}\) as x \(\in[0, \pi]\)

Now, evaluating the value of f at critical points x=\(\frac{\pi}{2}\) and x=\(\frac{\pi}{6}\) and the endpoints of the interval [0, \(\pi\)] (i.e., at x=0 and \(x=\pi\) ), we have:

⇒ \(f\left(\frac{\pi}{6}\right)=\cos ^2 \frac{\pi}{6}+\sin \frac{\pi}{6}=\left(\frac{\sqrt{3}}{2}\right)^2+\frac{1}{2}=\frac{5}{4}\)

⇒ \(\mathrm{f}(0)=\cos ^2 0+\sin 0=1+0=1\)

⇒ \(\mathrm{f}(\pi)=\cos ^2 \pi+\sin \pi-(-1)^3+0=1\)

⇒ \(\mathrm{f}\left(\frac{\pi}{2}\right)=\cos ^2 \frac{\pi}{2}+\sin \frac{\pi}{2}=0+1=1\)

Hence, the absolute maximum value of f is \(\frac{5}{4} \)occurring at x =\(\frac{\pi}{6}\) and the absolute minimum value of f is 1 occurring at x=0, \(\frac{\pi}{2}\)/ and \(\pi\).

Question 12. Show that the altitude of the right circular cone of maximum volume that can be inscribed in a sphere or radius r is \(\frac{4 r}{3}\).

Solution:

A sphere of fixed radius (r) is given.

Let R and h be the radius and the height of the cone respectively,

Applications Of Derivatives The Altitude Of The Right Circular Cone Of Maximum Volume

The volume \((\mathrm{V})\) of the cone is given by,V =\(\frac{1}{3} \pi \mathrm{R}^2 \mathrm{~h}\)

In \(\triangle\) B C D, B C=\(\sqrt{r^2-R^2}\)

⇒ \(\mathrm{h}=\mathrm{r}+\sqrt{\mathrm{r}^2-\mathrm{R}^2} \Rightarrow \mathrm{R}^2=2 h r-\mathrm{h}^2\)

⇒ \(\mathrm{V}=\frac{1}{3} \pi h\left(2 h r-h^2\right)=\frac{1}{3} \pi\left(2 \mathrm{~h}^2 \mathrm{r}-\mathrm{h}^3\right)\)

⇒ \(\frac{\mathrm{dV}}{\mathrm{dh}}=\frac{1}{3} \pi\left(4 \mathrm{hr}-3 \mathrm{~h}^2\right) and \frac{\mathrm{d}^2 \mathrm{~V}}{\mathrm{dh}^2}=\frac{1}{3} \pi(4 \mathrm{r}-6 \mathrm{~h})\)

for maxima or minima

⇒ \(\frac{\mathrm{dV}}{\mathrm{dh}}=0 \Rightarrow 4 \mathrm{hr}-3 \mathrm{~h}^2=0 \Rightarrow \mathrm{h}=0 or \mathrm{h}=\frac{4 \mathrm{r}}{3}\)

rejecting h 0 \(\mathrm{h}=\frac{4 \mathrm{r}}{3}\)

when h =\(\frac{4 \mathrm{r}}{3}, \frac{\mathrm{d}^2 \mathrm{~V}}{\mathrm{dh}^2}=\frac{1}{3} \pi(4 \mathrm{r}-8 \mathrm{r})=-\frac{4 \mathrm{r} \pi}{3}<0\)

V is maximum when \(h=\frac{4 \mathrm{r}}{3}\)

By second order derivative test, the volume of the cone is maximum when the altitude of the cone is \(\frac{4 \mathrm{r}}{3}\)

Question 13. Let f be a function defined on [a, b] such that \(f^{\prime}(x)>0\), for all x \(\in\)(a, b), then prove that f is an increasing function on (a, b).

Solution:

Since, \(f^{\prime}(x)>0\) on (a, b)

Then, f is a differentiable function on (a, b)

Also, every differentiable function is continuous,

Therefore, f is continuous on [a, b]

Let \(x_1, x_2 \in(a, b)\) such that, \(x_2>x_1\) then by LMV theorem, there exists c \(\in(a, b)\) s.t.

⇒ \(f^{\prime}(c)=\frac{f\left(x_2\right)-f\left(x_1\right)}{x_2-x_1}\)

⇒ \(f\left(x_2\right)-f\left(x_1\right)=\left(x_2-x_1\right) f^{\prime}(c)\)

⇒ \(f\left(x_2\right)-f\left(x_1\right)>0 \text { as } x_2>x_1 \text { and } f^{\prime}(x)>0\)

⇒ \(f\left(x_1\right)>f\left(x_1\right)\)

for \(\mathrm{x}_1<\mathrm{x}_2 \Rightarrow \mathrm{f}\left(\mathrm{x}_1\right)<\mathrm{f}\left(\mathrm{x}_2\right)\).

Therefore, f is an increasing function.

Question 14. A cylindrical tank of radius 10 m is being filled with wheat at the rate of 314 cubic meters per hour. Then the depth of the wheat is increasing at the rate of?

  1. 1 \(\mathrm{~m} / \mathrm{h}\)
  2. 0.1 \(\mathrm{~m} / \mathrm{h}\)
  3. 1.1 \(\mathrm{~m} / \mathrm{h}\)
  4. 0.5 \(\mathrm{~m} / \mathrm{h}\)

Solution: 1. 1\(\mathrm{~m} / \mathrm{h}\)

Let r be the radius of the cylinder

Then, volume \((\mathrm{V})\) of the cylinder is given by =\(\pi r^t \mathrm{~h}\),

V =\(\pi(\text { radius })^2 \times \text { height }\)

⇒ \(\left.=\pi(10)^2 \mathrm{~h} \quad \quad \text { (radius }=10 \mathrm{~m}\right)\)

=100 \(\pi \mathrm{h}\)

Differentiating with respect to time t, we have :

⇒ \(\frac{\mathrm{dV}}{\mathrm{dt}}=100 \pi \frac{\mathrm{dh}}{\mathrm{dt}}\)

The tank is being filled with wheat at the rate of 314 cubic meters per hour.

⇒ \(\frac{\mathrm{dV}}{\mathrm{dt}}=314 \mathrm{~m}^3 / \mathrm{h}\)

Thus, we have :

314=100 \(\pi \frac{\mathrm{dh}}{\mathrm{dt}} \Rightarrow \frac{\mathrm{dh}}{\mathrm{dt}}=\frac{314}{100(3.14)}=\frac{314}{314}=1\)

Hence, the depth of wheat is increasing at the rate of 1 \(\mathrm{~m} / \mathrm{h}\).

The correct answer is 1.

 

Continuity and Differentiability Class 12 Maths Important Questions Chapter 5

Continuity And Differentiability Exercise 5.1

Question 1. Prove that the function f(x) = 5x-3 is continuous at x = 0, at x = -3 and x = 5.

Solution:

The given function is f(x) = 5x-3

At x= 0, f(0) =5×0-3=-3

⇒ \(\lim _{x \rightarrow 0} f(x)=\lim _{x \rightarrow 0}(5 x-3)=5 \times\) 0-3=-3

⇒ \(\lim _{x \rightarrow 0}\) f(x)=f(0)

Therefore, f is continuous at x =0

At x=-3, f(-3)=5 \(\times(-3)\)-3=-18

⇒ \(\lim _{x \rightarrow-3} f(x)=\lim _{x \rightarrow-3}(5 x-3)=5 \times(-3)-3=-18 \Rightarrow \lim _{x \rightarrow-3} f(x)^{-}=f(-3)\)

Therefore, f is continuous at x=-3

At x=5, f(x)=f(5)=5 \(\times\) 5-3=25-3=22

⇒ \(\lim _{x \rightarrow 5} f(x)=\lim _{x \rightarrow 5}(5 x-3)=5 \times 5-3=22 \Rightarrow \lim _{x \rightarrow 5} f(x)=f(5)\)

Therefore, is continuous at x = 5

Question 2. Examine the continuity of the function \(\mathrm{f}(\mathrm{x})=2 \mathrm{x}^2-1\) at x=3.

Solution:

The given function is \(\mathrm{f}(\mathrm{x})=2 \mathrm{x}^2-1\)

At x=3, f(x)=f(3)=2 \(\times 3^2-1=17\)

⇒ \(\lim _{x \rightarrow 3} f(x)=\lim _{x \rightarrow 3}\left(2 x^2-1\right)=2 \times 3^2-1=17 \lim _{x \rightarrow 3} f(x)=f(3)\)

Thus, f is continuous at x =3

Question 3. Examine the following functions for continuity.

f(x)=x-5
f(x)=\(\frac{1}{x-5}, x \neq 5\)
f(x)=\(\frac{x^2-25}{x+5}, x \neq-5\)
f(x)=|x-5|

Solution :

1. The given function is f(x) = x- 5

It is evident that f is defined at every real number k and its value at k is k – 5

It is also observed that, \(\lim _{x \rightarrow k} f(x)=\lim _{x \rightarrow k}(x-5)=k-5=f(k)\)

⇒ \(\lim _{x \rightarrow k} f(x)=f(k)\)

Hence, f is continuous at every real number and therefore, it is a continuous function

Read and Learn More Class 12 Maths Chapter Wise with Solutions

2. The given function is f(x)=\(\frac{1}{x-5}, x \neq 5\)

For any real number k \(\neq 5\), we obtain \(\lim _{x \rightarrow k} f(x)=\lim _{x \rightarrow k}\left(\frac{1}{x-5}\right)=\frac{1}{k-5}\)

Also, f(k)=\(\frac{1}{k-5}( At k \neq 5) \Rightarrow \lim _{x \rightarrow k} f(x)=f(k)\)

Hence,f is continuous at every point in the domain of f and therefore, it is a continuous function.

3. The given function is f(x)=\(\frac{x^2-25}{x+5}\), x \(\neq-5\)

For any real number c \(\neq-5\), we obtain

⇒ \(\lim _{x \rightarrow c} f(x)=\lim _{x \rightarrow c} \frac{x^2-25}{x+5}=\lim _{x \rightarrow c} \frac{(x+5)(x-5)}{x+5}=\lim _{x \rightarrow c}(x-5)=(c-5)\)

Also, f(c)=\(\frac{(c+5)(c-5)}{c+5}=(c-5)(as c \neq-5)\)

\(\lim _{x \rightarrow c} f(x)=f(c)\)

Hence, f is continuous at every point in the domain of f and therefore, it is a continuous function.

4. The given function is f(x)=|x-5|=\(\begin{cases}{l}5-x, \text { if } x<5 \\ x-5, \text { if } x \geq 5\end{cases}\).

This function f is defined at all points of the real line.

Let c be a point on a real line. Then, c < 5 or c =5 or c > 5

Case 1: c < 5

Then, f (c)=5 – c

⇒ \(\lim _{x \rightarrow c} f(x)=\lim _{x \rightarrow c}(5-x)=5-c \Rightarrow \lim _{x \rightarrow c} f(x)=f(c)\)

Therefore, f is continuous at all real numbers less than 5.

Case 2: c=5

Then, f(c)=f(5)=(5-5)=0

⇒ \(\lim _{x \rightarrow 5} f(x)=\lim _{x \rightarrow 5}(5-x)=(5-5)=0 \Rightarrow \lim _{x \rightarrow 5^{+}} f(x)=\lim _{x \rightarrow 5}\)(x-5)=0

⇒ \(\lim _{x \rightarrow c^{-}} f(x)=\lim _{x \rightarrow c^{+}} f(x)=f(c)\)

Therefore, f is continuous at x = 5

Case 3: c > 5

Then, f(c)=f(5)=c-5

⇒ \(\lim _{x \rightarrow c} f(x)=\lim _{x \rightarrow c}(x-5)=c-5 \Rightarrow \lim _{x \rightarrow c} f(x)=f(c)\)

Therefore, f is continuous at all real numbers greater than 5.

Hence, f is continuous at every real number and therefore, it is a continuous function.

CBSE Class 12 Maths Chapter 5 Continuity And Differentiability Important Question And Answers

Question 4. Prove that the function f {x} =\(\mathrm{x}^{\mathrm{n}}\) is continuous at x= n, where n is a positive integer.

Solution:

The given function is \(\mathrm{f}(\mathrm{x})=\mathrm{x}^{\mathrm{n}}\)

It is evident that f is defined at all positive integers, n, and its value at n is n.

Then, \(\lim _{x \rightarrow \pi} f(x)=\lim _{x \rightarrow n}\left(x^n\right)=n^n \Rightarrow \lim _{x \rightarrow \pi}\)

f(x)=f(n)

Therefore, f is continuous at n, where n is a positive integer.

Question 5. Is the function f defined by f(x)=\(\begin{cases}{l}x,\text { if } x \leq 1 \\ 5, \text { if } x>1\end{cases}\).continuous at x=0 ? At x=1? At x=2?

Solution:

The given function f(x)=\(\begin{cases}{l}x, \text { if } x \leq 1 \\ 5, \text { if } x>1\end{cases}\).

⇒ \(\mathrm{f}(\mathrm{x})\) may be discontinuous at doubtful point x=1 and except this point \(\mathrm{f}(\mathrm{x})\) be continuous.

Now, at x =1,

The left hand {limit} of f at x=1 is, \(\lim _{x \rightarrow 1^{-}} f(x)=\lim _{x \rightarrow 1^{-}}\) x=1

The right hand limit of f at x=1 is, \(\lim _{x \rightarrow 1^{+}} f(x)=\lim _{x \rightarrow 1^{+}}(5)\)=5

⇒ \(\lim _{x \rightarrow 1^{-}} f(x) \neq \lim _{x \rightarrow 1^{+}} f(x)\)

Hence, f is not continuous at x = 1

Question 6. Find all points of discontinuity of f, where f is defined by f(x)=\(\begin{cases}{l}2 x+3, \text { if } x \leq 2 \\ 2 x-3, \text { if } x>2\end{cases}\).

Solution:

The given function f(x)=\(\begin{cases}{l}2 x+3, \text { if } x \leq 2 \\ 2 x-3, \text { if } x>2\end{cases}\). may be discontinuous at doubtful point x=2 and except this point \(\mathrm{f}(\mathrm{x}) \)be continuous.

At x=2

The left hand limit of f at x=2 is, \(\lim _{x \rightarrow 2^{-}} f(x)=\lim _{x \rightarrow 2^{-}}(2 x+3)=2 \times\) 2+3=7

The right hand limit of f at x=2 is \(\lim _{x \rightarrow 2^{+}} f(x)=\lim _{x \rightarrow 2^{+}}(2 x-3)=2 \times\) 2-3=1

⇒ \(\lim _{x \rightarrow 2^{-}} f(x) \neq \lim _{x \rightarrow 2^{+}} f(x)\)

Therefore, f is not continuous at x=2. Hence, x=2 is the only point of discontinuity of f.

Question 7. Find all points of discontinuity of f, where f is defined by f(x)=\(\begin{cases}{l}|x|+3, \text { if } x \leq-3 \\ -2 x, \text { if }-3<x<3 \\ 6 x+2, \text { if } x \geq 3\end{cases}\).

Solution:

The given function f is f(x)=\(\begin{cases}{l}|x|+3=-x+3, \text { if } x \leq-3 \\ -2 x, \text { if }-3<x<3 \\ 6 x+2, \text { if } x \geq 3\end{cases}\).

Given fn. f(x) may be discontinuous at doubtful points x=-3 and x=3 and except these two points f(x) be continuous.

at x=-3, f(-3)=-(-3)+3=6

L.H.L =\(\lim _{x \rightarrow-3^{-}} f(x)=\lim _{x \rightarrow-3^{-}}(-x+3)=-(-3)+3=6\)

R.H.L =\(\lim _{x \rightarrow-3^{+}} f(x)=\lim _{x \rightarrow-3^{+}}(-2 x)=-2 \times(-3)\)=6

⇒ \(\lim _{x \rightarrow-3} f(x)=f(-3)\)

Therefore, f is continuous at x = -3

Now at x=3, the left hand limit of f at x=3 is, \(\lim _{x \rightarrow 3^{-}} f(x)=\lim _{x \rightarrow 3^{-}}(-2 x)=-2 \times 3\)=-6

The right hand limit of f at x=3 is, \(\lim _{x \rightarrow 3^{+}} f(x)=\lim _{x \rightarrow 3^{+}}(6 x+2)=(6 \times 3)+2=20\)

It is observed that the left and right-hand limits of f at x=3 do not coincide.

Therefore, f is not continuous at x=3. Hence, x=3 is the only point of discontinuity of f.

Question 8. Find all points of discontinuity of f, where f is defined by f(x)= \(\begin{cases}|x| & \text { if } x \neq 0 \\ 0, & \text { if } x=0\end{cases}\)

Solution:

The given function f is f(x)=\(\begin{cases}{l}|x| \text { if } x \neq 0 \\ 0, \text { if }x=0\end{cases}\).

It is known that, x<0 \(\Rightarrow|x|\)=-x and x>0 \(\Rightarrow|x|\)=x

Therefore, the given function can be rewritten as f(x)=\(\left\{\begin{array}{l}
\frac{|x|}{x}=\frac{-x}{x}=-1 \text { if } x<0 \\
0, \text { if } x=0 \\
\frac{|x|}{x}=\frac{x}{x}=1, \text { if } x>0
\end{array}\right.\).

Given f. f(x) may be discontinuous at doubtful point x=0 and except these point f(x) be continuous.

Now at x=0, then the left hand limit of f at x=0 is, \(\lim _{x \rightarrow 0^{-}} f(x)=\lim _{x \rightarrow 0^{-}}(-1)\)=-1

The right hand limit of f at x=0 is, \(\lim _{x \rightarrow 0^{+}} f(x)=\lim _{x \rightarrow 0^{+}}(1)\)=1

It is observed that the left and right-hand limit of f at x=0 do not coincide.

Therefore, f is not continuous at x=0

Hence, x=0 is the only point of discontinuity of f.

Question 9. Find all points of discontinuity of f, where f is defined by f(x)= \(\begin{cases}\frac{x}{|x|} & \text { if } x<0 \\ -1, & \text { if } x \geq 0\end{cases}\)

Solution:

The given function f is f(x)= \(\begin{cases}\frac{x}{|x|}, & \text { if } x<0 \\ -1, & \text { if } x \geq 0\end{cases}\) It is known that, x<0 \(\Rightarrow|x|\)=-x

⇒ \(\mathrm{f}(\mathrm{x})\)=-1 for all x \(\in \mathrm{R}\)

Let c be any real number. Then, \(\lim _{x \rightarrow c} f(x)=\lim _{x \rightarrow c}(-1)\)=-1

Also, f(c)=\(-1=\lim _{x \rightarrow c} f(x)\)

Therefore, the given function is a continuous function.

Hence, the given function has no point of discontinuity.

Question 10. Find all points of discontinuity of f, where f is defined by f(x)=\(\begin{cases}{l}x+1, \text { if } x \geq 1 \\ x^2+1, \text { if } x<1\end{cases}\).

Solution:

The given function f is f(x)=\(\begin{cases}{l}x+1, \text { if } x \geq 1 \\ x^2+1, \text { if } x<1\end{cases}\).

Function \(\mathrm{f}(\mathrm{x})\) may be discontinuous at doubtful point x=1 and except these point f(x) be continuous.

Now at x=1, f(1)=1+1=2

The left hand limit of f at x=1 is, \(\lim _{x \rightarrow 1^{-}} f(x)=\lim _{x \rightarrow 1^{-}}\left(x^2+1\right)=1^2+1=2\)

The right hand limit of f at x=1 is, \(\lim _{x \rightarrow 1^{+}} f(x)=\lim _{x \rightarrow 1^{+}}(x+1)\)=1+1=2

⇒ \(\lim _{x \rightarrow 1} \mathrm{f}(\mathrm{x})=\mathrm{f}(1)\)

Therefore, f is continuous at x=1. Hence, the given function f has no point of discontinuity.

Question 11. Find all points of discontinuity of f, where f is defined by f(x)=\(\begin{cases}{l}x^3-3, \text { if } x \leq 2 \\ x^2+1, \text { if } x>2\end{cases}\).

Solution:

The given function f is f(x)=\(\left\{\begin{array}{l}x^3-3, \text { if } x \leq 2 \\ x^2+1, \text { if } x>2\end{array}\right.\)

Function \(\mathrm{f}(\mathrm{x})\) may be discontinuous at doubtful point x = 2 and except these point \(\mathrm{f}(\mathrm{x})\) be continuous.

Now at x=2, f(2)=\(2^3-3=5\)

L.H.L =\(\lim _{x \rightarrow 2^{-}} f(x)=\lim _{x \rightarrow 2^{-}}\left(x^3-3\right)=2^3-3=5\)

R.H.L =\(\lim _{x \rightarrow 2^{+}} f(x)=\lim _{x \rightarrow 2^2}\left(x^2+1\right)=2^2+1=5\)

L.H.L = R.H.L =5

⇒ \(\lim _{x \rightarrow 2} f(x)=f(2)\)

Therefore, f is continuous at x = 2. Thus, the given function f is continuous at every point on the real line, Hence, f has no point of discontinuity.

Question 12. Find all points of discontinuity of f, where f is defined by f(x)= \(\begin{cases}x^{10}-1, & \text { if } x \leq 1 \\ x^2, & \text { if } x>1\end{cases}\)

Solution:

The given function is f(x)= \(\begin{cases}x^{10}-1, & \text { if } x \leq 1 \\ x^2, & \text { if } x>1\end{cases}\)

The function \(\mathrm{f}(\mathrm{x})\) may be discontinuous at doubtful point x=1 and except this point f(x) be continuous.

Now, at x=1, then the left hand limit of f at x=1 is, \(\lim _{x \rightarrow 1^{-}} f(x)=\lim _{x \rightarrow 1^{-}}\left(x^{10}-1\right)=1^{10}-1=1-1=0\)

The right hand limit of f at x=1 is, \(\lim _{x \rightarrow 1^{+}} f(x)=\lim _{x \rightarrow 1^{+}}\left(x^2\right)=1^2=1\)

It is observed that the left and right-hand limit of f at x=1 do not coincide.

Therefore,f is not continuous at x=1

Thus, from the above observation, it can be concluded that x=1 is the only point of discontinuity of f.

Question 13. Is the function defined by f(x)=\(\begin{cases}{l}x+5, \text { if } x \leq 1 \\ x-5, \text { if } x>1\end{cases}\).. a continuous function?

Solution:

The given function is f(x)=\(\begin{cases}{l}x+5, \text { if } x \leq 1 \\ x-5, \text { if } x>1\end{cases}\).

Function \(\mathrm{f}(\mathrm{x})\) may be discontinuous at doubtful point x=1 and except these point \(\mathrm{f}(\mathrm{x})\) be continuous.

Now at x=1, f(1)=1+5=6

The left hand limit of f at x=1 is, \(\lim _{x \rightarrow 1^{-}} f(x)=\lim _{x \rightarrow 1^{-}}(x+5)=1+5=6\)

The right hand limit of f at x=1 is, \(\lim _{x \rightarrow 1^{+}} f(x)=\lim _{x \rightarrow 1^{+}}(x-5)\)=1-5=-4

It is observed that the left and right-hand limits of f at x 1 do not coincide.

Therefore, f is not continuous at x- 1

Thus, from the above observation, it can be concluded that x- 1 is the only point of discontinuity of f.

Question 14. Discuss the continuity of the function f, where f is defined by f(x)= \(\begin{cases}3, & \text { if } 0 \leq x \leq 1 \\ 4, & \text { if } 1<x<3 \\ 5, & \text { if } 3 \leq x \leq 10\end{cases}\)

Solution:

The given function is \(f(\mathrm{x})=\{\begin{array}{l}3, \text { if } 0 \leq \mathrm{x} \leq 1 \\ 4, \text { if } 1<x<3 \\ 5, \text { if } 3 \leq \mathrm{x} \leq 10\end{array}\).

Function \(\mathrm{f}(\mathrm{x})\) may be discontinuous at doubtful points x=1 and x=3 and except these two points \(\mathrm{f}(\mathrm{x})\) be continuous.

Now at x=1, {f}(1)=3

The left hand limit of f at x=1 is, \(\lim _{x \rightarrow 1^{-}} f(x)=\lim _{x \rightarrow 1^{-}}(3)\)=3

The right hand limit of f at x=1 is, \(\lim _{x \rightarrow 1^{+}} f(x)=\lim _{x \rightarrow 1^{+}}(4)=4\)

It is observed that the left and right-hand limits of f at x=1 do not coincide.

Therefore, f is not continuous at x=1 and at x=3, f(3)=5

The left hand limit of f at x=3 is, \(\lim _{x \rightarrow 3^{-}} f(x)=\lim _{x \rightarrow 3^{-}}(4)=4\)

The right hand limit of f at x=3 is, \(\lim _{x \rightarrow 3^{+}} f(x)=\lim _{x \rightarrow 3^{+}}(5)=5\)

It is observed that the left and right-hand limits of f at x=3 do not coincide.

Therefore. f is not continuous at x=3. Hence, f is not continuous at x=1 and x =3

Question 15. Discuss the continuity of the function f, where f is defined by f(x)= \(\begin{cases}2 x, & \text { if } x<0 \\ 0, & \text { if } 0 \leq x \leq 1 \\ 4 x, & \text { if } x>1\end{cases}\)

Solution:

The given function is f(x)=\(\{\begin{array}{l}2 x, \text { if } x<0 \\ 0, \text { if } 0 \leq x \leq 1 \\ 4 x, \text { if } x>1\end{array}\).

Function \(\mathrm{f}(\mathrm{x})\) may be discontinuous at doubtful points x=0 and x = 1 and except these two points \mathrm{f}(\mathrm{x}) be continuous.

Now, at x=0, f(0)=0

The left hand limit of f at x=0 is, \(\lim _{x \rightarrow 0^{-}} f(x)=\lim _{x \rightarrow 0^{-}}(2 x)=2 \times 0=0\)

The right hand limit of f at x=0 is, \(\lim _{x \rightarrow 0^{+}} f(x)=\lim _{x \rightarrow 0^{+}}(0)=0\)

⇒ \(\lim _{x \rightarrow 0^{-}} f(x)=\lim _{x \rightarrow 0^{+}} f(x) \Rightarrow \text { limit exist }\)

⇒ \(\lim _{x \rightarrow 0} f(x)=f(0)\)

Therefore, f is continuous at x=0 and at x=1, f(1)=0

The left hand limit of f at x=1 is, \(\lim _{x \rightarrow 1^{-}} f(x)=\lim _{x \rightarrow 1^{-}}(0)=0\)

The right hand limit of f at x=1 is, \(\lim _{x \rightarrow 1^{+}} f(x)=\lim _{x \rightarrow 1^{+}}(4 x)=4 \times\) 1=4

It is observed that the left and right-hand limits of fat x = 1 do not coincide.

Therefore, f is not continuous at x = 1. Hence, f is not continuous only at x. = 1

Question 16. Discuss the continuity of the function f, where f is defined by f(x)= \(\begin{cases}-2, & \text { if } x \leq-1 \\ 2 x, & \text { if }-1<x \leq 1 \\ 2, & \text { if } x>1\end{cases}\)

Solution:

The given function f is f(x)= \(\begin{cases}-2, & \text { if } x \leq-1 \\ 2 x, & \text { if }-1<x \leq 1 \\ 2, & \text { if } x>1\end{cases}\)

Function \(\mathrm{f}(\mathrm{x})\) may be discontinuous at doubtful points x=-1 and x=1 and except these two points \(\mathrm{f}(\mathrm{x})\) be continuous.

Now, at x=-1, f(-1)=-2

The right hand limit of f at x=-1 is, \(\lim _{x \rightarrow-1^{+}} f(x)=\lim _{x \rightarrow-1^{+}}(2 x)=2 x(-1)=-2\)

⇒ \(\lim _{x \rightarrow-1^{-}} f(x)=\lim _{x \rightarrow-1^{+}} f(x) \Rightarrow \text { limit exist } \)

⇒ \(\lim _{x \rightarrow-1^{-}} f(x)=f(-1)\)

Therefore, f is continuous at x =-1 and at x=1, \(\mathrm{f}(1)\)=2 \times 1=2

The left hand limit of f at x=1 is, \(\lim _{x \rightarrow 1^{-}} f(x)=\lim _{x \rightarrow 1^{-}}(2 x)=2 \times 1\)=2

The right hand limit of f at x=1 is, \(\lim _{x \rightarrow 1^{+}} f(x)=\lim _{x \rightarrow 1^{+}}\) 2=2

⇒ \(\lim _{x \rightarrow 1^{-}} f(x)=\lim _{x \rightarrow 1^{+}} f(x) \Rightarrow \text { limit exist }\)

⇒ \(\lim _{x \rightarrow 1} f(x)=f(1)\)

Thus, from the above observations, it can be concluded that is continuous at all points of the real line.

Question 17. Find the relationship between a and b so that the function f defined by f(x)= \(\begin{cases} an x+1, & \text { if } x \leq 3 \\ b x+3, & \text { if } x>3\end{cases}\) is continuous at x=3.

Solution: 

The given function f is f(x)=\(\{\begin{array}{ll}a x+1, & \text { if } x \leq 3 \\ b x+3, & \text { if } x>3\end{array}\).

If f is continuous at x=3, then

⇒ \(\lim _{x \rightarrow 3^{+}} f(x)=\lim _{x \rightarrow 3^{+}} f(x)=f(3)\)

Also, \(\lim _{x \rightarrow 3^{-}} f(x)=\lim _{x \rightarrow 3^{-}}(a x+1)=3 a+1\)

⇒ \(\lim _{x \rightarrow 3^{+}} f(x)=\lim _{x \rightarrow 3^{+}}(b x+3)=3 b+3 \Rightarrow f(3)\)=3 a+1

Therefore, from (1), we obtain.

3 a+1=3 b+3=3 a+1 \(\Rightarrow 3 a+1=3 b+3 \Rightarrow 3 a=3 b+2 \Rightarrow a-b=\frac{2}{3}\)

Therefore, the required relationship is given by a-b=\(\frac{2}{3}\)

Question 18. For what value of \(\lambda\) is the function defined by f(x)=\(\begin{cases} {cc}\lambda(x^2-2 x), & \text { if } x \leq 0 \\ 4 x+1, & \text { if } x>0\end{cases} \). continuous at x=0? What about continuity at x=1?

Solution:

The given function f is f(x)=\(\begin{cases}{cc}\lambda(x^2-2 x), & \text { if } x \leq 0 \\ 4 x+1, & \text { if } x>0\end{cases}\).

If f is continuous at x=0, then \(\lim _{x \rightarrow 0^{-}} f(x)=\lim _{x \rightarrow 0^{+}} f(x)=f(0)\)

\(\lim _{x \rightarrow 0^{-}} \lambda\left(x^2-2 x\right)=\lim _{x \rightarrow 0^{+}}(4 x+1)=\lambda\left(0^2-2 \times 0\right)\)

\(\lambda\left(0^2-2 \times 0\right)=4 \times 0+1=0 \Rightarrow 0=1=0\), which is not possible

Therefore, there is no value of \(\lambda\) for which f is continuous at x=0 At x=1, f(1)=4 x+1=4 \(\times \)1+1=5

⇒ \(\lim _{x \rightarrow 1}(4 x+1)=4 \times 1+1=5 \Rightarrow \lim _{x \rightarrow 1} f(x)=f(1)\)

Therefore, for any value of \lambda, f is continuous at x=1

Question 19. Show that the function defined by g(x)=x-[x] is discontinuous at all integral points. Here [x] denotes the greatest integer less than or equal to x.

Solution:

The given function is \(\mathrm{g}(\mathrm{x})=\mathrm{x}-[\mathrm{x}]\). It is evident that g is defined at all integral points.

Let n be an integer. Then, g(n)=n-[n]=n-n=0

The left hand limit of g at x=n is, \(\lim _{x \rightarrow n^{-}} g(x)=\lim _{x \rightarrow n^{-}}(x-[x])=\lim _{x \rightarrow n^{-}}(x)-\lim _{x \rightarrow n^{-}}[x]=n-(n-1)=1\)

The right hand limit of g at x=n is, \(\lim _{x \rightarrow n^{+}} g(x)=\lim _{x \rightarrow n^{+}}(x-[x])=\lim _{x \rightarrow n^{+}}(x)-\lim _{x \rightarrow n^{+}}[x]\)=n-n=0

It is observed that the left and right-hand limits of g at x=n do not coincide.

Therefore, g is not continuous at x=n

Hence,g is discontinuous at all integral points.

Question 20. Is the function defined by \(\mathrm{f}(\mathrm{x})=\mathrm{x}^2-\sin \mathrm{x}+5\) continuous at x=\(\pi\) ?

Solution:

The given function is f(x)=\(x^2-\sin x+5\)

At x=\(\pi, f(\pi)=\pi^2-\sin \pi+5=\pi^2-0+5=\pi^2+5\) and \(\lim _{x \rightarrow \pi} f(x)=\lim _{x \rightarrow \pi}\left(x^2-\sin x+5\right)\)

⇒ \(\lim _{x \rightarrow \pi} f(x)=\pi^2-\sin \pi+5 \Rightarrow \lim _{x \rightarrow \pi} f(x)=\pi^2+5\)

⇒ \(\lim _{x \rightarrow \pi} f(x)=f(\pi)=\pi^2+5\)

Question 21. Discuss the continuity of the following functions.

  1. f(x) = sin x + cos x
    f(x) = sin x- cos x
    f(x) = sin x x cos x

Solution:

By the property of continuity, we know that addition, subtraction, and multiplication of two continuous functions are always continuous.

Here, h(x) = sin x and g(x) = cos x are two continuous functions in R.

Therefore, sin x +cosx, sinx-cosx, and sin x.cosx are also continuous in R.

Question 22. Discuss the continuity of the cosine, cosecant, secant, and cotangent functions.

Solution:

It is known that if g and h are two continuous functions, then

⇒ \(\frac{\mathrm{h}(\mathrm{x})}{\mathrm{g}(\mathrm{x})}, \mathrm{g}(\mathrm{x}) \neq 0\) is continuous

⇒ \(\frac{1}{\mathrm{~g}(\mathrm{x})}, \mathrm{g}(\mathrm{x}) \neq 0\) is continuous

⇒ \(\frac{1}{h(x)}, h(x) \neq 0\) is continuous

It has to be proved first that g(x)=sin x and h(x)=cos x are continuous functions.

Let g(x)=sin x

It is evident that gx=sin x is defined for every real number.

Let c be a real number. Put x = c + h

If x \(\rightarrow\) c, then h \(\rightarrow\) 0

gc = sin c

⇒ \(\lim _{x \rightarrow c} g(x)=\lim _{x \rightarrow c} \sin x=\lim _{h \rightarrow 0} \sin (c+h)\)

= \(\lim _{h \rightarrow 0}[\sin c \cos h+\cos c \sin h]=\lim _{h \rightarrow 0}(\sin c \cos h)+\lim _{h \rightarrow 0}(\cos c \sin h) \)

= \(\sin \mathrm{c} \cos 0+\cos \mathrm{c} \sin 0=\sin \mathrm{c}+0=\sin \mathrm{c}\)

⇒ \(\lim _{x \rightarrow c} g(x)=g(c)\)

Therefore, g is a continuous function. Let \(\mathrm{h}(\mathrm{x})= cos x \)It is evident that h}(x)=cos x is defined for every real number.

Let c be a real number. Put x=c+h

If x \(\rightarrow\) c, then h \(\rightarrow\) 0, h{c}=cos c

⇒ \(\lim _{x \rightarrow c} h(x)=\lim _{x \rightarrow c} \cos x=\lim _{h \rightarrow 0} \cos (c+h)\)

=\(\lim _{h \rightarrow 0}[\cos c \cos h-\sin c \sin h]=\lim _{h \rightarrow 0} \cos c \cos h-\lim _{h \rightarrow 0} \sin c \sin h \)

=\(\cos c \cos 0-\sin c \sin 0=\cos c \times 1-\sin c \times 0=\cos c\)

⇒ \(\lim _{x \rightarrow c} h(x)=h(c)\)

Therefore, h(x)=cos x is a continuous function.

It can be concluded that,\({cosec} x=\frac{1}{\sin x}, \sin x \neq 0\) is continuous

⇒ \({cosec} x, x \neq n \pi(n \in Z)\) is continuous

Therefore, cosecant is continuous except at x=n p, n \(\in\) Z

sec x=\(\frac{1}{\cos x}, \cos x \neq 0\) is continuous

⇒ \(\sec x, x \neq(2 n+1) \frac{\pi}{2}(n \in Z)\) is continuous

Therefore, secant is continuous except at x=\((2 n+1) \frac{\pi}{2}(n \in Z)\)

⇒ \(\cot x=\frac{\cos x}{\sin x}, \sin x \neq 0\) is continuous

⇒ \(\cot x, x \neq n \pi(n \in Z)\) is continuous

Therefore, cotangent is continuous except at x=n \(\pi, n \in Z\)

Question 23. Find the points of discontinuity of f, where f(x)= \(\begin{cases}\frac{\sin x}{x}, & \text { if } x<0 \\ x+1, & \text { if } x \geq 0\end{cases}\)

Solution:

The given function f(x)= \(\begin{cases}\frac{\sin x}{x}, & \text { if } x<0 \\ x+1, & \text { if } x \geq 0\end{cases}\)

f(x) may be discontinuous at doubtful point x=0 and except this point $f(x)$ be continuous.

Now, at x=0, f(0)=0+1=1

The left hand limit of f at x=0 is, \(\lim _{x \rightarrow 0^{\circ}} f(x)=\lim _{x \rightarrow 0} \frac{\sin x}{x}=1\)

The right hand limit of f at x=0 is, \(\lim _{x \rightarrow 0} f(x)=\lim _{x \rightarrow 0^{+}}(x+1)=1\)

⇒ \(\lim _{x \rightarrow 0^{-}} f(x)=\lim _{x \rightarrow 0^{+}} f(x)=f(0)\)

Therefore, f is continuous at x =0

Thus, f has no point of discontinuity,

Question 24. Determine if f defined by f(x)=\(\begin{cases}{cl}x^2 \sin \frac{1}{x}, & \text { if } x \neq 0 \\ 0, & \text { if } x=0\end{cases}\). is a continuous function?

Solution:

The given function f is f(x)=\(\begin{cases}{cc}x^2 \sin \frac{1}{x}, & \text { if } x \neq 0 \\ 0 . & \text { if } x=0\end{cases}\).

f(x) may be discontinuous at doubtful point x=0 and except this point f(x) be continuous,

Now, at x =0, f(0)=0

⇒ \(\lim _{x \rightarrow 0} f(x)=\lim _{s \rightarrow 0} x^2 \sin \frac{1}{x}=0 \times(\text { Finite value from } 0 \text { to 1) }\)

⇒ \(\lim _{x \rightarrow 0} f(x)\)=0

Here, \(\lim _{x \rightarrow \infty} f(x)\)=f(0)

Therefore, f is continuous at x=0

Question 25. Examine the continuity of f, where f is defined by \(f(x)=\begin{cases}{cl}\sin x-\cos x & \text {, if } x \neq 0 \\ -1 & \text {, if } x=0\end{cases}\).

Solution:

The given function f(x)=\(\begin{cases}{cl}\sin x-\cos x & \text {, if } x \neq 0 \\ -1 & \text {, if } x=0\end{cases}\).

f(x) may be discontinuous at doubtful point x=0 and except this point f(x) be continuous. Now, at x=0, f(0)=-1

L.H.L =\(\lim _{x \rightarrow 0^{\circ}} f(x)=\lim _{x \rightarrow 0}(\sin x-\cos x)=\sin \theta-\cos 0=0-1=-1\)

R.H.L =\(\lim _{x \rightarrow x^2} f(x)=\lim _{x \rightarrow x^0}(\sin x-\cos x)=\sin 0-\cos 0=0-1=-1\)

⇒ \(\lim _{x \rightarrow 0^{-}} f(x)=\lim _{x \rightarrow+0^{+}} f(x)=f(0)\)

Therefore, f is continuous at x=0

Thus, f is a continuous function.

Question 26. Find the values of k so that the function f is continuous at the indicated point.

Solution:

f(x)=\(\begin{cases}{cc}\frac{k \cos x}{\pi-2 x} & \text {, if } x \neq \frac{\pi}{2} \\ 3 & \text {,if } x=\frac{\pi}{2}\end{cases}\). at x=\(\frac{\pi}{2}\)

The given function f(x)=\(\{\begin{cases}{cl}\frac{k \cos x}{\pi-2 x} & \text {, if } x \neq \frac{\pi}{2} \\ 3 & \text {, if } x=\frac{\pi}{2}\end{cases}\). is continuous at x=\(\frac{\pi}{2}\),

⇒ \(\lim _{x \rightarrow} f(x)=f\left(\frac{\pi}{2}\right)\)=3

Now, \(\lim _{\rightarrow \rightarrow-1} f(x)=\lim _{x \rightarrow \frac{\pi}{2}} \frac{k \cos x}{\pi-2 x}\)

Pet x=\(\frac{\pi}{2}+h, then, x \rightarrow \frac{\pi}{2} \Rightarrow h \rightarrow 0\)

⇒ \(\lim _{x \rightarrow+} f(x)=\lim _{x \rightarrow \frac{\pi}{2}} \frac{k \cos x}{\pi-2 x}=\lim _{x \rightarrow 4} \frac{k \cos \left(\frac{\pi}{2}+h\right)}{\pi-2\left(\frac{\pi}{2}+h\right)}\)

=k \(\lim _{b \rightarrow 0} \frac{-\sin h}{-2 h}=\frac{k}{2} \lim _{x \rightarrow \infty} \frac{\sin h}{h}=\frac{k}{2}, 1=\frac{k}{2}\)

⇒ \(\lim _{\rightarrow-\frac{1}{2}} f(x)=f\left(\frac{\pi}{2}\right) (From eq. (1))\)

⇒ \(\frac{k}{2}\)=3

k=6

Question 27. Find the values of k so that the function f is continuous at the indicated point:

Solution:

f(x)=\(\begin{cases}{l}
k x^2 & \text {, if } x \leq 2 \\
3 & \text {,if } x>2
\end{cases} \text { at } x=2t\).

The given function is f(x)=\(\begin{cases}{cl}k x^3 & \text { if } x \leq 2 \\ 3 & \text {, if } x>2\end{cases}\).

Is continuously at x=2,

⇒ \(\lim _{x \rightarrow x^2} f(x)=\lim _{x \rightarrow 0} f(x)=f(2) \Rightarrow \lim _{x \rightarrow 7}\left(k x^2\right)=\lim _{x \rightarrow 1}(3)=4 k\)

k \(\times 2^2=3=4 k \Rightarrow 4 k=3=4 k \Rightarrow 4 k=3 \Rightarrow k=\frac{3}{4}\)

Therefore, the required value of k is \(\frac{3}{4}\)

Question 28. Find the values of k so that the function f is continuous at the indicated point:

Solution:

f(x)=\(\begin{cases}{ll}
k x+1, & \text { if } x \leq \pi \\
\cos x, & \text { if } x>\pi
\end{cases} \text { at } x=\pi\).

The given function f(x)= \(\begin{cases}k x+1, & \text { if } x \leq \pi \\ \cos x, & \text { if } x>\pi\end{cases}\)

Is continuous at \(\mathrm{x}=\pi\)

⇒ \(\lim _{x \rightarrow 1^{-}} f(x)=\lim _{x \rightarrow m^{+}} f(x)=f(\pi) \Rightarrow \lim _{x \rightarrow z^{+}}(k x+1)=\lim _{x \rightarrow \pm^{-}} \cos x=k \pi+1 \)

k \(\pi+1=\cos \pi=k \pi+1 \Rightarrow k \pi+1=-1=k \pi+1 \Rightarrow\)

k=-\(\frac{2}{\pi}\)

Therefore, the required value of k is \(-\frac{2}{\pi}\)

Question 29. Find the values of k so that the function f is continuous at the indicated point:

Solution:

f(x)=\(\{\begin{array}{ll}
k x+1, & \text { if } x \leq 5 \\
3 x-5, & \text { if } x>5
\end{array}\) at x = 5

The given function f(x)= \(\begin{cases}k x+1, & \text { if } x \leq 5 \\ 3 x-5, & \text { if } x>5\end{cases}\)

Is continuous at x=5,

⇒ \(\lim _{x \rightarrow 5^5} f(x)=\lim _{x \rightarrow 5^5} f(x)=f(5) \Rightarrow \lim _{x \rightarrow 5^{(}}(k x+1)=\lim _{x \rightarrow 5^{(}}(3 x-5)=5 k+1\)

5 k+1=15-5=5 k+1 \(\Rightarrow 5 k+1=10 \Rightarrow 5 k=9 \Rightarrow k=\frac{9}{5}\)

Therefore, the required value of k is \(\frac{9}{5}\)

Question 30. Find the values of a and b such that the function defined by f(x)=\(\begin{cases}{cl}5, & \text { if } x \leq 2 \\ an x+b, & \text { if } 2<x<10 \\ 21, & \text { if } x \geq 10\end{cases}\) is a continuous function.

Solution:

The given function f(x)=\(\begin{cases}{cl}5, & \text { if } x \leq 2 \\ a x+b, & \text { if } 2<x<10 \text { is continuous function. } \\ 21, & \text { if } x \geq 10\end{cases}.\)

Since f is continuous at x=2, we obtain \(\lim _{x \rightarrow 2} f(x)=\lim _{x \rightarrow z^2} f(x)=f(2)\)

⇒ \(\lim _{x \rightarrow 5}(5)=\lim _{x \rightarrow 2}(a x+b)=5 \Rightarrow 5=2 a+b=5 \Rightarrow 2 a+b=5\)  → Equation 1

Since f is continuous at x=10, we obtain

⇒ \(\lim _{x \rightarrow 10^{-}} f(x)=\lim _{x \rightarrow 10^{+}} f(x)=f(10) \Rightarrow \lim _{x \rightarrow 10^{-}}(a x+b)=\lim _{x \rightarrow 10^0}(21)=21\) →  Equation 2

10 a+b=21=21 \(\Rightarrow\) 10 a+b=21

By subtracting equation (1) from equation (2), we obtain

8 \(\mathrm{a}=16 \Rightarrow \mathrm{a}=2\)

By putting a-2 in equation ( 1 ), we obtain

2 \(\approx 2+b=5 \Rightarrow 4+b=5 \Rightarrow\) b=1

Therefore, the values of a and b for which f is a continuous function are 2 and 1 respectively.

Question 31. Show that the function defined by f(x)=p\(\cos \left(x^2\right)\) is a continuous function.

Solution:

Given f(x)=\(\cos x^2\)

Let g(x)=\(\mathrm{x}^2 and h(x)=cos x\)

⇒ \(\mathrm{f}(\mathrm{x})=\cos (\mathrm{g}(\mathrm{x}))=\mathrm{h}(\mathrm{g}(\mathrm{x})) \Rightarrow f(\mathrm{x})={hog}(\mathrm{x})\)

g(x) is a polynomial function, which is continuous in its domain R so,g(x) is continuous.

Again h(x)=cos x is a trigonometric function that is continuous in {R}.

So, it is also continuous.

Since g(x) and h(x) is continuous.

i.e. h o g(x) is also continuous.

Hence, f(x) is a continuous function.

Question 32. Show that the function defined by f(x)=\(\mid \cos x\) is a continuous function.

Solution:

Given f(x)=|cos x|

Let g(x)= cos x and h(x)=|x|

⇒ \(\mathrm{f}(\mathrm{x})=|\mathrm{g}(\mathrm{x})|=\mathrm{h}(\mathrm{g}(\mathrm{x})) \Rightarrow \mathrm{f}(\mathrm{x})={hog}(\mathrm{x})\)

g(x) is a trigonometric function, which is continuous in its domain R.

So, g(x) is continuous

h}(x)=|x| is a modulus function, which is continuous in R.

So, it is also continuous.

Since g(x) and h(x) is continuous.

i.e. hog (x) is also continuous.

Hence, \(\mathrm{f}(\mathrm{x})=|\cos \mathrm{x}|\) is continuous function.

Question 33. Examine that sin x \(\mid\) is a continuous function.

Solution:

Let f(x)=sin |x|

This function f is defined for every real number and f can be written as the composition of two functions as,

Given f(x)=sin |x|

Let g(x)-|x| and h(x)=sin x

f(x)=sin (g(x))=h(g(x)) \(\Rightarrow\) f(x)=h o g(x)

g(x) is the module’s function, which is continuous in its domain R.

So, g(x) is continuous.

Again h (x) = sin x is a trigonometric function, which is continuous in R.

So, it is also continuous.

Since, g (x) and h (x) is continuous,

i.e. hog (x) is also continuous.

Hence, f (x)- sin jx] is a continuous function.

Question 34. Find all the points of discontinuity of f defined by f(x)=|x|-|x+1|

Solution:

Given that f(x)=|x|-|x+1|

f(x)=\(\begin{cases}{cc}
1 & \text { if } x<-1 \\
-2 x-1 & \text { if }-1 \leq x<0 \\
-1 & \text { if } x \geq 0
\end{cases}\)

Here, f(x) may be discontinuous at doubtful points x-1, x=0 and except these two points f(x) be continuous,

Now at x=-1

L.H.L =\(\lim _{x \rightarrow-1} f(x)=\lim _{x \rightarrow-1}(1)\)=1

R.H.L. =\(\lim _{x \rightarrow-1} f(x)=\lim _{x \rightarrow-1}(-2 x-1)\)=1

L.H.S. = R.H.S. \(\Rightarrow\) limit exist and {f}(-1)=1

Here \(\lim f(x)=f(-1)\)

So, f(x) is continuous at x=-1

Again at x=0

L.H.L. =\(\lim _{x \rightarrow 0^{+}} f(x)=\lim _{x \rightarrow 0^{+}}(-2 x-1)=-1 \)

R.H.L.=\(\lim _{x \rightarrow 0^{+}} f(x)=\lim _{x \rightarrow 0^{+}}(-1)=-1 \)

L.H.L = R.H.L \(\Rightarrow \text { limit exist and }\) f(0)=-1

Here \(\lim _{x \rightarrow 0} f(x)\)=f(0)

So, f(x) is continuous at x=0

Hence, there is no point of discontinuity for f(x).

Continuity And Differentiability Exercise 5.2

Question 1. \(\sin \left(x^2+5\right)\)

Solution:

⇒ \(\frac{d}{d x}\left[\sin \left(x^2+5\right)\right] =\cos \left(x^2+5\right) \cdot \frac{d}{d x}\left(x^2+5\right)\)

= \(\cos \left(x^4+5\right) \cdot\left[\frac{d}{d x}\left(x^2\right)+\frac{d}{d x}(5)\right]=\cos \left(x^2+5\right) \cdot[2 x+0]=2 x \cos \left(x^2+5\right)\)

Question 2. \(\cos (\sin x) \)

Solution:

⇒ \(\frac{d}{d x}[\cos (\sin x)]=-\sin (\sin x) \cdot \frac{d}{d x}(\sin x)\)

=-\(\sin (\sin x) \cdot \cos x=-\cos x \sin (\sin x)\)

Question 3. sin (a x+b)

Solution:

⇒ \(\frac{d}{d x}[\sin (a x+b)]=\cos (a x+b) \cdot \frac{d}{d x}(a x+b)\)

=\(\cos (a x+b) \cdot\left[\frac{d}{d x}(a x)+\frac{d}{d x}(b)\right]=\cos (a x+b) \cdot(a+0)=a \cos (a x+b)\)

Question 4. \(\sec (\tan (\sqrt{x}))\)

Solution:

⇒ \({\frac{d x}{d x}[\sec (\tan \sqrt{x})]} =\sec (\tan \sqrt{x}) \cdot \tan (\tan \sqrt{x}) \cdot \frac{d}{d x}(\tan \sqrt{x})\)

= \(\sec (\tan \sqrt{x}) \cdot \tan (\tan \sqrt{x}) \cdot \sec ^2(\sqrt{x}) \cdot \frac{d}{d x}(\sqrt{x}) \)

= \(\sec (\tan \sqrt{x}) \cdot \tan (\tan \sqrt{x}) \cdot \sec ^2(\sqrt{x}) \cdot \frac{1}{2 \sqrt{x}}\)

= \(\frac{\sec (\tan \sqrt{x}) \cdot \tan (\tan \sqrt{x}) \sec ^2(\sqrt{x})}{2 \sqrt{x}}\)

Question 5. \(\frac{\sin (a x+b)}{\cos (c x+d)}\)

Solution:

Let f(x)=\(\frac{\sin (a x+b)}{\cos (c x+d)}=\frac{u(x)}{v(x)}\)

⇒ \(f^{\prime}(x)=\frac{v(x)\left[u^{\prime}(x)\right]-u(x)\left[v^{\prime}(x)\right]}{[v(x)]^2}\)

⇒ \(\mathrm{f}(\mathrm{x})=\frac{\cos (c x+d) \cdot a \cos (a x+b)+\sin (a x+b) \cdot c \sin (c x+d)}{\cos ^2(c x+d)}\)

⇒ \(f^{\prime}(x)=a \cos (a x+b) \cdot \sec (c x+d)+c \sin (a x+b) \cdot \tan (c x+d) \cdot \sec (c x+d)\)

Question 6. \(\cos x^3 \cdot \sin ^2\left(x^5\right)\)

Solution:

The given function is \(\cos x^3-\sin ^2\left(x^3\right)\)

⇒ \(\frac{d}{d x}\left[\cos x^3 \cdot \sin ^2\left(x^5\right)\right]\)

= \(\sin ^2\left(x^5\right) \times \frac{d}{d x}\left(\cos x^3\right)+\cos x^3 \times \frac{d}{d x}\left[\sin ^2\left(x^5\right)\right]\)

= \(\sin ^2\left(x^5\right) \times\left(-\sin x^3\right) \times \frac{d}{d x}\left(x^3\right)+\cos x^5 \times 2 \sin \left(x^5\right) \cdot \frac{d}{d x}\left[\sin x^4\right]\)

=-\(\sin x^3 \sin ^2\left(x^5\right) \times 3 x^2+2 \sin x^5 \cos x^3 \cdot \cos x^5 \times \frac{d}{d x}\left(x^5\right)\)

=-\(3 x^2 \sin x^3 \cdot \sin ^2\left(x^5\right)+2 \sin x^5 \cos x^5 \cos x^3 \times 5 x^4 \)

= \(10 x^4 \sin x^4 \cos x^4 \cos x^3-3 x^7 \sin x^5 \sin ^2\left(x^5\right)\)

Question 7. 2 \(\sqrt{\cot \left(x^2\right)}\)

Solution:

⇒ \(\frac{d}{d x}\left[2 \sqrt{\cot \left(x^2\right)}\right]=2 \cdot \frac{1}{2 \sqrt{\cot \left(x^2\right)}} \times \frac{d}{d x}\left[\cot \left(x^2\right)\right]\)

=\(\sqrt{\frac{\sin \left(x^2\right)}{\cos \left(x^2\right)}} \times-{cosec}^2\left(x^2\right) \times \frac{d}{d x}\left(x^2\right)\)

=-\(\sqrt{\frac{\sin \left(x^2\right)}{\cos \left(x^2\right)}} \times \frac{1}{\sin ^2\left(x^2\right)} \times(2 x)\)

= \(\frac{-2 x}{\sqrt{\cos x^2 \sqrt{\sin x^2} \sin x^2}}=\frac{-2 \sqrt{2} x}{\sqrt{2 \sin x^2 \cos x^2}\left(\sin x^2\right)}\)

= \(\frac{-2 \sqrt{2} x}{\sin x^2 \sqrt{\sin 2 x^2}}\)

Question 8. \(\cos (\sqrt{x}\)

Solution:

⇒ \(\frac{d}{d x}[\cos (\sqrt{x})]=-\sin (\sqrt{x}) \cdot \frac{d}{d x}(\sqrt{x})=\frac{-\sin \sqrt{x}}{2 \sqrt{x}}\)

Question 9. Prove that the function f is given by f(x)=|x-1|, x \(\in \mathbf{R}\) is not differentiable at x=1.

Solution:

The given function is f(x)=\(|\mathrm{x}-1|, \mathrm{x} \in \mathbf{R}\)

It is known that a function f is differentiable at a point x=c in its domain if both \(\lim _{\mathrm{h} \rightarrow 0}\)

⇒ \(\frac{\mathrm{f}(\mathrm{c}-\mathrm{h})-\mathrm{f}(\mathrm{c})}{-\mathrm{h}} and \lim _{\mathrm{h} \rightarrow 0}\)

⇒ \(\frac{\mathrm{f}(\mathrm{c}+\mathrm{h})-\mathrm{f}(\mathrm{c})}{\mathrm{h}}\)are finite and equal.

To check the differentiability of the given function at x=1, consider the left-hand derivative of f at x=1

L.H.D. =\(\lim _{h \rightarrow 0} \frac{f(1-h)-f(1)}{-h}=\lim _{h \rightarrow 0} \frac{|1-h-1|-|1-1|}{-h}=\lim _{h \rightarrow 0}\left(\frac{h-0}{-h}\right)=-1\)

Consider the right-hand derivative of f at x=1

R.H.D. =\(\lim _{h \rightarrow 0} \frac{f(1+h)-f(1)}{h}=\lim _{h \rightarrow 0} \frac{|1+h-1|-|1-1|}{h}=\lim _{h \rightarrow 0}\left(\frac{h-0}{h}\right)=1\)

Since the left and right-hand derivatives of f at x=1 are not equal, f is not differentiable at x-1

Question 10. Prove that the greatest integer function defined by \(\mathrm{f}(\mathrm{x})=[\mathrm{x}], 0<\mathrm{x}<3\), is not differentiable at x-1 and x-2.

Solution:

The given function f is f(x)=[x], 0<x<3

It is known that a function f is differentiable at a point x=c in its domain if both \(\lim _{\mathrm{h} \rightarrow 0} \frac{f(c-h)-f(c)}{-h} and \lim _{x \rightarrow \infty} \frac{f(c+h)-f(c)}{h}\) are finite and equal.

To check the differentiability of the given function at x=1, consider the left-hand derivative of f at x=1,

L.H.D. =\(\lim _{h \rightarrow 0} \frac{f(1-h)-f(1)}{-h}=\lim _{h \rightarrow 0} \frac{[1-h]-[1]}{-h}=\lim _{h \rightarrow 0} \frac{0-1}{-h}=\infty\)

Consider the right-hand derivative of f at x=1

R.H.D. =\(\lim _{h \rightarrow h} \frac{f(1+h)-f(h)}{h}=\lim _{h \rightarrow 0} \frac{[1+h]-[t]}{h}=\lim _{h \rightarrow 1} \frac{1-r}{h}=\lim _{h \rightarrow 0}\) 0=0

Since the left and right-hand derivatives of f at x=1 are not equal, f is not differentiable at x=1

To check the differentiability of the given function at x=2, consider the left-hand derivative of f at x-2

L.H.D. =\(\lim _{h \rightarrow h} \frac{f(2-h)-f(2)}{-h}=\lim _{h \rightarrow 0} \frac{[2-h]-[2]}{-h}=\lim _{h \rightarrow 0} \frac{1-2}{-h}=\lim _{h \rightarrow 0} \frac{-1}{-h}=\infty\)

Consider the right-hand derivative of f at x-1.

R.H.D \(-\lim _{h \rightarrow 0} \frac{f(2+h)-f(2)}{h}=\lim _{h \rightarrow h} \frac{[2+h]-[2]}{h}=\lim _{h \rightarrow 0} \frac{2-2}{h}=\lim _{h \rightarrow 0}\) 0=0

Since the left and right-hand derivatives of f at x=2 are not equal, f is not differentiable at x=2

Continuity And Differentiability Exercise 5.3

Question 1.1. 2 x+3 y = sin x

Solution:

⇒ \(\frac{d}{d x}(2 x+3 y)=\frac{d}{d x}(\sin x)\)

⇒ \(\frac{d}{d x}(2 x)+\frac{d}{d x}(3 y)=\cos x\)

2+3 \(\frac{d y}{d x}=\cos x \)

3 \(\frac{d y}{d x}=\cos x-2 \)

⇒ \(\frac{d y}{d x}=\frac{\cos x-2}{3}\)

Question 2. 2 x+3 y=sin y

Solution:

⇒ \(\frac{d}{d x}(2 x)+\frac{d}{d x}(3 y)=\frac{d}{d x}(\sin y)\)

⇒ \(2+3 \frac{d y}{d x}=\cos y \frac{d y}{d x}\) [By using chain rule] }

⇒ \(2=(\cos y-3) \frac{d y}{d x} \Rightarrow \frac{d y}{d x}=\frac{2}{\cos y-3}\)

Question 3. a x+b \(y^2=\cos y\)

Solution:

⇒ \(\frac{d}{d x}(a x)+\frac{d}{d x}\left(b y^2\right)=\frac{d}{d x}(\cos y)\)

a+b \(\frac{d}{d x}\left(y^2\right)=\frac{d}{d x}(\cos y)\)

Using the chain rule, we obtain

⇒ \(\frac{d}{d x}\left(y^2\right)=2 y \frac{d y}{d x} and \frac{d}{d x}(\cos y)=-\sin y \frac{d y}{d x}\)

From (1) and (2), we obtain

a+b \(\times 2 y \frac{d y}{d x}=-\sin y \frac{d y}{d x}\)

⇒ \((2 b y+\sin y) \frac{d y}{d x}=-a\)

⇒ \(\frac{d y}{d x}=\frac{-a}{2 b y+\sin y}\)

Question 4. \(x y+y^2=\tan x+y\)

Solution :

⇒ \(\frac{d}{d x}\left(x y+y^2\right)=\frac{d}{d x}(\tan x+y) \Rightarrow \frac{d}{d x}(x y)+\frac{d}{d x}\left(y^2\right)\)

= \(\frac{d}{d x}(\tan x)+\frac{d y}{d x}\)

⇒ \(\left[y \cdot \frac{d}{d x}(x)+x \cdot \frac{d y}{d x}\right]+2 y \frac{d y}{d x}\)

= \(\sec ^2 x+\frac{d y}{d x}\) [Using product rule and chain rule]

y \(\cdot 1+x \cdot \frac{d y}{d x}+2 y \frac{d y}{d x}=\sec ^2 x+\frac{d y}{d x} \Rightarrow(x+2 y-1) \frac{d y}{d x}=\sec ^2 x-y\)

⇒ \(\frac{d y}{d x}=\frac{\sec ^2 x-y}{(x+2 y-1)}\)

Question 5. \( x^2+x y+y^2\)=100

Solution:

⇒ \(\frac{d}{d x}\left(x^2+x y+y^2\right)=\frac{d}{d x}(100)\)

⇒ \(\frac{d}{d x}\left(x^2\right)+\frac{d}{d x}(x y)+\frac{d}{d x}\left(y^2\right)\)=0 [Derivative of constant function is 0 ]

⇒ \(2 x+\left[y \cdot \frac{d}{d x}(x)+x \cdot \frac{d y}{d x}\right]+2 y \frac{d y}{d x}\)=0 [Using product rule and chain rule]

⇒ \(2 x+y \cdot 1+x \cdot \frac{d y}{d x}+2 y \frac{d y}{d x}\)=0

⇒ \(2 x+y+(x+2 y) \frac{d y}{d x}\)=0

⇒ \(\frac{d y}{d x}=-\frac{2 x+y}{x+2 y}\)

Question 6. \(x^3+x^2 y+x y^2+y^3=81\)

Solution:

⇒ \(\frac{d}{d x}\left(x^3+x^2 y+x y^2+y^3\right)=\frac{d}{d x}(81)\)

⇒ \(\frac{d}{d x}\left(x^3\right)+\frac{d}{d x}\left(x^2 y\right)+\frac{d}{d x}\left(x y^2\right)+\frac{d}{d x}\left(y^3\right)\)=0

⇒ \(3 x^2+\left[y \frac{d}{d x}\left(x^2\right)+x^2 \frac{d y}{d x}\right]+\left[y^2 \frac{d}{d x}(x)+x \frac{d}{d x}\left(y^2\right)\right]+3 y^2 \frac{d y}{d x}\)=0

⇒ \(3 x^2+\left[y \cdot 2 x+x^2 \frac{d y}{d x}\right]+\left[y^2 \cdot 1+x \cdot 2 y \cdot \frac{d y}{d x}\right]+3 y^2 \frac{d y}{d x}\)=0

⇒ \(\left(x^2+2 x y+3 y^2\right) \frac{d y}{d x}+\left(3 x^2+2 x y+y^2\right)\)=0

⇒ \(\frac{d y}{d x}=\frac{-\left(3 x^2+2 x y+y^2\right)}{\left(x^2+2 x y+3 y^2\right)}\)

Question 7. \(\sin ^2 y+\cos x y=k\)

Solution:

⇒ \(\frac{d}{d x}\left(\sin ^2 y+\cos x y\right)=\frac{d}{d x}(k)\)

⇒ \(\frac{d}{d x}\left(\sin ^2 y\right)+\frac{d}{d x}(\cos x y)=0\)

Using the chain rule, we obtain

⇒ \(\frac{d}{d x}\left(\sin ^2 y\right) =2 \sin y \frac{d}{d x}(\sin y)=2 \sin y \cos y \frac{d y}{d x}\)

⇒ \(\frac{d}{d x}(\cos x y)=-\sin x y \frac{d}{d x}(x y)=-\sin x y\left[y \frac{d}{d x}(x)+x \frac{d y}{d x}\right]\)

=-\(\sin x y\left[y \cdot 1+x \frac{d y}{d x}\right]=-y \sin x y-x \sin x y \frac{d y}{d x}\)

From (1), (2), and (3), we obtain

2 \(\sin y \cos y \frac{d y}{d x}-y \sin x y-x \sin x y \frac{d y}{d x}=0 \)

⇒ \((2 \sin y \cos y-x \sin x y) \frac{d y}{d x}=y \sin x y\)

⇒ \((\sin 2 y-x \sin x y) \frac{d y}{d x}=y \sin x y\)

⇒ \(\frac{d y}{d x}=\frac{y \sin x y}{\sin 2 y-x \sin x y}\)

Question 8. \(\sin ^2 x+\cos ^2 y=1\)

Solution:

⇒ \(\frac{d}{d x}\left(\sin ^2 x+\cos ^2 y\right)=\frac{d}{d x}(1)\)

⇒ \(\frac{d}{d x}\left(\sin ^2 x\right)+\frac{d}{d x}\left(\cos ^2 y\right)=0 \Rightarrow 2 \sin x \cdot \frac{d}{d x}(\sin x)+2 \cos y \cdot \frac{d}{d x}(\cos y)=0\)

2 \(\sin x \cos x+2 \cos y(-\sin y) \cdot \frac{d y}{d x}\)=0

⇒ \(\sin 2 x-\sin 2 y \frac{d y}{d x}\)=0

⇒ \(\frac{d y}{d x}=\frac{\sin 2 x}{\sin 2 y}\)

Question 9. y=\(\sin ^{-1}\left(\frac{2 x}{1+x^2}\right)\)

Solution:

y=\(\sin ^{-1}\left(\frac{2 x}{1+x^2}\right)\)

Put x=\(\tan \theta\)

⇒ \(\theta=\tan ^{-t}\) x

⇒ \(y=\sin ^{-1}\left(\frac{2 x}{1+x^2}\right)=\sin ^{-1}\left(\frac{2 \tan \theta}{1+\tan ^2 \theta}\right)\)

y=\(\sin ^{-1}(\sin 2 \theta)=2 \theta\)

y=2 \(\tan ^{-1} x\)

∴ \(\frac{d y}{d x}=\frac{2}{1+x^2}\)

Question 10. y=\(\tan ^{-1}\left(\frac{3 x-x^3}{1-3 x^2}\right),-\frac{1}{\sqrt{3}}<x<\frac{1}{\sqrt{3}} \)

Solution:

y=\(\tan ^{-1}\left(\frac{3 x-x^3}{1-3 x^2}\right)\)

Put x=\(\tan \theta \Rightarrow \theta=\tan ^{-1}x\)

y=\(\tan ^{-1}\left(\frac{3 \tan \theta-\tan ^3 \theta}{1-3 \tan ^2 \theta}\right)=\tan ^{-1}(\tan 3 \theta)\)

y=3 \(\theta=3 \tan ^{-1} x\)

⇒ \(\frac{d y}{d x}=\frac{3}{1+x^2}\)

Question 11. y=\(\cos ^{-1}\left(\frac{1-x^2}{1+x^2}\right), 0<x<1\)

Solution:

y=\(\cos ^{-1}\left(\frac{1-x^2}{1+x^2}\right)\)

Put x=\(\tan \theta \Rightarrow \theta=\tan ^{-1}\) x Equation 1

y=\(\cos ^{-1}\left(\frac{1-\tan ^2 \theta}{1+\tan ^2 \theta}\right) \left[ \cos 2 \theta=\frac{1-\tan ^2 \theta}{1+\tan ^2 \theta}\right]\)

y=\(\cos ^{-1}(\cos 2 \theta)=2 \theta \quad \Rightarrow y=2 \tan ^{-1}\) x From Equation 1

⇒ \(\frac{d y}{d x}=2 \cdot \frac{1}{1+x^2}\)

Question 12. y=\(\sin ^{-1}\left(\frac{1-x^2}{1+x^2}\right)\), 0<x<1

Solution:

y=\(\sin ^{-1}\left(\frac{1-x^2}{1+x^2}\right)\)

Put x=\(\tan \theta \Rightarrow \theta=\tan ^{-1} x\)

y=\(\sin ^{-1}\left(\frac{1-\tan ^2 \theta}{1+\tan ^2 \theta}\right) \Rightarrow y=\sin ^{-1}(\cos 2 \theta)\)

y=\(\sin ^{-1}\left\{\sin \left(\frac{\pi}{2}-2 \theta\right)\right\}\)

y=\(\frac{\pi}{2}-20=\frac{\pi}{2}-2 \tan ^{-1} x\)

⇒ \(\frac{d y}{d x}=-2 \cdot\left(\frac{1}{1+x^2}\right)=\frac{-2}{1+x^2}\)

Question 13. y=\(\cos ^{-1}\left(\frac{2 x}{1+x^2}\right),-1<x<1 \)

Solution:

y=\(\cos ^{-1}\left(\frac{2 x}{1+x^2}\right)\)

Put x=\(\tan \theta \Rightarrow \theta=\tan ^{-1} x\) Equation 1

y=\(\cos ^{-1}\left(\frac{2 \tan \theta}{1+\tan ^2 \theta}\right)\) \(\left( \sin 2 \theta=\frac{2 \tan \theta}{1+\tan ^2 \theta}\right)\)

y=\(\cos ^{-1}(\sin 2 \theta)\)

y=\(\cos ^{-1}\left\{\cos \left(\frac{\pi}{2}-2 \theta\right)\right\} \Rightarrow y=\frac{\pi}{2}-2 \theta\)

⇒ \( y=\frac{\pi}{2}-2 \tan ^{-1} x\) [From eq. (1)]

⇒ \(\frac{d y}{d x}=\frac{-2}{1+x^2}\)

Question 14. y=\(\sin ^{-1}\left(2 x \sqrt{1-x^2}\right),-\frac{1}{\sqrt{2}}<x<\frac{1}{\sqrt{2}}\)

Solution:

y=\(\sin ^{-1}\left(2 x \sqrt{1-x^2}\right)\)

Put x=\(\sin \theta \Rightarrow \theta=\sin ^{-1}\) x Equation 1

y=\(\sin ^{-1}\left(2 \sin \theta \sqrt{1-\sin ^2 \theta}\right)=\sin ^{-1}(2 \sin \theta \cos \theta)\)

y=\(\sin ^{-1}(\sin 2 \theta)=2 \theta \Rightarrow y=2 \sin ^{-1} x\) From Equation 1

⇒ \(\frac{d y}{d x}=\frac{2}{\sqrt{1-x^2}}\)

Question 15. y=\(\sec ^{-1}\left(\frac{1}{2 x^2-1}\right), 0<x<\frac{1}{\sqrt{2}}\)

Solution:

y=\(\sec ^{-1}\left(\frac{1}{2 x^2-1}\right)\)

Put x=\(\cos \theta \Rightarrow \theta=\cos ^{-1}\) x

y=\(\sec ^{-1}\left(\frac{1}{2 \cos ^2 \theta-1}\right)=\sec ^{-1}\left(\frac{1}{\cos 2 \theta}\right)=\sec ^{-1}(\sec 2 \theta)=2 \theta \)

y=2 \(\cos ^{-1}\) x (From eq. (1))

∴ \(\frac{d y}{d x}=\frac{-2}{\sqrt{1-x^2}}\)

Continuity And Differentiability Exercise 5.4

Question 1. \(\frac{e^x}{\sin x}\)

Solution:

Let y=\(\frac{e^4}{\sin x}\)

By using the quotient rule, we obtain

⇒ \(\frac{d y}{d x} =\frac{\sin x \frac{d}{d x}\left(e^x\right)-e^x \frac{d}{d x}(\sin x)}{\sin ^2 x}\)

=\(\frac{e^x(\sin x-\cos x)}{\sin ^2 x}, x \neq n \pi, n \in Z\) \(\left[\frac{d}{d x}\left(\frac{u}{v}\right)=\frac{v \frac{d}{d x}(u)-u \frac{d}{d x}(v)}{v^2}\right]\)

Question 2. \(e^{sin^{-1} x}\)

Solution:

Let y=\(e^{sin x^{-1}}\)

By using the chain rule, we obtain

⇒ \(\frac{d y}{d x}=\frac{d}{d x}\left(e^{n=-1}\right)\)

= \(\frac{d y}{d x}=e^{n+1}+\frac{d}{d x}\left(\sin ^{-1} x\right)\)

= \(\frac{e^{-n^{-1}} x}{\sqrt{1-x^2}}\)

⇒ \(\frac{d y}{d x}=\frac{e^{m n^{-1} x}}{\sqrt{1-x^2}}, x \in(-1,1)\)

Question 3. \(e^{x^2}\)

Solution:

Let y=\(e^{x^2}\) (By using the chain rule, we obtain)

⇒ \(\frac{d y}{d x}=\frac{d}{d x}\left(e^{x^{\prime}}\right)=e^{x^3} \cdot 3 x^2=3 x^2 e^{x^{\prime}}\)

Question 4. \(\sin \left(\tan ^{-1} e^{-x}\right)\)

Solution:

Let y=\(\sin \left(\tan ^{-1} e^{-\pi}\right)\) (By using chain rule, we obtain)

⇒ \(\frac{d y}{d x}=\frac{d}{d x}\left[\sin \left(\tan ^{-1} e^{-x}\right)\right]\)

=\(\cos \left(\tan ^{-1} e^{-x}\right) \cdot \frac{d}{d x}\left(\tan ^{-t} e^{-1}\right)\)

=\(\cos \left(\tan ^{-1} e^{-x}\right) \frac{1}{1+\left(e^{-1}\right)^2} \cdot \frac{d}{d x}\left(e^{-x}\right)\)

=\(\frac{\cos \left(\tan ^{-1} e^{-1}\right)}{1+e^{-2 x}} \cdot e^{-x} \cdot \frac{d}{d x}(-x)\)

=\(\frac{e^{-3} \cos \left(\tan ^{-1} e^{-1}\right)}{1+e^{-t x}} \times(-1)=\frac{-e^{-1} \cos \left(\tan ^{-1} e^{-t}\right)}{1+e^{-5 x}}\)

Question 5. \(\sin \left(\tan ^{-1} e^{-x}\right)\)

Solution:

Let y=\(\sin \left(\tan ^{-1} e^{-x}\right)\)(By using chain rule, we obtain)

⇒ \(\frac{d y}{d x} =\frac{d}{d x}\left[\sin \left(\tan ^{-1} e^{-1}\right)\right]\)

= \(\cos \left(\tan ^{-1} e^{-x}\right) \cdot \frac{d}{d x}\left(\tan ^{-1} e^{-1}\right)\)

= \(\cos \left(\tan ^{-1} e^{-1}\right) \frac{1}{1+\left(e^{-x}\right)} \cdot \frac{d}{d x}\left(e^{-x}\right)\)

= \(\frac{\cos \left(\tan ^{-1} e^{-1}\right)}{1+e^{-2 x}} \cdot e^{-x} \cdot \frac{d}{d x}(-x)\)

= \(\frac{e^{-1} \cos \left(\tan ^{-1} e^{-1}\right)}{1+e^{-3 x}} \times(-1)=\frac{-e^{-1} \cos \left(\tan ^{-1} e^{-x}\right)}{1+e^{-3 x}}\)

Question 6. \(e^x+e^{x^2}+\ldots .+e^{x^2}\)

Solution:

⇒ \(\frac{d}{d x}\left(e^x+e^{x^2}+\ldots .+e^{x^1}\right)\)

= \(\frac{d}{d x}\left(e^x\right)+\frac{d}{d x}\left(e^{x^2}\right)+\frac{d}{d x}\left(e^{x^3}\right)+\frac{d}{d x}\left(e^{x^4}\right)+\frac{d}{d x}\left(e^{x^2}\right)\)

=\(e^x+\left[e^{x^2} \times \frac{d}{d x}\left(x^2\right)\right]+\left[e^{x^1} \cdot \frac{d}{d x}\left(x^3\right)\right]+\left[e^{x^4} \cdot \frac{d}{d x}\left(x^4\right)\right]+\left[e^{x^3} \cdot \frac{d}{d x}\left(x^5\right)\right]\)

= \(e^x+\left(e^{x^1} \times 2 x\right)+\left(e^{x^3} \times 3 x^2\right)+\left(e^{x^4} \times 4 x^3\right)+\left(e^{x^3} \times 5 x^4\right)\)

=\(e^x+2 e^{x^1}+3 x^2 e^{x^1}+4 x^3 e^{x^1}+5 x^4 e^{x^5}\)

Question 7. \(\sqrt{e^{\sqrt{x}}}, x>0\)

Solution:

Let y=\(\sqrt{e^{\sqrt{x}}}\)

Then, \(y^2=e^{\sqrt{x}}\) (By differentiating this relationship with respect to x, we obtain)

2 y \(\frac{d y}{d x}=e^{\sqrt{x}} \frac{d}{d x}(\sqrt{x})\) [By applying chain rule]

2 y \(\frac{d y}{d x}=e^{\sqrt{x}} \frac{1}{2} \cdot \frac{1}{\sqrt{x}}\)

⇒ \(\frac{d y}{d x}\)

= \(\frac{e^{\sqrt{x}}}{4 y \sqrt{x}}\)

⇒ \(\frac{d y}{d x}=\frac{e^{\sqrt{x}}}{4 \sqrt{e^{\sqrt{x}}} \sqrt{x}}\)

⇒ \(\frac{d y}{d x}=\frac{e^{\sqrt{x}}}{4 \sqrt{x e^{\sqrt{x}}}}\), x>0

Question 8. \(\log (\log x)\), x>1

Solution:

Let y =\(\log (\log \mathrm{x})\) (By using chain rule, we obtain)

⇒ \(\frac{d y}{d x}=\frac{d}{d x}[\log (\log x)]=\frac{1}{\log x} \cdot \frac{d}{d x}(\log x)\)

=\(\frac{1}{\log x} \cdot \frac{1}{x}=\frac{1}{x \log x}, x>1\)

Question 9. \(\frac{\cos x}{\log x}\), x>0

Solution:

Let y=\(\frac{\cos x}{\log x}\)

By using the quotient rule, we obtain

⇒ \(\frac{d y}{d x}=\frac{\log x \frac{d}{d x}(\cos x)-\cos x \times \frac{d}{d x}(\log x)}{(\log x)^2}\)

=\(\frac{-\sin x \log x-\cos x \times \frac{1}{x}}{(\log x)^2}\)

=\(\frac{-[x \log x \cdot \sin x+\cos x]}{x(\log x)^2}\), x>0

Question 10. \(\cos \left(\log x+e^x\right)\), x>0

Solution:

Let y=\(\cos \left(\log x+e^x\right)\) (By using chain rule, we obtain)

⇒ \(\frac{d y}{d x} =\frac{d}{d x}\left\{\cos \left(\log x+e^x\right)\right\}\)

⇒ \(\frac{d y}{d x} =-\sin \left(\log x+e^x\right) \cdot \frac{d}{d x}\left(\log x+e^x\right)\)

=-\(\sin \left(\log x+e^x\right) \cdot\left(\frac{1}{x}+e^x\right)\)

=-\(\left(\frac{1}{x}+e^x\right) \sin \left(\log x+e^x\right)\), x>0

Continuity And Differentiability Exercise 5.5

Question 1. \(\cos \mathrm{x} \cdot \cos 2 \mathrm{x} \cdot \cos 3 \mathrm{x}\)

Solution:

Let y=\(\cos \mathrm{x} \cdot \cos 2 \mathrm{x} \cdot \cos 3 \mathrm{x}\)

Taking logarithm on both the sides \(\log y=\log (\cos x \cdot \cos 2 x \cdot \cos 3 x)\)

⇒ \(\log y=\log (\cos x)+\log (\cos 2 x)+\log (\cos 3 x)\)

Differentiating both sides with respect to x, we obtain

⇒ \(\frac{1}{y d x}=\frac{1}{\cos x} \cdot \frac{d}{d x}(\cos x)+\frac{1}{\cos 2 x} \cdot \frac{d}{d x}(\cos 2 x)+\frac{1}{\cos 3 x} \cdot \frac{d}{d x}(\cos 3 x)\)

⇒ \(\frac{d y}{d x}=y\left[-\frac{\sin x}{\cos x}-\frac{\sin 2 x}{\cos 2 x} \cdot \frac{d}{d x}(2 x)-\frac{\sin 3 x}{\cos 3 x} \cdot \frac{d}{d x}(3 x)\right]\)

⇒ \(\frac{d y}{d x}=-\cos x \cdot \cos 2 x \cdot \cos 3 x[\tan x+2 \tan 2 x+3 \tan 3 x]\)

Question 2. \(\sqrt{\frac{(x-1)(x-2)}{(x-3)(x-4)(x-5)}}\)

Solution:

Let y=\(\sqrt{\frac{(x-1)(x-2)}{(x-3)(x-4)(x-5)}}\)

Taking logarithm on both the sides \(\log y=\log \sqrt{\frac{(x-1)(x-2)}{(x-3)(x-4)(x-5)}}\)

⇒ \(\log y=\frac{1}{2} \log \left[\frac{(x-1)(x-2)}{(x-3)(x-4)(x-5)}\right]\)

⇒ \(\log y=\frac{1}{2}[\log \{(x-1)(x-2)\}-\log \{(x-3)(x-4)(x-5)\}]\)

⇒ \(\log y=\frac{1}{2}[\log (x-1)+\log (x-2)-\log (x-3)-\log (x-4)-\log (x-5)]\)

Differentiating both sides with respect to x, we obtain

⇒ \(\frac{1}{y} \frac{d y}{d x}=\frac{1}{2}\left[\begin{array}{l}
\frac{1}{x-1} \cdot \frac{d}{d x}(x-1)+\frac{1}{x-2} \cdot \frac{d}{d x}(x-2)-\frac{1}{x-3} \cdot \frac{d}{d x}(x-3) \\
-\frac{1}{x-4} \cdot \frac{d}{d x}(x-4)-\frac{1}{x-5} \cdot \frac{d}{d x}(x-5)
\end{array}\right]\)

⇒ \(\frac{d y}{d x}=\frac{y}{2}\left(\frac{1}{x-1}+\frac{1}{x-2}-\frac{1}{x-3}-\frac{1}{x-4}-\frac{1}{x-5}\right)\)

⇒ \(\frac{d y}{d x}=\frac{1}{2} \sqrt{(x-1)(x-2)}\left[\frac{1}{(x-3)(x-4)(x-5)}+\frac{1}{x-2}-\frac{1}{x-3}-\frac{1}{x-4}-\frac{1}{x-5}\right]\)

Question 3. \((\log x)^{\cos x}\)

Solution:

Let y=\((\log x)^{\cos x}\)

Taking logarithm on both the sides \(\log y=\cos x \cdot \log (\log x)\)

Differentiating both sides with respect to x, we obtain

⇒ \(\frac{1}{y} \cdot \frac{d y}{d x}=\frac{d}{d x}(\cos x) \times \log (\log x)+\cos x \times \frac{d}{d x}[\log (\log x)]\)

⇒ \(\frac{1}{y} \cdot \frac{d y}{d x}=-\sin x \log (\log x)+\cos x \times \frac{1}{\log x} \cdot \frac{d}{d x}(\log x)\)

⇒ \(\frac{d y}{d x}=y\left[-\sin x \log (\log x)+\frac{\cos x}{\log x} \times \frac{1}{x}\right]\)

∴ \(\frac{d y}{d x}=(\log x)^{\cos x}\left[\frac{\cos x}{x \log x}-\sin x \log (\log x)\right]\)

Question 4. \(x^x-2^{\sin x}\)

Solution:

Let y=\(x^x-2^{\sin x}\)

Also, let \(\mathrm{x}^{\mathrm{x}}=\mathrm{u} and 2^{\sin \mathrm{x}}=\mathrm{v}\)

y=\(\mathrm{u}-\mathrm{v}\)

⇒ \(\frac{\mathrm{dy}}{\mathrm{dx}}=\frac{\mathrm{du}}{\mathrm{dx}}-\frac{\mathrm{dv}}{\mathrm{dx}}\) →  Equation 1

⇒ \(\mathrm{u}=\mathrm{x}^{\mathrm{x}}\)

Taking logarithm on both sides log u = x log x

Differentiating both sides with respect to x, we obtain

⇒ \(\frac{1}{\mathrm{u}} \frac{\mathrm{du}}{\mathrm{dx}}=\left[\frac{\mathrm{d}}{\mathrm{dx}}(\mathrm{x}) \times \log \mathrm{x}+\mathrm{x} \times \frac{\mathrm{d}}{\mathrm{dx}}(\log x)\right]\)

⇒ \(\frac{\mathrm{du}}{\mathrm{dx}}=\mathrm{u}\left[1 \times \log \mathrm{x}+\mathrm{x} \times \frac{1}{\mathrm{x}}\right]\)

⇒ \(\frac{\mathrm{du}}{\mathrm{dx}}=\mathrm{x}^{\mathrm{x}}(1+\log \mathrm{x})\) →  Equation 2

⇒ \(\mathrm{v}=2^{\sin x}\)

⇒ \(\frac{1}{\mathrm{u} d \mathrm{du}}=\left[\frac{\mathrm{d}}{\mathrm{dx}}(\mathrm{x}) \times \log \mathrm{x}+\mathrm{x} \times \frac{\mathrm{d}}{\mathrm{dx}}(\log x)\right]\)

⇒ \(\frac{\mathrm{du}}{\mathrm{dx}}=\mathrm{u}\left[1 \times \log \mathrm{x}+\mathrm{x} \times \frac{1}{\mathrm{x}}\right]\)

⇒ \(\frac{\mathrm{du}}{\mathrm{dx}}=\mathrm{x}^{\mathrm{x}}(1+\log \mathrm{x})\)

⇒ \(\mathrm{v}=2^{\sin x}\)

Taking logarithm on both the sides \(\log v=\sin x \cdot \log 2\)

Differentiating both sides with respect to x, we obtain

⇒ \(\frac{1}{v} \cdot \frac{d v}{d x}=\log 2 \cdot \frac{d}{d x}(\sin x)\)

⇒ \(\frac{d v}{d x}=v(\log 2 \cos x) \Rightarrow \frac{d v}{d x}=2^{\sin x}(\cos x \log 2)\) Equation 3

Therefore, from equations (1), (2) and (3), we obtain

⇒ \(\frac{\mathrm{dy}}{\mathrm{dx}}=\mathrm{x}^{\mathrm{x}}(1+\log \mathrm{x})-2^{\sin \mathrm{x}} \cos \mathrm{x} \log 2\)

Question 5. \((x+3)^2 \cdot(x+4)^3 \cdot(x+5)^4\)

Solution:

Let \(y=(x+3)^2 \cdot(x+4)^3 \cdot(x+5)^4\)

Taking logarithms on both sides

⇒ \(\log y=\log (x+3)^2+\log (x+4)^3+\log (x+5)^4\)

∴ \(\log y=2 \log (x+3)+3 \log (x+4)+4 \log (x+5)\)

Differentiating both sides with respect to x, we obtain

⇒ \(\frac{1}{y} \cdot \frac{d y}{d x}=2 \cdot \frac{1}{x+3} \cdot \frac{d}{d x}(x+3)+3 \cdot \frac{1}{x+4} \cdot \frac{d}{d x}(x+4)+4 \cdot \frac{1}{x+5} \cdot \frac{d}{d x}(x+5)\)

⇒ \(\frac{d y}{d x}=y\left[\frac{2}{x+3}+\frac{3}{x+4}+\frac{4}{x+5}\right]\)

⇒ \(\frac{d y}{d x}=(x+3)^2(x+4)^3(x+5)^4 \cdot\left[\frac{2}{x+3}+\frac{3}{x+4}+\frac{4}{x+5}\right]\)

⇒ \(\frac{d y}{d x}=(x+3)^2(x+4)^3(x+5)^4 \cdot\left[\frac{2(x+4)(x+5)+3(x+3)(x+5)+4(x+3)(x+4)}{(x+3)(x+4)(x+5)}\right]\)

⇒ \(\frac{d y}{d x}=(x+3)(x+4)^2(x+5)^3 \cdot\left[2\left(x^2+9 x+20\right)+3\left(x^2+8 x+15\right)+4\left(x^2+7 x+12\right)\right]\)

⇒ \(\frac{d y}{d x}=(x+3)(x+4)^2(x+5)^3\left(9 x^2+70 x+133\right)\)

∴ \(\left.\left(x+\frac{1}{x}\right)^x+x^x+\frac{1}{x}\right)\)

Question 6. \(\left(x+\frac{1}{x}\right)^x+x^{\left(1+\frac{1}{x}\right)}\)

Solution:

Let y=\(\left(x+\frac{1}{x}\right)^x+x^{\left(1+\frac{1}{x}\right)}\)

Also, let \(mathrm{u}=\left(\mathrm{x}+\frac{1}{\mathrm{x}}\right)^{\mathrm{x}} and \mathrm{v}=\mathrm{x}^{\left(1+\frac{1}{\mathrm{x}}\right)}\)

y=u+v

⇒ \(\frac{d y}{d x} =\frac{d u}{d x}+\frac{d v}{d x}\)

u =\(\left(x+\frac{1}{x}\right)^x\)

Taking logarithms on both sides

⇒ \(\log u=\log \left(x+\frac{1}{x}\right)^x \Rightarrow \log u=x \log \left(x+\frac{1}{x}\right)\)

Differentiating both sides with respect to x, we obtain

⇒ \(\frac{1}{u} \cdot \frac{d u}{d x}=\frac{d}{d x}(x) \times \log \left(x+\frac{1}{x}\right)+x \times \frac{d}{d x}\left[\log \left(x+\frac{1}{x}\right)\right]\)

⇒ \(\frac{1}{u} \frac{d u}{d x}=1 \times \log \left(x+\frac{1}{x}\right)+x \times \frac{1}{\left(x+\frac{1}{x}\right)} \cdot \frac{d}{d x}\left(x+\frac{1}{x}\right)\)

⇒ \(\frac{d u}{d x}=u\left[\log \left(x+\frac{1}{x}\right)+\frac{x}{\left(x+\frac{1}{x}\right)} \times\left(1-\frac{1}{x^2}\right)\right]\)

⇒ \(\frac{d u}{d x}=\left(x+\frac{1}{x}\right)^x\left[\log \left(x+\frac{1}{x}\right)+\frac{\left(x-\frac{1}{x}\right)}{\left(x+\frac{1}{x}\right)}\right]\)

⇒ \(\frac{d u}{d x}=\left(x+\frac{1}{x}\right)^x\left[\log \left(x+\frac{1}{x}\right)+\frac{x^2-1}{x^2+1}\right]\)

⇒ \(\frac{d u}{d x}=\left(x+\frac{1}{x}\right)^x\left[\frac{x^2-1}{x^2+1}+\log \left(x+\frac{1}{x}\right)\right]\)

⇒ \(x^{\left(1+\frac{1}{x}\right)}\)

Taking logarithms on both the sides \(\log v=\log \left[x^{\left(1+\frac{1}{x}\right)}\right]\)

⇒ \(\log v=\left(1+\frac{1}{x}\right) \log x\) Differentiating both the sides with respect to x, we obtain

⇒ \(\frac{1}{v} \cdot \frac{d v}{d x}=\left[\frac{d}{d x}\left(1+\frac{1}{x}\right)\right] \times \log x+\left(1+\frac{1}{x}\right) \cdot \frac{d}{d x}(\log x)\)

⇒ \(\frac{1}{v} \frac{d v}{d x}=\left(-\frac{1}{x^2}\right) \log x+\left(1+\frac{1}{x}\right) \cdot \frac{1}{x} \Rightarrow \frac{1}{v} \frac{d v}{d x}=-\frac{\log x}{x^2}+\frac{1}{x}+\frac{1}{x^2}\)

⇒ \(\frac{d v}{d x}=v\left[\frac{-\log x+x+1}{x^2}\right] \Rightarrow \frac{d v}{d x}=x^{\left(1+\frac{1}{v}\right)}\left(\frac{x+1-\log x}{x^2}\right)\)

Therefore, from (1), (2), and (3), we obtain

∴ \(\frac{d y}{d x}=\left(x+\frac{1}{x}\right)^x\left[\frac{x^2-1}{x^2+1}+\log \left(x+\frac{1}{x}\right)\right]+x^{\left(1+\frac{1}{x}\right)}\left(\frac{x+1-\log x}{x^2}\right)\)

Question 7. \((\log x)^x+x^{\log x}\)

Solution:

Let y=\((\log x)^x+x^{\log x}\)

Also, let u=\((\log x)^x and v=x^{\log x}\)

y=u+v

⇒ \(\frac{d y}{d x}=\frac{d u}{d x}+\frac{d v}{d x}\)

u=\((\log x)^x\)

Taking logarithm on both the sides \(\log u=\log \left[(\log x)^x\right]\)

⇒ \(\log u=x \log (\log x)\) Differentiating both sides with respect to x, we obtain

⇒ \(\frac{1}{u} \frac{d u}{d x}=\frac{d}{d x}(x) \times \log (\log x)+x \cdot \frac{d}{d x}[\log (\log x)]\)

⇒ \(\frac{d u}{d x}=u\left[1 \times \log (\log x)+x \cdot \frac{1}{\log x} \cdot \frac{d}{d x}(\log x)\right]\)

⇒ \(\frac{d u}{d x}=(\log x)^x\left[\log (\log x)+\frac{x}{\log x} \cdot \frac{1}{x}\right]=(\log x)^x\left[\log (\log x)+\frac{1}{\log x}\right]\)

⇒ \(\frac{d u}{d x}=(\log x)^x\left[\frac{\log (\log x) \cdot \log x+1}{\log x}\right]=(\log x)^{x-1}[1+\log x \cdot \log (\log x)]\)

v=\(x^{\log x}\)

Taking logarithm on both the sides \(\log v=\log \left(x^{\log x}\right)\)

⇒ \(\log v=\log x \log x=(\log x)^2\)

Differentiating both sides w.r.t.x, we obtain

⇒ \(\frac{1}{v} \cdot \frac{d v}{d x}=\frac{d}{d x}\left[(\log x)^2\right]\)

⇒ \(\frac{1}{v} \cdot \frac{d v}{d x}=2(\log x) \cdot \frac{d}{d x}(\log x) \Rightarrow \frac{d v}{d x}=2 v(\log  x) \cdot \frac{1}{x}\)

⇒ \(\frac{d v}{d x}=2 x^{\log x} \frac{\log x}{x} \Rightarrow \frac{d v}{d x}=2 x^{\log x-1} \cdot \log x\)

Therefore, from (1), (2), and (3), we obtain

∴ \(\frac{d y}{d x}=(\log x)^{x-1}[1+\log x \cdot \log (\log x)]+2 x^{\log x-1} \cdot \log x\)

Question 8. \((\sin x)^x+\sin ^{-1} \sqrt{x}\)

Solution:

Let y=\((\sin x)^x+\sin ^{-1} \sqrt{x}\)

⇒Also, let u=\((\sin x)^x and v=\sin ^{-1} \sqrt{x}\)

\(\mathrm{y}=\mathrm{u}+\mathrm{v}\)

⇒ \(\frac{\mathrm{dy}}{\mathrm{dx}}=\frac{\mathrm{du}}{\mathrm{dx}}+\frac{\mathrm{dv}}{\mathrm{dx}}\)  → Equation 1

⇒ \(\mathrm{u}=(\sin \mathrm{x})^{\mathrm{x}}\)

Taking logarithm on both the sides \(\log \mathrm{u}=\log (\sin \mathrm{x})^{\mathrm{x}} \Rightarrow \log \mathrm{u}=\mathrm{x} \log (\sin \mathrm{x})\)

Differentiating both sides with respect to x, we obtain

⇒ \(\frac{1}{u} \frac{d u}{d x}=\frac{d}{d x}(x) \times \log (\sin x)+x \times \frac{d}{d x}[\log (\sin x)]\)

⇒ \(\frac{d u}{d x}=u\left[1 \cdot \log (\sin x)+x \cdot \frac{1}{\sin x} \cdot \frac{d}{d x}(\sin x)\right]\)

⇒ \(\frac{d u}{d x}=(\sin x)^x\left[\log (\sin x)+\frac{x}{\sin x} \cdot \cos x\right]=(\sin x)^x(x \cot x+\log \sin x)\)  → Equation 2

v=\(\sin ^{-1} \sqrt{x}\)

Differentiating both sides with respect to x, we obtain

⇒ \(\frac{\mathrm{dv}}{\mathrm{dx}}=\frac{1}{\sqrt{1-(\sqrt{\mathrm{x}})^2}} \cdot \frac{\mathrm{d}}{\mathrm{dx}}(\sqrt{\mathrm{x}})\)

=\(\frac{1}{\sqrt{1-\mathrm{x}}} \cdot \frac{1}{2 \sqrt{\mathrm{x}}}=\frac{1}{2 \sqrt{\mathrm{x}-\mathrm{x}^2}}\) ⇒ Equation 3

Therefore, from (1), (2), and (3), we obtain \(\frac{d y}{d x}=(\sin x)^x(x \cot x+\log \sin x)+\frac{1}{2 \sqrt{x-x^2}}\)

Question 9. \(x^{\sin x}+(\sin x)^{\cos x}\)

Solution:

Let y=\(x^{\sin x}+(\sin x)^{\cos x}\)

Also, let \(\mathrm{u}=\mathrm{x}^{\sin \mathrm{x}}\) and \(\mathrm{v}=(\sin \mathrm{x})^{\cos \mathrm{x}}\)

⇒ \(\mathrm{y}=\mathrm{u}+\mathrm{v}\)

⇒ \(\frac{d y}{d x}=\frac{d u}{d x}+\frac{d v}{d x}\) →  Equation 1

⇒ \(u=x^{\sin x}\)

Taking logarithm on both the sides \(\log \mathrm{u}=\log \left(\mathrm{x}^{\sin \mathrm{x}}\right)\)

⇒ \(\log \mathrm{u}=\sin \mathrm{x} \log \mathrm{x}\) Differentiating both sides with respect to x, we obtain

⇒ \(\frac{1}{\mathrm{u} d \mathrm{du}}=\frac{\mathrm{d}}{\mathrm{dx}}(\sin \mathrm{x}) \cdot \log \mathrm{x}+\sin x \cdot \frac{\mathrm{d}}{\mathrm{dx}}(\log x)\)

⇒ \(\frac{\mathrm{du}}{\mathrm{dx}}=\mathrm{u}\left[\cos x \log x+\sin x \cdot \frac{1}{x}\right]\)

⇒ \(\frac{\mathrm{du}}{\mathrm{dx}}=x^{\sin x}\left[\cos x \log x+\frac{\sin x}{x}\right]\) . Equation 2

⇒ \(\mathrm{v}=(\sin x)^{\cos x}\)

Taking logarithm on both the sides \(\log \mathrm{v}=\log (\sin x)^{\cos x} \log \mathrm{y}=\cos x \log (\sin x)\)

Differentiating both sides with respect to x, we obtain

⇒ \(\frac{1}{v d v}=\frac{d}{d x}(\cos x) \times \log (\sin x)+\cos x \times \frac{d}{d x}[\log (\sin x)]\)

⇒ \(\frac{d v}{d x}=v\left[-\sin x \cdot \log (\sin x)+\cos x \cdot \frac{1}{\sin x} \cdot \frac{d}{d x}(\sin x)\right]\)

⇒ \(\frac{d v}{d x}=(\sin x)^{\cos x}\left[-\sin x \log \sin x+\frac{\cos x}{\sin x} \cos x\right]=(\sin x)^{\cos x}[-\sin x \log \sin x+\cot x \cos x]\) →  Equation 3

⇒ \(\frac{d v}{d x}=(\sin x)^{\cos x}[\cot x \cos x-\sin x \log \sin x]]\)

From (1), (2), and (3), we obtain

∴ \(\frac{d y}{d x}=x^{\sin x}\left(\cos x \log x+\frac{\sin x}{x}\right)+(\sin x)^{\cos x}[\cos x \cot x-\sin x \log \sin x]\)

Question 10. \(x^{x \cos x}+\frac{x^2+1}{x^2-1}\)

Solution:

Let y=\(x^{x \cos x}+\frac{x^2+1}{x^2-1}\)

Also, let \(\mathrm{u}=\mathrm{x}^{\mathrm{x} \cos \mathrm{x}} and \mathrm{v}=\frac{\mathrm{x}^2+1}{\mathrm{x}^2-1}\)  → Equation 1

⇒ \(\mathrm{y}=\mathrm{u}+\mathrm{v}\)

⇒ \(\frac{\mathrm{dy}}{\mathrm{dx}}=\frac{\mathrm{du}}{\mathrm{dx}}+\frac{\mathrm{dv}}{\mathrm{dx}}\)

⇒ \(\mathrm{u}=\mathrm{x}^{\mathrm{xec} x}\)

Taking logarithm on both the sides \(\log \mathrm{u}=\log \left(\mathrm{x}^{\mathrm{x} {cosx}}\right)\)

⇒ \(\log \mathrm{u}=\mathrm{x} \cos \mathrm{x} \log \mathrm{x} \)

Differentiating both sides with respect to x, we obtain

⇒ \(\frac{1}{u} \frac{d u}{d x}=\frac{d}{d x}(x) \cdot \cos x \cdot \log x+x \cdot \frac{d}{d x}(\cos x) \cdot \log x+x \cos x \cdot \frac{d}{d x}(\log x)\)

⇒ \(\frac{d u}{d x}=u\left[1 \cdot \cos x \cdot \log x+x \cdot(-\sin x) \log x+x \cos x \cdot \frac{1}{x}\right]\)

⇒ \(\frac{d u}{d x}=x^{x \cos x}(\cos x \log x-x \sin x \log x+\cos x)\)  → Equation 2

⇒ \(\frac{d u}{d x}=x^{x \cos x}[\cos x(1+\log x)-x \sin x \log x]\)

v =\(\frac{x^2+1}{x^2-1}\)

Taking logarithm on both the sides \(\log v=\log \left(x^2+1\right)-\log \left(x^2-1\right)\)

Differentiating both sides with respect to x, we obtain

⇒ \(\frac{1}{v} \frac{d v}{d x}=\frac{2 x}{x^2+1}-\frac{2 x}{x^2-1}\)

⇒ \(\frac{d v}{d x}=v\left[\frac{2 x\left(x^2-1\right)-2 x\left(x^2+1\right)}{\left(x^2+1\right)\left(x^2-1\right)}\right]\)

⇒ \(\frac{d v}{d x}=\frac{x^2+1}{x^2-1} \times\left[\frac{-4 x}{\left(x^2+1\right)\left(x^2-1\right)}\right]\)

⇒ \(\frac{d v}{d x}=\frac{-4 x}{\left(x^2-1\right)^2}\)

Therefore, from (1), (2), and (3), we obtain

⇒ \(\frac{d y}{d x}=x^{x \cos x}[\cos x(1+\log x)-x \sin x \log x]-\frac{4 x}{\left(x^2-1\right)^2}\)

Question 11. \((x \cos x)^x+(x \sin x)^{\frac{1}{x}}\)

Solution:

Let y=\((x \cos x)^x+(x \sin x)^{\frac{1}{x}}\)

Also, let u=\((x \cos x)^x and v=(x \sin x)^{\frac{1}{x}}\)

⇒ \(\mathrm{y}=\mathrm{u}+\mathrm{v}\)

⇒ \(\frac{\mathrm{dy}}{\mathrm{dx}}=\frac{\mathrm{du}}{\mathrm{dx}}+\frac{\mathrm{dv}}{\mathrm{dx}}\)

⇒ \(\mathrm{u}=(\mathrm{x} \cos \mathrm{x})^{\mathrm{x}}\)(Differentiating both sides w.r.t. x, we obtain)

Taking logarithms on both sides

⇒ \(\log u =\log (x \cos x)^x\)

⇒ \(\log u =x \log (x \cos x) \Rightarrow \log u=x[\log x+\log \cos x] \log u=x \log x+x \log \cos x\)

Differentiating both sides with respect to x, we obtain

⇒ \(\frac{1}{\mathrm{u}}\)

⇒ \(\frac{\mathrm{du}}{\mathrm{dx}}=\frac{\mathrm{d}}{\mathrm{dx}}(\mathrm{x} \log \mathrm{x})+\frac{\mathrm{d}}{\mathrm{dx}}(\mathrm{x} \log \cos \mathrm{x})\) .

⇒ \(\frac{\mathrm{du}}{\mathrm{dx}}=\mathrm{u}\left[\left\{\log \mathrm{x} \cdot \frac{\mathrm{d}}{\mathrm{dx}}(\mathrm{x})+\mathrm{x} \cdot \frac{\mathrm{d}}{\mathrm{dx}}(\log \mathrm{x})\right\}+\left\{\log \cos \mathrm{x} \cdot \frac{\mathrm{d}}{\mathrm{dx}}(\mathrm{x})+\mathrm{x} \cdot \frac{\mathrm{d}}{\mathrm{dx}}(\log \cos \mathrm{x})\right\}\right]\)

⇒ \(\frac{\mathrm{du}}{\mathrm{dx}}=(\mathrm{x} \cos \mathrm{x})^{\mathrm{x}}\left[\left(\log \mathrm{x} \cdot 1+\mathrm{x} \cdot \frac{1}{\mathrm{x}}\right)+\left\{\log \cos \mathrm{x} \cdot 1+\mathrm{x} \cdot \frac{1}{\cos \mathrm{x}} \cdot \frac{\mathrm{d}}{\mathrm{dx}}(\cos \mathrm{x})\right\}\right]\)

⇒ \(\frac{\mathrm{du}}{\mathrm{dx}}=(\mathrm{x} \cos \mathrm{x})^{\mathrm{x}}\left[(\log \mathrm{x}+1)+\left\{\log \cos \mathrm{x}+\frac{\mathrm{x}}{\cos \mathrm{x}} \cdot(-\sin \mathrm{x})\right\}\right]\)

⇒ \(\frac{\mathrm{du}}{\mathrm{dx}}=(\mathrm{x} \cos \mathrm{x})^{\mathrm{x}}[(1+\log \mathrm{x})+(\log \cos \mathrm{x}-\mathrm{x} \tan \mathrm{x})]\)

⇒ \(\frac{\mathrm{du}}{\mathrm{dx}}=(\mathrm{x} \cos \mathrm{x})^{\mathrm{x}}[1-\mathrm{x} \tan \mathrm{x}+(\log \mathrm{x}+\log \cos \mathrm{x})]\)

⇒ \(\frac{d u}{d x}=(x \cos x)^x[1-x \tan x+\log (x \cos x)]\)

⇒ \(\mathrm{v}=(\mathrm{x} \sin \mathrm{x})^{\frac{1}{\mathrm{x}}}\)

Taking logarithms on both sides

⇒ \(\log \mathrm{v}=\log (\mathrm{x} \sin \mathrm{x})^{\frac{1}{\mathrm{x}}}\)

⇒ \(\log \mathrm{v}=\frac{1}{\mathrm{x}} \log (\mathrm{x} \sin \mathrm{x}) \Rightarrow \log \mathrm{v}\)

=\(\frac{1}{\mathrm{x}}(\log \mathrm{x}+\log \sin \mathrm{x})\)

⇒ \(\log \mathrm{v}=\frac{1}{\mathrm{x}} \log \mathrm{x}+\frac{1}{\mathrm{x}} \log \sin \mathrm{x}\)

Differentiating both sides with respect to x

⇒ \(\frac{1}{v} \frac{d v}{d x}=\frac{d}{d x}\left(\frac{1}{x} \log x\right)+\frac{d}{d x}\left[\frac{1}{x} \log (\sin x)\right]\)

⇒ \(\frac{1}{v} \frac{d v}{d x}=\left[\log x \cdot \frac{d}{d x}\left(\frac{1}{x}\right)+\frac{1}{x} \cdot \frac{d}{d x}(\log x)\right]+\left[\log (\sin x) \cdot \frac{d}{d x}\left(\frac{1}{x}\right)+\frac{1}{x} \cdot \frac{d}{d x}\{\log (\sin x)\}\right]\)

⇒ \(\frac{1}{v} \frac{d v}{d x}=\left[\log x \cdot\left(-\frac{1}{x^2}\right)+\frac{1}{x} \cdot \frac{1}{x}\right]+\left[\log (\sin x) \cdot\left(-\frac{1}{x^2}\right)+\frac{1}{x} \cdot \frac{1}{\sin x} \cdot \frac{d}{d x}(\sin x)\right]\)

⇒ \(\frac{1}{v} \frac{d v}{d x}=\frac{1}{x^2}(1-\log x)+\left[-\frac{\log (\sin x)}{x^2}+\frac{1}{x \sin x} \cdot \cos x\right]\)

⇒ \(\frac{d v}{d x}=(x \sin x)^{\frac{1}{x}}\left[\frac{1-\log x}{x^2}+\frac{-\log (\sin x)+x \cot x}{x^2}\right]\)

⇒ \(\frac{d v}{d x}=(x \sin x)^{\frac{1}{x}}\left[\frac{1-\log x-\log (\sin x)+x \cot x}{x^2}\right]\)

⇒ \(\frac{d v}{d x}=(x \sin x)^{\frac{1}{x}}\left[\frac{1-\log (x \sin x)+x \cot x}{x^2}\right]\)

From (1), (2), and (3), we obtain

∴ \(\frac{d y}{d x}=(x \cos x)^x[1-x \tan x+\log (x \cos x)]+(x \sin x)^{\frac{1}{x}}\left[\frac{x \cot x+1-\log (x \sin x)}{x^2}\right]\)

Question 12. \(x^y+y^x=1\)

Solution:

The given function is \(\mathrm{x}^y+\mathrm{y}^{\mathrm{x}}\)=1.

Let \(\mathrm{x}^{\mathrm{y}}=\mathrm{u} and\mathrm{y}^{\mathrm{x}}=\mathrm{v}\)

Then, the function becomes \mathrm{u}+\mathrm{v}=1

⇒ \(\frac{\mathrm{du}}{\mathrm{dx}}+\frac{\mathrm{dv}}{\mathrm{dx}}\)=0

⇒ \(\mathrm{u} =\mathrm{x}^{\mathrm{y}}\)

Taking logarithm on both the sides \(\log u=\log \left(x^y\right) \Rightarrow \log u=y \log x \)

Differentiating both sides with respect to x, we obtain

⇒ \(\mathrm{v}=\mathrm{y}^{\mathrm{x}}\)

⇒ \(\mathrm{v}=\mathrm{y}^{\mathrm{x}}\)

Taking logarithm on both the sides \(\log v=\log \left(y^x\right) \Rightarrow \log v=x \log y\)

Differentiating both sides with respect to x, we obtain

⇒ \(\frac{1 \mathrm{du}}{\mathrm{u} x}=\log x \frac{\mathrm{dy}}{\mathrm{dx}}+\mathrm{y} \cdot \frac{\mathrm{d}}{\mathrm{dx}}(\log \mathrm{x}) \Rightarrow \frac{\mathrm{du}}{\mathrm{dx}}\)

= \(\mathrm{u}\left[\log \mathrm{x} \frac{\mathrm{dy}}{\mathrm{dx}}+\mathrm{y} \cdot \frac{1}{\mathrm{x}}\right] \Rightarrow \frac{\mathrm{du}}{\mathrm{dx}}\)

= \(\mathrm{x}^y\left(\log \mathrm{x} \frac{\mathrm{dy}}{\mathrm{dx}}+\frac{\mathrm{y}}{\mathrm{x}}\right)\)

⇒ \(\mathrm{v}=\mathrm{y}^{\mathrm{x}}\)

Taking logarithm on both the sides \(\log v=\log \left(y^x\right) \Rightarrow \log v=x \log y\)

Differentiating both sides with respect to x, we obtain

⇒ \(\frac{1}{v} \cdot \frac{d v}{d x}=\log y \cdot \frac{d}{d x}(x)+x \cdot \frac{d}{d x}(\log y)\)

⇒ \(\frac{d v}{d x}=v\left(\log y \cdot 1+x \cdot \frac{1}{y} \cdot \frac{d y}{d x}\right)\)

⇒ \(\frac{d v}{d x}=y^x\left(\log y+\frac{x d y}{y}\right)\)

From (1), (2), and (3), we obtain

⇒ \(x^y\left(\log x \frac{d y}{d x}+\frac{y}{x}\right)+y^x\left(\log y+\frac{x d y}{y d x}\right)\)=0

⇒ \(\left(x^y \log x+x y^{x-1}\right) \frac{d y}{d x}=-\left(y x^{y-1}+y^x \log y\right)\)

∴ \(\frac{d y}{d x}=-\frac{y x^{y-1}+y^x \log y}{x^y \log x+x y^{x-1}}\)

Question 13. \(\mathrm{y}^{\mathrm{x}}=\mathrm{x}^y\)

Solution:

The given function is \(\mathrm{y}^{\mathrm{x}}=\mathrm{x}^y\)

Taking logarithm on both sides, we obtain \(\log y^\pi=\log x^5 \Rightarrow x \log y=y \log x\)

Differentiating both sides with respect to x, we obtain

⇒ \(\log y \cdot \frac{d}{d x}(x)+x \cdot \frac{d}{d x}(\log y)=\log x \cdot \frac{d}{d x}(y)+y \cdot \frac{d}{d x}(\log x)\)

⇒ \(\log y \cdot 1+x \cdot \frac{1}{y} \cdot \frac{d y}{d x}=\log x \cdot \frac{d y}{d x}+y \cdot \frac{1}{x}\)

⇒ \(\log y+\frac{x}{y} \frac{d y}{d x}=\log x \frac{d y}{d x}+\frac{y}{x}\)

⇒ \(\left(\frac{x}{y}-\log x\right) \frac{d y}{d x}=\frac{y}{x}-\log y \Rightarrow\left(\frac{x-y \log x}{y}\right) \frac{d y}{d x}=\frac{y-x \log y}{x}\)

∴ \(\frac{d y}{d x}=\frac{y}{x}\left(\frac{y-x \log y}{x-y \log x}\right)\)

Question 14. \((\cos x)^y=(\cos y)^x\)

Solution:

The given function is \((\cos x)^y=(\cos y)^x\)

Taking logarithms on both sides, we obtain

y \(\log \cos x=x \log \cos y\)

⇒ \(\log \cos x \cdot \frac{d y}{d x}+y \cdot \frac{d}{d x}(\log \cos x)=\log \cos y \cdot \frac{d}{d x}(x)+x \cdot \frac{d}{d x}(\log \cos y)\)

⇒ \(\log \cos x \frac{d y}{d x}+y \cdot \frac{1}{\cos x} \cdot \frac{d}{d x}(\cos x)=\log \cos y \cdot 1+x \cdot \frac{1}{\cos y} \cdot \frac{d}{d x}(\cos y)\)

⇒ \(\log \cos x \frac{d y}{d x}+\frac{y}{\cos x} \cdot(-\sin x)=\log \cos y+\frac{x}{\cos y}(-\sin y) \cdot \frac{d y}{d x}\)

⇒ \(\log \cos x \frac{d y}{d x}-y \tan x=\log \cos y-x \tan y \frac{d y}{d x}\)

⇒ \((\log \cos x+x \tan y) \frac{d y}{d x}=y \tan x+\log \cos y\)

∴ \(\frac{d y}{d x}=\frac{y \tan x+\log \cos y}{x \tan y+\log \cos x}\)

Question 15. \(x y=e^{(x-y)}\)

Solution:

The given function is x y=\(e^{(x-y)}\)

⇒ Taking logarithm on both the sides \(\log (x y)=\log \left(e^{x-y}\right)\)

⇒ \(\log x+\log y=(x-y) \log e \Rightarrow \log x+\log y=(x-y) \times 1\)

⇒ \(\log x+\log y=x-y\)

Differentiating both sides with respect to x, we obtain

⇒ \(\frac{d}{d x}(\log x)+\frac{d}{d x}(\log y)=\frac{d}{d x}(x)-\frac{d y}{d x}\)

⇒ \(\frac{1}{x}+\frac{1}{y} \frac{d y}{d x}=1-\frac{d y}{d x}\)

⇒ \(\left(1+\frac{1}{y}\right) \frac{d y}{d x}=1-\frac{1}{x}\)

⇒ \(\left(\frac{y+1}{y}\right) \frac{d y}{d x}=\frac{x-1}{x}\)

∴ \(\frac{d y}{d x}=\frac{y(x-1)}{x(y+1)}\)

Equation 16. Find the derivative of the function given by \(f(x)=(1+x)\left(1+x^2\right)\left(1+x^4\right)\left(1+x^8\right)\) and hence find \(\mathrm{f}^{\prime}(1)\).

Solution:

The given relationship is \(f(x)=(1+x)\left(1+x^2\right)\left(1+x^4\right)\left(1+x^3\right)\)

Taking logarithms on both sides

⇒ \(\log f(x)=\log (1+x)+\log \left(1+x^2\right)+\log \left(1+x^4\right)+\log \left(1+x^x\right)\)

Differentiating both sides with respect to x, we obtain

⇒ \(\frac{1}{f(x)} \cdot \frac{d}{d x}[f(x)]=\frac{d}{d x} \log (1+x)+\frac{d}{d x} \log \left(1+x^2\right)+\frac{d}{d x} \log \left(1+x^4\right)+\frac{d}{d x} \log \left(1+x^8\right)\)

⇒ \(\frac{1}{f(x)} \cdot f^{\prime}(x)=\frac{1}{1+x} \cdot \frac{d}{d x}(1+x)+\frac{1}{1+x^2} \cdot \frac{d}{d x}\left(1+x^2\right)+\frac{1}{1+x^4} \cdot \frac{d}{d x}\left(1+x^4\right)+\frac{1}{1+x^8} \cdot \frac{d}{d x}\left(1+x^3\right)\)

⇒ \(f^{\prime}(x)=f(x)\left[\frac{1}{1+x}+\frac{1}{1+x^2} \cdot 2 x+\frac{1}{1+x^4} \cdot 4 x^3+\frac{1}{1+x^3} \cdot 8 x^7\right]\)

⇒ \(f^{\prime}(x)=(1+x)\left(1+x^2\right)\left(1+x^4\right)\left(1+x^8\right)\left[\frac{1}{1+x}+\frac{2 x}{1+x^7}+\frac{4 x^3}{1+x^4}+\frac{8 x^7}{1+x^5}\right]\)

Hence,\(f^{\prime}(1)=(1+1)\left(1+1^2\right)\left(1+1^4\right)\left(1+1^3\right)\left[\frac{1}{1+1}+\frac{2 \times 1}{1+1^2}+\frac{4 \times 1^3}{1+1^4}+\frac{8 \times 1^7}{1+1^1}\right]\)

=\(2 \times 2 \times 2 \times 2\left[\frac{1}{2}+\frac{2}{2}+\frac{4}{2}+\frac{8}{2}\right]=16 \times\left(\frac{1+2+4+8}{2}\right)=16 \times \frac{15}{2}=120\)

Question 17. If u, v and w are functions of x, then show that \(\frac{d}{d x}(u, v, w)=\frac{d u}{d x} v \cdot w+u, \frac{d v}{d x}, w+u, v, \frac{d w}{d x}\) in two ways-first by repeated application of product rule, second by logarithmic differentiation.

Solution:

Let y= u.v.w. =\(\mathrm{u} .(\mathrm{v} . \mathrm{w})\)

By applying the product rule, we obtain

⇒ \(\frac{d y}{d x}=\frac{d u}{d x} \cdot(v \cdot w)+u \cdot \frac{d}{d x}(v, w)\)

⇒ \(\frac{d y}{d x}=\frac{d u}{d x} v \cdot w+u\left[\frac{d v}{d x} \cdot w+v \cdot \frac{d w}{d x}\right]\) (Again applying product rule)

⇒ \(\frac{d y}{d x}=\frac{d u}{d x} \cdot v \cdot w+u \cdot \frac{d v}{d x} \cdot w+u \cdot v \cdot \frac{d w}{d x}\)

By taking logarithm on both sides of the equation y = u.v.w, we obtain

⇒ \(\log y=\log u+\log v+\log w\)

Differentiating both sides with respect to x, we obtain

⇒ \(\frac{1}{y} \cdot \frac{d y}{d x}=\frac{d}{d x}(\log u)+\frac{d}{d x}(\log v)+\frac{d}{d x}(\log w)\)

⇒ \(\frac{1}{y} \cdot \frac{d y}{d x}=\frac{1}{u} \frac{d u}{d x}+\frac{1}{v} \frac{d v}{d x}+\frac{1}{w} \frac{d w}{d x}\)

⇒ \(\frac{d y}{d x}=y\left(\frac{1}{u} \frac{d u}{d x}+\frac{1}{v} \frac{d v}{d x}+\frac{1}{w} \frac{d w}{d x}\right)\)

⇒ \(\frac{d y}{d x}=u \cdot v \cdot w \cdot\left(\frac{1}{u d x}+\frac{1 d v}{v d x}+\frac{1}{w} \frac{d w}{d x}\right)\)

∴ \(\frac{d y}{d x}=\frac{d u}{d x} \cdot v \cdot w+u \cdot \frac{d v}{d x} \cdot w+u \cdot v \cdot \frac{d w}{d x}\)

Continuity And Differentiability Exercise 5.6

Question 1. \(\mathrm{x}=2 \mathrm{at} \mathrm{t}^2, \mathrm{y}=\mathrm{at}^4\)

Solution:

The given equations are \(\mathrm{x}=2 \mathrm{at}^2\) and \(\mathrm{y}=a \mathrm{t}^4\)

Then, \(\frac{\mathrm{dx}}{\mathrm{dt}}=\frac{\mathrm{d}}{\mathrm{dt}}\left(2 \mathrm{at}^2\right)=2 \mathrm{a} \cdot \frac{\mathrm{d}}{\mathrm{dt}}\left(\mathrm{t}^2\right)\)

=\(2 \mathrm{a} \cdot 2 \mathrm{t}=4 \mathrm{at}\) and \(\frac{\mathrm{dy}}{\mathrm{dt}}=\frac{\mathrm{d}}{\mathrm{dt}}\left(\mathrm{at}^4\right)\)

=\(\mathrm{a} \cdot \frac{\mathrm{d}}{\mathrm{dt}}\left(\mathrm{t}^4\right)=\mathrm{a} \cdot 4 \cdot \mathrm{t}^3=4 \mathrm{at}^5\)

∴ \(\frac{d y}{d x}=\frac{\left(\frac{d y}{d t}\right)}{\left(\frac{d x}{d t}\right)}=\frac{4 a t^3}{4 a t}=t^2\)

Question 2. \(x=a \cos \theta, y=b \cos \theta\)

Solution:

The given equations are x=a \(\cos \theta and y=b \cos \theta\)

Then, \(\frac{d x}{d \theta}=\frac{d}{d \theta}(a \cos \theta)=a(-\sin \theta)=-a \sin \theta and \frac{d y}{d \theta}\)

=\(\frac{d}{d \theta}(b \cos \theta)=b(-\sin \theta)=-b \sin \theta\)

∴ \(\frac{d y}{d x}=\frac{\left(\frac{d y}{d \theta}\right)}{\left(\frac{d x}{d \theta}\right)}=\frac{-b \sin \theta}{-a \sin \theta}=\frac{b}{a}\)

Question 3. x=\(\sin t, y=\cos\) 2 t

Solution:

The given equations are x=\(\sin t and y=\cos\) 2 t

Then, \(\frac{\mathrm{dx}}{\mathrm{dt}}=\frac{\mathrm{d}}{\mathrm{dt}}(\sin \mathrm{t})=\cos \mathrm{t}\) and

⇒ \(\frac{\mathrm{dy}}{\mathrm{dt}}=\frac{\mathrm{d}}{\mathrm{dt}}(\cos 2 \mathrm{t})=-\sin 2 t \cdot \frac{\mathrm{d}}{\mathrm{dt}}(2 \mathrm{t})=-2 \sin 2 \mathrm{t}\)

∴ \(\frac{d y}{d x}=\frac{\left(\frac{d y}{d t}\right)}{\left(\frac{d x}{d t}\right)}=\frac{-2 \sin 2 t}{\cos t}=\frac{-2 \cdot 2 \sin t \cos t}{\cos t}=-4 \sin t\)

Question 4. x=4 t, y=\(\frac{4}{t}\)

Solution:

The given equations are x=4 t and y=\(\frac{4}{t}\)

⇒ \(\frac{d x}{d t}=\frac{d}{d t}(4 t)=4 and \frac{d y}{d t}=\frac{d}{d t}\left(\frac{4}{t}\right)\)

=\(4 \cdot \frac{d}{d t}\left(\frac{1}{t}\right)=4 \cdot\left(\frac{-1}{t^2}\right)=\frac{-4}{t^2}\)

∴ \(\frac{d y}{d x}=\frac{\left(\frac{d y}{d t}\right)}{\left(\frac{d x}{d t}\right)}=\frac{\left(\frac{-4}{t^2}\right)}{4}=\frac{-1}{t^2}\)

Question 5. x=\(\cos \theta-\cos 2 \theta, y=\sin \theta-\sin 2 \theta\)

Solution:

The given equations are x=\(\cos \theta-\cos 2 \theta and y=\sin \theta-\sin 2 \theta\)

Then,\(\frac{d x}{d \theta}=\frac{d}{d \theta}(\cos \theta-\cos 2 \theta)=\frac{d}{d \theta}(\cos \theta)-\frac{d}{d \theta}(\cos 2 \theta)\)

=-\(\sin \theta-(-2 \sin 2 \theta)=2 \sin 2 \theta-\sin \theta\)

and \(\frac{d y}{d \theta} =\frac{d}{d \theta}(\sin \theta-\sin 2 \theta)=\frac{d}{d \theta}(\sin \theta)-\frac{d}{d \theta}(\sin 2 \theta)\)

=\(\cos \theta-2 \cos 2 \theta\)

\(\frac{d y}{d x} =\frac{\left(\frac{d y}{d \theta}\right)}{\left(\frac{d x}{d \theta}\right)}\)

= \(\frac{\cos \theta-2 \cos 2 \theta}{2 \sin 2 \theta-\sin \theta}\)

Question 6. x=a\((\theta-\sin \theta), y=a(1+\cos \theta)\)

Solution:

The given equations are x=a\((\theta-\sin \theta) and y=a(1+\cos \theta)\)

Then, \(\frac{d x}{d \theta}=a\left[\frac{d}{d \theta}(\theta)-\frac{d}{d \theta}(\sin \theta)\right]=a(1-\cos \theta)\)

and \(\left.\frac{d y}{d \theta}=a\left[\frac{d}{d \theta}(1)+\frac{d}{d \theta}(\cos \theta)\right]=a[0+(-\sin \theta)]\right]=-a \sin \theta\)

⇒ \(\frac{d y}{d x}=\frac{\left(\frac{d y}{d \theta}\right.}{\left(\frac{d x}{d \theta}\right)}\)

=\(\frac{-a \sin \theta}{a(1-\cos \theta)}=\frac{-2 \sin \frac{\theta}{2} \cos \frac{\theta}{2}}{2 \sin ^2 \frac{\theta}{2}}\)

= \(\frac{-\cos \frac{\theta}{2}}{\sin \frac{\theta}{2}}=-\cot \frac{\theta}{2}\)

Question 7. x=\(\frac{\sin ^3 t}{\sqrt{\cos 2 t}}, y=\frac{\cos ^3 t}{\sqrt{\cos 2 t}}\)

Solution:

The given equations are x=\(\frac{\sin ^3 t}{\sqrt{\cos 2 t}} and y=\frac{\cos ^3 t}{\sqrt{\cos 2 t}}\)

Then,\(\frac{d x}{d t}=\frac{d}{d t}\left[\frac{\sin ^3 t}{\sqrt{\cos 2 t}}\right]\)

=\(\frac{\sqrt{\cos 2 t} \cdot \frac{d}{d t}\left(\sin ^3 t\right)-\sin ^3 t \cdot \frac{d}{d t} \sqrt{\cos 2 t}}{(\sqrt{\cos 2 t})^2}\)

=\(\frac{\sqrt{\cos 2 t} \cdot 3 \sin ^2 t \cdot \frac{d}{d t}(\sin t)-\sin ^3 t \times \frac{1}{2 \sqrt{\cos 2 t}} \cdot \frac{d}{d t}(\cos 2 t)}{\cos 2 t}\)

=\(\frac{3 \sqrt{\cos 2 t} \cdot \sin ^2 t \cos t-\frac{\sin ^3 t}{2 \sqrt{\cos 2 t}} \cdot(-2 \sin 2 t)}{\cos 2 t}\)

= \(\frac{3 \cos 2 t \sin ^2 t \cos t+\sin ^3 t \sin 2 t}{\cos ^2 t \sqrt{\cos 2 t}}\)

and \(\frac{d y}{d t} =\frac{d}{d t}\left[\frac{\cos ^3 t}{\sqrt{\cos 2 t}}\right]=\frac{\sqrt{\cos 2 t} \cdot \frac{d}{d t}\left(\cos ^3 t\right)-\cos ^3 t \cdot \frac{d}{d t}(\sqrt{\cos 2 t})}{\cos 2 t}\)

= \(\frac{\sqrt{\cos 2 t} \cdot 3 \cos ^2 t \cdot \frac{d}{d t}(\cos t)-\cos ^3 t \cdot \frac{1}{2 \sqrt{\cos 2 t}} \cdot \frac{d}{d t}(\cos 2 t)}{\cos 2 t}\)

=\(\frac{3 \sqrt{\cos 2 t} \cdot \cos ^2 t(-\sin t)-\cos ^3 t \cdot \frac{1}{2 \sqrt{\cos 2 t}} \cdot(-2 \sin 2 t)}{\cos 2 t}\)

=\(\frac{-3 \cos 2 t \cdot \cos ^2 t \cdot \sin t+\cos ^3 t \sin 2 t}{\cos ^2 t \cdot \sqrt{\cos 2 t}}\)

\(\frac{d y}{d x} =\frac{\left(\frac{d y}{d t}\right)}{\left(\frac{d x}{d t}\right)}=\frac{-3 \cos 2 t \cdot \cos ^2 t \cdot \sin t+\cos ^3 t \sin 2 t}{3 \cos 2 t \sin ^2 t \cos t+\sin ^3 t \sin 2 t}\)

=\(\frac{-3 \cos 2 t \cdot \cos ^2 t \cdot \sin t+\cos ^2 t(2 \sin t \cos t)}{3 \cos 2 t \sin ^2 t \cos t+\sin ^3 t(2 \sin t \cos t)}\)

=\(\frac{\sin t \cos t\left[-3 \cos 2 t \cdot \cos t+2 \cos ^3 t\right]}{\sin t \cos t\left[3 \cos 2 t \sin t+2 \sin ^3 t\right]}\)

=\(\frac{\left[-3\left(2 \cos ^2 t-1\right) \cos t+2 \cos ^3 t\right]}{\left[3\left(1-2 \sin ^2 t\right) \sin t+2 \sin ^3 t\right]}\)

⇒ \(\left[\begin{array}{l}
\cos 2 t=\left(2 \cos ^2 t-1\right)
\cos 2 t=\left(1-2 \sin ^2 t\right)
\end{array}\right]\)

=\(\frac{-4 \cos ^3 t+3 \cos t}{3 \sin t-4 \sin ^3 t}\)

∴ \(\frac{d y}{d x}=\frac{-\cos 3 t}{\sin 3 t}\left[\begin{array}{l}
\cos 3 t=4 \cos ^3 t-3 \cos t \sin 3 t=3 \sin t-4 \sin ^3 t
\end{array}\right]=-\cot 3 t\)

Question 8. x=a\(\left(\cos t+\log \tan \frac{t}{2}\right), y=a \sin t\)

Solution:

The given equations are x=a\(\left(\cos t+\log \tan \frac{t}{2}\right) and y=a \sin t\)

Then, \(\frac{\mathrm{dx}}{\mathrm{dt}}=\mathrm{a} \cdot\left[\frac{\mathrm{d}}{\mathrm{dt}}(\cos \mathrm{t})+\frac{\mathrm{d}}{\mathrm{dt}}\left(\log \tan \frac{\mathrm{t}}{2}\right)\right]\)

=\(\mathrm{a}\left[-\sin \mathrm{t}+\frac{\mathrm{t}}{\tan \frac{\mathrm{t}}{2}} \cdot \frac{\mathrm{d}}{\mathrm{dt}}\left(\tan \frac{\mathrm{t}}{2}\right)\right]\)

=a\(\left[-\sin t+\cot \frac{t}{2} \cdot \sec ^2 \frac{t}{2} \cdot \frac{d}{d t}\left(\frac{t}{2}\right)\right]=a\left[-\sin t+\frac{\cos \frac{t}{2}}{\sin \frac{t}{2}} \times \frac{1}{\cos ^2 \frac{t}{2}} \times \frac{1}{2}\right]\)

=a\(\left[-\sin t+\frac{1}{2 \sin \frac{t}{2} \cos \frac{t}{2}}\right]\)

=a\(\left(-\sin t+\frac{1}{\sin t}\right)=a\left(\frac{-\sin ^2 t+1}{\sin t}\right)\)

=a\(\left(\frac{\cos ^2 t}{\sin t}\right)\)

and \(\frac{d y}{d t}=a \frac{d}{d t}(\sin t)=a \cos t\)

∴ \(\frac{d y}{d x}=\frac{\left(\frac{d y}{d t}\right)}{\left(\frac{d x}{d t}\right)}=\frac{a \cos t}{\left(a \frac{\cos ^2 t}{\sin t}\right)}=\frac{\sin t}{\cos t}=\tan t\)

Question 9. x=a \(\sec \theta, y=b \tan \theta\)

Solution:

The given equations are x=a \(\sec \theta and y=b \tan \theta\)

Then, \(\frac{d x}{d \theta}=a \cdot \frac{d}{d \theta}(\sec \theta)=a \sec \theta \tan \theta\)

⇒ \(\frac{d y}{d \theta}=b \cdot \frac{d}{d \theta}(\tan \theta)=b \sec ^2 \theta\)

⇒ \(\frac{d y}{d x}=\frac{\left(\frac{d y}{d \theta}\right)}{\left(\frac{d x}{d \theta}\right)}\)

= \(\frac{b \sec ^2 \theta}{{asec} \theta \tan \theta}=\frac{b}{a} \sec \theta \cot \theta\)

= \(\frac{b \cos \theta}{a \cos \theta \sin \theta}=\frac{b}{a} \times \frac{1}{\sin \theta}=\frac{b}{a} {cosec} \theta\)

Question 10. x=a\((\cos \theta+\theta \sin \theta), y=a(\sin \theta-\theta \cos \theta)\)

Solution:

The given equations are x=a\((\cos \theta+\theta \sin \theta) and y=a(\sin \theta-\theta \cos \theta)\)

Then, \(\frac{d x}{d \theta} =a\left[\frac{d}{d \theta}(\cos \theta)+\frac{d}{d \theta}(\theta \sin \theta)\right]\)

=a\(\left[-\sin \theta+\theta \frac{d}{d \theta}(\sin \theta)+\sin \theta \frac{d}{d \theta}(\theta)\right]\)

=\(a(-\sin \theta+\theta \cos \theta+\sin \theta)=a \theta \cos \theta \)

and \(\frac{d y}{d \theta} =a\left[\frac{d}{d \theta}(\sin \theta)-\frac{d}{d \theta}(\theta \cos \theta)\right]\)

=\(a\left[\cos \theta-\left\{\theta \frac{d}{d \theta}(\cos \theta)+\cos \theta \cdot \frac{d}{d \theta}(\theta)\right\}\right]\)

=\(a[\cos \theta+\theta \sin \theta-\cos \theta]=a \theta \sin \theta \)

∴ \(\frac{d y}{d x} =\frac{\left(\frac{d y}{d \theta}\right)}{\left(\frac{d x}{d \theta}\right)}=\frac{a \theta \sin \theta}{a \theta \cos \theta}=\tan \theta\)

Question 11. If x=\(\sqrt{a^{\sin ^{-1} 1}}\), y=\(\sqrt{a^{\alpha x^{-1} t}}\), show that \(\frac{d y}{d x}=-\frac{y}{x}\) ?

Solution:

The given equations are x=\(\sqrt{a^{\sin ^{-1} t}}\) and y=\(\sqrt{a^{\cos ^{-1} t}}\)

x=\(\left(a^{\sin ^{-1} t}\right)^{\frac{1}{2}}\) and y=\(\left(a^{\cos ^{-1} t}\right)^{\frac{1}{2}}\)

x=\(a^{\frac{1}{2} {tin}^{-1} t}\) and y=\(a^{\frac{1}{2} \cos ^{-1} t}\)

Consider \(\mathrm{x}=\mathrm{a}^{\frac{1}{2} \sin ^{-1} \mathrm{t}}\)

Taking logarithms on both sides, we obtain

⇒ \(\log x=\frac{1}{2} \sin ^{-1} t \log a\)

⇒ \(\frac{1}{x} \cdot \frac{d x}{d t}=\frac{1}{2} \log a \cdot \frac{d}{d t}\left(\sin ^{-1} t\right)\)

⇒ \(\frac{d x}{d t}=\frac{x}{2} \log a \cdot \frac{1}{\sqrt{1-t^2}}\)

⇒ \(\frac{d x}{d t}=\frac{x \log a}{2 \sqrt{1-t^2}}\)

Then, consider y=\(a^{\frac{1}{2} \cos ^{-1} t}\)

Taking logarithms on both sides, we obtain

⇒ \(\log y=\frac{1}{2} \cos ^{-1} t \log a\)

⇒ \(\frac{1}{y} \cdot \frac{d y}{d t}=\frac{1}{2} \log a \cdot \frac{d}{d t}\left(\cos ^{-1} t\right)\)

⇒ \(\frac{d y}{d t}=\frac{y \log a}{2} \cdot\left(\frac{-1}{\sqrt{1-t^2}}\right)\)

⇒ \(\frac{d y}{d t}=\frac{-y \log a}{2 \sqrt{1-t^2}}\)

⇒ \(\frac{d y}{d x}=\frac{\left(\frac{d y}{d t}\right)}{\left(\frac{d x}{d t}\right)}\)

=\(\frac{\left(\frac{-y \log a}{2 \sqrt{1-t^2}}\right)}{\left(\frac{x \log a}{2 \sqrt{1-t^2}}\right)}=-\frac{y}{x}\)

Hence, proved.

Continuity And Differentiability Exercise 5.7

Question 1. \(x^2+3 x+2\)

Solution:

Let y=\(x^2+3 x+2\)

Differentiating w.r.t. x on both the sides

Then,\(\frac{\mathrm{dy}}{\mathrm{dx}}=\frac{\mathrm{d}}{\mathrm{dx}}\left(\mathrm{x}^2\right)+\frac{\mathrm{d}}{\mathrm{dx}}(3 \mathrm{x})+\frac{\mathrm{d}}{\mathrm{dx}}(2)\)

=2 \(\mathrm{x}+3+0=2 \mathrm{x}+3\)

Again differentiating with respect to x on both sides

⇒ \(\frac{d^2 y}{d x^2}=\frac{d}{d x}(2 x+3)=\frac{d}{d x}(2 x)+\frac{d}{d x}(3)=2+0=2\)

Question 2. \(\mathrm{x}^{20}\)

Solution:

Let \(\mathrm{y}=\mathrm{x}^{20}\)

Then,\(\frac{\mathrm{dy}}{\mathrm{dx}}=\frac{\mathrm{d}}{\mathrm{dx}}\left(\mathrm{x}^{20}\right)=20 \mathrm{x}^{19}\)

Again differentiating with respect to x on both sides

∴ \(\frac{d^2 y}{d x^2}=\frac{d}{d x}\left(20 x^{19}\right)=20 \frac{d}{d x}\left(x^{19}\right)=20 \cdot 19 \cdot x^{18}=380 x^{18}\)

Question 3. x \(\cdot \cos x\)

Solution:

Let y=x \(\cdot \cos x\)

Then, \(\frac{d y}{d x}=\frac{d}{d x}(x \cdot \cos x)=\cos x \cdot \frac{d}{d x}(x)+x \frac{d}{d x}(\cos x)=\cos x \cdot 1+x(-\sin x)=\cos x-x \sin x\)

Again differentiating with respect to x on both sides

⇒ \(\frac{d^2 y}{d x^2} =\frac{d}{d x}[\cos x-x \sin x]=\frac{d}{d x}(\cos x)-\frac{d}{d x}(x \sin x)\)

=-\(\sin x-\left[\sin x \cdot \frac{d}{d x}(x)+x \cdot \frac{d}{d x}(\sin x)\right]=-\sin x-(\sin x+x \cos x)=-(x \cos x+2 \sin x)\)

Question 4. log x

Solution:

Let y=\(\log \mathrm{x}\)

Then, \(\frac{d y}{d x}=\frac{d}{d x}(\log x)=\frac{1}{x}\)

Again differentiating with respect to x on both sides

⇒ \(\frac{d^2 y}{d x^2}=\frac{d}{d x}\left(\frac{1}{x}\right)=\frac{-1}{x^2}\)

Question 5. \(x^3 \log x\)

Solution:

Let y=\(x^3 \log x\)

Then,\(\frac{d y}{d x} =\frac{d}{d x}\left[x^3 \log x\right]=\log x \cdot \frac{d}{d x}\left(x^3\right)+x^3 \cdot \frac{d}{d x}(\log x)\)

=\(\log x \cdot 3 x^2+x^3 \cdot \frac{1}{x}=\log x \cdot 3 x^2+x^2=x^2(1+3 \log x)\)

Again differentiating with respect to x on both sides

⇒ \(\frac{d^2 y}{d x^2} =\frac{d}{d x}\left[x^2(1+3 \log x)\right]=(1+3 \log x) \cdot \frac{d}{d x}\left(x^2\right)+x^2 \frac{d}{d x}(1+3 \log x)\)

= \((1+3 \log x) \cdot 2 x+x^2 \cdot \frac{3}{x}\)

= 2 x+6 x \(\log x+3 x=5 x+6 x \log x=x(5+6 \log x)\)

Question 6. \(e^x \sin 5 x\)

Solution:

Let y=\(e^x \sin 5 x\)

⇒ \(\frac{d y}{d x}= \frac{d}{d x}\left(e^x \sin 5 x\right)\)

=\(\sin 5 x \cdot \frac{d}{d x}\left(e^x\right)+e^x \frac{d}{d x}(\sin 5 x)\)

=\(\sin 5 x \cdot e^x+e^x \cdot \cos 5 x \cdot \frac{d}{d x}(5 x)\)

=\(e^x \sin 5 x+e^x \cos 5 x \cdot 5=e^x(\sin 5 x+5 \cos 5 x)\)

Again differentiating with respect to x on both sides

⇒ \(\frac{d^2 y}{d x^2} =\frac{d}{d x}\left[e^x(\sin 5 x+5 \cos 5 x)\right] \)

=\((\sin 5 x+5 \cos 5 x) \cdot \frac{d}{d x}\left(e^x\right)+e^x \cdot \frac{d}{d x}(\sin 5 x+5 \cos 5 x)\)

= \((\sin 5 x+5 \cos 5 x) e^x+e^x\left[\cos 5 x \cdot \frac{d}{d x}(5 x)+5(-\sin 5 x) \cdot \frac{d}{d x}(5 x)\right]\)

=\(e^x(\sin 5 x+5 \cos 5 x)+e^x(5 \cos 5 x-25 \sin 5 x)\)

=\(e^x(10 \cos 5 x-24 \sin 5 x)=2 e^x(5 \cos 5 x-12 \sin 5 x)\)

Question 7. \(e^{4 x} \cos 3 x\)

Solution:

Let y=\(e^{6 x} \cos 3 x\)

Then, \(\frac{d y}{d x}=\frac{d}{d x}\left(e^{6 x} \cdot \cos 3 x\right)\)

=\(\cos 3 x \cdot \frac{d}{d x}\left(e^{6 x}\right)+e^{4 x} \cdot \frac{d}{d x}(\cos 3 x)\)

=\(\cos 3 x \cdot e^{6 x} \cdot \frac{d}{d x}(6 x)+e^{6 x} \cdot(-\sin 3 x) \cdot \frac{d}{d x}(3 x)\)

=\(6 e^{6 x} \cos 3 x-3 e^{6 x} \sin 3 x\)

Again differentiating with respect to x on both sides

⇒ \(\frac{d^2 y}{d x^2} =\frac{d}{d x}\left(6 e^{6 x} \cos 3 x-3 e^{6 x} \sin 3 x\right)\)

= \(6 \cdot \frac{d}{d x}\left(e^{6 x} \cos 3 x\right)-3 \cdot \frac{d}{d x}\left(e^{6 x} \sin 3 x\right)\)

=\(6 \cdot\left[6 e^{6 x} \cos 3 x-3 e^{6 x} \sin 3 x\right]-3 \cdot\left[\sin 3 x \cdot \frac{d}{d x}\left(e^{6 x}\right)+e^{6 x} \cdot \frac{d}{d x}(\sin 3 x)\right] [U {sing}(1)]\)

=\(36 e^{6 x} \cos 3 x-18 e^{6 x} \sin 3 x-3\left[\sin 3 x \cdot e^{6 x} \cdot 6+e^{6 x} \cdot \cos 3 x \cdot 3\right]\)

=\(36 e^{6 x} \cos 3 x-18 e^{6 x} \sin 3 x-18 e^{6 x} \sin 3 x-9 e^{6 x} \cos 3 x\)

=\(27 e^{6 x} \cos 3 x-36 e^{6 x} \sin 3 x=9 e^{6 x}(3 \cos 3 x-4 \sin 3 x)\)

Question 8. \(\tan ^{-1} x\)

Solution:

Let y=\(\tan ^{-1} x\)

Then, \(\frac{d y}{d x}=\frac{d}{d x}\left(\tan ^{-1} x\right)=\frac{1}{1+x^2}\)

Again differentiating with respect to x on both sides, use obtain

⇒ \(\frac{d^2 y}{d x^2}=\frac{d}{d x}\left(\frac{1}{1+x^2}\right)=\frac{d}{d x}\left(1+x^2\right)^{-1}\)

= \((-1) \cdot\left(1+x^2\right)^{-2} \cdot \frac{d}{d x}\left(1+x^2\right)\)

= \(\frac{-1}{\left(1+x^2\right)^2} \times 2 x=\frac{-2 x}{\left(1+x^2\right)^2}\)

Question 9. \(\log (\log x)\)

Solution:

Let y=\(\log (\log x)\)

Then, \(\frac{d y}{d x}=\frac{d}{d x}[\log (\log x)]=\frac{1}{\log x} \cdot \frac{d}{d x}(\log x)\)

=\(\frac{1}{x \log x}=(x \log x)^{-1}\)

Again differentiating with respect to x on both sides

⇒ \(\frac{d^2 y}{d x^2}=\frac{d}{d x}\left[(x \log x)^{-1}\right]=(-1) \cdot(x \log x)^{-2} \cdot \frac{d}{d x}(x \log x)\)

=-\(\frac{1}{(x \log x)^2}\left[\log x \frac{d(x)}{d x}+x \cdot \frac{d}{d x}(\log x)\right]\)

=-\(\frac{1}{(x \log x)^2} \cdot\left[\log x \cdot 1+x \cdot \frac{1}{x}\right]=\frac{-(1+\log x)}{(x \log x)^2}\)

Question 10. \(\sin (\log x)\)

Solution:

Let \(\mathrm{y}=\sin (\log \mathrm{x})\)

Then, \(\frac{d y}{d x}=\frac{d}{d x}[\sin (\log x)]=\cos (\log x) \cdot \frac{d}{d x}(\log x)=\frac{\cos (\log x)}{x}\)

Again differentiating with respect to x on both sides

⇒ \(\frac{d^2 y}{d x^2} =\frac{d}{d x}\left[\frac{\cos (\log x)}{x}\right]\)

= \(\frac{x \cdot \frac{d}{d x}[\cos (\log x)]-\cos (\log x) \frac{d}{d x}(x)}{x^2}\)

=  \(\frac{x \cdot\left[-\sin (\log x) \cdot \frac{d}{d x}(\log x)\right]-\cos (\log x) \cdot 1}{x^2}\)

=\(\frac{-x \sin (\log x) \cdot \frac{1}{x}-\cos (\log x)}{x^2}\)

=\(\frac{-[\sin (\log x)+\cos (\log x)]}{x^2}\)

Question 11. If y=5 \(\cos x-3 \sin x, prove that \frac{d^2 y}{d x^2}+y=0\)

Solution:

It is given that, y=5 \(\cos x-3 \sin x\)

Then,\(\frac{d y}{d x}=\frac{d}{d x}(5 \cos x)-\frac{d}{d x}(3 \sin x)=5 \frac{d}{d x}(\cos x)-3 \frac{d}{d x}(\sin x)\)

=\(5(-\sin x)-3 \cos x=-(5 \sin x+3 \cos x)\)

Again differentiating with respect to \(\mathrm{x}\) on both the sides

⇒ \(\frac{d^2 y}{d x^2} =\frac{d}{d x}[-(5 \sin x+3 \cos x)]\)

=-\(\left[5 \cdot \frac{d}{d x}(\sin x)+3 \cdot \frac{d}{d x}(\cos x)\right]\)

=-\([5 \cos x+3(-\sin x)]=-[5 \cos x-3 \sin x]\)=-y

⇒ \(\frac{d^2 y}{d x^2}+y\) =0

Hence, proved.

Question 12. If y=\(\cos ^{-1}\) x, find \(\frac{d^2 y}{d x^2}\) in terms of y alone?

Solution:

It is given that, y=\(\cos ^{-1} x \Rightarrow x=\cos y\)

⇒ \(\frac{d y}{d x}=-\frac{1}{\sqrt{1-x^2}}=-\frac{1}{\sqrt{1-\cos ^2 y}}=-{cosec} y \)

⇒ \(\frac{d^2 y}{d x^2}=\frac{d}{d x}(-{cosec} y)\)

⇒ \(\frac{d^2 y}{d x^2}={cosec} y \cot y \frac{d y}{d x}\)

=-\({cosec} y \cot y\)

Alternate Method:

It is given that, y=\(\cos ^{-1} x\)

Then, \(\frac{d y}{d x}=\frac{d}{d x}\left(\cos ^{-1} x\right)=\frac{-1}{\sqrt{1-x^2}}=-\left(1-x^2\right)^{\frac{-1}{2}}\)

Again differentiating with respect to x on both sides

⇒ \(\frac{d^2 y}{d x^2}=\frac{d}{d x}\left[-\left(1-x^2\right)^{\frac{-1}{2}}\right]\)

=-\(\left(-\frac{1}{2}\right) \cdot\left(1-x^2\right)^{\frac{-3}{2}} \cdot \frac{d}{d x}\left(1-x^2\right)\)

=\(\frac{1}{2 \sqrt{\left(1-x^2\right)^3}} \times(-2 x)\)

⇒ \(\frac{d^2 y}{d x^2}=\frac{-x}{\sqrt{\left(1-x^2\right)^2}}\)

⇒ \(y=\cos ^{-1} x \Rightarrow x=\cos y\)

Putting x=\(\cos y\) in equation (1), we obtain

⇒ \(\frac{d^2 y}{d x^2}=\frac{-\cos y}{\sqrt{\left(1-\cos ^2 y\right)^3}}=\frac{-\cos y}{\sqrt{\left(\sin ^2 y\right)^3}}\)

=\(\frac{-\cos y}{\sin ^3 y}=\frac{-\cos y}{\sin y} \times \frac{1}{\sin ^2 y}\)

∴ \(\frac{d^2 y}{d x^2}=-\cot y \cdot {cosec}^2 y\)

Question 13. If y=3 \(\cos (\log x)+4 \sin (\log x)\), show that \(x^2 y_2+x y_1+y\)=0

Solution:

It is given that, y=3 \(\cos (\log x)+4 \sin (\log x)\)

Then, \(y_1 =3 \cdot \frac{d}{d x}[\cos (\log x)]+4 \cdot \frac{d}{d x}[\sin (\log x)]\)

=3 \(\cdot\left[-\sin (\log x) \cdot \frac{d}{d x}(\log x)\right]+4 \cdot\left[\cos (\log x) \cdot \frac{d}{d x}(\log x)\right]\)

⇒ \(y_1 =\frac{-3 \sin (\log x)}{x}+\frac{4 \cos (\log x)}{x}=\frac{4 \cos (\log x)-3 \sin (\log x)}{x}\)

⇒ \(y_1 =4 \cos (\log x)-3 \sin (\log x)\)

Again differentiating with respect to x on both sides

x \(\frac{d\left(y_1\right)}{d x}+y_1 \frac{d(x)}{d x}=4(-\sin (\log x)) \frac{d}{d x}(\log x)-3 \cos (\log x) ⇒ \frac{d}{d x}(\log x)\)

⇒ \([latex]x y_2+y_1=\frac{-4 \sin (\log x)}{x}-\frac{3 \cos (\log x)}{x} \)

⇒ \(x^2 y_2+x y_1=-(4 \sin (\log x)+3 \cos (\log x))\)

⇒ \(x^2 y_2+x y_1=-y or x^2 y_2+x y_1+y=0\) [From eq. (1)]

Hence, proved.

Question 14. If y=500 \(e^{7 x}+600 e^{-7 x}\), show that \(\frac{d^2 y}{d x^2}\)=49 y

Solution:

It is given that, y=500 \(e^{1 x}+600 e^{-3 x}\)

Then, \(\frac{d y}{d x} =500 \cdot \frac{d}{d x}\left(e^{7 x}\right)+600 \cdot \frac{d}{d x}\left(e^{-7 x}\right)\)

=500 \(\cdot e^{7 x} \cdot \frac{d}{d x}(7 x)+600 \cdot e^{-7 x} \cdot \frac{d}{d x}(-7 x)=3500 e^{7 x}-4200 e^{-7 x}\)

Again differentiating with respect to x on both sides

⇒ \(\frac{d^2 y}{d x^2} =3500 \cdot \frac{d}{d x}\left(e^{7 x}\right)-4200 \cdot \frac{d}{d x}\left(e^{-7 x}\right)\)

=3500 \(\cdot e^{7 x} \cdot \frac{d}{d x}(7 x)-4200 \cdot e^{-7 x} \cdot \frac{d}{d x}(-7 x)\)

=7 \(\times 3500 \cdot e^{7 x}+7 \times 4200 \cdot e^{-7 x}=49 \times 500 e^{7 x}+49 \times 600 e^{-7 x} \)

=49\(\left(500 e^{7 x}+600 e^{-7 x}\right)\)=49 y

Hence, proved.

Question 15. If \(\mathrm{e}^y(\mathrm{x}+1)=1\), show that \(\frac{\mathrm{d}^2 \mathrm{y}}{\mathrm{dx}^2}=\left(\frac{\mathrm{dy}}{\mathrm{dx}}\right)^2\)

Solution:

The given relationship is \(e^y(x+1)=1 \Rightarrow e^y=\frac{1}{x+1}\)

Taking logarithm on both the sides \(\log e^y=\log \frac{1}{(x+1)} \Rightarrow y=\log \frac{1}{(x+1)}\)

Differentiating this relationship with respect to x

⇒ \(\frac{\mathrm{dy}}{\mathrm{dx}}=(\mathrm{x}+1) \frac{\mathrm{d}}{\mathrm{dx}}\left(\frac{1}{\mathrm{x}+1}\right)\)

=\((\mathrm{x}+1) \cdot \frac{-1}{(\mathrm{x}+1)^2}=\frac{-1}{\mathrm{x}+1}\)

Again differentiating with respect to x on both sides

⇒ \(\frac{d^2 y}{d x^2}=-\frac{d}{d x}\left(\frac{1}{x+1}\right)=-\left(\frac{-1}{(x+1)^2}\right)\)

=\(\frac{1}{(x+1)^2}=\left(\frac{-1}{x+1}\right)^2=\left(\frac{d y}{d x}\right)^2\)

Hence, proved.

Question 16. If y=\(\left(\tan ^{-1} x\right)^2\), show that \(\left(x^2+1\right)^2 y_2+2 ⇒ x\left(x^2+1\right) y_1\)=2

Solution:

The given relationship is y=\(\left(\tan ^{-1} x\right)^2\).

Then, \(y_1=2 \tan ^{-1} x \frac{d}{d x}\left(\tan ^{-1} x\right)\)

⇒ \(y_1=2 \tan ^{-1} x \cdot \frac{1}{1+x^2}\)

⇒ \(\left(1+x^2\right) y_1=2 \tan ^{-1} x\)

Again differentiating with respect to x on both sides

⇒ \(\left(1+x^2\right) y_2+2 x y_1=2\left(\frac{1}{1+x^2}\right)\)

⇒\(\left(1+x^2\right)^2 y_2+2 x\left(1+x^2\right) y_1=2\)

Hence, proved.

Continuity And Differentiability Miscellaneous Exercise

Question 1. \(\left(3 x^2-9 x+5\right)^6\)

Solution:

Let y=\(\left(3 x^2-9 x+5\right)^{\prime}\)

Using the chain rule, we obtain

⇒ \(\frac{d y}{d x} =\frac{d}{d x}\left(3 x^2-9 x+5\right)^9=9\left(3 x^2-9 x+5\right)^1 \cdot \frac{d}{d x}\left(3 x^2-9 x+5\right)\)

=\(9\left(3 x^2-9 x+5\right)^3 \cdot(6 x-9)\)

=9\(\left(3 x^2-9 x+5\right)^3 \cdot 3(2 x-3)=27\left(3 x^2-9 x+5\right)^3(2 x-3)\)

Question 2. \(\sin ^3 x+\cos ^6 x\)

Solution:

Let y=\(\sin ^3 x+\cos ^4 x \)

⇒ \(\frac{d y}{d x} =\frac{d}{d x}\left(\sin ^3 x\right)+\frac{d}{d x}\left(\cos ^6 x\right)\)

=\(3 \sin ^2 x \cdot \frac{d}{d x}(\sin x)+6 \cos ^5 x \cdot \frac{d}{d x}(\cos x)\)

=3 \(\sin ^2 x \cdot \cos x+6 \cos ^5 x \cdot(-\sin x)\)

=3 \(\sin x \cos x\left(\sin x-2 \cos ^4 x\right)\)

Question 3. \((5 x)^{3 \mathrm{mon} t}\)

Solution: 

Let y=\((5 x)^{2 \cos 2 x}\)

Taking logarithm on both sides, we obtain, \(\log y=3 \cos 2 x \log 5 x\)

Differentiating both sides with respect to x, we obtain

⇒ \(\frac{1}{y} \frac{d y}{d x}=3\left[\log 5 x \cdot \frac{d}{d x}(\cos 2 x)+\cos 2 x \cdot \frac{d}{d x}(\log 5 x)\right]\)

⇒ \(\frac{d y}{d x}=3 y\left[\log 5 x(-\sin 2 x) \cdot \frac{d}{d x}(2 x)+\cos 2 x-\frac{1}{5 x} \cdot \frac{d}{d x}(5 x)\right]\)

⇒ \(\frac{d y}{d x}=3 y\left[-2 \sin 2 x \log 5 x+\frac{\cos 2 x}{x}\right]\)

⇒ \(\frac{d y}{d x}=y\left[\frac{3 \cos 2 x}{x}-6 \sin 2 x \log 5 x\right]\)

∴ \(\frac{d y}{d x}=(5 x)^{3 \cos 2 x}\left[\frac{3 \cos 2 x}{x}-6 \sin 2 x \log 5 x\right]\)

Question 4. \(\sin ^{-1}(x \sqrt{x}), 0 \leq x \leq 1\)

Solution:

Let \(y=\sin ^{-1}(x \sqrt{x})\)

Using the chain rule, we obtain

⇒ \(\frac{d y}{d x} =\frac{d}{d x} \sin ^{-1}(x \sqrt{x})=\frac{1}{\sqrt{1-(x \sqrt{x})^2}} \times \frac{d}{d x}(x \sqrt{x})\)

=\(\frac{1}{\sqrt{1-x^3}} \cdot \frac{d}{d x}\left(x^{\frac{3}{2}}\right)\)

=\(\frac{1}{\sqrt{1-x^2}} \times \frac{3}{2} \cdot x^{\frac{1}{2}}=\frac{3 \sqrt{x}}{2 \sqrt{1-x^3}}\)

=\(\frac{3}{2} \sqrt{\frac{x}{1-x^3}}\)

Question 5. \(\frac{\cos ^{-1} \frac{x}{2}}{\sqrt{2 x+7}},-2<x<2\)

Solution:

Let \(\mathrm{y}=\frac{\cos ^{-1} \frac{\mathrm{x}}{2}}{\sqrt{2 \mathrm{x}+7}}\)

By the quotient rule, we obtain,

⇒ \(\frac{d y}{d x} =\frac{\sqrt{2 x+7} \frac{d}{d x}\left(\cos ^{-1} \frac{x}{2}\right)-\left(\cos ^{-1} \frac{x}{2}\right) \frac{d}{d x}(\sqrt{2 x+7})}{(\sqrt{2 x+7})^2}\)

=\(\frac{\sqrt{2 x+7}\left[\frac{-1}{\sqrt{1-\left(\frac{x}{2}\right)^2}} \cdot \frac{d}{d x}\left(\frac{x}{2}\right)\right]-\left(\cos ^{-1} \frac{x}{2}\right) \frac{1}{2 \sqrt{2 x+7}} \cdot \frac{d}{d x}(2 x+7)}{2 x+7}\),

=\(\frac{\sqrt{2 x+7} \frac{-1}{\sqrt{4-x^2}}-\left(\cos ^{-1} \frac{x}{2}\right) \frac{2}{2 \sqrt{2 x+7}}}{2 x+7}\)

=-\(\frac{\sqrt{2 x+7}}{\sqrt{4-x^2} \times(2 x+7)}-\frac{\cos ^{-1} \frac{x}{2}}{(\sqrt{2 x+7})(2 x+7)}\)

=-\(\left[\frac{1}{\sqrt{4-x^2} \sqrt{2 x+7}}+\frac{\cos ^{-1} \frac{x}{2}}{(2 x+7)^{\frac{3}{2}}}\right]\)

Question 6. \(\cot ^{-1}\left[\frac{\sqrt{1+\sin x}+\sqrt{1-\sin x}}{\sqrt{1+\sin x}-\sqrt{1-\sin x}}\right], 0<x<\frac{\pi}{2}\)

Solution:

Let y=\(\cot ^{-1}\left[\frac{\sqrt{1+\sin x}+\sqrt{1-\sin x}}{\sqrt{1+\sin x}-\sqrt{1-\sin x}}\right]\)

Then,\(\left[\frac{\sqrt{1+\sin x}+\sqrt{1-\sin x}}{\sqrt{1+\sin x}-\sqrt{1-\sin x}}\right]\)

=\(\frac{(\sqrt{1+\sin x}+\sqrt{1-\sin x})^2}{(\sqrt{1+\sin x}-\sqrt{1-\sin x})(\sqrt{1+\sin x}+\sqrt{1-\sin x})}\)

=\(\frac{(1+\sin x)+(1-\sin x)+2 \sqrt{(1-\sin x)(1+\sin x)}}{(1+\sin x)-(1-\sin x)}\)

=\(\frac{2+2 \sqrt{1-\sin ^2 x}}{2 \sin x}=\frac{1+\cos x}{\sin x}\)

=\(\frac{2 \cos ^2 \frac{x}{2}}{2 \sin \frac{x}{2} \cos \frac{x}{2}}=\cot \frac{x}{2}\)

Therefore, equation (1) becomes, y=\(\cot ^{-1}\left(\cot \frac{x}{2}\right)\)

⇒ \(y=\frac{x}{2}\)

⇒ \(\frac{d y}{d x}=\frac{1}{2} \frac{d}{d x}(x)\)

∴ \(\frac{d y}{d x}=\frac{1}{2}\)

Question 7. \((\log x)^{\log x}, x>1\)

Solution:

Let y=\((\log x)^{\log x}\)

Taking logarithms on both sides, we obtain

⇒ \(\log y=\log (\log x)^{\log x}\)

⇒ \(\log y=\log x \cdot \log (\log x)\)

Differentiating both sides with respect to x, we obtain

⇒ \(\frac{1}{y d y}=\frac{d}{d x}[\log x \cdot \log (\log x)]\)

⇒ \(\frac{1}{y d y}=\log (\log x) \cdot \frac{d}{d x}(\log x)+\log x \cdot \frac{d}{d x}[\log (\log x)]\)

⇒ \(\frac{d y}{d x}=y\left[\log (\log x) \cdot \frac{1}{x}+\log x \cdot \frac{1}{\log x} \cdot \frac{d}{d x}(\log x)\right]\)

⇒ \(\frac{d y}{d x}=y\left[\frac{1}{x} \cdot \log (\log x)+\frac{1}{x}\right] \Rightarrow \frac{d y}{d x}\)

=\((\log x)^{\log x}\left[\frac{1}{x}+\frac{\log (\log x)}{x}\right]\)

Question 8. \(\cos (a \cos x+b \sin x)\), for some constant a and b.

Solution:

Let y=\(\cos (a \cos x+b \sin x)\)

By using the chain rule, we obtain

⇒ \(\frac{d y}{d x} =\frac{d}{d x} \cos (a \cos x+b \sin x)\)

⇒ \(\frac{d y}{d x} =-\sin (a \cos x+b \sin x) \cdot \frac{d}{d x}(a \cos x+b \sin x)\)

=-\(\sin (a \cos x+b \sin x) \cdot[a(-\sin x)+b \cos x]=(a \sin x-b \cos x) \cdot \sin [a \cos x+b \sin x]\)

Question 9. \((\sin x-\cos x)^{(\sin x-\cos x)}, \frac{\pi}{4}<x<\frac{3 \pi}{4}\)

Solution:

Let y=\((\sin x-\cos x)^{(\sin x-606 x)}\)

Taking logarithms on both sides, we obtain

⇒ \(\log y=\log \left[(\sin x-\cos x)^{\sin x-\cos x)}\right]\)

⇒ \(\log y=(\sin x-\cos x) \cdot \log (\sin x-\cos x)\)

Differentiating both sides with respect to x, we obtain

⇒ \(\frac{1}{y d y}=\frac{d}{d x}[(\sin x-\cos x) \log (\sin x-\cos x)]\)

⇒ \(\frac{1 d y}{y d x}=\log (\sin x-\cos x) \cdot \frac{d}{d x}(\sin x-\cos x)+(\sin x-\cos x) \cdot \frac{d}{d x} \log (\sin x-\cos x)\)

⇒ \(\frac{1}{y} \frac{d y}{d x}=\log (\sin x-\cos x) \cdot(\cos x+\sin x)+(\sin x-\cos x) \cdot \frac{1}{(\sin x-\cos x)} \cdot \frac{d}{d x}(\sin x-\cos x)\)

⇒ \(\frac{d y}{d x}=y[(\cos x+\sin x) \cdot \log (\sin x-\cos x)+(\cos x+\sin x)]\)

∴ \(\frac{d y}{d x}=(\sin x-\cos x)^{(\sin x-\cos x)}(\cos x+\sin x)[1+\log (\sin x-\cos x)]\)

Question 10. Find \(\frac{d y}{d x}, if y=12(1-\cos t), x=10(t-\sin t),-\frac{\pi}{2}<t<\frac{\pi}{2}\)

Solution:

It is given that, y=12\((1-\cos t), x=10(t-\sin t)\)

⇒ \(\frac{\mathrm{dx}}{\mathrm{dt}}=\frac{\mathrm{d}}{\mathrm{dt}}[10(\mathrm{t}-\sin t)]=10 \cdot \frac{\mathrm{d}}{\mathrm{dt}}(\mathrm{t}-\sin t)=10(1-\cos \mathrm{t})\)

and\(\frac{\mathrm{dy}}{\mathrm{dt}}=\frac{\mathrm{d}}{\mathrm{dt}}[12(1-\cos \mathrm{t})]\)

=12 \(\cdot \frac{\mathrm{d}}{\mathrm{dt}}(1-\cos \mathrm{t})=12 \cdot[0-(-\sin t)]=12 \sin t\)

⇒ \(\frac{\mathrm{dy}}{\mathrm{dx}}=\frac{\left(\frac{\mathrm{dy}}{\mathrm{dt}}\right)}{\left(\frac{\mathrm{dx}}{\mathrm{dt}}\right)}\)

=\(\frac{12 \sin \mathrm{t}}{10(1-\cos \mathrm{t})}=\frac{12 \cdot 2 \sin \frac{\mathrm{t}}{2} \cdot \cos \frac{\mathrm{t}}{2}}{10 \cdot 2 \sin ^2 \frac{\mathrm{t}}{2}}\)

= \(\frac{6}{5} \cot \frac{\mathrm{t}}{2}\)

Question 11. If x \(\sqrt{1+y}+y \sqrt{1+x}=0, for -1<x<1\), prove that \(\frac{d y}{d x}=-\frac{1}{(1+x)^2}\) It is given that, x \(\sqrt{1+y}+y \sqrt{1+x}=0 \Rightarrow x \sqrt{1+y}=-y \sqrt{1+x}\)

Solution:

Squaring both sides, we obtain

⇒ \(x^2(1+y)=y^2(1+x) \Rightarrow x^2+x^2 y=y^2+x y^2\)

⇒ \(x^2-y^2=x y^2-x^2 y\)

⇒ \(x^2-y^2=x y(y-x)\)

(x+y)(x-y)=x y(y-x)

⇒ \(x+y=-x y \)

(1+x) y=-x

⇒ \(y=\frac{-x}{(1+x)}\)

Differentiating both sides with respect to x, we obtain

⇒ \(\frac{d y}{d x}=-\frac{(1+x) \frac{d}{d x}(x)-x \frac{d}{d x}(1+x)}{(1+x)^2}=-\frac{(1+x)-x}{(1+x)^2}=-\frac{1}{(1+x)^2}\)

Hence, proved.

Question 12. If \(\cos y=x \cos (a+y) with \cos a \neq \pm 1\), prove that \(\frac{d y}{d x}=\frac{\cos ^2(a+y)}{\sin a}\).

Solution:

⇒ \(\cos y=x \cos (a+y) \Rightarrow x=\frac{\cos y}{\cos (a+y)}\)

Differentiating both sides with respect to y. we obtain

⇒ \(\frac{d x}{d y}=\frac{\cos (a+y) \frac{d}{d y}(\cos y)-\cos y \frac{d}{d y}(\cos (a+y))}{\cos ^2(a+y)}\)

=\(\frac{-\cos (a+y) \sin y+\cos y \sin (a+y)}{\cos ^2(a+y)} \)

⇒ \(\frac{d x}{d y}=\frac{\sin (a+y-y)}{\cos ^2(a+y)}=\frac{\sin a}{\cos ^2(a+y)} (\sin (A-B)=\sin A \cos B-\cos A \sin B)\)

∴ \(\frac{d y}{d x}=\frac{\cos ^2(a+y)}{\sin a}\)

Question 13. If x=a\((\cos t+t \sin t)\) and y=a\((\sin t-t \cos t)\), find \(\frac{d^2 y}{d x^2}\)

Solution:

It is given that, x=a\((\cos t+t \sin t) and y=a(\sin t-t \cos t)\)

⇒ \(\frac{\mathrm{dx}}{\mathrm{dt}}=\mathrm{a} \cdot \frac{\mathrm{d}}{\mathrm{dt}}(\cos \mathrm{t}+\mathrm{t} \sin \mathrm{t})\)

=a\(\left[-\sin t+\sin t \cdot \frac{d}{d t}(t)+t \cdot \frac{d}{d t}(\sin t)\right]=a[-\sin t+\sin t+t \cos t]=a t \cos t \)

and \(\frac{d y}{d t}=a \cdot \frac{d}{d t}(\sin t-t \cos t)\)

=a\(\left[\cos t-\left\{\cos t \cdot \frac{d}{d t}(t)+t \cdot \frac{d}{d t}(\cos t)\right\}\right]=a[\cos t-\{\cos t-t \sin t\}]=a t \sin t \)

⇒ \(\frac{d y}{d x}=\frac{\left(\frac{d y}{d t}\right)}{\left(\frac{d x}{d t}\right)}=\frac{a t \sin t}{a t \cos t}=\tan t\)

Then, \(\frac{d^2 y}{d x^2}=\frac{d}{d x}\left(\frac{d y}{d x}\right)=\frac{d}{d x}(\tan t)\)

=\(\sec ^2 t \cdot \frac{d t}{d x}=\sec ^2 t \cdot \frac{1}{a t \cos t}[\frac{d x}{d t}=a t \cos t\)

⇒ \(\frac{d t}{d x}=\frac{1}{a t \cos t}]\)

=\(\frac{\sec ^3 \mathrm{t}}{\mathrm{at}}, 0<\mathrm{t}<\frac{\pi}{2}\)

Question 14. If f(x)=\(|x|^3\), show that \(f^{\prime \prime}(x)\) exists for all real x, and find it.

Solution:

It is known that, \(|\mathrm{x}|=\begin{cases}{cl}\mathrm{x}, & \text { if } \mathrm{x} \geq 0 \\ -\mathrm{x}, & \text { if } \mathrm{x}<0\end{cases}\).

Therefore when x \(\geq 0, f(x)=|x|^3=x^3\).

In this case, \(f(x)=3 x^2\) and hence, \(f^{\prime}(x)=6 x\)

When x<0, \(f(x)=|x|^3=(-x)^2=-x^3\)

In this case, \(f(x)=-3 x^2 and hence, f^{\prime \prime}(x)=-6 x\)

Thus, for \(f(x)=|x|^3, f^{\prime}(x)\) exists for all real x and is given by, \(f^{\prime \prime}(x)=\begin{cases}{cl}6 x, & \text { if } x \geq 0 \\ -6 x, & \text { if } & x<0\end{cases}\).

Question 15. Using the fact that \(\sin (A+B)=\sin A \cos B+\cos A \sin B\) and the differentiation, obtain the sum formula for cosines.

Solution:

⇒ \(\sin (A+B)=\sin A \cos B+\cos A \sin B\)

Differentiating both sides with respect to X, we obtain

⇒ \(\frac{d}{d x}[\sin (A+B)]=\frac{d}{d x}(\sin A \cos B)+\frac{d}{d x}(\cos A \sin B)\)

⇒ \(\cos (A+B) \cdot \frac{d}{d x}(A+B)\)

= \(\cos B \cdot \frac{d}{d x}(\sin A)+\sin A \cdot \frac{d}{d x}(\cos B)+\sin B \cdot \frac{d}{d x}(\cos A)+\cos A \cdot \frac{d}{d x}(\sin B)\)

⇒ \(\cos (A+B) \cdot \frac{d}{d x}(A+B)\)

=\(\cos B \cdot \cos A \frac{d A}{d x}+\sin A(-\sin B) \frac{d B}{d x}+\sin B(-\sin A) \cdot \frac{d A}{d x}+\cos A \cos B \frac{d B}{d x}\)

⇒ \(\cos (A+B) \cdot\left[\frac{d A}{d x}+\frac{d B}{d x}\right]=(\cos A \cos B-\sin A \sin B) \cdot\left[\frac{d A}{d x}+\frac{d B}{d x}\right]\)

⇒ \(\cos (A+B)=\cos A \cos B-\sin A \sin B\)

Question 16. If y=\(\left|\begin{array}{ccc}f(x) & g(x) & h(x) \\ \ell & m & n \\ a & b & c\end{array}\right|\), prove that \(\frac{d y}{d x}=\left|\begin{array}{ccc}f^{\prime}(x) & g^{\prime}(x) & h^{\prime}(x) \\ \ell & m & n \\ a & b & c\end{array}\right|\)

Solution:

y=\(\left|\begin{array}{ccc}
f(x) & g(x) & h(x) \\
\ell & m & n \\
a & b & c
\end{array}\right|\)

y=\((m c-n b) f(x)-(\ell c-n a) g(x)+(c b-m a) h(x)\)

Then, \(\frac{d y}{d x} =\frac{d}{d x}[(m c-n b) f(x)]-\frac{d}{d x}[(c-n a) g(x)]+\frac{d}{d x}[(f b-m a) h(x)]\)

=\((m c-n b) f^{\prime}(x)-(f c-n a) g^{\prime}(x)+((b-m a) h^{\prime}(x)\).

=\(\left|\begin{array}{ccc}
f^{\prime}(x) & g^{\prime}(x) & h^{\prime}(x) \\
f & m & n \\
a & b & c
\end{array}\right|\)

Thus, \(\frac{d y}{d x}=\left|\begin{array}{ccc}
f^{\prime}(x) & g^{\prime}(x) & h^{\prime}(x) \\
c & m & n \\
a & b & c
\end{array}\right|\)

Question 17. If y=\(e^{3 \cos ^{-1} x},-1 \leq x \leq 1\), show that \(\left(1-x^2\right) \frac{d^2 y}{d x^2}-x \frac{d y}{d x}-a^2 y\)=0.

Solution:

It is given that, y=\(e^{0.06 E^{-1} x}\)

Taking logarithms on both sides, we obtain.

⇒ \(\log y=\log e^{a \cos ^{-1} x}\)

⇒ \(\log y=a \cos ^{-1} x \log e \Rightarrow \log y=a \cos ^{-1} x\)

Differentiating both sides with respect to x, we obtain

⇒ \(\frac{1}{y} \frac{d y}{d x}=a \times \frac{-1}{\sqrt{1-x^2}}\)

⇒ \(\frac{d y}{d x}=\frac{-a y}{\sqrt{1-x^2}}\)

By squaring both the sides, we obtain

⇒ ⇒ \(\left(\frac{d y}{d x}\right)^2=\frac{a^2 y^2}{1-x^2}\)

⇒ \(\left(1-x^2\right)\left(\frac{d y}{d x}\right)^2=a^2 y^2\)

Again differentiating both sides with respect to x, we obtain

⇒ \(\left(\frac{d y}{d x}\right)^2 \frac{d}{d x}\left(1-x^2\right)+\left(1-x^2\right) \times \frac{d}{d x}\left[\left(\frac{d y}{d x}\right)^2\right]\)

=\(a^2 \frac{d}{d x}\left(y^2\right)\)

⇒ \(\left(\frac{d y}{d x}\right)^2(-2 x)+\left(1-x^2\right) \times 2 \frac{d y}{d x} \cdot \frac{d^2 y}{d x^2}=a^2 \cdot 2 y \cdot \frac{d y}{d x}\)

-x\( \frac{d y}{d x}+\left(1-x^2\right) \frac{d^2 y}{d x^2}=a^2 \cdot y \left[\frac{d y}{d x} \neq 0\right]\)

⇒ \(\left(1-x^2\right) \frac{d^2 y}{d x^2}-x \frac{d y}{d x}-a^2 y\) = 0

Hence Proved

 

Matrices Class 12 Maths Important Questions Chapter 3

Matrices Exercise 3.1

Question1. In the matrix \(\left[\begin{array}{cccc}
2 & 5 & 19 & -7 \\
35 & -2 & \frac{5}{2} & 12 \\
\sqrt{3} & 1 & -5 & 17
\end{array}\right]\)
, Write

The order of the matrix (2) the number of elements (3) write the elements \(a_{13}, a_{21}, a_{33}, a_{24}, a_{23}\)

Solution:

1. In the given matrix, the number of rows is 3 and the number of columns is 4. Therefore, the order of the matrix is 3 x 4.

2. Since, the order of the matrix is 3 x 4, so there are 3 x 4 = 12 elements in it

3. Let \(\left[\begin{array}{cccc}
2 & 5 & 19 & -7 \\
35 & -2 & \frac{5}{2} & 12 \\
\sqrt{3} & 1 & -5 & 17
\end{array}\right]=\left[\begin{array}{llll}
a_{11} & a_{12} & a_{13} & a_{14} \\
a_{21} & a_{22} & a_{23} & a_{24} \\
a_{31} & a_{32} & a_{33} & a_{34}
\end{array}\right]\)

On comparing the corresponding elements, we get \(a_{13}=19; a_{21}=35; a_{33}=-5, a_{24}=12; a_{23}=\frac{5}{2}\)

Question 2. If a matrix has 24 elements, what are the possible orders it can have? What, if it has 1 3 elements? Sol. We know that if a matrix is of the order m x n, it has mn elements. Hence, a matrix containing 24 elements can have any one of the following orders:

Solution:

1 x 24, 2 x 12, 3 x 8, 4 x 6, 6 x 4, 8 x 3, 12 x 2 or 24 x 1.

Similarly, a matrix containing 13 elements can have an order of 1 x 13 or 13 x 1.

Question 3. If a matrix has 1 8 elements, what are the possible orders it can have? What, if it has 5 elements?

Solution: A matrix containing 1 8 elements can have any one of the following orders:

1 X 18, 18 x 1,2 x 9,9 x 2,3 x 6,6 x 3

Similarly, a matrix containing 5 elements can have an order of 1 x 5 or 5 x 1.

Question 4. Construct 2 \(\times matrix, A=\left[a_{i j}\right]\), whose elements are given by:

  1. \(\mathrm{a}_{\mathrm{ij}}=\frac{(\mathrm{i}+\mathrm{j})^2}{2}\)
  2. \(\mathrm{a}_{\mathrm{ij}}=\frac{\mathrm{i}}{\mathrm{j}}\)
  3. \(\mathrm{a}_{\mathrm{ij}}=\frac{(\mathrm{i}+2 \mathrm{j})^2}{2}\)

Solution:

(1) The order of the given matrix is 2 \(\times\) 2, so A=\(\left[\begin{array}{ll}a_{11} & a_{12} \\ a_{21} & a_{22}\end{array}\right]_{2 \times 2}\)

where \(a_{i j}=\frac{(i+j)^2}{2}\).

⇒ \(\mathrm{a}_{11}=\frac{(1+1)^2}{2}=2, \mathrm{a}_{12}=\frac{(1+2)^2}{2}=\frac{9}{2}, \mathrm{a}_{21}=\frac{(2+1)^2}{2}=\frac{9}{2}, \mathrm{a}_{22}=\frac{(2+2)^2}{2}=8\)

Read and Learn More Class 12 Maths Chapter Wise with Solutions

Hence, the required matrix is A=\(\left[\begin{array}{cc}2 & 9 / 2 \\ 9 / 2 & 8\end{array}\right]_{2 \times 2}\)

2. Here, A=\(\left[\begin{array}{ll}
a_{11} & a_{12} \\
a_{21} & a_{22}
\end{array}\right]_{2 \times 2}\) where \(a_{i j}=\frac{i}{j}\)

⇒ \(a_{11}=\frac{1}{1}=1, a_{12}=\frac{1}{2}, a_{21}=\frac{2}{1}=2, a_{22}=\frac{2}{2}\)=1

Hence, the required matrix is A=\(\left[\begin{array}{cc}
1 & 1 / 2 \\
2 & 1
\end{array}\right]_{2 \times 2}\)

3. Here A =\(\left[\begin{array}{ll}
a_{11} & a_{12} \\
a_{21} & a_{22}
\end{array}\right]_{2 \times 2}\) where \(a_{i j}=\frac{(i+2 j)^2}{2}\)

⇒ \(a_{11}=\frac{(1+2)^2}{2}=\frac{9}{2}, a_{12}=\frac{(1+4)^2}{2}=\frac{25}{2}, a_{21}=\frac{(2+2)^2}{2}=8, a_{22}=\frac{(2+4)^2}{2}=18\)

Hence The Required Matrix A=\(\left[\begin{array}{cc}
9 / 2 & 25 / 2 \\
8 & 18
\end{array}\right]_{2 \times 2}\)

Question 5. Construct a 3 x 4 matrix whose elements are given by:

  1. \(a_{i j}=\frac{1}{2}|-3 i+j|\)
  2. \(\mathrm{a}_{\mathrm{ij}}=2 \mathrm{i}-\mathrm{j}\)

Solution:

1. The order of the given matrix is 3 x 4, so the required matrix is

A=\(\left[\begin{array}{llll}
a_{11} & a_{12} & a_{13} & a_{14} \\
a_{21} & a_{22} & a_{23} & a_{24} \\
a_{31} & a_{32} & a_{33} & a_{34}
\end{array}\right]_{3 \times 4}\) where \(a_{i j}=\frac{1}{2}|-3 i+j|\)

⇒ \(\mathrm{a}_{11}=\frac{1}{2}|-3+1|=1, \mathrm{a}_{12}=\frac{1}{2}|-3+2|=\frac{1}{2}, \mathrm{a}_{13}=\frac{1}{2}|-3+3|=0, \mathrm{a}_{14}=\frac{1}{2}|-3+4|=\frac{1}{2}\)

⇒ \(\mathrm{a}_{21}=\frac{1}{2}|-6+1|=\frac{5}{2}, \mathrm{a}_{22}=\frac{1}{2}|-6+2|=2, \mathrm{a}_{23}=\frac{1}{2}|-6+3|=\frac{3}{2}, \mathrm{a}_{24}=\frac{1}{2}|-6+4|=1 \)

⇒ \(\mathrm{a}_{31}=\frac{1}{2}|-9+1|=4, \mathrm{a}_{32}=\frac{1}{2}|-9+2|=\frac{7}{2}, \mathrm{a}_{33}=\frac{1}{2}|-9+3|=3 and \mathrm{a}_{34}=\frac{1}{2}|-9+4|=\frac{5}{2}\)

Here the Required Matrix Is A=\(\left[\begin{array}{cccc}
1 & 1 / 2 & 0 & 1 / 2 \\
5 / 2 & 2 & 3 / 2 & 1 \\
4 & 7 / 2 & 3 & 5 / 2
\end{array}\right]_{3 \times 4}\)

2. Here, \(\left[\begin{array}{llll}
a_{11} & a_{12} & a_{13} & a_{14} \\
a_{21} & a_{22} & a_{23} & a_{24} \\
a_{31} & a_{32} & a_{33} & a_{34}
\end{array}\right]_{3 \times 4}\) Where, \(a_{i j}\)=2 i-j

⇒ \(a_{11}=2-1=1, a_{12}=2-2=0, a_{13}=2-3=-1, a_{14}=2-4\)=-2,

⇒ \(a_{21}=4-1=3, a_{22}=4-2=2, a_{23}=4-3=1, a_{24}=4-4\)=0,

⇒ \(a_{31}=6-1=5, a_{32}=6-2=4, a_{33}=6-3\)=3 and \(a_{34}=6-4\)=2

Here, The Required Matrix A=\(\left[\begin{array}{cccc}
1 & 0 & -1 & -2 \\
3 & 2 & 1 & 0 \\
5 & 4 & 3 & 2
\end{array}\right]_{3 \times 4}\)

Question 6. Find the values of x, y, and z from the following equations :

1. \(\left[\begin{array}{ll}
4 & 3 \\
\mathrm{x} & 5
\end{array}\right]=\left[\begin{array}{ll}
\mathrm{y} & \mathrm{z} \\
1 & 5
\end{array}\right]\)

2. \(\left[\begin{array}{cc}
x+y & 2 \\
5+z & x y
\end{array}\right]=\left[\begin{array}{ll}
6 & 2 \\
5 & 8
\end{array}\right]\)

3. \(\left[\begin{array}{c}
x+y+z \\
x+z \\
y+z
\end{array}\right]=\left[\begin{array}{l}
9 \\
5 \\
7
\end{array}\right]\)

Solution:

1. Given, \(\left[\begin{array}{ll}
4 & 3 \\
\mathrm{x} & 5
\end{array}\right]=\left[\begin{array}{ll}
\mathrm{y} & \mathrm{z} \\
1 & 5
\end{array}\right]\) By definition of equality of matrix as the given matrices are equal, their corresponding elements are also equal. Comparing the corresponding elements, we get 4- % 3- z and x = 1 ⇒ x = 1., y = 4 and z = 3

2. Given, \(\left[\begin{array}{cc}
x+y & 2 \\
5+z & x y
\end{array}\right]=\left[\begin{array}{ll}
6 & 2 \\
5 & 8
\end{array}\right]\) By definition of equality of matrix as the given matrices are equal, their corresponding elements are also equal. Comparing the corresponding elements,

we get x + y- 6  → Equation 1

5 + 2=5  → Equation 2

xy- 8 → Equation 3

From Eq. (2), we get z = 0

From Eq, (1), y = 6- x Equation 4

Substituting this value of y in Eq. (3), we obtain

x(6- x) – 8 ⇒ x‘- 6x – 8 – 0 (x- 2)(x- 4) = 0 ⇒ x = 2 or x = 4

When x = 2, then from Eq, (4), y = 6- 2 = 4 and

when x = 4, then from Eq. (4), y = 6- 4 = 2.

So, either x = 2, y = 4 and z = 0 or x = 4, y = 2 and z = 0

3. \(\left[\begin{array}{c}
x+y+z \\
x+z \\
y+z
\end{array}\right]=\left[\begin{array}{l}
9 \\
5 \\
7
\end{array}\right]\) By definition of equality of matrix as the given matrices are equal, their corresponding elements are also equal. Comparing the corresponding elements,

we get x + y + z = 9 →  Equation 1

x + z = 5  → Equation 2

y + z = 7  →Equation3

Subtracting Eq. (2) from Eq. ( 1 ), we get y = 4

Subtracting Eq. (3) from Eq.(l), we get x = 2

Substituting y = 4 in Eq. (3), we get 4 +z = 7 ⇒ z = 7-4 = 3

Hence; x = 2, y = 4, z = 3

Question 7. Find the values of a, b. c and d from the equation \(\left[\begin{array}{cc}
a-b & 2 a+c \\
2 a-b & 3 c+d
\end{array}\right]=\left[\begin{array}{cc}
-1 & 5 \\
0 & 13
\end{array}\right]\)

Solution:

Given, \(\left[\begin{array}{cc}
a-b & 2 a+c \\
2 a-b & 3 c+d
\end{array}\right]=\left[\begin{array}{cc}
-1 & 5 \\
0 & 13
\end{array}\right]\)

By definition of equality of matrix, as the given matrices are equal, their corresponding elements are also equal. Comparing the corresponding elements, we get:

a – b = – 1 → Equation 1

2a- b = 0  → Equation 2

2a + c = 5  → Equation 3

and 3c + d= 13

Subtracting Eq. (1) from Eq.(2), we get a =1

Putting a = 1 in Eq. (1) and Eq.(3), we get 1 – b =- 1 and 2 c 5 => b 2 and c = 3

Substituting c = 3 in Eq. (4),

We obtain 3 x 3 + d = 13 => d- 13 – 9 = 4

Hence; a = 1 , b = 2, c = 3 and d = 4.

Question 8. A=\(\left[a_1\right]_{n \rightarrow n} \text { is a square matrix, if ? }\)

  1. m n
  2. m > n
  3. m = n
  4. None of these

Solution: 3.

Since, in a square matrix, the number of rows is equal to the number of columns; therefore, we must have m = n

Question 9. Which of the given values of x and y make the following pairs of matrices equal?
\(\left[\begin{array}{cc}
3 x+7 & 5 \\
y+1 & 2-3 x
\end{array}\right] \cdot\left[\begin{array}{cc}
0 & y-2 \\
8 & 4
\end{array}\right]\)

  1. x=\(\frac{-1}{3}\), y=7
  2. Not possible to find
  3. y=7, x=\(\frac{-2}{3}\)
  4. x=\(\frac{-1}{3}, y=\frac{-2}{3}\)

Solution: 2.  Not possible to find

y=7, x=\(\frac{-2}{3}\)

x=\(\frac{-1}{3}, y=\frac{-2}{3}\)

According to the question, \(\left[\begin{array}{cc}
3 x+7 & 5 \\
y+1 & 2-3 x
\end{array}\right] \cdot\left[\begin{array}{cc}
0 & y-2 \\
8 & 4
\end{array}\right]\)

By definition of equality of matrices, we have

3x +7 = 0  → Equation 1

5 = y – 2  → Equation 2

y + 1 = 8  → Equation 3

2- 3x = 4  → Equation 4

From Eq. (2), y=7

From Eq. (1), 3 x+7=0 \(\Rightarrow x=\frac{-7}{3}\)

From Eq. (4), 2-3 x=4 \(\Rightarrow\) x=\(\frac{-2}{3}\)

‘x’ can have only one value at a time.

Hence, it is not possible to find the values of x and y for which the given matrices are equal.

Question 10. The number of all possible matrices of order 3 x 3 with each entry or 0 or 1 is

  1. 27
  2. 18
  3. 81
  4. 512

Solution: 4. 512

We know that a matrix having an order of 3 x 3 contains 9 elements.

Each element can be selected in 2 ways (it can be either 0 or 1).

Hence, All the nine entries can be chosen by 2° ways =512 ways.

Matrices Exercise 3.2

Question 1. Let A= \(\left[\begin{array}{ll}
2 & 4 \\
3 & 2
\end{array}\right]\)
, B= \(\left[\begin{array}{cc}
1 & 3 \\
-2 & 5
\end{array}\right]\)
, C=\(\left[\begin{array}{cc}
-2 & 5 \\
3 & 4
\end{array}\right]\)
.Find Each Of The Following

  1. A + B
  2. A – B
  3. 3A – C
  4. AB
  5. BA

Solution:

1. A + B = \(\left[\begin{array}{ll}
2 & 4 \\
3 & 2
\end{array}\right]+\left[\begin{array}{cc}
1 & 3 \\
-2 & 5
\end{array}\right]=\left[\begin{array}{cc}
2+1 & 4+3 \\
3+(-2) & 2+5
\end{array}\right]=\left[\begin{array}{ll}
3 & 7 \\
1 & 7
\end{array}\right]\)

2. A – B =\(\left[\begin{array}{ll}
2 & 4 \\
3 & 2
\end{array}\right]-\left[\begin{array}{cc}
1 & 3 \\
-2 & 5
\end{array}\right]=\left[\begin{array}{cc}
2-1 & 4-3 \\
3-(-2) & 2-5
\end{array}\right]=\left[\begin{array}{cc}
1 & 1 \\
5 & -3
\end{array}\right]\)

3. 3A – C = \(3\left[\begin{array}{ll}
2 & 4 \\
3 & 2
\end{array}\right]-\left[\begin{array}{cc}
-2 & 5 \\
3 & 4
\end{array}\right]=\left[\begin{array}{cc}
6 & 12 \\
9 & 6
\end{array}\right]-\left[\begin{array}{cc}
-2 & 5 \\
3 & 4
\end{array}\right]=\left[\begin{array}{ll}
8 & 7 \\
6 & 2
\end{array}\right]\)

4. AB= \(\left[\begin{array}{ll}
2 & 4 \\
3 & 2
\end{array}\right]\left[\begin{array}{cc}
1 & 3 \\
-2 & 5
\end{array}\right]=\left[\begin{array}{ll}
2 \times 1+4 \times(-2) & 2 \times 3+4 \times 5 \\
3 \times 1+2 \times(-2) & 3 \times 3+2 \times 5
\end{array}\right]=\left[\begin{array}{cc}
-6 & 26 \\
-1 & 19
\end{array}\right]\)

5. BA =\(\left[\begin{array}{cc}
1 & 3 \\
-2 & 5
\end{array}\right]\left[\begin{array}{ll}
2 & 4 \\
3 & 2
\end{array}\right]=\left[\begin{array}{cc}
1 \times 2+3 \times 3 & 1 \times 4+3 \times 2 \\
(-2) \times 2+5 \times 3 & (-2) \times 4+5 \times 2
\end{array}\right]=\left[\begin{array}{cc}
11 & 10 \\
11 & 2
\end{array}\right]\)

Question 2. Compute the following:

(1) \(\left[\begin{array}{cc}a & b \\ -b &a\end{array}\right]+\left[\begin{array}{ll}a & b \\ b & a\end{array}\right]\)

(2) \(\left[\begin{array}{ll}a^2+b^2 & b^2+c^2 \\ a^2+c^2 & a^2+b^2\end{array}\right]+\left[\begin{array}{cc}2 a b & 2 b c \\ -2 a c & -2 a b\end{array}\right]\)

(3) \(\left[\begin{array}{ccc}-1 & 4 & -6 \\ 8 & 5 & 16 \\ 2 & 8 & 5\end{array}\right]+\left[\begin{array}{ccc}12 & 7 & 6 \\ 8 & 0 & 5 \\ 3 & 2 & 4\end{array}\right]\)

(4) \(\left[\begin{array}{ll}\cos ^2 x & \sin ^2 x \\ \sin ^2 x & \cos ^2 x\end{array}\right]+\left[\begin{array}{ll}\sin ^2 x & \cos ^2 x \\ \cos ^2 x & \sin ^2 x\end{array}\right]\)

Solution:

1. \(\left[\begin{array}{cc}
a & b \\
-b & a
\end{array}\right]+\left[\begin{array}{ll}
a & b \\
b & a
\end{array}\right]=\left[\begin{array}{cc}
a+a & b+b \\
-b+b & a+a
\end{array}\right]=\left[\begin{array}{cc}
2 a & 2 b \\
0 & 2 a
\end{array}\right]\)

2. \(\left[\begin{array}{ll}
a^2+b^2 & b^2+c^2 \\
a^2+c^2 & a^2+b^2
\end{array}\right]+\left[\begin{array}{cc}
2 a b & 2 b c \\
-2 a c & -2 a b
\end{array}\right]=\left[\begin{array}{ll}
a^2+b^2+2 a b & b^2+c^2+2 b c \\
a^2+c^2-2 a c & a^2+b^2-2 a b
\end{array}\right] \)

= \(\left[\begin{array}{ll}
(a+b)^2 & (b+c)^2 \\
(a-c)^2 & (a-b)^2
\end{array}\right] \quad\left\{\begin{array}{ll}
& (a+b)^2=a^2+2 a b+b^2 \\
\text { and } & (a-b)^2=a^2-2 a b+b^2
\end{array}\right\}\)

3. \(\left[\begin{array}{ccc}
-1 & 4 & -6 \\
8 & 5 & 16 \\
2 & 8 & 5
\end{array}\right]+\left[\begin{array}{ccc}
12 & 7 & 6 \\
8 & 0 & 5 \\
3 & 2 & 4
\end{array}\right]=\left[\begin{array}{ccc}
-1+12 & 4+7 & -6+6 \\
8+8 & 5+0 & 16+5 \\
2+3 & 8+2 & 5+4
\end{array}\right]=\left[\begin{array}{ccc}
11 & 11 & 0 \\
16 & 5 & 21 \\
5 & 10 & 9
\end{array}\right]\)

4. \(\left[\begin{array}{ll}
\cos ^2 x & \sin ^2 x \\
\sin ^2 x & \cos ^2 x
\end{array}\right]+\left[\begin{array}{cc}
\sin ^2 x & \cos ^2 x \\
\cos ^2 x & \sin ^2 x
\end{array}\right]=\left[\begin{array}{ll}
\cos ^2 x+\sin ^2 x & \sin ^2 x+\cos ^2 x \\
\sin ^2 x+\cos ^2 x & \cos ^2 x+\sin ^2 x
\end{array}\right]=\left[\begin{array}{ll}
1 & 1 \\
1 & 1
\end{array}\right]\)

Question 3. Compute identical products

1. \(\left[\begin{array}{cc}
a & b \\
-b & a
\end{array}\right]\left[\begin{array}{cc}
a & -b \\
b & a
\end{array}\right]\)

2. \(\left[\begin{array}{l}
1 \\
2 \\
3
\end{array}\right]\left[\begin{array}{lll}
2 & 3 & 4
\end{array}\right]\)

3. \(\left[\begin{array}{cc}
1 & -2 \\
2 & 3
\end{array}\right]\left[\begin{array}{lll}
1 & 2 & 3 \\
2 & 3 & 1
\end{array}\right]\)

4. \(\left[\begin{array}{lll}
2 & 3 & 4 \\
3 & 4 & 5 \\
4 & 5 & 6
\end{array}\right]\left[\begin{array}{ccc}
1 & -3 & 5 \\
0 & 2 & 4 \\
3 & 0 & 5
\end{array}\right]\)

5. \(\left[\begin{array}{cc}
2 & 1 \\
3 & 2 \\
-1 & 1
\end{array}\right]\left[\begin{array}{ccc}
1 & 0 & 1 \\
-1 & 2 & 1
\end{array}\right]\)

6. \(\left[\begin{array}{ccc}
3 & -1 & 3 \\
-1 & 0 & 2
\end{array}\right]\left[\begin{array}{cc}
2 & -3 \\
1 & 0 \\
3 & 1
\end{array}\right]\)

Solution:

1. \(\left[\begin{array}{cc}
a & b \\
-b & a
\end{array}\right]\left[\begin{array}{cc}
a & -b \\
b & a
\end{array}\right]=\left[\begin{array}{cc}
a \times a+b \times b & a \times(-b)+b \times a \\
(-b) \times a+a \times b & (-b) \times(-b)+a \times a
\end{array}\right]=\left[\begin{array}{cc}
a^2+b^2 & 0 \\
0 & b^2+a^2
\end{array}\right]\)

2. \(\left[\begin{array}{l}
1 \\
2 \\
3
\end{array}\right]\left[\begin{array}{lll}
2 & 3 & 4
\end{array}\right]=\left[\begin{array}{lll}
1 \times 2 & 1 \times 3 & 1 \times 4 \\
2 \times 2 & 2 \times 3 & 2 \times 4 \\
3 \times 2 & 3 \times 3 & 3 \times 4
\end{array}\right]=\left[\begin{array}{ccc}
2 & 3 & 4 \\
4 & 6 & 8 \\
6 & 9 & 12
\end{array}\right]\)

3. \(\left[\begin{array}{cc}
1 & -2 \\
2 & 3
\end{array}\right]\left[\begin{array}{ccc}
1 & 2 & 3 \\
2 & 3 & 1
\end{array}\right]=\left[\begin{array}{ccc}
1 \times 1+(-2) \times 2 & 1 \times 2+(-2) \times 3 & 1 \times 3+(-2) \times 1 \\
2 \times 1+3 \times 2 & 2 \times 2+3 \times 3 & 2 \times 3+3 \times 1
\end{array}\right]\)

=\(\left[\begin{array}{ccc}
-3 & -4 & 1 \\
8 & 13 & 9
\end{array}\right]\)

4. \(\left[\begin{array}{lll}
2 & 3 & 4 \\
3 & 4 & 5 \\
4 & 5 & 6
\end{array}\right]\left[\begin{array}{ccc}
1 & -3 & 5 \\
0 & 2 & 4 \\
3 & 0 & 5
\end{array}\right]=\left[\begin{array}{ccc}
2+0+12 & -6+6+0 & 10+12+20 \\
3+0+15 & -9+8+0 & 15+16+25 \\
4+0+18 & -12+10+0 & 20+20+30
\end{array}\right]\)

=\(\left[\begin{array}{ccc}
14 & 0 & 42 \\
18 & -1 & 56 \\
22 & -2 & 70
\end{array}\right]\)

5. \(\left[\begin{array}{cc}
2 & 1 \\
3 & 2 \\
-1 & 1
\end{array}\right]\left[\begin{array}{ccc}
1 & 0 & 1 \\
-1 & 2 & 1
\end{array}\right]=\left[\begin{array}{ccc}
2-1 & 0+2 & 2+1 \\
3-2 & 0+4 & 3+2 \\
-1-1 & 0+2 & -1+1
\end{array}\right]=\left[\begin{array}{ccc}
1 & 2 & 3 \\
1 & 4 & 5 \\
-2 & 2 & 0
\end{array}\right]\)

6. \(\left[\begin{array}{ccc}
3 & -1 & 3 \\
-1 & 0 & 2
\end{array}\right]\left[\begin{array}{cc}
2 & -3 \\
1 & 0 \\
3 & 1
\end{array}\right]=\left[\begin{array}{cc}
6-1+9 & -9+0+3 \\
-2+0+6 & 3+0+2
\end{array}\right]=\left[\begin{array}{cc}
14 & -6 \\
4 & 5
\end{array}\right]\)

Question 4. If A =\(\left[\begin{array}{ccc}
1 & 2 & -3 \\
5 & 0 & 2 \\
1 & -1 & 1
\end{array}\right]+\left[\begin{array}{ccc}
3 & -1 & 2 \\
4 & 2 & 5 \\
2 & 0 & 3
\end{array}\right]\)
, B = \(\left[\begin{array}{ccc}
3 & -1 & 2 \\
4 & 2 & 5 \\
2 & 0 & 3
\end{array}\right]\)
, C =\(\left[\begin{array}{ccc}
4 & 1 & 2 \\
0 & 3 & 2 \\
1 & -2 & 3
\end{array}\right]\)
, then compute (A + B) and (B – C).

Also, verify that A + (B- C) = (A + B)- C.

Solution:

A+B=\(\left[\begin{array}{ccc}
1 & 2 & -3 \\
5 & 0 & 2 \\
1 & -1 & 1
\end{array}\right]+\left[\begin{array}{ccc}
3 & -1 & 2 \\
4 & 2 & 5 \\
2 & 0 & 3
\end{array}\right]=\left[\begin{array}{ccc}
4 & 1 & -1 \\
9 & 2 & 7 \\
3 & -1 & 4
\end{array}\right]\)

And B-c = \(\left[\begin{array}{ccc}
3 & -1 & 2 \\
4 & 2 & 5 \\
2 & 0 & 3
\end{array}\right]-\left[\begin{array}{ccc}
4 & 1 & 2 \\
0 & 3 & 2 \\
1 & -2 & 3
\end{array}\right]=\left[\begin{array}{ccc}
-1 & -2 & 0 \\
4 & -1 & 3 \\
1 & 2 & 0
\end{array}\right]\)

⇒ \(A+(B-C)=\left[\begin{array}{ccc}
1 & 2 & -3 \\
5 & 0 & 2 \\
1 & -1 & 1
\end{array}\right]+\left[\begin{array}{ccc}
-1 & -2 & 0 \\
4 & -1 & 3 \\
1 & 2 & 0
\end{array}\right]=\left[\begin{array}{ccc}
0 & 0 & -3 \\
9 & -1 & 5 \\
2 & 1 & 1
\end{array}\right]\)

⇒ \((A+B)-C=\left[\begin{array}{ccc}
4 & 1 & -1 \\
9 & 2 & 7 \\
3 & -1 & 4
\end{array}\right]-\left[\begin{array}{ccc}
4 & 1 & 2 \\
0 & 3 & 2 \\
1 & -2 & 3
\end{array}\right]=\left[\begin{array}{ccc}
0 & 0 & -3 \\
9 & -1 & 5 \\
2 & 1 & 1
\end{array}\right]\)

Hence, A + (B- C) = (A + B) ~ C is verified.

Question 5. If A= \(=\left[\begin{array}{ccc}
2 / 3 & 1 & 5 / 3 \\
1 / 3 & 2 / 3 & 4 / 3 \\
7 / 3 & 2 & 2 / 3
\end{array}\right]\)
and B=\(\left[\begin{array}{ccc}
2 / 5 & 3 / 5 & 1 \\
1 / 5 & 2 / 5 & 4 / 5 \\
7 / 5 & 6 / 5 & 2 / 5
\end{array}\right]\)
,Then Compute 3A-5B

Solution:

⇒ \(3 \mathrm{~A}-5 \mathrm{~B}=3\left[\begin{array}{ccc}
2 / 3 & 1 & 5 / 3 \\
1 / 3 & 2 / 3 & 4 / 3 \\
7 / 3 & 2 & 2 / 3
\end{array}\right]-5\left[\begin{array}{ccc}
2 / 5 & 3 / 5 & 1 \\
1 / 5 & 2 / 5 & 4 / 5 \\
7 / 5 & 6 / 5 & 2 / 5
\end{array}\right]\)

= \(\left[\begin{array}{lll}
2 & 3 & 5 \\
1 & 2 & 4 \\
7 & 6 & 2
\end{array}\right]-\left[\begin{array}{lll}
2 & 3 & 5 \\
1 & 2 & 4 \\
7 & 6 & 2
\end{array}\right]=\left[\begin{array}{lll}
0 & 0 & 0 \\
0 & 0 & 0 \\
0 & 0 & 0
\end{array}\right]\)=0 (Zero matrix)

Question 6. Simplify \(\cos \theta\left[\begin{array}{cc}
\cos \theta & \sin \theta \\
-\sin \theta & \cos \theta
\end{array}\right]+\sin \theta\left[\begin{array}{cc}
\sin \theta & -\cos \theta \\
\cos \theta & \sin \theta
\end{array}\right]\)
.

Solution:

⇒ \(\cos \theta\left[\begin{array}{cc}
\cos \theta & \sin \theta \\
-\sin \theta & \cos \theta
\end{array}\right]+\sin \theta\left[\begin{array}{cc}
\sin \theta & -\cos \theta \\
\cos \theta & \sin \theta
\end{array}\right]\)

= \(\left[\begin{array}{cc}
\cos ^2 \theta & \cos \theta \sin \theta \\
-\cos \theta \sin \theta & \cos ^2 \theta
\end{array}\right]+\left[\begin{array}{cc}
\sin ^2 \theta & -\sin \theta \cos \theta \\
\sin \theta \cos \theta & \sin ^2 \theta
\end{array}\right] \)

= \(\left[\begin{array}{cc}
\cos ^2 \theta+\sin ^2 \theta & 0 \\
0 & \cos ^2 \theta+\sin ^2 \theta
\end{array}\right]=\left[\begin{array}{ll}
1 & 0 \\
0 & 1
\end{array}\right]\) (\(\cos ^2 \theta+\sin ^2 \theta\)=1)

Question 7. Find X and Y if:

X+Y=X+Y=\(\left[\begin{array}{ll}
7 & 0 \\
2 & 5
\end{array}\right]\) and X-Y=\(\left[\begin{array}{ll}
3 & 0 \\
0 & 3
\end{array}\right]\)

\(\left[\begin{array}{ll}
2 & 3 \\
4 & 0
\end{array}\right]\) and 3 X+2 Y=\(\left[\begin{array}{cc}
2 & -2 \\
-1 & 5
\end{array}\right]\)

Solution:

Given, X+Y=\(\left[\begin{array}{ll}7 & 0 \\ 2 & 5\end{array}\right]\)  → Equation 1

and X-Y=\(\left[\begin{array}{ll}3 & 0 \\ 0 & 3\end{array}\right]\)  →  Equation 2

Adding equation (1) and (2), we get

1. 2 \(\mathrm{X}=\left[\begin{array}{ll}
7 & 0 \\
2 & 5
\end{array}\right]+\left[\begin{array}{ll}
3 & 0 \\
0 & 3
\end{array}\right]=\left[\begin{array}{cc}
10 & 0 \\
2 & 8
\end{array}\right] \Rightarrow \mathrm{X}=\frac{1}{2}\left[\begin{array}{cc}
10 & 0 \\
2 & 8
\end{array}\right]=\left[\begin{array}{ll}
5 & 0 \\
1 & 4
\end{array}\right]\)

Subtracting equation (2) from equation (1), we get

2 \(\mathrm{Y}=\left[\begin{array}{ll}
7 & 0 \\
2 & 5
\end{array}\right]-\left[\begin{array}{ll}
3 & 0 \\
0 & 3
\end{array}\right]=\left[\begin{array}{ll}
4 & 0 \\
2 & 2
\end{array}\right] \Rightarrow \mathrm{Y}=\frac{1}{2}\left[\begin{array}{ll}
4 & 0 \\
2 & 2
\end{array}\right]=\left[\begin{array}{ll}
2 & 0 \\
1 & 1
\end{array}\right]\)

2. Given, 2 \(\mathrm{X}+3 \mathrm{Y}=\left[\begin{array}{ll}2 & 3 \\ 4 & 0\end{array}\right]\)

and 3 X+2 Y=\(\left[\begin{array}{cc}2 & -2 \\ -1 & 5\end{array}\right]\)

Multiplying equation (1) by 2, \mathrm{Eq}. (2) by 3 and then subtracting, we get

⇒ \( 2(2 \mathrm{X}+3 \mathrm{Y})-3(3 \mathrm{X}+2 \mathrm{Y})=2\left[\begin{array}{ll}
2 & 3 \\
4 & 0
\end{array}\right]-3\left[\begin{array}{cc}
2 & -2 \\
-1 & 5
\end{array}\right]\)

⇒ \(\Rightarrow 4 \mathrm{X}+6 \mathrm{Y}-9 \mathrm{X}-6 \mathrm{Y}=\left[\begin{array}{cc}
4 & 6 \\
8 & 0
\end{array}\right]-\left[\begin{array}{cc}
6 & -6 \\
-3 & 15
\end{array}\right]\)

⇒ \(\Rightarrow-5 \mathrm{X}=\left[\begin{array}{cc}
4-6 & 6+6 \\
8+3 & 0-15
\end{array}\right]=\left[\begin{array}{cc}
-2 & 12 \\
11 & -15
\end{array}\right]\)

⇒ \(\Rightarrow \mathrm{X}=-\frac{1}{5}\left[\begin{array}{cc}
-2 & 12 \\
11 & -15
\end{array}\right]=\left[\begin{array}{cc}
2 / 5 & -12 / 5 \\
-11 / 5 & 3
\end{array}\right]\)

Then, from Eq. (1),

⇒ \(3 \mathrm{Y}=\left[\begin{array}{ll}
2 & 3 \\
4 & 0
\end{array}\right]-2 \mathrm{X}\)

=\(\left[\begin{array}{ll}
2 & 3 \\
4 & 0
\end{array}\right]-2\left[\begin{array}{cc}
2 / 5 & -12 / 5
-11 / 5 & 3
\end{array}\right]\)

=\(\left[\begin{array}{cc}
2-\frac{4}{5} & 3+\frac{24}{5}
4+\frac{22}{5} & 0-6
\end{array}\right]=\left[\begin{array}{cc}
6 / 5 & 39 / 5
42 / 5 & -6
\end{array}\right]\)

⇒ \(\mathrm{Y}=\frac{1}{3}\left[\begin{array}{cc}
6 / 5 & 39 / 5
42 / 5 & -6
\end{array}\right]=\left[\begin{array}{cc}
2 / 5 & 13 / 5
14 / 5 & -2
\end{array}\right]\)

Question 8. Find X , if Y=\(\left[\begin{array}{ll}
3 & 2 \\
1 & 4
\end{array}\right]\)
and  2 X+Y=\(\left[\begin{array}{cc}
1 & 0 \\
-3 & 2
\end{array}\right]\)

Solution:

⇒ \(2 X=\left[\begin{array}{cc}
1 & 0 \\
-3 & 2
\end{array}\right]-Y=\left[\begin{array}{cc}
1 & 0 \\
-3 & 2
\end{array}\right]-\left[\begin{array}{ll}
3 & 2 \\
1 & 4
\end{array}\right]=\left[\begin{array}{cc}
1-3 & 0-2 \\
-3-1 & 2-4
\end{array}\right]=\left[\begin{array}{cc}
-2 & -2 \\
-4 & -2
\end{array}\right]\)

X=\(\frac{1}{2}\left[\begin{array}{ll}
-2 & -2 \\
-4 & -2
\end{array}\right]=\left[\begin{array}{cc}
-1 & -1 \\
-2 & -1
\end{array}\right]\)

Question 9. Find x and y ,if 2\(\left[\begin{array}{ll}
1 & 3 \\
0 & x
\end{array}\right]+\left[\begin{array}{ll}
y & 0 \\
1 & 2
\end{array}\right]=\left[\begin{array}{ll}
5 & 6 \\
1 & 8
\end{array}\right]\)
.

Solution:

Given, \(2\left[\begin{array}{ll}
1 & 3 \\
0 & \mathrm{x}
\end{array}\right]+\left[\begin{array}{ll}
\mathrm{y} & 0 \\
1 & 2
\end{array}\right]=\left[\begin{array}{ll}
5 & 6 \\
1 & 8
\end{array}\right] \Rightarrow\left[\begin{array}{cc}
2+\mathrm{y} & 6+0 \\
0+1 & 2 \mathrm{x}+2
\end{array}\right]=\left[\begin{array}{ll}
5 & 6 \\
1 & 8
\end{array}\right]\)

By definition of equality of matrix, as the given matrices are equal, their corresponding elements are also equal. Comparing these corresponding elements, we get

2+Y = S  →  Equation (1)

and 2x+2=8  →  Equation (2)

y=5-2=3 and 2 x=8-2 \(\Rightarrow\) y=3 and x=\(\frac{6}{2}\)=3

Question 10. Solve the equation for x, y, z And t,- if 2\(\left[\begin{array}{ll}
x & z \\
y & t
\end{array}\right]+3\left[\begin{array}{cc}
1 & -1 \\
0 & 2
\end{array}\right]=3\left[\begin{array}{ll}
3 & 5 \\
4 & 6
\end{array}\right]\)

Solution:

By definition of equality of matrix, as the given matrices are equal, their corresponding elements are also equal. Comparing the corresponding elements, we get

2x + 3 = 9,2y + 0 = 12,2z – 3 = 15 and 2t + 6 = 18

⇒ \(x=\frac{9-3}{2}, y=\frac{12}{2}, z=\frac{15+3}{2} \text { and } t=\frac{18-6}{2}\)

x=3, y=6, 2=9 and t=6.

Question 11. If  \(x\left[\begin{array}{l}
2 \\
3
\end{array}\right]+y\left[\begin{array}{c}
-1 \\
1
\end{array}\right]=\left[\begin{array}{c}
10 \\
5
\end{array}\right]\)
,Then find the values of x And y.

Solution:

Given that \(x\left[\begin{array}{l}
2 \\
3
\end{array}\right]+y\left[\begin{array}{c}
-1 \\
1
\end{array}\right]=\left[\begin{array}{c}
10 \\
5
\end{array}\right] \Rightarrow\left[\begin{array}{c}
2 x-y \\
3 x+y
\end{array}\right]=\left[\begin{array}{c}
10 \\
5
\end{array}\right]\)

By definition of equality of matrix as the given matrices are equal, their corresponding elements are also equal” Comparing the corresponding elements, we get

2x – y = 19 Equation (1)

and 3x + Y = 5 Equation (2)

Adding Eq. (1) and (2), we get; 5x = 15 =* x = 3

Substituting x= 3 in Eq. (1), we get : 2 x 3 -y = 10 + y =6 -10 =-4

Question 12. \(3\left[\begin{array}{cc}
x & y \\
z & w
\end{array}\right]=\left[\begin{array}{cc}
x & 6 \\
-1 & 2 w
\end{array}\right]+\left[\begin{array}{cc}
4 & x+y \\
z+w & 3
\end{array}\right]\)
,Find the values of x,y,z And w.

Solution:

Given, \(3\left[\begin{array}{ll}
x & y \\
z & w
\end{array}\right]=\left[\begin{array}{cc}
x & 6 \\
-1 & 2 w
\end{array}\right]+\left[\begin{array}{cc}
4 & x+y \\
z+w & 3
\end{array}\right] \Rightarrow\left[\begin{array}{cc}
3 x & 3 y \\
3 z & 3 w
\end{array}\right]=\left[\begin{array}{cc}
x+4 & 6+x+y \\
-1+z+w & 2 w+3
\end{array}\right]\)

By definition of equality of matrix, as the given matrices are equal, their corresponding elements are also equal. Comparing the corresponding elements, we get

3 x=x+4 \(\Rightarrow 2 x=4 \Rightarrow\) x=2 and 3 y=6+x+y \(\Rightarrow 2 y=6+x \Rightarrow y=\frac{6+x}{2}=\frac{6+2}{2}=\frac{8}{2}\)=4

Now, 3 z=-1+z+w, 2 z=-1+w \(\Rightarrow z=\frac{-1+w}{2}\)

Now, 3 w=2 w+3 \(\Rightarrow\) w=3

Putting the value of w in Eq. (1), we get: z=\(\frac{-1+3}{2}\)=1

Hence, the values of x, y, z, and w are 2,4,1 and 3, respectively.

Question 13. If F(x)= \(\left[\begin{array}{ccc}
\cos x & -\sin x & 0 \\
\sin x & \cos x & 0 \\
0 & 0 & 1
\end{array}\right]\)
 , then show that F(x) \(\cdot \)F(y)=F(x+y).

Solution:

⇒ \(F(x) F(y)=\left[\begin{array}{ccc}
\cos x & -\sin x & 0 \\
\sin x & \cos x & 0 \\
0 & 0 & 1
\end{array}\right]\)

⇒ \(\left[\begin{array}{ccc}
\cos y & -\sin y & 0 \\
\sin y & \cos y & 0 \\
0 & 0 & 1
\end{array}\right]\)

= \(\left[\begin{array}{ccc}
\cos x \cos y-\sin x \sin y & -\sin y \cos x-\sin x \cos y & 0 \\
\sin x \cos y+\cos x \sin y & -\sin x \sin y+\cos x \cos y & 0 \\
0 & 0 & 1
\end{array}\right]\)

= \(\left[\begin{array}{ccc}
\cos (x+y) & -\sin (x+y) & 0 \\
\sin (x+y) & \cos (x+y) & 0 \\
0 & 0 & 1
\end{array}\right] \left\{\begin{array}{c}
\cos (A+B)=\cos A \cos B-\sin A \sin B \\
\sin (A+B)=\sin A \cos B+\sin B \cos A
\end{array}\right\}\)

F(x), F(y)=F(x+y)

Question 14. Show That

1. \(\left[\begin{array}{cc}
5 & -1 \\
6 & 7
\end{array}\right]\left[\begin{array}{ll}
2 & 1 \\
3 & 4
\end{array}\right] \neq\left[\begin{array}{ll}
2 & 1 \\
3 & 4
\end{array}\right]\left[\begin{array}{cc}
5 & -1 \\
6 & 7
\end{array}\right]\)

2. \(\left[\begin{array}{lll}
1 & 2 & 3 \\
0 & 1 & 0 \\
1 & 1 & 0
\end{array}\right]\left[\begin{array}{ccc}
-1 & 1 & 0 \\
0 & -1 & 1 \\
2 & 3 & 4
\end{array}\right] \neq\left[\begin{array}{ccc}
-1 & 1 & 0 \\
0 & -1 & 1 \\
2 & 3 & 4
\end{array}\right]\left[\begin{array}{lll}
1 & 2 & 3 \\
0 & 1 & 0 \\
1 & 1 & 0
\end{array}\right]\)

Solution:

1. \({\left[\begin{array}{cc}
5 & -1 \\
6 & 7
\end{array}\right]\left[\begin{array}{ll}
2 & 1 \\
3 & 4
\end{array}\right]=\left[\begin{array}{cc}
10-3 & 5-4 \\
12+21 & 6+28
\end{array}\right]=\left[\begin{array}{cc}
7 & 1 \\
33 & 34
\end{array}\right]}\)

⇒ \({\left[\begin{array}{cc}
2 & 1 \\
3 & 4
\end{array}\right]\left[\begin{array}{cc}
5 & -1 \\
6 & 7
\end{array}\right]=\left[\begin{array}{cc}
10+6 & -2+7 \\
15+24 & -3+28
\end{array}\right]=\left[\begin{array}{cc}
16 & 5 \\
39 & 25
\end{array}\right]}\)

Hence, \(\left[\begin{array}{cc}
5 & -1 \\
6 & 7
\end{array}\right]\left[\begin{array}{ll}
2 & 1 \\
3 & 4
\end{array}\right] \neq\left[\begin{array}{ll}
2 & 1 \\
3 & 4
\end{array}\right]\left[\begin{array}{cc}
5 & -1 \\
6 & 7
\end{array}\right]\)

2. Here, \(\left[\begin{array}{ccc}
1 & 2 & 3 \\
0 & 1 & 0 \\
1 & 1 & 0
\end{array}\right]\left[\begin{array}{ccc}
-1 & 1 & 0 \\
0 & -1 & 1 \\
2 & 3 & 4
\end{array}\right]\)

=\(\left[\begin{array}{ccc}
-1+0+6 & 1-2+9 & 0+2+12 \\
0+0+0 & 0+(-1)+0 & 0+1+0 \\
-1+0+0 & 1-1+0 & 0+1+0
\end{array}\right]\)

=\(\left[\begin{array}{ccc}
5 & 8 & 14 \\
0 & -1 & 1 \\
-1 & 0 & 1
\end{array}\right]\) and

⇒ \(\left[\begin{array}{ccc}
-1 & 1 & 0 \\
0 & -1 & 1 \\
2 & 3 & 4
\end{array}\right]\left[\begin{array}{lll}
1 & 2 & 3 \\
0 & 1 & 0 \\
1 & 1 & 0
\end{array}\right]\)

= \(\left[\begin{array}{ccc}
-1+0+0 & -2+1+0 & -3+0+0 \\
0+0+1 & 0-1+1 & 0+0+0 \\
2+0+4 & 4+3+4 & 6+0+0
\end{array}\right]\)

= \(\left[\begin{array}{ccc}
-1 & -1 & -3 \\
1 & 0 & 0 \\
6 & 11 & 6
\end{array}\right]\)

⇒ \(\left[\begin{array}{lll}
1 & 2 & 3 \\
0 & 1 & 0 \\
1 & 1 & 0
\end{array}\right]\left[\begin{array}{ccc}
-1 & 1 & 0 \\
0 & -1 & 1 \\
2 & 3 & 4
\end{array}\right] \neq\left[\begin{array}{ccc}
-1 & 1 & 0 \\
0 & -1 & 1 \\
2 & 3 & 4
\end{array}\right]\left[\begin{array}{lll}
1 & 2 & 3 \\
0 & 1 & 0 \\
1 & 1 & 0
\end{array}\right]\)

Hence, the required result is verified.

Question 15. Find \(A^2-5 \)A+6 I if  A=\(\left[\begin{array}{ccc}
2 & 0 & 1 \\
2 & 1 & 3 \\
1 & -1 & 0
\end{array}\right]\)

Solution:

Here, \(A^2=A \cdot A\)=\(\left[\begin{array}{ccc}
2 & 0 & 1 \\
2 & 1 & 3 \\
1 & -1 & 0
\end{array}\right]\left[\begin{array}{ccc}
2 & 0 & 1 \\
2 & 1 & 3 \\
1 & -1 & 0
\end{array}\right]\)

= \(\left[\begin{array}{ccc}
4+0+1 & 0+0-1 & 2+0+0 \\
4+2+3 & 0+1-3 & 2+3+0 \\
2-2+0 & 0-1+0 & 1-3+0
\end{array}\right]\)

= \(\left[\begin{array}{ccc}
5 & -1 & 2 \\
9 & -2 & 5 \\
0 & -1 & -2
\end{array}\right]\)

⇒ \(\mathrm{A}^2-5 \mathrm{~A}+6 \mathrm{I}=\left[\begin{array}{ccc}
5 & -1 & 2 \\
9 & -2 & 5 \\
0 & -1 & -2
\end{array}\right]-5\left[\begin{array}{ccc}
2 & 0 & 1 \\
2 & 1 & 3 \\
1 & -1 & 0
\end{array}\right]+6\left[\begin{array}{lll}
1 & 0 & 0 \\
0 & 1 & 0 \\
0 & 0 & 1
\end{array}\right]\)

= \(\left[\begin{array}{ccc}
5 & -1 & 2 \\
9 & -2 & 5 \\
0 & -1 & -2
\end{array}\right]-\left[\begin{array}{ccc}
10 & 0 & 5 \\
10 & 5 & 15 \\
5 & -5 & 0
\end{array}\right]+\left[\begin{array}{lll}
6 & 0 & 0 \\
0 & 6 & 0 \\
0 & 0 & 6
\end{array}\right]\)

= \(\left[\begin{array}{ccc}
5-10+6 & -1-0+0 & 2-5+0 \\
9-10+0 & -2-5+6 & 5-15+0 \\
0-5+0 & -1+5+0 & -2-0+6
\end{array}\right]\)

= \(\left[\begin{array}{ccc}
1 & -1 & -3 \\
-1 & -1 & -10 \\
-5 & 4 & 4
\end{array}\right]\)

Question 16. If A=\(\left[\begin{array}{lll}
1 & 0 & 2 \\
0 & 2 & 1 \\
2 & 0 & 3
\end{array}\right]\)
 , prove that \(A^3-6 A^2+7\) A+2 I=0

Solution:

⇒ \(A^2=A \times A=\left[\begin{array}{lll}
1 & 0 & 2 \\
0 & 2 & 1 \\
2 & 0 & 3
\end{array}\right]\left[\begin{array}{lll}
1 & 0 & 2 \\
0 & 2 & 1 \\
2 & 0 & 3
\end{array}\right]\)

= \(\left[\begin{array}{lll}
1+0+4 & 0+0+0 & 2+0+6 \\
0+0+2 & 0+4+0 & 0+2+3 \\
2+0+6 & 0+0+0 & 4+0+9
\end{array}\right]\)

= \(\left[\begin{array}{ccc}
5 & 0 & 8 \\
2 & 4 & 5 \\
8 & 0 & 13
\end{array}\right]\)

⇒ \(\mathrm{A}^3=\mathrm{A}^2-\mathrm{A}=\left[\begin{array}{ccc}
5 & 0 & 8 \\
2 & 4 & 5 \\
8 & 0 & 13
\end{array}\right]\left[\begin{array}{lll}
1 & 0 & 2 \\
0 & 2 & 1 \\
2 & 0 & 3
\end{array}\right]\)

= \(\left[\begin{array}{ccc}
5+0+16 & 0+0+0 & 10+0+24 \\
2+0+10 & 0+8+0 & 4+4+15 \\
8+0+26 & 0+0+0 & 16+0+39
\end{array}\right]\)

= \(\left[\begin{array}{ccc}
21 & 0 & 34 \\
12 & 8 & 23 \\
34 & 0 & 55
\end{array}\right]\)

⇒\(\mathrm{A}^3-6 \mathrm{~A}^2+7 \mathrm{~A}+2 \mathrm{I}=\left[\begin{array}{lll}
21 & 0 & 34 \\
12 & 8 & 23 \\
34 & 0 & 55
\end{array}\right]-6\left[\begin{array}{ccc}
5 & 0 & 8 \\
2 & 4 & 5 \\
8 & 0 & 13
\end{array}\right]+7\left[\begin{array}{ccc}
1 & 0 & 2 \\
0 & 2 & 1 \\
2 & 0 & 3
\end{array}\right]+2\left[\begin{array}{lll}
1 & 0 & 0 \\
0 & 1 & 0 \\
0 & 0 & 1
\end{array}\right]\)

= \(\left[\begin{array}{lll}
21 & 0 & 34 \\
12 & 8 & 23 \\
34 & 0 & 55
\end{array}\right]-\left[\begin{array}{ccc}
30 & 0 & 48 \\
12 & 24 & 30 \\
48 & 0 & 78
\end{array}\right]+\left[\begin{array}{ccc}
7 & 0 & 14 \\
0 & 14 & 7 \\
14 & 0 & 21
\end{array}\right]+\left[\begin{array}{lll}
2 & 0 & 0 \\
0 & 2 & 0 \\
0 & 0 & 2
\end{array}\right]\)

= \(\left[\begin{array}{ccc}
21-30+7+2 & 0-0+0+0 & 34-48+14+0 \\
12-12+0+0 & 8-24+14+2 & 23-30+7+0 \\
34-48+14+0 & 0-0+0+0- & 55-78+21+2
\end{array}\right]\)

= \(\left[\begin{array}{lll}
0 & 0 & 0 \\
0 & 0 & 0 \\
0 & 0 & 0
\end{array}\right]\)=0

Question 17. If A=\(\left[\begin{array}{ll}
3 & -2 \\
4 & -2
\end{array}\right]\) and  I=\(\left[\begin{array}{ll}
1 & 0 \\
0 & 1
\end{array}\right]\)
, then find k so that \(A^2\)=k A-2 I.

Solution:

Given; \(A^2-k A-2 I \Rightarrow A \cdot A=k A-2 I\)

⇒ \(\Rightarrow\left[\begin{array}{cc}
3 & -2 \\
4 & -2
\end{array}\right]\left[\begin{array}{ll}
3 & -2 \\
4 & -2
\end{array}\right]\)

= \(k\left[\begin{array}{ll}
3 & -2 \\
4 & -2
\end{array}\right]-2\left[\begin{array}{cc}
1 & 0 \\
0 & 1
\end{array}\right]\)

⇒ \(\left[\begin{array}{cc}
9-8 & -6+4 \\
12-8 & -8+4
\end{array}\right]\)

= \(\left[\begin{array}{ll}
3 k & -2 k \\
4 k & -2 k
\end{array}\right]-\left[\begin{array}{ll}
2 & 0 \\
0 & 2
\end{array}\right] \Rightarrow\left[\begin{array}{cc}
1 & -2 \\
4 & -4
\end{array}\right]\)

= \(\left[\begin{array}{cc}
3 k-2 & -2 k \\
4 k & -2 k-2
\end{array}\right]\)

By definition of equality of matrix, as the given matrices are equal, their corresponding elements are also equal. Comparing the corresponding elements, we get.

3 \(\mathrm{k}-2=1 \Rightarrow \mathrm{k}=1 ;-2 \mathrm{k}=-2 \Rightarrow \mathrm{k}=1 ; 4 \mathrm{k}=4 \Rightarrow \mathrm{k}=1 ;-4=-2 \mathrm{k}-2 \Rightarrow \mathrm{k}\)=1

Hence, k=1

Question 18. If A=\(\left[\begin{array}{cc}
0 & -\tan \frac{\alpha}{2} \\
\tan \frac{\alpha}{2} & 0
\end{array}\right]\)
And I is the Identity matrix Of order 2, show that I+A=(I-A) \(\left[\begin{array}{cc}
\cos \alpha & -\sin \alpha \\
\sin \alpha & \cos \alpha
\end{array}\right]\)

Solution:

Here, \(\mathrm{A}=\left[\begin{array}{cc}
0 & -\mathrm{t} \\
\mathrm{t} & 0
\end{array}\right]\) where \(\mathrm{t}=\tan \left(\frac{\alpha}{2}\right)\)

Now, \(\cos \alpha=\frac{1-\tan ^2\left(\frac{\alpha}{2}\right)}{1+\tan ^2\left(\frac{\alpha}{2}\right)}=\frac{1-t^2}{1+t^2} and \sin \alpha=\frac{2 \tan \left(\frac{\alpha}{2}\right)}{1+\tan ^2\left(\frac{\alpha}{2}\right)}=\frac{2 t}{1+t^2}\)

RHS =(I-A)\(\left[\begin{array}{cc}
\cos \alpha & -\sin \alpha \\
\sin \alpha & \cos \alpha
\end{array}\right]\)

=\(\left[\left(\begin{array}{cc}
1 & 0 \\
0 & 1
\end{array}\right)-\left(\begin{array}{cc}
0 & -t \\
t & 0
\end{array}\right)\right]\left[\begin{array}{cc}
\frac{1-t^2}{1+t^2} & \frac{-2 t}{1+t^2} \\
\frac{2 t}{1+t^2} & \frac{1-t^2}{1+t^2}
\end{array}\right]\)

= \(\left[\begin{array}{cc}
1 & t \\
-t & 1
\end{array}\right]\left[\begin{array}{cc}
\frac{1-t^2}{1+t^2} & \frac{-2 t}{1+t^2} \\
\frac{2 t}{1+t^2} & \frac{1-t^2}{1+t^2}
\end{array}\right]\)

= \(\left[\begin{array}{cc}
\frac{1-t^2+2 t^2}{1+t^2} & \frac{-2 t+t\left(1-t^2\right)}{1+t^2} \\
\frac{-t\left(1-t^2\right)+2 t}{1+t^2} & \frac{2 t^2+1-t^2}{1+t^2}
\end{array}\right]\)

= \(\left[\begin{array}{cc}
\frac{1+t^2}{1+t^2} & \frac{-2 t+t-t^3}{1+t^2} \\
\frac{-t+t^3+2 t}{1+t^2} & \frac{2 t^2+1-t^2}{1+t^2}
\end{array}\right]\)

=\(\left[\begin{array}{cc}
\frac{1+t^2}{1+t^2} & \frac{-t\left(1+t^2\right)}{1+t^2} \\
\frac{t\left(1+t^2\right)}{r+t^2} & \frac{1+t^2}{1+t^2}
\end{array}\right]\)

= \(\left[\begin{array}{cc}
1 & -t \\
t & 1
\end{array}\right]=\left[\begin{array}{cc}
1 & -\tan \frac{\alpha}{2} \\
\tan \frac{\alpha}{2} & 1
\end{array}\right]\)

and L H S=I+A=\(\left[\begin{array}{ll}
1 & 0 \\
0 & 1
\end{array}\right]+\left[\begin{array}{cc}
0 & -t \\
t & 0
\end{array}\right]\)

=\(\left[\begin{array}{cc}
1 & -t \\
t & 1
\end{array}\right]=\left[\begin{array}{cc}
1 & -\tan \frac{\alpha}{2} \\
\tan \frac{\alpha}{2} & 1
\end{array}\right]\)=R H S

Question 19. A trust fund has (30,000 that must be invested in two different types of bonds. The first bond pays 5% interest per year and the second bond pays 7% interest per year. Using matrix multiplication. determine how to divide ₹30,000 among the two types of bonds, if the trust fund must obtain an annual total interest of (a) ₹1800 and (b) ₹2000.

Solution:

Let the amount invested in the first type of bond is x, and then that invested in the second type of bonds
will he ₹(30000 – x).

1. According to given condition,

⇒ \(\left[\begin{array}{ll}
\mathrm{x} & 30000-\mathrm{x}
\end{array}\right]\left[\begin{array}{c}
\frac{5}{100} \\
\frac{7}{100}
\end{array}\right]=[1800] \Rightarrow\left[\frac{5 \mathrm{x}}{100}+\frac{(30000-\mathrm{x}) 7}{100}\right]=[1800]\)

⇒ \(\left[\frac{5 x+210000-7 x}{100}\right]=[1800] \Rightarrow 210000-2 x=180000 \Rightarrow 30000=2 x \Rightarrow x\)=15000

⇒ Hence, the amounts invested in the two types of bonds are respectively 15000 and ₹(30000 – 15000) = ₹15000

2. According to the given condition.

⇒ \({[\mathrm{x} 30000-\mathrm{x}]\left[\begin{array}{c}
\frac{5}{100} \\
\frac{7}{100}
\end{array}\right]=[2000] }\)

⇒ \({\left[\frac{5 \mathrm{x}}{100}+\frac{(30000-\mathrm{x}) 7}{100}\right]=[2000] \Rightarrow\left[\frac{5 \mathrm{x}+210000-7 \mathrm{x}}{100}\right]=\{2000] }\)

⇒ \(-2 \mathrm{x}+210000=200000 \Rightarrow-2 \mathrm{x}=-10000 \Rightarrow \mathrm{x}\)=5000

Hence, the amounts invested in two types of bonds are respectively ₹ 5000 and ₹(30000-5000)=₹ 25000.

Question 20. The bookshop of a particular school has 10 dozen chemistry books, I dozen physics books, and 10 dozen economics books. Their selling prices are ₹80, ₹60, and ₹40 for one book of, each subject respectively. Find the total amount that the bookshop will receive from selling the books, using matrix algebra.

Solution:

Let A=\(\left[\begin{array}{lll}10 \times 12 & 8 \times 12 & 10 \times 12\end{array}\right]\) and B=\(\left[\begin{array}{l}80 \\ 60 \\ 40\end{array}\right]\)

The amount received by the bookseller on selling these types of tools can be computed by evaluating products A and B.

Now, A B =\([\begin{array}{lll}
120 & 96 & 120
\end{array}][\begin{array}{l}
80 \\
60 \\
40
\end{array}\)

=\([120 \times 80+96 \times 60+120 \times 40]_{1 \times 1}\)

=[9600+5760+4800]\(_{|\times|}=[20160]_{1 \times 1}\)

The amount received by the bookseller =₹ 20160

Question 21. The restrictions on n, k, and p so that PY + WY will be defined are

  1. k=3,p=n
  2. k is arbitrary, p=2
  3. p is arbitrary, k=3
  4. k = 2, p = 3

Solution:

1. Matrices P and Y are of the orders p x k and 3 x k respectively.

Therefore, matrix PY will be defined if k=3

Consequently, P Y will be of the order p x k. Matrices W and Y are of the orders n x 3 and 3 x k respectively.

Since the number of columns in W is equal to the number of rows in Y, matrix WY is well-defined and is of the order n x k.

Matrices PY and WY can be added only when their orders are the same.

However, PY is of the order p x k, therefore we must have p=n.

Thus, k=3 and p= n are the restrictions on n, k and p so that PY+WY will be defined.

Question 22. If \(\mathrm{n}=\mathrm{p}\) then the order of the matrix 7 \(\mathrm{X}\)-52 is

  1. \(\mathrm{p} \times 2\)
  2. \(2 \times \mathrm{n}\)
  3. \(n \times 3\)
  4. \(\mathrm{p} \times \mathrm{n}\)

Solution: 2. \(2 \times \mathrm{n}\)

Matrix \(\mathrm{X}\) is of the order 2 \(\times \mathrm{n}\).

Therefore, matrix 7 \(\mathrm{X}\) is also of the same order.

Matrix Z is of the order 2 \(\times pi.e. 2 \times n\)

[Since, \(\mathrm{n}=\mathrm{p} ]\)

Therefore, matrix 5 \(\mathrm{Z}\) is also of the same order.

Now, both the matrices 7 X and 5 Z are of the order 2 \(\times\) n

Thus, matrix 7 X-5 Z is well-defined and is of the order 2 \(\times\)n.

Matrices Exercise 3.3

Question 1. Find the transpose of Each of the following matrices.

  1. \(\left[\begin{array}{c}5 \\ 1 / 2 \\ -1\end{array}\right]\)
  2. \(\left[\begin{array}{cc}1 & -1 \\ 2 & 3\end{array}\right]\)
  3. \(\left[\begin{array}{ccc}-1 & 5 & 6 \\ \sqrt{3} & 5 & 6 \\ 2 & 3 & -1\end{array}\right]\)

Solution:

Let A=\(\left[\begin{array}{c}5 \\ 1 / 2 \\ -1\end{array}\right]_{3-1}, then A^{\prime}=\left[\begin{array}{lll}5 & 1 / 2 & -1\end{array}\right]_{1-3}\)

Let A=\(\left[\begin{array}{cc}1 & -1 \\ 2 & 3\end{array}\right]_{2 \times 2}, then A^{\prime}=\left[\begin{array}{cc}1 & 2 \\ -1 & 3\end{array}\right]_{2 \times 2}\)

Let A=\(\left[\begin{array}{ccc}-1 & 5 & 6 \\ \sqrt{3} & 5 & 6 \\ 2 & 3 & -1\end{array}\right]_{3 \ldots 5}, then A^{\prime}=\left[\begin{array}{ccc}-1 & \sqrt{3} & 2 \\ 5 & 5 & 3 \\ 6 & 6 & -1\end{array}\right]_{3 \times 1}\)

Question 2. If A=\(\left[\begin{array}{ccc}-1 & 2 & 3 \\ 5 & 7 & 9 \\ -2 & 1 & 1\end{array}\right]\) and B=\(\left[\begin{array}{ccc}-4 & 1 & -5 \\ 1 & 2 & 0 \\ 1 & 3 & 1\end{array}\right]\), then verify that

  1. \((A+B)^{\prime}=A^{\prime}+B^{\prime}\)
  2. \((\mathrm{A}-\mathrm{B})^{\prime}=\mathrm{A}^{\prime}-\mathrm{B}^{\prime}\)

Solution:

1. Here, A+B=\(\left[\begin{array}{ccc}
-1 & 2 & 3 \\
5 & 7 & 9 \\
-2 & 1 & 1
\end{array}\right]+\left[\begin{array}{ccc}
-4 & 1 & -5 \\
1 & 2 & 0 \\
1 & 3 & 1
\end{array}\right]\)

= \(\left[\begin{array}{ccc}
-5 & 3 & -2 \\
6 & 9 & 9 \\
-1 & 4 & 2
\end{array}\right]\)

⇒ \((A+B)^{\prime}=\left[\begin{array}{ccc}
-5 & 6 & -1 \\
3 & 9 & 4 \\
-2 & 9 & 2
\end{array}\right]\)

Also \(A^{\prime}+B^{\prime}=\left[\begin{array}{ccc}
-1 & 2 & 3 \\
5 & 7 & 9 \\
-2 & 1 & 1
\end{array}\right]^{\prime}+\left[\begin{array}{ccc}
-4 & 1 & -5 \\
1 & 2 & 0 \\
1 & 3 & 1
\end{array}\right]^{\prime}\)

=\(\left[\begin{array}{ccc}
-1 & 5 & -2 \\
2 & 7 & 1 \\
3 & 9 & 1
\end{array}\right]+\left[\begin{array}{ccc}
-4 & 1 & 1 \\
1 & 2 & 3 \\
-5 & 0 & 1
\end{array}\right]\)

= \(\left[\begin{array}{ccc}
-5 & 6 & -1 \\
3 & 9 & 4 \\
-2 & 9 & 2
\end{array}\right]\)

From Eq. (1) and (2), it is verified that }\((A+B)^{\prime}=A^{\prime}+B^{\prime}\).

2. \( Here, A-B=\left[\begin{array}{ccc}
-1 & 2 & 3 \\
5 & 7 & 9 \\
-2 & 1 & 1
\end{array}\right]-\left[\begin{array}{ccc}
-4 & 1 & -5 \\
1 & 2 & 0 \\
1 & 3 & 1
\end{array}\right]\)

= \(\left[\begin{array}{ccc}
-1+4 & 2-1 & 3+5 \\
5-1 & 7-2 & 9-0 \\
-2-1 & 1-3 & 1-1
\end{array}\right]=\left[\begin{array}{ccc}
3 & 1 & 8 \\
4 & 5 & 9 \\
-3 & -2 & 0
\end{array}\right]\)

⇒ \(\Rightarrow(A-B)^{\prime}=\left[\begin{array}{ccc}
3 & 4 & -3 \\
1 & 5 & -2 \\
8 & 9 & 0
\end{array}\right]\)

Also, \(A^{\prime}-B^{\prime}=\left[\begin{array}{ccc}-1 & 5 & -2 \\ 2 & 7 & 1 \\ 3 & 9 & 1\end{array}\right]-\left[\begin{array}{ccc}-4 & 1 & 1 \\ 1 & 2 & 3 \\ -5 & 0 & 1\end{array}\right]\)

= \(\left[\begin{array}{ccc}-1+4 & 5-1 & -2-1 \\ 2-1 & 7-2 & 1-3 \\ 3-(-5) & 9-0 & 1-1\end{array}\right]=\left[\begin{array}{ccc}3 & 4 & -3 \\ 1 & 5 & -2 \\ 8 & 9 & 0\end{array}\right]\)

From Eqs. (1) and (2), it is verified that \((A-B)^{\prime}=A^{\prime}-B^{\prime}\).

Question 3. If \(A^{\prime}=\left[\begin{array}{cc}3 & 4 \\ -1 & 2 \\ 0 & 1\end{array}\right]\) and B=\(\left[\begin{array}{ccc}-1 & 2 & 1 \\ 1 & 2 & 3\end{array}\right]\), then verify that

1.\((\mathrm{A}+\mathrm{B})^{\prime}=\mathrm{A}^{\prime}+\mathrm{B}^{\prime}\)

2.\((\mathrm{A}-\mathrm{B})^{\prime}=\mathrm{A}^{\prime}-\mathrm{B}^{\prime}\)

Solution:

A=\(\left(A^{\prime}\right)^{\prime}=\left[\begin{array}{cc}
3 & 4 \\
-1 & 2 \\
0 & 1
\end{array}\right]^{\prime}=\left[\begin{array}{ccc}
3 & -1 & 0 \\
4 & 2 & 1
\end{array}\right]\)

Here,A+B=\(\left[\begin{array}{ccc}
3 & -1 & 0 \\
4 & 2 & 1
\end{array}\right]+\left[\begin{array}{ccc}
-1 & 2 & 1 \\
1 & 2 & 3
\end{array}\right]=\left[\begin{array}{lll}
2 & 1 & 1 \\
5 & 4 & 4
\end{array}\right]\)

⇒ \((A+B)^{\prime}=\left[\begin{array}{lll}
2 & 1 & 1 \\
5 & 4 & 4
\end{array}\right]^{\prime}=\left[\begin{array}{ll}
2 & 5 \\
1 & 4 \\
1 & 4
\end{array}\right]\)  → Equation 1

⇒ \(B^{\prime}=\left[\begin{array}{ccc}
-1 & 2 & 1 \\
1 & 2 & 3
\end{array}\right]=\left[\begin{array}{cc}
-1 & 1 \\
2 & 2 \\
1 & 3
\end{array}\right], A^{\prime}=\left[\begin{array}{cc}
3 & 4 \\
-1 & 2 \\
0 & 1
\end{array}\right]\)

\(A^{\prime}+B^{\prime}=\left[\begin{array}{cc}
3 & 4 \\
-1 & 2 \\
0 & 1
\end{array}\right]+\left[\begin{array}{cc}
-1 & 1 \\
2 & 2 \\
1 & 3
\end{array}\right]=\left[\begin{array}{ll}
2 & 5 \\
1 & 4 \\
1 & 4
\end{array}\right]\)  → Equation 2

From equations (1) and (2), it is verified that \((A+B)^{\prime}=A^{\prime}+B^{\prime}\).

RHS =\(A^{\prime}-B^{\prime}=\left[\begin{array}{cc}
3 & 4 \\
-1 & 2 \\
0 & 1
\end{array}\right]-\left[\begin{array}{cc}
-1 & 1 \\
2 & 2 \\
1 & 3
\end{array}\right]\)

=\(\left[\begin{array}{cc}
3+1 & 4-1 \\
-1-2 & 2-2 \\
0-1 & 1-3
\end{array}\right]\)

=\(\left[\begin{array}{cc}
4 & 3 \\
-3 & 0 \\
-1 & -2
\end{array}\right]\)

LHS =\((A-B)^{\prime}=\left(\left[\begin{array}{ccc}
3 & -1 & 0 \\
4 & 2 & 1
\end{array}\right]-\left[\begin{array}{ccc}
-1 & 2 & 1 \\
1 & 2 & 3
\end{array}\right]\right)\)

=\(\left[\begin{array}{ccc}
4 & -3 & -1 \\
3 & 0 & -2
\end{array}\right]=\left[\begin{array}{cc}
4 & 3 \\
-3 & 0 \\
-1 & -2
\end{array}\right]\)

From equation (1) and (2), it is verified that \((A-B)^{\prime}=A^{\prime}-B^{\prime}\)

Question 4. If \(A^{\prime}=\left[\begin{array}{cc}
-2 & 3 \\
1 & 2
\end{array}\right]\)
and B=\(\left[\begin{array}{cc}
-1 & 0 \\
1 & 2
\end{array}\right]\)
then find A+2 \(B)^{\prime}\)

Solution:

⇒ \( Given,A^{\prime}=\left[\begin{array}{cc}
-2 & 3 \\
1 & 2
\end{array}\right]\)

A=\(\left(A^{\prime}\right)^{\prime}=\left[\begin{array}{cc}
-2 & 3 \\
1 & 2
\end{array}\right]^{\prime}=\left[\begin{array}{cc}
-2 & 1 \\
3 & 2
\end{array}\right]\)

and \(2 B=2\left[\begin{array}{cc}
-1 & 0 \\
1 & 2
\end{array}\right]=\left[\begin{array}{cc}
-2 & 0 \\
2 & 4
\end{array}\right]\)

A+2 B=\(\left[\begin{array}{cc}
-2 & 1 \\
3 & 2
\end{array}\right]+\left[\begin{array}{cc}
-2 & 0 \\
2 & 4
\end{array}\right]=\left[\begin{array}{cc}
-4 & 1 \\
5 & 6
\end{array}\right]\)

⇒ \((A+2 B)^{\prime}=\left[\begin{array}{cc}
-4 & 1 \\
5 & 6
\end{array}\right]=\left[\begin{array}{cc}
-4 & 5 \\
1 & 6
\end{array}\right]\)

Question 5. For the matrices, A and B, verify that \((A B)^{\prime}=B^{\prime} A^{\prime}\), where

  1. \(\mathrm{A}=\left[\begin{array}{c}1 \\ -4 \\ 3\end{array}\right], \mathrm{B}=\left[\begin{array}{lll}-1 & 2 & 1\end{array}\right]\)
  2. \(\mathrm{A}=\left[\begin{array}{l}0 \\ 1 \\ 2\end{array}\right], \mathrm{B}=\left[\begin{array}{lll}1 & 5 & 7\end{array}\right]\)

Solution:

1. Here, A B=\(\left[\begin{array}{c}1 \\ -4 \\ 3\end{array}\right]\left[\begin{array}{lll}-1 & 2 & 1\end{array}\right]=\left[\begin{array}{ccc}-1 & 2 & 1 \\ 4 & -8 & -4 \\ -3 & 6 & 3\end{array}\right]\)

\((\mathrm{AB})^{\prime}=\left[\begin{array}{ccc}
-1 & 2 & 1 \\
4 & -8 & -4 \\
-3 & 6 & 3
\end{array}\right]^{\prime}=\left[\begin{array}{ccc}
-1 & 4 & -3 \\
2 & -8 & 6 \\
1 & -4 & 3
\end{array}\right]\)  →Equation 1

Also, \(\mathrm{B}^{\prime} \mathrm{A}^{\prime}=\left[\begin{array}{lll}-1 & 2 & 1\end{array}\right]^{\prime}\left[\begin{array}{c}1 \\ -4 \\ 3\end{array}\right]^{\prime}=\left[\begin{array}{c}-1 \\ 2 \\ 1\end{array}\right]\left[\begin{array}{lll}1 &-4 & 3\end{array}\right]\)

= \(\left[\begin{array}{ccc}-1 & 4 & -3 \\ 2 & -8 & 6 \\ 1 & -4 & 3\end{array}\right]\)  → Equation 2

From Eqs. (1) and (2), we find that \((A B)^{\prime}=B^{\prime} A^{\prime}\)

2. Here, \((\mathrm{AB})=\left[\begin{array}{l}0 \\ 1 \\ 2\end{array}\right]\left[\begin{array}{lll}1 & 5 & 7\end{array}\right]=\left[\begin{array}{ccc}0 & 0 & 0 \\ 1 & 5 & 7 \\ 2 & 10 & 14\end{array}\right]\)

⇒ \((\mathrm{AB})^{\prime}=\left[\begin{array}{ccc}0 & 0 & 0 \\ 1 & 5 & 7 \\ 2 & 10 & 14\end{array}\right]^{\prime}=\left[\begin{array}{ccc}0 & 1 & 2 \\ 0 & 5 & 10 \\ 0 & 7 & 14\end{array}\right]\)  → Equation 1

Also, \(B’\mathrm{A}^{\prime}=\left[\begin{array}{lll}1 & 5 & 7\end{array}\right]^{\prime}\left[\begin{array}{l}0 \\ 1 \\ 2\end{array}\right]^{\prime}=\left[\begin{array}{l}1 \\ 5 \\ 7\end{array}\right]\left[\begin{array}{lll}0 & 1 & 2\end{array}\right]=\left[\begin{array}{lll}0 & 1 & 2 \\ 0 & 5 & 10 \\ 0 & 7 & 14\end{array}\right]\)  → Equation 2

From Eq, (1) and (2), it is verified that \((A B)^{\prime}=B^{\prime} A^{\prime}\).

Question 6. 1. If A=\(\left[\begin{array}{cc}\cos \alpha & \sin \alpha \\ -\sin \alpha & \cos \alpha\end{array}\right]\), then verify that \(A^{\prime}\) A=I.

2. If A=\(\left[\begin{array}{cc}\sin \alpha & \cos \alpha \\ -\cos \alpha & \sin \alpha\end{array}\right]\), then verify that \(A^{\prime}\) A=1.

Solution:

1. Here, \(\mathrm{A}=\left[\begin{array}{cc}
\cos \alpha & \sin \alpha \\
-\sin \alpha & \cos \alpha
\end{array}\right] \)

⇒ \(\mathrm{A}^{\prime}=\left[\begin{array}{cc}
\cos \alpha & \sin \alpha \\
-\sin \alpha & \cos \alpha
\end{array}\right]^{\prime}=\left[\begin{array}{cc}
\cos \alpha & -\sin \alpha \\
\sin \alpha & \cos \alpha
\end{array}\right]\)

⇒ \(\mathrm{A}^{\prime} \mathrm{A}=\left[\begin{array}{cc}
\cos \alpha & -\sin \alpha \\
\sin \alpha & \cos \alpha
\end{array}\right]\left[\begin{array}{cc}
\cos \alpha & \sin \alpha \\
-\sin \alpha & \cos \alpha
\end{array}\right] \)

= \(\left[\begin{array}{cc}
(\cos \alpha)(\cos \alpha)+(\sin \alpha)(\sin \alpha) & (\cos \alpha)(\sin \alpha)+(-\sin \alpha)(\cos \alpha) \\
(\sin \alpha)(\cos \alpha)+(\cos \alpha)(-\sin \alpha) & (\sin \alpha)(\sin \alpha)+(\cos \alpha)(\cos \alpha)
\end{array}\right]\)

= \(\left[\begin{array}{cc}
\cos ^2 \alpha+\sin ^2 \alpha & 0 \\
0 & \sin ^2 \alpha+\cos ^2 \alpha
\end{array}\right]\)

=\(\left[\begin{array}{ll}
1 & 0 \\
0 & 1
\end{array}\right]=I\left[\sin ^2 \alpha+\cos ^2 \alpha=1\right]\)

2. Here, A=\(\left[\begin{array}{cc}
\sin \alpha & \cos \alpha \\
-\cos \alpha & \sin \alpha
\end{array}\right] \)

⇒ \(A^{\prime}=\left[\begin{array}{cc}
\sin \alpha & \cos \alpha \\
-\cos \alpha & \sin \alpha
\end{array}\right]^{\prime}=\left[\begin{array}{cc}
\sin \alpha & -\cos \alpha \\
\cos \alpha & \sin \alpha
\end{array}\right]\)

⇒ \(\mathrm{A}^{\prime} \mathrm{A}=\left[\begin{array}{cc}
\sin \alpha & -\cos \alpha \\
\cos \alpha & \sin \alpha
\end{array}\right]\left[\begin{array}{cc}
\sin \alpha & \cos \alpha \\
-\cos \alpha & \sin \alpha
\end{array}\right]\)

=\(\left[\begin{array}{cc}
(\sin \alpha)(\sin \alpha)+(-\cos \alpha)(-\cos \alpha) & (\sin \alpha)(\cos \alpha)+(-\cos \alpha)(\sin \alpha) \\
(\sin \alpha)(\cos \alpha)+(\cos \alpha)(-\sin \alpha) & (\cos \alpha)(\cos \alpha)+(\sin \alpha)(\sin \alpha)
\end{array}\right]\)

= \(\left[\begin{array}{cc}
\sin ^2 \alpha+\cos ^2 \alpha & \sin \alpha \cos \alpha-\cos \alpha \sin \alpha \\
\sin \alpha \cos \alpha-\sin \alpha \cos \alpha & \cos ^2 \alpha+\sin ^2 \alpha
\end{array}\right]\)

=\(\left[\begin{array}{ll}
1 & 0 \\
0 & 1
\end{array}\right]=1\left[\sin ^2 \alpha+\cos ^2 \alpha=1\right]\)

Hence, we verified that \(A^{\prime}\) A=1

Question 7. 1. Show that the matrix, A=\(\left[\begin{array}{ccc}1 & -1 & 5 \\ -1 & 2 & 1 \\ 5 & 1 & 3\end{array}\right]\) is a Symmetric mâtrix.

2. Show that the matrix, A=\(\left[\begin{array}{ccc}0 & 1 & -1 \\ -1 & 0 & 1 \\ 1 & -1 & 0\end{array}\right]\) is a Skew-symmetric matrix.

Solution:

1. Here, A=\(\left[\begin{array}{ccc}1 & -1 & 5 \\ -1 & 2 & 1 \\ 5 & 1 & 3\end{array}\right]\)

⇒ \(A^{\prime}=\left[\begin{array}{ccc}1 & -1 & 5 \\ -1 & 2 & 1 \\ 5 & 1 & 3\end{array}\right]=\left[\begin{array}{ccc}1 & -1 & 5 \\ -1 & 2 & 1 \\ 5 & 1 & 3\end{array}\right]\)=A

⇒ \(\mathrm{A}^{\prime}=\mathrm{A}\). Hence, A is a symmetric matrix.

2. Here, \(\mathrm{A}=\left[\begin{array}{ccc}0 & 1 & -1 \\ -1 & 0 & 1 \\ 1 & -1 & 0\end{array}\right]\)

\(\mathrm{A}^{\prime}=\left[\begin{array}{ccc}0 & 1 & -1 \\ -1 & 0 & 1 \\ 1 & -1 & 0\end{array}\right]=\left[\begin{array}{ccc}0 & -1 & 1 \\ 1 & 0 & -1 \\ -1 & 1 & 0\end{array}\right]\)

=-\(\left[\begin{array}{ccc}0 & 1 & -1 \\ -1 & 0 & 1 \\ 1 & -1 & 0\end{array}\right]=-\mathrm{A}\)

∴ \(\mathrm{A}^{+}=-\mathrm{A}\). Hence, \(\mathrm{A}\) is skew-symmetric matrix.

Question 8. For the matrix A=\(\left[\begin{array}{ll}1 & 5 \\ 6 & 7\end{array}\right]\), verify that:

1. \(\left(\mathrm{A}+\mathrm{A}^{\prime}\right)\) is a symmetric matrix.

2.\(\left(\mathrm{A}-\mathrm{A}^{\prime}\right)\) is a skew-symmetric matrix.

Solution:

Given, A=\(\left[\begin{array}{ll}
1 & 5 \\
6 & 7
\end{array}\right]\)

⇒ \(A+A^{\prime}=\left[\begin{array}{ll}
1 & 5 \\
6 & 7
\end{array}\right]+\left[\begin{array}{ll}
1 & 5 \\
6 & 7
\end{array}\right]^{\prime}\)

=\(\left[\begin{array}{ll}
1 & 5 \\
6 & 7
\end{array}\right]+\left[\begin{array}{ll}
1 & 6 \\
5 & 7
\end{array}\right]=\left[\begin{array}{cc}
2 & 11 \\
11 & 14
\end{array}\right]\)

⇒ \(\Rightarrow A+A^{\prime}=\left[\begin{array}{ll}
2 & 11 \\
11 & 14
\end{array}\right] and \left(A+A^{\prime}\right)^{\prime}=\left[\begin{array}{cc}
2 & 11 \\
11 & 14
\end{array}\right]=\left[\begin{array}{cc}
2 & 11 \\
11 & 14
\end{array}\right]=A+A^{\prime}\)

⇒ \((\mathrm{A}+\mathrm{A}^{\prime})^{\prime}=\mathrm{A}+\mathrm{A}^{\prime}\), So,\((\mathrm{A}+\mathrm{A}^{\prime})\) is a symmetric matrix.

2. \(A-A^{\prime}=\left[\begin{array}{ll}
1 & 5 \\
6 & 7
\end{array}\right]-\left[\begin{array}{ll}
1 & 5 \\
6 & 7
\end{array}\right]^{\prime}\)

= \(\left[\begin{array}{ll}
1 & 5 \\
6 & 7
\end{array}\right]-\left[\begin{array}{ll}
1 & 6 \\
5 & 7
\end{array}\right]=\left[\begin{array}{cc}
0 & -1 \\
1 & 0
\end{array}\right]\)

⇒ \(A-A^{\prime}=\left[\begin{array}{cc}
0 & -1 \\
1 & 0
\end{array}\right] and \left(A-A^{\prime}\right)^{\prime}=\left[\begin{array}{cc}
0 & -1 \\
1 & 0
\end{array}\right]^{\prime}=\left[\begin{array}{cc}
0 & 1 \\
-1 & 0
\end{array}\right]\)

=-\(\left[\begin{array}{cc}
0 & -1 \\
1 & 0
\end{array}\right]=-\left(A-A^{\prime}\right)\)

⇒ \((A-A)^{\prime}=-\left(A-A^{\prime}\right)\)

Hence, \(\left(\mathrm{A}-\mathrm{A}^{\prime}\right)\) is a skew-symmetric matrix.

Question 9. Find \(\frac{1}{2}\left(A+A^{\prime}\right) and \frac{I}{2}\left(A-A^{\prime}\right)\), when A=\(\left[\begin{array}{ccc}0 & a & b \\ -a & 0 & c \\ -b & -c & 0\end{array}\right]\)

Solution:

Given A=\(\left[\begin{array}{ccc}0 & a & b \\ -a & 0 & c \\ -b & -c & 0\end{array}\right]\)

Now,\(\frac{1}{2}\left(A+A^{\prime}\right)=\frac{1}{2}\left(\left[\begin{array}{ccc}0 & a & b \\ -a & 0 & c \\ -b & -c & 0\end{array}\right]+\left[\begin{array}{ccc}0 & a & b \\ -a & 0 & c \\ -b & -c & 0\end{array}\right]\right)\)

=\(\frac{1}{2}\left(\left[\begin{array}{ccc}
0 & a & b \\
-a & 0 & c \\
-b & -c & 0
\end{array}\right]+\left[\begin{array}{ccc}
0 & -a & -b \\
a & 0 & -c \\
b & c & 0
\end{array}\right]\right)\)

=\(\left[\begin{array}{ccc}
0 & 0 & 0 \\
0 & 0 & 0 \\
0 & 0 & 0
\end{array}\right]\)

and \(\frac{1}{2}\left(A-A^{\prime}\right)=\frac{1}{2}\left(\left[\begin{array}{ccc}
0 & a & b \\
-a & 0 & c \\
-b & -c & 0
\end{array}\right]-\left[\begin{array}{ccc}
0 & a & b \\
-a & 0 & c \\
-b & -c & 0
\end{array}\right]\right)\)

= \(\frac{1}{2}\left(\left[\begin{array}{ccc}
0 & a & b \\
-a & 0 & c \\
-b & -c & 0
\end{array}\right]-\left[\begin{array}{ccc}
0 & -a & -b \\
a & 0 & -c \\
b & c & 0
\end{array}\right]\right)\)

=\(\frac{1}{2}\left[\begin{array}{ccc}
0 & 2 a & 2 b \\
-2 a & 0 & 2 c \\
-2 b & -2 c & 0
\end{array}\right]=\left[\begin{array}{ccc}
0 & a & b \\
-a & 0 & c \\
-b & -c & 0
\end{array}\right]\)

Question 10. Express the following matrices as the sum of a symmetric and a skew-symmetric matrix.

  1. \(\left[\begin{array}{cc}3 & 5 \\ 1 & -1\end{array}\right]\)
  2. \(\left[\begin{array}{ccc}6 & -2 & 2 \\ -2 & 3 & -1 \\ 2 & -1 & 3\end{array}\right]\)
  3. \(\left[\begin{array}{ccc}3 & 3 & -1 \\ -2 & -2 & 1 \\ -4 & -5 & 2\end{array}\right]\)
  4. \(\left[\begin{array}{cc}1 & 5 \\ -1 & 2\end{array}\right]\)

Solution:

1. A square matrix A can be expressed as the sum of a symmetric and skew-symmetric matrix.

⇒ \(\mathrm{A}=\frac{1}{2}\left[\mathrm{~A}+\mathrm{A}^{\prime}\right]+\frac{1}{2}\left[\mathrm{~A}-\mathrm{A}^{\prime}\right]\)

Let A=\(\left[\begin{array}{cc}3 & 5 \\ 1 & -1\end{array}\right], A^{\prime}=\left[\begin{array}{cc}3 & 1 \\ 5 & -1\end{array}\right]\)

Now,\(\frac{1}{2}\left(A+A^{\prime}\right)=\frac{1}{2}\left(\left[\begin{array}{cc}3 & 5 \\ 1 & -1\end{array}\right]+\left[\begin{array}{cc}3 & 1 \\ 5 & -1\end{array}\right]\right)\)

= \(\frac{1}{2}\left[\begin{array}{cc}6 & 6 \\ 6 & -2\end{array}\right]=\left[\begin{array}{cc}3 & 3 \\ 3 & -1\end{array}\right]=P( let )\)

⇒ \(P^{\prime}=\left[\begin{array}{cc}
3 & 3 \\
3 & -1
\end{array}\right]^{\prime}=\left[\begin{array}{cc}
3 & 3 \\
3 & -1
\end{array}\right]=\mathrm{P}\)

Thus,\(\mathrm{P}=\frac{1}{2}\left(\mathrm{~A}+\mathrm{A}^{\prime}\right)\) is a symmetric matrix.

Again,\(\frac{1}{2}\left(\mathrm{~A}-\mathrm{A}^{\prime}\right)=\frac{1}{2}\left(\left[\begin{array}{cc}3 & 5 \\ 1 & -1\end{array}\right]-\left[\begin{array}{cc}3 & 1 \\ 5 & -1\end{array}\right]\right)\)

=\(\frac{1}{2}\left[\begin{array}{cc}0 & 4 \\ -4 & 0\end{array}\right]=\left[\begin{array}{cc}0 & 2 \\ -2 & 0\end{array}\right]=\mathrm{Q}((let)\)

\(Q^{\prime}=\left[\begin{array}{cc}0 & 2 \\ -2 & 0\end{array}\right]^{\prime}=\left[\begin{array}{cc}0 & -2 \\ 2 & 0\end{array}\right]=-\left[\begin{array}{cc}0 & 2 \\ -2 & 0\end{array}\right]\)=-Q

Thus,\(\mathrm{Q}=\frac{1}{2}\left(\mathrm{~A}-\mathrm{A}^{\prime}\right)\) is a skew – symmetric matrix.

Now, P+Q=\(\left[\begin{array}{cc}3 & 3 \\ 3 & -1\end{array}\right]+\left[\begin{array}{cc}0 & 2 \\ -2 & 0\end{array}\right]=\left[\begin{array}{cc}3 & 5 \\ 1 & -1\end{array}\right]=\mathrm{A}\).

Hence, matrix A is the sum of a symmetric matrix and a Skew-Symmetric matrix.

2. A square matrix A can be expressed as the sum of a symmetric and skew symmetric matrices.

Let A=\(\left[\begin{array}{ccc}6 & -2 & 2 \\ -2 & 3 & -1 \\ 2 & -1 & 3\end{array}\right]\), then \(A^{\prime}=\left[\begin{array}{ccc}6 & -2 & 2 \\ -2 & 3 & -1 \\ 2 & -1 & 3\end{array}\right]=\left[\begin{array}{ccc}6 & -2 & 2 \\ -2 & 3 & -1 \\ 2 & -1 & 3\end{array}\right]\)

Now, \(A+A^{\prime}=\left[\begin{array}{ccc}6 & -2 & 2 \\ -2 & 3 & -1 \\ 2 & -1 & 3\end{array}\right]+\left[\begin{array}{ccc}6 & -2 & 2 \\ -2 & 3 & -1 \\ 2 & -1 & 3\end{array}\right]\)

=\(\left[\begin{array}{ccc}12 & -4 & 4 \\ -4 & 6 & -2 \\ 4 & -2 & 6\end{array}\right]\)

Let,P=\(\frac{1}{2}\left(A+A^{\prime}\right)=\frac{1}{2}\left[\begin{array}{ccc}12 & -4 & 4 \\ -4 & 6 & -2 \\ 4 & -2 & 6\end{array}\right]=\left[\begin{array}{ccc}6 & -2 & 2 \\ -2 & 3 & -1 \\ 2 & -1 & 3\end{array}\right]\)

⇒ \(P^{\prime}=\left[\begin{array}{ccc}
6 & -2 & 2 \\
-2 & 3 & -1 \\
2 & -1 & 3
\end{array}\right]\)

= \(\left[\begin{array}{ccc}
6 & -2 & 2 \\
-2 & 3 & -1 \\
2 & -1 & 3
\end{array}\right]\)=P

Thus,\(\mathrm{P}=\frac{1}{2}\left(\mathrm{~A}+\mathrm{A}^{\prime}\right)\) is a symmetric matrix.

Now, \(A-A^{\prime}=\left[\begin{array}{ccc}6 & -2 & 2 \\ -2 & 3 & -1 \\ 2 & -1 & 3\end{array}\right]-\left[\begin{array}{ccc}6 & -2 & 2 \\ -2 & 3 & -1 \\ 2 & -1 & 3\end{array}\right]\)

= \(\left[\begin{array}{lll}0 & 0 & 0 \\ 0 & 0 & 0 \\ 0 & 0 & 0\end{array}\right]\)

LetQ=\(\frac{1}{2}\left(A-A^{\prime}\right)=\frac{1}{2}\left[\begin{array}{lll}0 & 0 & 0 \\ 0 & 0 & 0 \\ 0 & 0 & 0\end{array}\right]=\left[\begin{array}{lll}0 & 0 & 0 \\ 0 & 0 & 0 \\ 0 & 0 & 0\end{array}\right]\)

⇒ \(Q^{\prime}=\left[\begin{array}{lll}
0 & 0 & 0 \\
0 & 0 & 0 \\
0 & 0 & 0
\end{array}\right]\)=-Q

Thus,\(\mathrm{Q}=\frac{1}{2}\left(\mathrm{~A}-\mathrm{A}^{\prime}\right)\) is a skew-symmetric matrix.

Now, P+Q=\(\left[\begin{array}{ccc}6 & -2 & 2 \\ -2 & 3 & -1 \\ 2 & -1 & 3\end{array}\right]+\left[\begin{array}{lll}0 & 0 & 0 \\ 0 & 0 & 0 \\ 0 & 0 & 0\end{array}\right]=\left[\begin{array}{ccc}6 & -2 & 2 \\ -2 & 3 & -1 \\ 2 & -1 & 3\end{array}\right]\)=A

Hence, matrix A is the sum of a symmetric matrix and a Skew-Symmetric matrix”

3. A square matrix A can be expressed as the sum of symmetric and skew-symmetric matrices

\(\mathrm{A}=\frac{1}{2}\left[\mathrm{~A}+\mathrm{A}^{\prime}\right]+\frac{1}{2}\left[\mathrm{~A}-\mathrm{A}^{\prime}\right]\)

Let A=\(\left[\begin{array}{ccc}3 & 3 & -1 \\ -2 & -2 & 1 \\ -4 & -5 & 2\end{array}\right], then A^{\prime}=\left[\begin{array}{ccc}3 & -2 & -4 \\ 3 & -2 & -5 \\ -1 & 1 & 2\end{array}\right]\)

Now, \(A+A^{\prime}=\left[\begin{array}{ccc}3 & 3 & -1 \\ -2 & -2 & 1 \\ -4 & -5 & 2\end{array}\right]+\left[\begin{array}{ccc}3 & -2 & -4 \\ 3 & -2 & -5 \\ -1 & 1 & 2\end{array}\right]=\left[\begin{array}{ccc}6 & 1 & -5 \\ 1 & -4 & -4 \\ -5 & -4 & 4\end{array}\right]\)

⇒ \(\mathrm{P}=\frac{1}{2}\left(\mathrm{~A}+\mathrm{A}^{\prime}\right)=\frac{1}{2}\left[\begin{array}{ccc}
6 & 1 & -5 \\
1 & -4 & -4 \\
-5 & -4 & 4
\end{array}\right]\)

=\(\left[\begin{array}{ccc}
3 & 1 / 2 & -5 / 2 \\
1 / 2 & -2 & -2 \\
-5 / 2 & -2 & 2
\end{array}\right]\)

⇒ \(\mathrm{P}^{\prime}=\left[\begin{array}{ccc}
3 & 1 / 2 & -5 / 2 \\
1 / 2 & -2 & -2 \\
-5 / 2 & -2 & 2
\end{array}\right]=\left[\begin{array}{ccc}
3 & 1 / 2 & -5 / 2 \\
1 / 2 & -2 & -2 \\
-5 / 2 & -2 & 2
\end{array}\right]=\mathrm{P}\)

Thus,\(\mathrm{P}=\frac{1}{2}\left(\mathrm{~A}+\mathrm{A}^{\prime}\right)\) is a symmetric matrix.

Now,\(A-A^{\prime}=\left[\begin{array}{ccc}3 & 3 & -1 \\ -2 & -2 & 1 \\ -4 & -5 & 2\end{array}\right]-\left[\begin{array}{ccc}3 & -2 & -4 \\ 3 & -2 & -5 \\ -1 & 1 & 2\end{array}\right]=\left[\begin{array}{ccc}0 & 5 & 3 \\ -5 & 0 & 6 \\ -3 & -6 & 0\end{array}\right]\)

Let,Q=\(\frac{1}{2}\left(A-A^{\prime}\right)=\frac{1}{2}\left[\begin{array}{ccc}0 & 5 & 3 \\ -5 & 0 & 6 \\ -3 & -6 & 0\end{array}\right]=\left[\begin{array}{ccc}0 & 5 / 2 & 3 / 2 \\ -5 / 2 & 0 & 3 \\ -3 / 2 & -3 & 0\end{array}\right]\)

⇒ \(Q^{\prime}=\left[\begin{array}{ccc}
0 & 5 / 2 & 3 / 2 \\
-5 / 2 & 0 & 3 \\
-3 / 2 & -3 & 0
\end{array}\right]^{\prime}=\left[\begin{array}{ccc}
0 & -5 / 2 & -3 / 2 \\
5 / 2 & 0 & -3 \\
3 / 2 & 3 & 0
\end{array}\right]=-\mathrm{Q}\)

Thus, Q=\(\frac{1}{2}\left(A-A^{\prime}\right)\) is a skew – symmetric matrix.

Now, \(\mathrm{P}+\mathrm{Q}=\left[\begin{array}{ccc}3 & 1 / 2 & -5 / 2 \\ 1 / 2 & -2 & -2 \\ -5 / 2 & -2 & 2\end{array}\right]+\left[\begin{array}{ccc}0 & 5 / 2 & 3 / 2 \\ -5 / 2 & 0 & 3 \\ -3 / 2 & -3 & 0\end{array}\right]=\left[\begin{array}{ccc}3 & 3 & -1 \\ -2 & -2 & 1 \\ -4 & -5 & 2\end{array}\right]=\mathrm{A}\)

Hence, matrix A is the sum of a symmetric matrix and a Skew-Symmetric matrix.

A square matrix A can be expressed as the sum of symmetric and skew-symmetric matrices

\(\mathrm{A}=\frac{1}{2}\left[\mathrm{~A}+\mathrm{A}^{\prime}\right]+\frac{1}{2}\left[\mathrm{~A}-\mathrm{A}^{\prime}\right]\)

Let A=\(\left[\begin{array}{cc}1 & 5 \\ -1 & 2\end{array}\right]\) Then,\( A^{\prime}=\left[\begin{array}{cc}1 & 5 \\ -1 & 2\end{array}\right]^{\prime}=\left[\begin{array}{cc}1 & -1 \\ 5 & 2\end{array}\right]\)

Now, \(A+A^{\prime}=\left[\begin{array}{cc}1 & 5 \\ -1 & 2\end{array}\right]+\left[\begin{array}{cc}1 & -1 \\ 5 & 2\end{array}\right]=\left[\begin{array}{ll}2 & 4 \\ 4 & 4\end{array}\right]\)

Let P=\(\frac{1}{2}\left(A+A^{\prime}\right)=\frac{1}{2}\left[\begin{array}{ll}2 & 4 \\ 4 & 4\end{array}\right]=\left[\begin{array}{ll}1 & 2 \\ 2 & 2\end{array}\right]\)

⇒ \(P^{\prime}=\left[\begin{array}{ll}
1 & 2 \\
2 & 2
\end{array}\right]^{\prime}=\left[\begin{array}{ll}
1 & 2 \\
2 & 2
\end{array}\right]\)=P

Thus, P=\(\frac{1}{2}\left(A+A^{\prime}\right)\) is a symmetric matrix.

Now, \(A-A^{\prime}=\left[\begin{array}{cc}1 & 5 \\ -1 & 2\end{array}\right]-\left[\begin{array}{cc}1 & -1 \\ 5 & 2\end{array}\right]=\left[\begin{array}{cc}0 & 6 \\ -6 & 0\end{array}\right]\)

Let\(Q=\frac{1}{2}\left(A-A^{\prime}\right)=\frac{1}{2}\left[\begin{array}{cc}0 & 6 \\ -6 & 0\end{array}\right]=\left[\begin{array}{cc}0 & 3 \\ -3 & 0\end{array}\right]\)

⇒ \(Q^{\prime}=\left[\begin{array}{cc}
0 & 3 \\
-3 & 0
\end{array}\right]=\left[\begin{array}{cc}
0 & -3 \\
3 & 0
\end{array}\right]=-Q\)

Thus,\(\mathrm{Q}=\frac{1}{2}\left(\mathrm{~A}-\mathrm{A}^{\prime}\right)\) is a skew – symmetric matrix.

⇒ \(\mathrm{P}+\mathrm{Q}=\left[\begin{array}{ll}
1 & 2 \\
2 & 2
\end{array}\right]+\left[\begin{array}{cc}
0 & 3 \\
-3 & 0
\end{array}\right]=\left[\begin{array}{cc}
1 & 5 \\
-1 & 2
\end{array}\right]=\mathrm{A}\)

Hence, matrix A is the sum of a symmetric matrix and a Skew-symmetric matrix,

Question 11. If A and B are symmetric matrices of the same order, then AB-BA is a:

  1. Skew-symmetric matrix
  2. Symmetric matrix
  3. Zero matrix
  4. Identity matrix

Solution 1. Skew-symmetric matrix

1. Given, that A and B are symmetric matrices.

⇒ \(A^{\prime}\)=A and \(B^{\prime}\) =B

⇒ \((A B-B A)^{\prime} =(A B)^{\prime}-(B A)^{\prime}{\left[(A-B)^{\prime}=A^{\prime}-B^{\prime}\right]}\)

=\(B^{\prime} A^{\prime}-A^{\prime} B^{\prime} {\left[(A B)^{\prime}=B^{\prime} A^{\prime}\right]}\)

= B A-A B \({\left[ A^{\prime}=A and B^{\prime}=B\right]} \)

= -(A B-B A)

⇒ \((A B-B A)^{\prime}\)=-(A B-B A)

Thus, \((\mathrm{AB}-\mathrm{BA})\) is a skew – symmetric matrix.

Question 12. If \(\mathrm{A}=\left[\begin{array}{cc}\cos \alpha & -\sin \alpha \\ \sin \alpha & \cos \alpha\end{array}\right]\), then \(\mathrm{A}+\mathrm{A}^{\prime}=\mathrm{I}\), if the value of \(\alpha\) is ?

  1. \(\pi / 6\)
  2. \(\pi / 3\)
  3. \(\pi\)
  4. 3 \(\pi / 2\)

Solution: 2. \(\pi / 3\)

Here, \(\mathrm{A}=\left[\begin{array}{cc}\cos \alpha & -\sin \alpha \\ \sin \alpha & \cos \alpha\end{array}\right]\)

and \(\mathrm{A}^{\prime}=\left[\begin{array}{cc}\cos \alpha & -\sin \alpha \\ \sin \alpha & \cos \alpha\end{array}\right]^{\prime}=\left[\begin{array}{cc}\cos \alpha & \sin \alpha \\ -\sin \alpha & \cos \alpha\end{array}\right]\)

Given. \(\mathrm{A}+\mathrm{A}^{\prime}=\mathrm{I}\)

⇒ \(\left[\begin{array}{cc}
\cos \alpha & -\sin \alpha \\
\sin \alpha & \cos \alpha
\end{array}\right]+\left[\begin{array}{cc}
\cos \alpha & \sin \alpha \\
-\sin \alpha & \cos \alpha
\end{array}\right]\)

= \(\left[\begin{array}{ll}
1 & 0 \\
0 & 1
\end{array}\right] \Rightarrow\left[\begin{array}{cc}
2 \cos \alpha & 0 \\
0 & 2 \cos \alpha
\end{array}\right]=\left[\begin{array}{ll}
1 & 0 \\
0 & 1
\end{array}\right]\)

Comparing the corresponding elements of the above matrices, we have

⇒ \(2 \cos \alpha=1 \Rightarrow \cos \alpha=\frac{1}{2}=\cos \frac{\pi}{3} \Rightarrow \alpha=\frac{\pi}{3}\)

Matrices Exercise 3.4

Question 1. Matrices A and B will be inverse of each other only if

  1. AB=BA
  2. AB=0,BA=I
  3. AB=BA=0
  4. AB=BA=I

Solution: 4. AB=BA=I

We know that if A is a square matrix of order ‘n’ and if there exists another square matrix B

of the same order ‘n’, such that AB = BA = I., then B is said to be the inverse of A’

In this case, it is clear that A is the inverse of B’

Thus, matrices A and B will be inverse of each other only if AB = BA = I

Matrices Miscellaneous Exercise

Question 1. If A and B are symmetric matrices prove that AB – BA is a skew-symmetric matrix.

Solution:

Here, A and B are symmetric matrices” then \(A^{\prime}-A \) and \( B^{\prime}=B\)

Now,\((A B-B A)^{\prime} =(A B)^{\prime}-(B A)^{\prime}\) \(((A-B)^{\prime}=A^{\prime}-B^{\prime}[(A B)^{\prime}=B^{\prime} A^{\prime}]\)

=\(B^{\prime} A^{\prime}-A^{\prime} B^{\prime}=B A-A B( B^{\prime}=B\) and \(A^{\prime}=A)\)

=-(A B-B A)

⇒ \((A B-B A)^{\prime}\)=-(A B-B A)

Thus, \((\mathrm{AB}-\mathrm{BA})\) is a skew-symmetric matrix.

Question 2. Show that the matrix B’AB is symmetric or skew-symmetric according as A is symmetric or skewsymmetric.

Solution:

We suppose that A is a symmetric matrix, then \(A^{\prime}=A\)

Consider \(\left(B^{\prime} A B\right)^{\prime}=\left\{B^{\prime}(A B)\right\}^{\prime}=(A B)^{\prime}\left(B^{\prime}\right)^{\prime} \)

⇒ \({\left[(\mathrm{AB})^{\prime}=\mathrm{B}^{\prime} \mathrm{A}^{\prime}\right]} \)

=\(\left(\mathrm{B}^{\prime} \mathrm{A}^{\prime}\right) \mathrm{B}\)

⇒ \({\left[\left(B^{\prime}\right)^{\prime}=\mathrm{B}, \mathrm{A}^{\prime}=\mathrm{A}\right]}\)

=\(B^{\prime}\left(A^{\prime} B\right)=B^{\prime}(A B) \)

⇒ \(\left(B^{\prime} A B\right)^{\prime}=B^{\prime}\) A B

Which shows that \(B^{\prime}\) A B is a symmetric matrix.

Now, we suppose that A is a skew-symmetric matrix.

Then, \(\mathrm{A}^{+}=-\mathrm{A}\)

Consider, \(\left(B^{\prime} A B\right)^{\prime} =\left[B^{\prime}(A B)\right]^{\prime}=(A B)^{\prime}\left(B^{\prime}\right)^{\prime} \left[\left(A B^{\prime}\right)=B^{\prime} A^{\prime}\right]\)

=\(\left(B^{\prime} A^{\prime}\right) B=B^{\prime}(-A) B=-B^{\prime} A B \left[\left(A^{\prime}\right)^{\prime}=A\right]\left[ A^{\prime}=-A\right]\)

∴ \(\left(B^{\prime} A B\right)^{\prime}=-B^{\prime} A B\), which shows that \(B^{\prime}\) A B is a skew – symmetric matrix.

Question 3. Find the values of x, y and z if the matrix A= \(\left[\begin{array}{ccc}
0 & 2 y & z \\
x & y & -z \\
x & -y & z
\end{array}\right]\)
 satisfies the equation \(A^{\prime}\) A=1

Solution:

⇒ Given, \(A^{\prime} A=I\)

⇒ \(\left[\begin{array}{ccc}
0 & x & x \\
2 y & y & -y \\
z & -z & z
\end{array}\right]\left[\begin{array}{ccc}
0 & 2 y & z \\
x & y & -z \\
x & -y & z
\end{array}\right]\)

=\(\left[\begin{array}{ccc}
1 & 0 & 0 \\
0 & 1 & 0 \\
0 & 0 & 1
\end{array}\right]\)

⇒ \(\left[\begin{array}{ccc}
0+x^2+x^2 & 0+x y-x y & 0-x z+x z \\
0+y x-y x & 4 y^2+y^2+y^2 & 2 y z-y z-y z \\
0-z x+z x & 2 y z-y z-y z & z^2+z^2+z^2
\end{array}\right]=\left[\begin{array}{lll}
1 & 0 & 0 \\
0 & 1 & 0 \\
0 & 0 & 1
\end{array}\right]\)

⇒ \(\left[\begin{array}{ccc}
2 x^2 & 0 & 0 \\
0 & 6 y^2 & 0 \\
0 & 0 & 3 z^2
\end{array}\right]=\left[\begin{array}{ccc}
1 & 0 & 0 \\
0 & 1 & 0 \\
0 & 0 & 1
\end{array}\right]\)

On comparing the corresponding elements, we have

2. \(\mathrm{x}^2=1,6 \mathrm{y}^2=1,3 \mathrm{z}^2=1 \Rightarrow \mathrm{x}^2=\frac{1}{2}, \mathrm{y}^2=\frac{1}{6}, \mathrm{z}^2=\frac{1}{3}\)

\(\mathrm{x}= \pm \frac{1}{\sqrt{2}}, \mathrm{y}= \pm \frac{1}{\sqrt{6}}, \mathrm{z}= \pm \frac{1}{\sqrt{3}}\)

Question 4. For what value of x ;\(\left[\begin{array}{lll}1 & 2 & 1\end{array}\right]\left[\begin{array}{lll}1 & 2 & 0 \\ 2 & 0 & 1 \\ 1 & 0 & 2\end{array}\right]\left[\begin{array}{l}0 \\ 2 \\ x\end{array}\right]\)=0

Solution:

Given \(\left[\begin{array}{lll}1 & 2 & 1\end{array}\right]\left[\begin{array}{lll}1 & 2 & 0 \\ 2 & 0 & 1 \\ 1 & 0 & 2\end{array}\right]\left[\begin{array}{l}0 \\ 2 \\ x\end{array}\right]\)=0

⇒ \(\left[\begin{array}{lll}1 & 2 & 1\end{array}\right]\left[\begin{array}{c}0+4+0 \\ 0+0+x \\ 0+0+2 x\end{array}\right]\)=0

⇒ \(\left[\begin{array}{lll}1 & 2 & 1\end{array}\right]\left[\begin{array}{c}4 \\ x \\ 2 x\end{array}\right]\)=0

[4+2 x+2 x]=[0] \(\Rightarrow 4+4 x=0 \Rightarrow\) x=-1

Question 5. If A=\(\left[\begin{array}{cc}3 & 1 \\ -1 & 2\end{array}\right]\), show that \(A^2-5 \)A+71=0

Solution:

Given, A=\(\left[\begin{array}{cc}3 & 1 \\ -1 & 2\end{array}\right]\)

Now, \(A^2=A. A=\left[\begin{array}{cc}3 & 1 \\ -1 & 2\end{array}\right]\left[\begin{array}{cc}3 & 1 \\ -1 & 2\end{array}\right]\)

=\(\left[\begin{array}{cc}9-1 & 3+2 \\ -3-2 & -1+4\end{array}\right]=\left[\begin{array}{cc}8 & 5 \\ -5 & 3\end{array}\right]\)

⇒ \(A^2-5\) A+7 =\(\left[\begin{array}{cc}
8 & 5 \\
-5 & 3
\end{array}\right]-5\left[\begin{array}{cc}
3 & 1 \\
-1 & 2
\end{array}\right]+7\left[\begin{array}{ll}
1 & 0 \\
0 & 1
\end{array}\right]\)

=\(\left[\begin{array}{cc}
8 & 5 \\
-5 & 3
\end{array}\right]-\left[\begin{array}{cc}
15 & 5 \\
-5 & 10
\end{array}\right]+\left[\begin{array}{ll}
7 & 0 \\
0 & 7
\end{array}\right]\)

= \(\left[\begin{array}{cc}
8-15+7 & 5-5+0 \\
-5+5+0 & 3-10+7
\end{array}\right]=\left[\begin{array}{ll}
0 & 0 \\
0 & 0
\end{array}\right]\)=0

Question 6. Find x, if \(\left[\begin{array}{lll}x & -5 & -1\end{array}\right]\left[\begin{array}{lll}1 & 0 & 2 \\ 0 & 2 & 1 \\ 2 & 0 & 3\end{array}\right]\left[\begin{array}{l}x \\ 4 \\ 1\end{array}\right]\)=0

Solution:

Here, \(\left[\begin{array}{lll}x & -5 & -1\end{array}\right]\left[\begin{array}{lll}1 & 0 & 2 \\ 0 & 2 & 1 \\ 2 & 0 & 3\end{array}\right]\left[\begin{array}{l}x \\ 4 \\ 1\end{array}\right]\)=0

⇒ \(\left[\begin{array}{lll}x & -5 & -1\end{array}\right]\left(\left[\begin{array}{lll}1 & 0 & 2 \\ 0 & 2 & 1 \\ 2 & 0 & 3\end{array}\right]\left[\begin{array}{l}x \\ 4 \\ 1\end{array}\right]\right)\)=0

⇒ \(\left[\begin{array}{lll}
x & -5 & -1
\end{array}\right]\left[\begin{array}{c}
x+0+2 \\
0+8+1 \\
2 x+0+3
\end{array}\right]\)=0

⇒ \([x(x+2)+(-5)(9)+(-1)(2 x+3)]=[0]\)

⇒ \(\left[x^2-48\right]=[0]\)

⇒ \(x^2-48=0 \Rightarrow x= \pm \sqrt{48}= \pm 4 \sqrt{3}\)

Question 7. A manufacturer produces three products x,y, and z which he sells in two markets, and the annual sales are indicated below.

Market:                            1                                    2

Products:  10,000 2,000 18,000                       6,000 20,000 8,000

1. If unit sale prices of x, and are ₹2.50, ₹1.50, and ₹1.00 respectively; find the total revenue in each market with the help of matrix algebra.

2. If the unit costs of the above three commodities are ₹2.00, ₹1.00, and 50 paise respectively; find the gross profit.

Solution:

Matrix representing the sales is A=\(\left[\begin{array}{ccc}10000 & 2000 & 18000 \\ 6000 & 20000 & 8000\end{array}\right]\)

1. Matrix representing the sale price per unit is \(\mathrm{B}=\left[\begin{array}{B}
5 / 2 \\
3 / 2 \\
1
\end{array}\right]\)

Total revenue in each market is given by the product;

⇒ \(\mathrm{AB} =\left[\begin{array}{ccc}
10000 & 2000 & 18000 \\
6000 & 20000 & 8000
\end{array}\right]\left[\begin{array}{B}
5 / 2 \\
3 / 2 \\
1
\end{array}\right]\)

= \(\left[\begin{array}{l}
\left(10000 \times \frac{5}{2}\right)+\left(2000 \times \frac{3}{2}\right)+(18000 \times 1) \\
\left(6000 \times \frac{5}{2}\right)+\left(20000 \times \frac{3}{2}\right)+(8000 \times 1)
\end{array}\right]\)

= \(\left[\begin{array}{c}
46000 \\
53000
\end{array}\right]\)

Hence, the total revenue in Market 1 is ₹ 46000, and that in Market 2 is ₹ 53000.

The matrix representing the cost price per unit is \(\mathrm{C}=\left[\begin{array}{l}2.00 \\ 1.00 \\ 0.50\end{array}\right]\)

The total cost in the two markets is given by the product

⇒ \(\mathrm{AC}=\left[\begin{array}{ccc}
10000 & 2000 & 18000 \\
6000 & 20000 & 8000
\end{array}\right]\left[\begin{array}{c}
2 \\
1 \\
1 / 2
\end{array}\right]\)

= \(\left[\begin{array}{l}
(10000 \times 2)+(2000 \times 1)+\left(18000 \times \frac{1}{2}\right) \\
(6000 \times 2)+(20000 \times 1)+\left(8000 \times \frac{1}{2}\right)
\end{array}\right]=\left[\begin{array}{l}
31000 \\
36000
\end{array}\right]\)

Profit in market \(\mathrm{I}\)=₹(46000-31000)=₹ 15000

and profit in market 2 =₹(53000-36000)=₹ 17000

The gross profit =₹(15000+17000)=₹ 32000.

Question 8. Find the matrix X so that \(X\left[\begin{array}{lll}1 & 2 & 3 \\ 4 & 5 & 6\end{array}\right]=\left[\begin{array}{ccc}-7 & -8 & -9 \\ 2 & 4 & 6\end{array}\right]\).

Solution:

Here, \(X\left[\begin{array}{lll}1 & 2 & 3 \\ 4 & 5 & 6\end{array}\right]=\left[\begin{array}{ccc}-7 & -8 & -9 \\ 2 & 4 & 6\end{array}\right]\)

The matrix given on the RHS of the equation is a 2 \(\times\) 3 matrix and the one given on the LHS of the equation is also a 2 \(\times 3\) matrix. Therefore, X has to be a 2 \(\times\) 2 matrix. Now, Let X=\(\left[\begin{array}{ll}a & c \\ b & d\end{array}\right]\)

Therefore; we have \(\left[\begin{array}{ll}\mathrm{a} & \mathrm{c} \\ \mathrm{b} & \mathrm{d}\end{array}\right]\left[\begin{array}{lll}\mathrm{l} & 2 & 3 \\ 4 & 5 & 6\end{array}\right]=\left[\begin{array}{ccc}-7 & -8 & -9 \\ 2 & 4 & 6\end{array}\right]\)

⇒ \(\left[\begin{array}{ccc}
a+4 c & 2 a+5 c & 3 a+6 c \\
b+4 d & 2 b+5 d & 3 b+6 d
\end{array}\right]=\left[\begin{array}{ccc}
-7 & -8 & -9 \\
2 & 4 & 6
\end{array}\right]\)

Equating the corresponding elements of the two matrices, we leave

a+4 c=-7, 2 a+5 c=-8, 3 a+6 c=-9

b+4 d=2, 2 b+5 d=4, 3 b+6 d=6

Now, a+4 c=-7 \(\Rightarrow\) a=-7-4 c

2 \(\mathrm{a}+5 \mathrm{c}=-8 \Rightarrow-14-8 \mathrm{c}+5 \mathrm{c}=-8 \Rightarrow-3 \mathrm{c}=6 \Rightarrow \mathrm{c}\)=-2

a=-7-4(-2)=-7+8=1 also; 3(1)+6(-2)=3-12=-9

Also, b+4 d=2 \(\Rightarrow\) b=2-4 d and 2 b+5 d=4 \(\Rightarrow\) 4-8 d+5 d=4

-3 d=0 \(\Rightarrow\) d=0

b=2-4(0)=2 Also; 3(2)+6 \(\times\) 0=6

Thus, a=1, b=2, c=-2 and d=0

Hence, the required matrix X is \(\left[\begin{array}{cc}1 & -2 \\ 2 & 0\end{array}\right]\)

Question 9. If A=\(\left[\begin{array}{cc}\alpha & \beta \\ \gamma & -\alpha\end{array}\right] \)is such that \(A^2=\)1, then

  1. 1+\(\alpha^2+\beta \gamma\)=0
  2. 1-\(\alpha^2+\beta \gamma\)=0
  3. 1-\(\alpha^2-\beta \gamma\)=0
  4. 1+\(\alpha^2-\beta \gamma\)=0

Solution:

Given, \(A^2\)=1

⇒ \(A \cdot A=T \Rightarrow\left[\begin{array}{cc}
\alpha & \beta \\
\gamma & -\alpha
\end{array}\right]\left[\begin{array}{cc}
\alpha & \beta \\
\gamma & -\alpha
\end{array}\right]=\left[\begin{array}{ll}
1 & 0 \\
0 & 1
\end{array}\right]\)

⇒ \(\left[\begin{array}{cc}
\alpha^2+\beta \gamma & \alpha \beta-\alpha \beta \\
\alpha \gamma-\gamma \alpha & \gamma \beta+\alpha^2
\end{array}\right]=\left[\begin{array}{ll}
1 & 0 \\
0 & 1
\end{array}\right]\)

On comparing the corresponding elements, we have

∴ \(\alpha^2+\beta \gamma=1 \Rightarrow \alpha^2+\beta \gamma-1=0 \Rightarrow 1-\alpha^2-\beta \gamma\)=0

Question 10. If a matrix n is both a symmetric and skew-symmetric matrix, then

  1. A is a diagonal matrix
  2. A is a zero matrix
  3. A is a square matrix
  4. None of these

Solution: 2. A is a zero matrix

Let A be a square matrix such that A is both a symmetric and skew-symmetric matrix.

⇒ \(\mathrm{A}^{\prime}=\mathrm{A}\) {A is symmetric }

and \(A^{\prime}\)=-A {[ A is skew- symmetric] }

A=-A

A+A=O \(\Rightarrow 2 A=O \Rightarrow\) A=0

Question 11. If A is square matrix such that \(\mathrm{A}^2=\mathrm{A}\), then \((\mathrm{I}+\mathrm{A})^3-7 \mathrm{~A}\) is equal to

  1. A
  2. I – A
  3. 1
  4. 3 A

Solution: 3. 1

Here, \(\mathrm{A}^2\)= A

⇒ \((I+A)^2\)=(I+A)(I+A) [A I=A=\([A, A^2=A]\).

⇒ \((I+A)^3 =(I+A)^2 \cdot(I+A)=(I+3 A) \cdot(I+A)\)

= I+I A+3(A I)+3(A A)

= I+A+3 A+3 A=I+7 A \(\left[ A^2=A\right]\)

∴ \((I+A)^3\)-7 A=I

Inverse Trigonometric Functions Class 12 Maths Important Questions Chapter 2

Inverse Trigonometric Functions Exercise 2.1

Find the principal values of the following.

Question 1. \(\sin ^{-1}\left(\frac{-1}{2}\right)\)

Solution:

Let \(\sin ^{-1}\left(\frac{-1}{2}\right)=\theta \Rightarrow \sin \theta=-\frac{1}{2}\)

We know that the range of principal value of \(\sin ^{-1} \mathrm{x} is \left[-\frac{\pi}{2}, \frac{\pi}{2}\right]\).

⇒ \(\sin \theta=-\frac{1}{2}=-\sin \frac{\pi}{6}=\sin \left(-\frac{\pi}{6}\right)  ( \sin (-\theta)=-\sin \theta)\)

⇒  \(\theta=-\frac{\pi}{6}\), where \(\theta \in\left[-\frac{\pi}{2}, \frac{\pi}{2}\right] \Rightarrow \sin ^{-1}\left(-\frac{1}{2}\right)=-\frac{\pi}{6}\)

Question 2. \(\cos ^{-1}\left(\frac{\sqrt{3}}{2}\right)\)

Solution:

Let \(\cos ^{-1}\left(\frac{\sqrt{3}}{2}\right)=\theta \Rightarrow \cos \theta=\frac{\sqrt{3}}{2}\)

We know that the range of principal value of \(\cos ^{-1} \mathrm{x} is [0, \pi]\).

∴  \(\cos \theta=\frac{\sqrt{3}}{2}=\cos \frac{\pi}{6} \Rightarrow \theta=\frac{\pi}{6}\), where \(\theta \in[0, \pi] \Rightarrow \cos ^{-1}\left(\frac{\sqrt{3}}{2}\right)=\frac{\pi}{6}\)

Question 3.\({cosec}^{-1}(2)\)

Solution:

Let \({cosec}^{-1}(2)=\theta \Rightarrow {cosec} \theta\)=2

We know that the range of principal value of \({cosec}^{-1} \mathrm{x} is \left[-\frac{\pi}{2}, \frac{\pi}{2}\right]-\{0\}\).

⇒  \({cosec} \theta=2={cosec} \frac{\pi}{6} \Rightarrow \theta=\frac{\pi}{6}\), where \(\theta \in[-\frac{\pi}{2}\), \(\frac{\pi}{2}-{0}\)

⇒ \({cosec}^{-1}(2)\)

= \(\frac{\pi}{6}\)

Read and Learn More Class 12 Maths Chapter Wise with Solutions

Question 4. \(\tan ^{-1}(-\sqrt{3})\)

Solution:

Let \(\tan ^{-1}(-\sqrt{3})=\theta \Rightarrow \tan \theta=-\sqrt{3}\)

We know that the range of principal value of \(\tan ^{-1} \mathrm{x} is \left(-\frac{\pi}{2}, \frac{\pi}{2}\right)\)

⇒  \(\tan \theta=-\sqrt{3}=-\tan \frac{\pi}{3}=\tan \left(-\frac{\pi}{3}\right) (\tan (-\theta)=-\tan \theta)\)

⇒  \(\theta=-\frac{\pi}{3}\),where \(\theta \in\left(-\frac{\pi}{2}, \frac{\pi}{2}\right) \Rightarrow \tan ^{-1}(-\sqrt{3})=-\frac{\pi}{3}\)

Question 5. \(\cos ^{-1}\left(-\frac{1}{2}\right)\)

Solution:

Let \(\cos ^{-1}\left(-\frac{1}{2}\right)=\theta \Rightarrow \cos \theta=-\frac{1}{2}\)

We know that the range of principal value of \(\cos ^{-1} \mathrm{x} is [0, \pi]\)

⇒  \(\cos \theta=-\frac{1}{2}=-\cos \frac{\pi}{3}=\cos \left(\pi-\frac{\pi}{3}\right)=\cos \frac{2 \pi}{3}(\cos (\pi-\theta)=-\cos \theta) \)

⇒  \(\theta=\frac{2 \pi}{3}\) ; where \(\theta \in[0, \pi] \Rightarrow \cos ^{-1}\left(-\frac{1}{2}\right)=\frac{2 \pi}{3}\)

CBSE Class 12 Maths Chapter 2 Inverse Trigonometric Functions Important Question And Answers

Question 6. \(\tan ^{-1}(-1)\)

Solution:

Let \(\tan ^{-1}(-1)=\theta \Rightarrow \tan \theta\)=-1

We know that the range of principal value of \(\tan ^{-1} \mathrm{x} is \left(-\frac{\pi}{2}, \frac{\pi}{2}\right)\)

⇒  \(\tan \theta=-1=-\tan \frac{\pi}{4}=\tan \left(-\frac{\pi}{4}\right)\)

⇒  \((\tan (-\theta)=-\tan \theta)\)

⇒ \(\theta=-\frac{\pi}{4} ; where \theta \in\left(-\frac{\pi}{2}, \frac{\pi}{2}\right)\)

∴ \(\tan ^{-1}(-1)=-\frac{\pi}{4}\)

Question 7. \(\sec ^{-1}\left(\frac{2}{\sqrt{3}}\right)\)

Solution:

Let \(\sec ^{-1}\left(\frac{2}{\sqrt{3}}\right)=\theta \Rightarrow \sec \theta=\frac{2}{\sqrt{3}}\)

We know that the range of principal value of \(\sec ^{-1} \mathrm{x} is [0, \pi]-\left\{\frac{\pi}{2}\right\}\)

∴ \(\sec \theta=\frac{2}{\sqrt{3}}=\sec \left(\frac{\pi}{6}\right) \Rightarrow \theta=\frac{\pi}{6} \text { where } \theta \in[0, \pi]-\left\{\frac{\pi}{2}\right\} \)

⇒  \(\sec ^{-1}\left(\frac{2}{\sqrt{3}}\right)=\frac{\pi}{6}\)

Question 8. \(\cot ^{-1}(\sqrt{3})\)

Solution:

Let \(\cot ^{-1}(\sqrt{3})=\theta \Rightarrow \cot \theta=\sqrt{3}\)

We know that the range of principal value of \(\cot ^{-1} \mathrm{x} is (0, \pi)\)

∴ \(\cot \theta=\sqrt{3}=\cot \frac{\pi}{6} \Rightarrow \theta=\frac{\pi}{6} ; where \theta \in(0, \pi) \Rightarrow \cot ^{-1}(\sqrt{3})=\frac{\pi}{6} \)

Question 9. \(\cos ^{-1}\left(-\frac{1}{\sqrt{2}}\right)\)

Solution:

Let \(\cos ^{-1}\left(-\frac{1}{\sqrt{2}}\right)=\theta \Rightarrow \cos \theta=-\frac{1}{\sqrt{2}}\)

We know that the range of principal value of \(\cos ^{-1} \mathrm{x} is [0, \pi]\)

∴ \(\cos \theta=-\frac{1}{\sqrt{2}}=-\cos \frac{\pi}{4}=\cos \left(\pi-\frac{\pi}{4}\right)\) \([\cos (\pi-\theta)=-\cos \theta] \)

⇒  \(\theta=\frac{3 \pi}{4}\), where \(\theta \in[0, \pi] \Rightarrow \cos ^{-1}\left(-\frac{1}{\sqrt{2}}\right)=\frac{3 \pi}{4}\)

Question 10. \({cosec}^{-1}(-\sqrt{2})\)

Solution:

Let \({cosec}^{-1}(-\sqrt{2})=\theta \Rightarrow {cosec} \theta=-\sqrt{2}\)

We know that the range of principal value of \({cosec}^{-1} \mathrm{x} is \left[-\frac{\pi}{2}, \frac{\pi}{2}\right]-\{0\}\)

∴ \({cosec} \theta=-\sqrt{2}=-{cosec} \frac{\pi}{4}={cosec}\left(-\frac{\pi}{4}\right)\)      \(({cosec}(-\theta)=-{cosec} \theta) \)

⇒  \({cosec}^{-1}(-\sqrt{2})=-\frac{\pi}{4}\)

Question 11. \(\tan ^{-1}(1)+\cos ^{-1}\left(-\frac{1}{2}\right)+\sin ^{-1}\left(-\frac{1}{2}\right)\)

Solution:

Let \(\tan ^{-1}(1)=x \Rightarrow \tan x=1=\tan \frac{\pi}{4} \Rightarrow x=\frac{\pi}{4}\)

where principal value of x \(\in\left(-\frac{\pi}{2}, \frac{\pi}{2}\right) \tan ^{-1}(1)=\frac{\pi}{4}\)

Let \(\cos ^{-1}\left(-\frac{1}{2}\right)=y \Rightarrow \cos y=-\frac{1}{2}=-\cos \left(\frac{\pi}{3}\right)=\cos \left(\pi-\frac{\pi}{3}\right)=\cos \left(\frac{2 \pi}{3}\right)(\cos (\pi-\theta)=-\cos \theta)\)

⇒  y=\(\frac{2 \pi}{3}\), where principal value of y \(\in[0, \pi]\)

∴ \(\cos ^{-1}\left(-\frac{1}{2}\right)=\frac{2 \pi}{3}\)

Let \(\sin ^{-1}\left(-\frac{1}{2}\right)=z \Rightarrow \sin z=-\frac{1}{2}=-\sin \left(\frac{\pi}{6}\right)=\sin \left(-\frac{\pi}{6}\right)\)        (\(\sin (-\theta)=-\sin \theta)\)

⇒  z=-\(\frac{\pi}{6}\), where principal value of z \(\in\left[-\frac{\pi}{2}, \frac{\pi}{2}\right]\)

∴ \(\sin ^{-1}\left(-\frac{1}{2}\right)=-\frac{\pi}{6}\)

∴ \(\tan ^{-1}(1)+\cos ^{-1}\left(-\frac{1}{2}\right)+\sin ^{-1}\left(-\frac{1}{2}\right)=x+y+z=\frac{\pi}{4}+\frac{2 \pi}{3}-\frac{\pi}{6}=\frac{3 \pi+8 \pi-2 \pi}{12}=\frac{9 \pi}{12}=\frac{3 \pi}{4}\)

Question 12. \(\cos ^{-1}\left(\frac{1}{2}\right)+2 \sin ^{-1}\left(\frac{1}{2}\right)\)

Solution:

Let \(\cos ^{-1}\left(\frac{1}{2}\right)=x \Rightarrow \cos x=\frac{1}{2}=\cos \frac{\pi}{3}\)

⇒  \(\mathrm{x}=\frac{\pi}{3} \in[0, \pi]\) (Principal interval)

Again, let \(\sin ^{-1}\left(\frac{1}{2}\right)=y \Rightarrow \sin y=\frac{1}{2}=\sin \frac{\pi}{6}\)

⇒  \(\mathrm{y}=\frac{\pi}{6} \in\left[-\frac{\pi}{2}, \frac{\pi}{2}\right]\) (Principal interval)

∴ \(\cos ^{-1}\left(\frac{1}{2}\right)+2 \sin ^{-1}\left(\frac{1}{2}\right)=x+2 y=\frac{\pi}{3}+\left(2 \times \frac{\pi}{6}\right)=\frac{2 \pi}{3}\)

Question 13. If \(\sin ^{-1}\) x=y, then

  1. 0 \(\leq \mathrm{y} \leq \pi\)
  2. –\(\frac{\pi}{2} \leq y \leq \frac{\pi}{2}\)
  3. \(0<y<\pi\)
  4. –\(\frac{\pi}{2}<y<\frac{\pi}{2}\)

Solution: 2. –\(\frac{\pi}{2} \leq y \leq \frac{\pi}{2}\)

As range of \(\sin ^{-1} x is \left[-\frac{\pi}{2}, \frac{\pi}{2}\right], therefore -\frac{\pi}{2} \leq y \leq \frac{\pi}{2}\).

Question 14. \(\tan ^{-1} \sqrt{3}-\sec ^{-1}(-2)\) is equal to

  1. \(\pi\)
  2. –\(\frac{\pi}{3}\)
  3. \(\frac{\pi}{3}\)
  4. \(\frac{2 \pi}{3}\)

Solution: 2. –\(\frac{\pi}{3}\)

Let \(\tan ^{-1} \sqrt{3}=x \Rightarrow \tan x=\sqrt{3} \Rightarrow \tan x=\tan \frac{\pi}{3}\)

⇒ \(\mathrm{x}=\frac{\pi}{3} \in\left(-\frac{\pi}{2}, \frac{\pi}{2}\right)\) (Principal interval)

Let \(\sec ^{-1}(-2)=y \Rightarrow \sec\) y=-2

⇒ \(\sec y=-\sec \frac{\pi}{3} \Rightarrow \sec y=\sec \left(\pi-\frac{\pi}{3}\right)\) [ \(\sec (\pi-\theta)=-\sec \theta\)]

⇒ \(\sec y=\sec \left(\frac{2 \pi}{3}\right) \Rightarrow y=\frac{2 \pi}{3} \in[0, \pi]-\left\{\frac{\pi}{2}\right\}\) (Principal interval)

∴ \(\tan ^{-1} \sqrt{3}-\sec ^{-1}(-2)=x-y=\frac{\pi}{3}-\frac{2 \pi}{3}=-\frac{\pi}{3}\)

Inverse Trigonometric Functions Exercise 2.2

Question 1. 3 \(\sin ^{-1} x=\sin ^{-1}\left(3 x-4 x^3\right), x \in\left[-\frac{1}{2}, \frac{1}{2}\right]\)

Solution:

Let \(\sin ^{-1} x=\theta \Rightarrow x=\sin \theta\), then

RHS =\(\sin ^{-1}\left(3 x-4 x^3\right)\)

= \(\sin ^{-1}\left[3 \sin \theta-4 \sin ^3 \theta\right]=\sin ^{-1}[\sin 3 \theta]=3 \theta \)      \([ \sin 3 \theta=3 \sin \theta-4 \sin ^3 \theta] \)

= 3 \(\sin ^{-1}\) x= LHS

Question 2. 3 \(\cos ^{-1} x=\cos ^{-1}\left(4 x^3-3 x\right), x \in\left[\frac{1}{2}, 1\right]\)

Solution:

Let \(\cos ^{-1} x=\theta \Rightarrow x=\cos \theta\), then

RHS =\(\cos ^{-1}\left(4 x^3-3 x\right)\)

= \(\cos ^{-1}\left[4 \cos ^3 \theta-3 \cos \theta\right]=\cos ^{-1}[\cos 3 \theta]=3 \theta\)      \([ \cos 3 \theta=4 \cos ^3 \theta-3 \cos \theta]\)

= 3 \(\cos ^{-1} x\)= LHS

Question 3. \(\tan ^{-1}\left(\frac{\sqrt{1+x^2}-1}{x}\right), x \neq 0\)

Solution:

Put x=\(\tan \theta \Rightarrow \theta=\tan ^{-1}\)x

∴  \(\tan ^{-1}\left(\frac{\sqrt{1+x^2}-1}{x}\right)=\tan ^{-1}\left(\frac{\sqrt{1+\tan ^2 \theta}-1}{\tan \theta}\right)=\tan ^{-1}\left(\frac{\sqrt{\sec ^2 \theta}-1}{\tan \theta}\right) \)        \(\left[ 1+\tan ^2 \theta=\sec ^2 \theta\right] \)

= \(\tan ^{-1}\left[\frac{\sec \theta-1}{\tan \theta}\right]=\tan ^{-1}\left[\frac{\frac{1}{\cos \theta}-1}{\frac{\sin \theta}{\cos \theta}}\right]=\tan ^{-1}\left[\frac{\frac{1-\cos \theta}{\cos \theta}}{\frac{\sin \theta}{\cos \theta}}\right]=\tan ^{-1}\left[\frac{1-\cos \theta}{\cos \theta} \times \frac{\cos \theta}{\sin \theta}\right]\)

= \(\tan ^{-1}[\frac{1-\cos \theta}{\sin \theta}]=\tan ^{-1}[\frac{2 \sin ^2 \frac{\theta}{2}}{2 \sin \frac{\theta}{2} \cos \frac{\theta}{2}}]\)

1-\(\cos \theta=2 \sin ^2 \frac{\theta}{2}\)

and \(\sin \theta=2 \sin \frac{\theta}{2} \cos \frac{\theta}{2}\)

= \(\tan ^{-1}\left[\frac{2 \sin \frac{\theta}{2}}{2 \cos \frac{\theta}{2}}\right]=\tan ^{-1}\left[\tan \frac{\theta}{2}\right]=\frac{\theta}{2}=\frac{\tan ^{-1} x}{2}\) [from equation (1)]

∴ \(\tan ^{-1}\left(\frac{\sqrt{1+x^2}-1}{x}\right)=\frac{1}{2} \tan ^{-1} x\)

Question 4. \(\tan ^{-1}\left(\sqrt{\frac{1-\cos x}{1+\cos x}}\right), 0<x<\pi \)

Solution:

⇒ \(\tan ^{-1}\left(\sqrt{\frac{1-\cos x}{1+\cos x}}\right)=\tan ^{-1}\left(\sqrt{\frac{2 \sin ^2(x / 2)}{2 \cos ^2(x / 2)}}\right)\) \((1-\cos x=2 \sin ^2(x / 2) and 1+\cos x=2 \cos ^2(x / 2)\)

= \(\tan ^{-1}\left(\sqrt{\tan ^2 \frac{x}{2}}\right)=\tan ^{-1}\left(\tan \frac{x}{2}\right)=\frac{x}{2}\)

Question 5. \(\tan ^{-1}\left(\frac{\cos x-\sin x}{\cos x+\sin x}\right), \frac{-\pi}{4}<x<\frac{3 \pi}{4} \)

Solution:

⇒ \(\tan ^{-1}(\frac{\cos x-\sin x}{\cos x+\sin x})=\tan ^{-1}(\frac{\frac{\cos x}{\cos x}-\frac{\sin x}{\cos x}}{\frac{\cos x}{\cos x}+\frac{\sin x}{\cos x}})\) (inside the bracket divide numerator and denominator by \(\cos x)\)

= \(\tan ^{-1}\left(\frac{1-\tan x}{1+\tan x}\right)=\tan ^{-1}\left[\tan \left(\frac{\pi}{4}-x\right)\right]=\frac{\pi}{4}-x \left[\tan \left(\frac{\pi}{4}-x\right)=\frac{\tan \frac{\pi}{4}-\tan x}{1+\tan \frac{\pi}{4} \cdot \tan x}=\frac{1-\tan x}{1+\tan x}\right]\)

Question 6. \(\tan ^{-1}\left(\frac{x}{\sqrt{a^2-x^2}}\right),|x|<a\)

Solution:

Put x=a \(\sin \theta \Rightarrow \frac{x}{a}=\sin \theta \Rightarrow \sin ^{-1}\left(\frac{x}{a}\right)=\theta\)

∴ \(\tan ^{-1}\left(\frac{x}{\sqrt{a^2-x^2}}\right) =\tan ^{-1}\left(\frac{a \sin \theta}{\sqrt{a^2-a^2 \sin ^2 \theta}}\right) \)

= \(\tan ^{-1}\left(\frac{a \sin \theta}{a \sqrt{1-\sin ^2 \theta}}\right)=\tan ^{-1}\left(\frac{\sin \theta}{\cos \theta}\right) \)

= \(\tan ^{-1}(\tan \theta)=\theta=\sin ^{-1}\left(\frac{x}{a}\right)\)

⇒ \(\sin ^2 x+\cos ^2 x=1\)

∴ \(\cos x=\sqrt{1-\sin ^2 x}\) [from equation (1)]

Question 7. \(\tan ^{-1}\left(\frac{3 a^2 x-x^3}{a^3-3 a x^2}\right), a>0 ; \frac{-a}{\sqrt{3}}<x<\frac{a}{\sqrt{3}}\)

Solution:

Put \(\mathrm{x}=\mathrm{a} \tan \theta\)

⇒ \(\frac{\mathrm{x}}{\mathrm{a}}=\tan \theta \Rightarrow \theta=\tan ^{-1}\left(\frac{\mathrm{x}}{\mathrm{a}}\right)\)

∴ \(\tan ^{-1}\left(\frac{3 a^2 x-x^3}{a^3-3 a x^2}\right)=\tan ^{-1}\left[\frac{3 a^2(a \tan \theta)-(a \tan \theta)^3}{a^3-3 a(a \tan \theta)^2}\right]=\tan ^{-1}\left[\frac{a^3\left(3 \tan \theta-\tan ^3 \theta\right)}{a^3\left(1-3 \tan ^2 \theta\right)}\right]\)

=\(\tan ^{-1}\left[\frac{3 \tan \theta-\tan ^3 \theta}{1-3 \tan ^2 \theta}\right]=\tan ^{-1}(\tan 3 \theta)\left(\tan 3 \theta=\frac{3 \tan \theta-\tan ^3 \theta}{1-3 \tan ^2 \theta}\right)\)

=3 \(\theta=3 \tan ^{-1}\left(\frac{\mathrm{x}}{\mathrm{a}}\right)\) [From equation (1)]

Question 8. \(\tan ^{-1}\left[2 \cos \left(2 \sin ^{-1} \frac{1}{2}\right)\right]\)

Solution:

⇒ \(\tan ^{-1}\left[2 \cos \left(2 \sin ^{-1} \frac{1}{2}\right)\right]\)

= \(\tan ^{-1}\left[2 \cos \left\{2 \sin ^{-1}\left(\sin \frac{\pi}{6}\right)\right\}\right] \quad\left(\sin \frac{\pi}{6}=\frac{1}{2}\right)\)

= \(\tan ^{-1}\left[2 \times \frac{1}{2}\right]=\tan ^{-1}(1) \)

= \(\tan ^{-1}\left[2 \cos \left(2 \times \frac{\pi}{6}\right)\right]=\tan ^{-1}\left[2 \cos \frac{\pi}{3}\right]\)    \(\left(\cos \frac{\pi}{3}=\frac{1}{2}\right)\)

= \(\tan ^{-1}\left(\tan \frac{\pi}{4}\right)=\frac{\pi}{4} \)   \(\left(\tan \frac{\pi}{4}=1\right) \)

Question 9. \(\tan \frac{1}{2}\left[\sin ^{-1}\left(\frac{2 x}{1+x^2}\right)+\cos ^{-1}\left(\frac{1-y^2}{1+y^2}\right)\right],|x|<1, y \cdot>0 \text { and } x y<1\)

Solution:

= \(\tan ^{-1}\left[2 \times \frac{1}{2}\right]=\tan ^{-1}(1) \) \(\tan \frac{1}{2}\left[\sin ^{-1}\left(\frac{2 x}{1+x^2}\right)+\cos ^{-1}\left(\frac{1-y^2}{1+y^2}\right)\right]\)

Put x=\(\tan \theta and y=\tan \phi\)

Now, \(\tan \frac{1}{2}\left[\sin ^{-1}\left(\frac{2 \tan \theta}{1+\tan ^2 \theta}\right)+\cos ^{-1}\left(\frac{1-\tan ^2 \phi}{1+\tan ^2 \phi}\right)\right] \Rightarrow \tan \frac{1}{2}\left[\sin ^{-1}(\sin 2 \theta)+\cos ^{-1}(\cos 2 \phi)\right] \)

∴ \(\tan \frac{1}{2}[2 \theta+2 \phi] \Rightarrow \tan (\theta+\phi)=\frac{\tan \theta+\tan \phi}{1-\tan \theta \tan \phi}=\frac{x+y}{1-x y}\)

Question 10. \(\sin ^{-1}\left(\sin \frac{2 \pi}{3}\right)\)

Solution:

⇒ \(\sin ^{-1}\left(\sin \frac{2 \pi}{3}\right) =\sin ^{-1}\left[\sin \left(\pi-\frac{\pi}{3}\right)\right]=\sin ^{-1}\left(\sin \frac{\pi}{3}\right) \)

= \(\frac{\pi}{3} \in\left[-\frac{\pi}{2}, \frac{\pi}{2}\right]\)      \((\sin ^{-1}(\sin \theta)=\theta, \theta \in[-\frac{\pi}{2}, \frac{\pi}{2}])\)

Question 11. \(\tan ^{-1}\left(\tan \frac{3 \pi}{4}\right)\)

Solution:

⇒ \(\tan ^{-1}\left(\tan \frac{3 \pi}{4}\right)=\tan ^{-1}\left[\tan \left(\pi-\frac{\pi}{4}\right)\right]=\tan ^{-1}\left(-\tan \frac{\pi}{4}\right) {[-\tan \theta=\tan (-\theta)]}\)

= \(\tan ^{-1}\left[\tan \left(-\frac{\pi}{4}\right)\right]=-\frac{\pi}{4} \in\left(-\frac{\pi}{2}, \frac{\pi}{2}\right) {\left[ \tan ^{-1}(\tan \theta)=\theta, \theta \in\left(-\frac{\pi}{2}, \frac{\pi}{2}\right)\right]}\)

Question 12. \(\tan \left(\sin ^{-1} \frac{3}{5}+\cot ^{-1} \frac{3}{2}\right)\)

Solution:

⇒ \(\tan \left(\sin ^{-1} \frac{3}{5}+\cot ^{-1} \frac{3}{2}\right)\)

Put \(\sin ^{-1} \frac{3}{5}=\theta \Rightarrow \sin \theta=\frac{3}{5} and \cot ^{-1} \frac{3}{2}=\phi \Rightarrow \cot \phi=\frac{3}{2} \)

Thus, \(\tan (\theta+\phi)\)  → Equation 1

Now, \(\tan \theta=\frac{3}{4}\) and \(\tan \phi=\frac{2}{3}(\sin ^{-1} x=\tan ^{-1} \frac{x}{\sqrt{1-x^2}})\) Since, \(\tan (\theta+\phi)=\frac{\tan \theta+\tan \phi}{1-\tan \theta \tan \phi}=\left[\frac{\frac{3}{4}+\frac{2}{3}}{1-\left(\frac{3}{4} \times \frac{2}{3}\right)}\right]=\frac{17}{6} \Rightarrow(\theta+\phi)=\tan ^{-1} \frac{17}{6} \ldots \)  → Equation 2

Putting the value from eq.(2) in (1), \(\tan \left(\tan ^{-1} \cdot \frac{17}{6}\right)=\frac{17}{6}\)

Question 13. \(\cos ^{-1}\left(\cos \frac{7 \pi}{6}\right)\) is equal to?

  1. \(\frac{7 \pi}{6}\)
  2. \(\frac{5 \pi}{6}\)
  3. \(\frac{\pi}{3}\)
  4. \(\frac{\pi}{6}\)

Solution: 2. \(\frac{5 \pi}{6}\)

⇒ \(\cos ^{-1}\left(\cos \frac{7 \pi}{6}\right)=\cos ^{-1}\left[\cos \left(2 \pi-\frac{5 \pi}{6}\right)\right]\)

∴ \(\cos ^{-1}\left(\cos \frac{7 \pi}{6}\right)=\cos ^{-1}\left[\cos \left(\frac{5 \pi}{6}\right)\right]=\frac{5 \pi}{6} \in[0, \pi] \left[ \cos ^{-1}(\cos \theta)=\theta, \theta \in[0, \pi]\right]\)

Question 14. \(\sin \left[\frac{\pi}{3}-\sin ^{-1}\left(-\frac{1}{2}\right)\right]\) is equal to

  1. \(\frac{1}{2}\)
  2. \(\frac{1}{3}\)
  3. \(\frac{1}{4}\)
  4. 1

Solution: 4.

⇒ \(\sin \left[\frac{\pi}{3}-\sin ^{-1}\left(-\frac{1}{2}\right)\right]=\sin \left[\frac{\pi}{3}-\sin ^{-1}\left(-\sin \frac{\pi}{6}\right)\right] \quad\left[ \sin \frac{\pi}{6}=\frac{1}{2}\right][ \sin (-x)=-\sin x]\)

=\(\sin \left[\frac{\pi}{3}-\sin ^{-1}\left\{\sin \left(-\frac{\pi}{6}\right)\right\}\right]=\sin \left[\frac{\pi}{3}-\left(-\frac{\pi}{6}\right)\right]=\sin \left[\frac{\pi}{3}+\frac{\pi}{6}\right]=\sin \frac{\pi}{2}\)=1

Question 15. \(\tan ^{-1}(\sqrt{3})-\cot ^{-1}(-\sqrt{3})\) is equal to

  1. \(\pi\)
  2. –\(\frac{\pi}{2}\)
  3. zero
  4. 2 \(\sqrt{3}\)

Solution: 2. –\(\frac{\pi}{2}\)

⇒ \(\tan ^{-1}(\sqrt{3})-\cot ^{-1}(-\sqrt{3}) \)

= \(\tan ^{-1}(\sqrt{3})-\left[\pi-\cot ^{-1}(\sqrt{3})\right]\) \([\cot ^{-1}(-x)=\pi-\cot ^{-1} x] \)

= \(\tan ^{-1}(\sqrt{3})-\pi+\cot ^{-1}(\sqrt{3})=\frac{\pi}{3}-\pi+\frac{\pi}{6}=-\frac{\pi}{2}\)

Inverse Trigonometric Functions Miscellaneous Exercise

Question 1. \(\cos ^{-1}\left(\cos \frac{13 \pi}{6}\right)\)

Solution:

⇒ \(\cos ^{-1}(\cos \frac{13 \pi}{6})=\cos ^{-1}[\cos (2 \pi+\frac{\pi}{6})]\)

= \(\cos ^{-1}\left[\cos \left(\frac{\pi}{6}\right)\right]=\frac{\pi}{6} \in[0, \pi]    [ \cos ^{-1}(\cos \theta)=\theta, \theta \in[0, \pi] and \cos (2 \pi+\theta)=\cos \theta]\)

Question 2. \(\tan ^{-1}\left(\tan \frac{7 \pi}{6}\right)\)

Solution:

⇒ \(\tan ^{-1}(\tan \frac{7 \pi}{6})=\tan ^{-1}[\tan (\pi+\frac{\pi}{6}) [\tan ^{-1}(\tan \theta)=\theta, \theta \in(-\frac{\pi}{2}, \frac{\pi}{2})\) and \(\tan (\pi+\theta)=\tan \theta]]\)

∴ \(\tan ^{-1}\left(\tan \frac{7 \pi}{6}\right)=\tan ^{-1}\left[\tan \frac{\pi}{6}\right]=\frac{\pi}{6} \in\left(-\frac{\pi}{2}, \frac{\pi}{2}\right)\)

Question 3. 2 \(\sin ^{-1}\left(\frac{3}{5}\right)=\tan ^{-1}\left(\frac{24}{7}\right)\)

solution:

LHS =\(2 \sin ^{-1}(\frac{3}{5})=\sin ^{-1}[2 \times \frac{3}{5} \sqrt{1-(\frac{3}{5})^2}]\)   [\( 2 \sin ^{-1} x=\sin ^{-1}(2 x \sqrt{1-x^2})]\)

= \(\sin ^{-1}[2 \times \frac{3}{5} \times \frac{4}{5}]=\sin ^{-1}(\frac{24}{25})=\tan ^{-1}[\frac{\frac{24}{25}}{\sqrt{1-(\frac{24}{25})^2}}] (\sin ^{-1} x=\tan ^{-1} \frac{x}{\sqrt{1-x^2}})\)

= \(\tan ^{-1}\left[\frac{\frac{24}{25}}{\sqrt{1-\frac{576}{625}}}\right]=\tan ^{-1}\left[\frac{24}{25} \times \frac{25}{7}\right]=\tan ^{-1}\left[\frac{24}{7}\right]\)= RHS. Hence, proved.

Question 4. \(\sin ^{-1}\left(\frac{8}{17}\right)+\sin ^{-1}\left(\frac{3}{5}\right)=\tan ^{-1}\left(\frac{77}{36}\right)\)

Solution:

L.H.S. =\(\sin ^{-1}\left(\frac{8}{17}\right)+\sin ^{-1}\left(\frac{3}{5}\right)\)

Let \(\sin ^{-1} \frac{8}{17}=x \Rightarrow \sin x=\frac{8}{17}\)

Now, \(\cos x=\sqrt{1-\sin ^2 x}=\sqrt{1-\left(\frac{8}{17}\right)^2}=\frac{15}{17}\)

Thus, \(\tan x=\frac{\sin x}{\cos x}=\frac{8}{15}\)

Similarly Let \(\sin ^{-1} \frac{3}{5}=y \Rightarrow \sin y=\frac{3}{5}\)

Now, \(\cos y=\sqrt{1-\sin ^2 y}=\sqrt{1-\left(\frac{3}{5}\right)^2}=\frac{4}{5}\)

Thus, \(\tan y=\frac{\sin y}{\cos y}=\frac{3}{4}\)

Now, \(\tan (x+y)=\frac{\tan x+\tan y}{1-\tan x \tan y}=\left[\frac{\frac{8}{15}+\frac{3}{4}}{1-\frac{8}{15} \times \frac{3}{4}}\right]=\frac{\frac{32+45}{60}}{\frac{60-24}{60}}=\frac{77}{36}\)

Hence, \(\tan (x+y)=\frac{77}{36} \Rightarrow x+y=\tan ^{-1}\left(\frac{77}{36}\right)\)

Putting the values of x and y; we get :

∴ \(\sin ^{-1}\left(\frac{8}{17}\right)+\sin ^{-1}\left(\frac{3}{5}\right)=\tan ^{-1}\left(\frac{77}{36}\right)\)= RHS Hence, proved.

Question 5. \(\cos ^{-1}\left(\frac{4}{5}\right)+\cos ^{-1}\left(\frac{12}{13}\right)=\cos ^{-1}\left(\frac{33}{65}\right)\)

Solution:

LHS =\(\cos ^{-1}\left(\frac{4}{5}\right)+\cos ^{-1}\left(\frac{12}{13}\right)\)

Let \(\cos ^{-1} \frac{4}{5}=x \Rightarrow \cos x=\frac{4}{5}\)

Now, \(\sin x=\sqrt{1-\cos ^2 x}=\sqrt{1-\left(\frac{4}{5}\right)^2}=\frac{3}{5}\)

Let y=\(\cos ^{-1} \frac{12}{13} \Rightarrow \cos y=\frac{12}{13}\)

Now, \(\sin y=\sqrt{1-\left(\frac{12}{13}\right)^2}=\frac{5}{13}\)

We know that \(\cos (x+y)=\cos x \cos y-\sin x \sin y\)

⇒ \(\cos (x+y)=\left(\frac{4}{5} \times \frac{12}{13}\right)-\left(\frac{3}{5} \times \frac{5}{13}\right)=\frac{48}{65}-\frac{15}{65}=\frac{33}{65}\)

x+y =\(\cos ^{-1}\left(\frac{33}{65}\right)\)

Putting the values of x and y

⇒ \(\cos ^{-1}\left(\frac{4}{5}\right)+\cos ^{-1}\left(\frac{12}{13}\right)=\cos ^{-1}\left(\frac{33}{65}\right)\)=RHS

∴ Hence proved.

Question 6. \(\cos ^{-1}\left(\frac{12}{13}\right)+\sin ^{-1}\left(\frac{3}{5}\right)=\sin ^{-1}\left(\frac{56}{65}\right)\)

Solution:

LHS =\(\cos ^{-1}\left(\frac{12}{13}\right)+\sin ^{-1}\left(\frac{3}{5}\right) \)

Let \(\cos ^{-1}\left(\frac{12}{13}\right)=x \Rightarrow \cos x=\frac{12}{13}. Now, \sin x=\sqrt{1-\cos ^2 x}=\sqrt{1-\left(\frac{12}{13}\right)^2}=\frac{5}{13}\)

Let y=\(\sin ^{-1} \frac{3}{5} \Rightarrow \sin y=\frac{3}{5}. Now, \cos y=\sqrt{1-\sin ^2 y}=\sqrt{1-\left(\frac{3}{5}\right)^2}=\frac{4}{5}\)

We know that \(\sin (x+y)=\sin x \cos y+\cos x \sin y=\left(\frac{5}{13} \times \frac{4}{5}\right)+\left(\frac{12}{13} \times \frac{3}{5}\right)=\frac{20}{65}+\frac{36}{65}\)

⇒ \(\sin (x+y)=\frac{56}{65} \Rightarrow x+y=\sin ^{-1}\left(\frac{56}{65}\right)\)

Putting values of x and y

∴ \(\cos ^{-1}\left(\frac{12}{13}\right)+\sin ^{-1}\left(\frac{3}{5}\right)=\sin ^{-1}\left(\frac{56}{65}\right)\)= RHS

Question 7. \(\tan ^{-1}\left(\frac{63}{16}\right)=\sin ^{-1}\left(\frac{5}{13}\right)+\cos ^{-1}\left(\frac{3}{5}\right)\)

Solution:

LHS =\(\sin ^{-1}\left(\frac{5}{13}\right)+\cos ^{-1}\left(\frac{3}{5}\right)\)

Let \(\sin ^{-1}\left(\frac{5}{13}\right)=x \Rightarrow \sin x=\frac{5}{13}\). Now, \(\cos x=\sqrt{1-\sin ^2 x}=\sqrt{1-\left(\frac{5}{13}\right)^2}=\frac{12}{13}\)

⇒ \(\tan x=\frac{\sin x}{\cos x} \Rightarrow \tan x=\frac{\left(\frac{5}{13}\right)}{\left(\frac{12}{13}\right)}=\frac{5}{12}\)

Let \(\cos ^{-1}\left(\frac{3}{5}\right)=y \Rightarrow \cos y=\frac{3}{5}\). Now, \(\sin y=\sqrt{1-\cos ^2 y}=\sqrt{1-\left(\frac{3}{5}\right)^2}=\frac{4}{5}\)

Let \(\cos ^{-1}\left(\frac{3}{5}\right)=y \Rightarrow \cos y=\frac{3}{5}\). Now, \(\sin y=\sqrt{1-\cos ^2 y}=\sqrt{1-\left(\frac{3}{5}\right)^2}=\frac{4}{5}\)

⇒ \(\tan y=\frac{\sin y}{\cos y} \Rightarrow \tan y=\frac{\left(\frac{4}{5}\right)}{\left(\frac{3}{5}\right)}=\frac{4}{3}\)

Now, \(\tan (x+y)=\frac{\tan x+\tan y}{1-\tan x \tan y}=\left[\frac{\frac{5}{12}+\frac{4}{3}}{1-\frac{5}{12} \times \frac{4}{3}}\right]=\left[\frac{\frac{15+48}{12 \times 3}}{\frac{12 \times 3-20}{12 \times 3}}\right]=\left(\frac{63}{16}\right)\)

Hence, \(\tan (x+y)=\frac{63}{16} \Rightarrow x+y=\tan ^{-1}\left(\frac{63}{16}\right)\)

Putting the values of x and y; we get \(\sin ^{-1}\left(\frac{5}{13}\right)+\cos ^{-1}\left(\frac{3}{5}\right)=\tan ^{-1}\left(\frac{63}{16}\right)\)= R.H.S.

Question 8. Prove that : \(\tan ^{-1} \sqrt{x}=\frac{1}{2} \cos ^{-1}\left(\frac{1-x}{1+x}\right), x \in[0,1]\)

Solution:

LHS =\(\tan ^{-1} \sqrt{x}=\frac{1}{2}\left(2 \tan ^{-1} \sqrt{x}\right)=\frac{1}{2} \cos ^{-1}\left(\frac{1-(\sqrt{x})^2}{1+(\sqrt{x})^2}\right) \quad\left[2 \tan ^{-1} x=\cos ^{-1}\left(\frac{1-x^2}{1+x^2}\right)\right]\)

= \(\frac{1}{2} \cos ^{-1}\left(\frac{1-x}{1+x}\right)\)= RHS

Question 9. \(\cot ^{-1}\left(\frac{\sqrt{1+\sin x}+\sqrt{1-\sin x}}{\sqrt{1+\sin x}-\sqrt{1-\sin x}}\right)=\frac{x}{2}, x \in\left(0, \frac{\pi}{4}\right)\)

Solution:

LHS =\(\cot ^{-1}\left(\frac{\sqrt{1+\sin x}+\sqrt{1-\sin x}}{\sqrt{1+\sin x}-\sqrt{1-\sin x}}\right)\) →     Equation 1

Now, we can write

⇒ \(\sqrt{1+\sin x}=\sqrt{\sin ^2 \frac{x}{2}+\cos ^2 \frac{x}{2}+2 \sin \frac{x}{2} \cos \frac{x}{2}}=\sqrt{\left(\sin \frac{x}{2}+\cos \frac{x}{2}\right)^2}\)

= \(\left|\sin \frac{x}{2}+\cos \frac{x}{2}\right|=\sin \frac{x}{2}+\cos \frac{x}{2}\)

Similarly, we can get \(\sqrt{1-\sin x}=\left|\cos \frac{x}{2}-\sin \frac{x}{2}\right|=\cos \frac{x}{2}-\sin \frac{x}{2}\)

On substituting the above two values in Equation (1), we get

LHS = \(\cot ^{-1}\left(\frac{\left(\sin \frac{x}{2}+\cos \frac{x}{2}\right)+\left(\cos \frac{x}{2}-\sin \frac{x}{2}\right)}{\left(\sin \frac{x}{2}+\cos \frac{x}{2}\right)-\left(\cos \frac{x}{2}-\sin \frac{x}{2}\right)}\right)=\cot ^{-1}\left(\frac{2 \cos \frac{x}{2}}{2 \sin \frac{x}{2}}\right)=\cot ^{-1}\left(\cot \frac{x}{2}\right)=\frac{x}{2}\)=RHS

Question 10. \(\tan ^{-1}\left(\frac{\sqrt{1+x}-\sqrt{1-x}}{\sqrt{1+x}+\sqrt{1-x}}\right)=\frac{\pi}{4}-\frac{1}{2} \cos ^{-1} x,-\frac{1}{\sqrt{2}} \leq x \leq 1\)

Solution:

Put x=\(\cos \theta \Rightarrow \cos ^{-1} x=\theta\)

L.H.S.= \(\tan ^{-1}\left(\frac{\sqrt{1+x}-\sqrt{1-x}}{\sqrt{1+x}+\sqrt{1-x}}\right)=\tan ^{-1}\left(\frac{\sqrt{1+\cos \theta}-\sqrt{1-\cos \theta}}{\sqrt{1+\cos \theta}+\sqrt{1-\cos \theta}}\right)\)

= \(\tan ^{-1}(\frac{\sqrt{2} \cos \frac{\theta}{2}-\sqrt{2} \sin \frac{\theta}{2}}{\sqrt{2} \cos \frac{\theta}{2}+\sqrt{2} \sin \frac{\theta}{2}}) ( 1+\cos \theta=2 \cos ^2 \frac{\theta}{2} and 1-\cos \theta=2 \sin ^2 \frac{\theta}{2})\)

= \(\tan ^{-1}(\frac{\cos \frac{\theta}{2}-\sin \frac{\theta}{2}}{\cos \frac{\theta}{2}+\sin \frac{\theta}{2}})=\tan ^{-1}(\frac{1-\tan \frac{\theta}{2}}{1+\tan \frac{\theta}{2}})\)

(inside the bracket divide numerator and denominator by \(\cos (\theta / 2)\) )

= \(\tan ^{-1}\left(\tan \left(\frac{\pi}{4}-\frac{\theta}{2}\right)\right) {\left[ \tan (A-B)=\frac{\tan A-\tan B}{1+\tan A \tan B}\right]}\)

= \(\frac{\pi}{4}-\frac{\theta}{2}=\frac{\pi}{4}-\frac{1}{2} \cos ^{-1} x\)= R.HS.

Question 11. \(2 \tan ^{-1}(\cos x)=\tan ^{-1}(2 {cosec} x)\)

Solution:

Given, 2 \(\tan ^{-1}(\cos x)=\tan ^{-1}(2 {cosec} x)\)

⇒ \(\tan \left[2 \tan ^{-1}(\cos x)\right]=2 {cosec} x [ Let \tan ^{-1}(\cos x)=\theta]\)

⇒ \(\tan 2 \theta=2 {cosec}\) x

⇒ \(\frac{2 \tan \theta}{1-\tan ^2 \theta}=2 {cosec}\) x

⇒ \(\frac{2 \tan ^{-1}\left(\tan ^{-1} \cos x\right)}{1-\tan ^2\left(\tan ^{-1} \cos x\right)}=2 {cosec} x\)

∴ \(\frac{2 \cos x}{1-\cos ^2 x}=\frac{2}{\sin x} \Rightarrow \cot x=1 \Rightarrow \cot x=\cot \frac{\pi}{4} \Rightarrow x=\frac{\pi}{4}\)

Question 12. \(\tan ^{-1}\left(\frac{1-x}{1+x}\right)=\frac{1}{2} \tan ^{-1} x,(x>0)\)

Solution:

Given; \(\tan ^{-1}\left(\frac{1-x}{1+x}\right)=\frac{1}{2} \tan ^{-1} x \Rightarrow 2 \tan ^{-1}\left(\frac{1-x}{1+x}\right)=\tan ^{-1} x \Rightarrow \tan \left\{2 \tan ^{-1}\left(\frac{1-x}{1+x}\right)\right\}\)=x

⇒ \(\tan 2 \theta=x \Rightarrow \frac{2 \tan \theta}{1-\tan ^2 \theta}\)=x

⇒ \(\frac{2 \tan [\tan ^{-1}(\frac{1-x}{1+x})]}{1-\tan ^2[\tan ^{-1}(\frac{1-x}{1+x})]}=x [\frac{\frac{2(1-x)}{(1+x)}}{\frac{(1+x)^2-(1-x)^2}{(1+x)^2}}]=x [\frac{2(1-x)(1+x)}{(1+x)^2-(1-x)^2}]\)=x

⇒ \({\left[\frac{2\left(1-x^2\right)}{1+x^2+2 x-1-x^2+2 x}\right]=x \Rightarrow\left[\frac{2\left(1-x^2\right)}{4 x}\right]=x}\)

⇒ \(\left(\frac{\left(1-x^2\right)}{2 x}\right)=x \Rightarrow \frac{1-x^2}{2 x}=x \Rightarrow 1-x^2=2 x^2 \Rightarrow 1=3 x^2 \Rightarrow x^2=\frac{1}{3} \Rightarrow x= \pm \frac{1}{\sqrt{3}}\)

x= \(\frac{1}{\sqrt{3}}\) [x>0 given, so we do not take x=-\(\frac{1}{\sqrt{3}}]\)

Question 13. \(\sin \left(\tan ^{-1} x\right) ;|x|<1\) is equal to

  1. \(\frac{x}{\sqrt{1-x^2}}\)
  2. \(\frac{1}{\sqrt{1-x^2}}\)
  3. \(\frac{1}{\sqrt{1+x^2}}\)
  4. \(\frac{x}{\sqrt{1+x^2}}\)

Solution: 4. \(\frac{x}{\sqrt{1+x^2}}\)

∴ \(\sin \left(\tan ^{-1} \mathrm{x}\right)=\sin \left[\sin ^{-1}\left(\frac{\mathrm{x}}{\sqrt{1+\mathrm{x}^2}}\right)\right]=\frac{\mathrm{x}}{\sqrt{1+\mathrm{x}^2}} \left[ \tan ^{-1} \mathrm{x}=\sin ^{-1}\left(\frac{\mathrm{x}}{\sqrt{1+\mathrm{x}^2}}\right)\right]\)

Question 14. If \(\sin ^{-1}(1-x)-2 \sin ^{-1} x=\frac{\pi}{2}\), then x is equal to

  1. 0, \(\frac{1}{2}\)
  2. 1, \(\frac{1}{2}\)
  3. 0
  4. \(\frac{1}{2}\)

Solution: 3. 0

Given, \(\sin ^{-1}(1-x)-2 \sin ^{-1} x=\frac{\pi}{2}\)

-2 \(\sin ^{-1} x=\frac{\pi}{2}-\sin ^{-1}(1-x) \Rightarrow-2 \sin ^{-1} x\)

= \(\cos ^{-1}(1-x) \left[ \sin ^{-1}(1-x)+\cos ^{-1}(1-x)=\frac{\pi}{2}\right]\)

⇒ \(\cos \left(-2 \sin ^{-1} \mathrm{x}\right)=1-\mathrm{x}\)

[Multiplying both sides by \(\cos x\) ]

⇒ \(\cos \left(2 \sin ^{-1} x\right)=1-x [\cos (-x)=\cos x]\)

⇒ \({\left[1-2 \sin ^2\left(\sin ^{-1} x\right)\right]=1-x} {\left[\cos 2 x=1-2 \sin ^2 x\right]}\)

⇒ \(1-2\left[\sin \left(\sin ^{-1} x\right)\right]^2=1-x {\left[\sin ^2 x=(\sin x)^2\right]} \)

⇒ \(1-2 x^2=1-x \Rightarrow 2 x^2-x\)=0

⇒ x(2 x-1)=0 \(\Rightarrow x\)=0 or 2 x-1=0 \(\Rightarrow\) x=0 or \(\frac{1}{2}\)

∴ But x=\(\frac{1}{2}\) does not satisfy the given equation, so x=0

Relations and Functions Class 12 Maths Important Questions Chapter 1

Relations And Functions Exercise 1.1

Question 1. Determine whether each of the following relations is reflexive, symmetric, or transitive :

  1.  Relation R in the set A = {1,2, 3…13, 14} defined as R = {(x, y): 3x – y = 0}
  2.  Relation R in the set N of natural numbers defined as R = {(x, y): y = x + 5 and x < 4}
  3. Relation R in the set A = (1, 2, 3, 4, 5, 6} as R = {(x, y); y is divisible by x}
  4.  Relation R in the set Z of all integers defined as R = {(x, y): x – y is an integer}
  5.  Relation R in the set A of human beings in a town at a particular time given by
    1.  R = {(x, y): x and y work at the same place}
    2. R = {(x, y): x and y live in the same locality}
    3.  R – {(x, y): x is exactly 7 cm taller than y}
    4.  R = {(x, y): x is wife of y)
    5.  R = {(x, y): x is father of y}

Solution:

1. A = {1,2, 3 … 13, 14}; R = {(x, y): 3x -y = 0}

Reflexive: R = {(1, 3), (2, 6), (3, 9), (4, 12)}

(1,1) ∉ R, So, R is not reflexive.

Symmetric: (1, 3) ∈ R, but (3, 1) ∉ R. So, R is not symmetric

Transitive: (1,3), (3, 9) ∈ R, but (1, 9) ∉ R. So, R is not transitive

Hence, R is neither reflexive, nor symmetric, nor transitive.

2. R = {(x, y): y = x + 5 and x < 4} ⇒ R ~ {(1, 6), (2, 7), (3, 8)}

Reflexive:  (1, 1) ∉ R. So, R is not reflexive.

Symmetric : (1,6) ∈ R, but (6, 1) ∉ R. R is not symmetric.

Transitive: Since there are no three elements x, y, z ∈ N such that (x, y) ∈ R, z) ∈ R but (x, z) ∉ R

∴ R is transitive. Hence, R is neither reflexive nor symmetric but it is transitive.

3. A = {1, 2, 3,4, 5, 6}; R = {(x, y): y is divisible by x}

Reflexive: Let x∈ A such that if (x, x) ∈ R x is divisible by x, which is true, ∀ x∈ A

∴ R is reflexive.

Read and Learn More Class 12 Maths Chapter Wise with Solutions

Symmetric: Let x, y ∈ A such that if (x, y) ∈ R => y is divisible by x

⇒ x is not divisible by => (y, x) ∉R

For example: (2, 4) ∈ R but (4, 2) ∉R.

∴ R is not symmetric.

Transitive: Let x, y, z ∈ A such that if (x, y) ∈ R and (y, z) ∈ R

⇒ y is divisible by x and z is divisible by y.

⇒ \(\frac{y}{x}=k_1 \in I \)  → Equation 1

⇒ \((y, z) \in R \Rightarrow \frac{z}{y}=k_2 \in\) I  → Equation 2

Equation (1) x (2) gives

⇒ \(\frac{y}{x} \times \frac{z}{y}=k_1 \times k_2 \Rightarrow \frac{z}{x}=\left(k_1 k_2\right) \in\) I

(x, z)\( \in\)

∴ R . R is transitive.

Hence, R is reflexive and transitive but not symmetric.

(4) R = {(x, y): x – y is an integer}

Reflexive: If (x, x) ∈R ⇒ x – x = 0, which is an integer, ∀ x ∈ Z.

∴ R is reflexive.

Symmetric: If (x, y) ∈ R ⇒ (x – y) is an integer. ⇒ (y – x) is also an integer.

⇒ (v. x) ∈R

∴ R is symmetric.

Transitive: If (x, y) ∉ R ⇒ x -y = k1 ∈  (1)

and (y, z) ∈ R ⇒ y – z = k2 ∈  (2)

Adding equation (1) and (2), we get

(x – y) + (y – z) = (k1 + k2) ∈

x – z = (k1 + k2) ∈ I

⇒ (x, z) ∈ R

∴ R is transitive.

Hence, R is reflexive, symmetric, and transitive.

5. Given A = (x : x is a human being in a town}

(1.) R = {(x, y): x and y work at the same place}

Reflexive: If (x, x) e R ⇒ x and x work at the same place, which is true, V x ∈ A R is reflexive.

Symmetric: If (x, y) ∈ R ⇒ x and y work at the same place.

⇒ y and x also work at the same place. ⇒ (y, x)∈R.

∴R is symmetric.

Transitive: If (x, y) ∈ R and (y, z) ∈ R

⇒ x and y work at the same place and y and z work at the same place.

⇒ x and z work at the same place. ⇒ (x, z) ∈ R

∴ R is transitive.

Hence, R is reflexive, symmetric, and transitive.

(2) R = {(x, y): x and y live in the same locality}

Reflexive: If (x, x) e R => x and x live in the same locality, which is true, ∀ x ∈ A

∴ R is reflexive.

Symmetric: If (x, y) ∈ R ⇒ x and y live in the same locality.

⇒ y and x also live in the same locality ⇒ (y, x) ∈ R

∴ R is symmetric.

Transitive: Let (x, y) ∈ R and (y, z) ∈ R.

x and y live in the same locality and y and z live in the same locality.

⇒ x and z live in the same locality. ⇒ (x, z) ∈ R

R is transitive. Hence, R is reflexive, symmetric, and transitive.

(3.) R = {(x, y): x is exactly 7 cm taller than y}

Reflexive: If (x, x) ∈ R ⇒ x is exactly 7 cm taller than x, which is not true, for any x ∈ A. Since human beings x cannot be taller than themselves.

∴ So, R is not reflexive.

Symmetric: If (x, y) ∈ R ⇒ x is exactly 7 cm taller than y ⇒ y is exactly 7 cm smaller

⇒ (y, x)∉ R

∴ R is not symmetric.

Transitive: If (x, y), (y, z) ∈ R

⇒ x is exactly 7 cm taller than y and y is exactly 7 cm taller than z.

⇒ x is exactly 14 cm taller than z.

∴ (x, z) ∉ R

∴ R is not transitive.

Hence, R is neither reflexive, nor symmetric, nor transitive.

(4) R = ((x, y): x is wife of y)

Reflexive: If (x, x) ∈ R ⇒ x is wife of x, which is not true, for any x ∈ A

∴ R is not reflexive.

Symmetric; If (x, y) ∈ R ⇒ x is the wife of y ⇒ y is the husband of x.

(y, x) ∉ R

R is not symmetric.

Transitive: There are no three elements x, y, z ∈ A such that (x, y) R and (y, z) ∈ R but (x, z) ∉ R

So, R is transitive. Hence, R is neither reflexive, nor symmetric, but R is transitive.

(5) R = {(x, y): x is father of y}

Reflexive: If (x, x) ∈ R ⇒ x is the father of x, which, is not true, for any x ∈ A

∴ R is not reflexive.

Symmetric: If (x, y) ∈ R ⇒ x is the father of y ⇒ his son or daughter of x.

(y,x) ∉ R R is not symmetric.

Transitive: If (x, y) ∈ R and (y, z) ∈ R,

x is the father of y, and y is the father of z ⇒ x is the grandfather of z.

(x, z) ∉ R

R is not transitive. Hence, R is neither reflexive, symmetric, nor transitive.

CBSE Class 12 Maths Chapter 1 Relations And Functions Important Question And Answers

Question 2. Show that the relation R in the set R of real numbers, defined as R = {(a, b): a < b} is neither reflexive nor symmetric nor transitive.

Solution:

Reflexive: R = {(a, b): a \(\leq b^2\)}

Let a ∈ R such that if (a, a) ∈ R ⇒ a ≤ a² is not true for all a ∈ R

i.e. (a, a) \(\notin R, \forall a \in\) R

Let a=\(\frac{1}{2} \in\) R

⇒ \(\frac{1}{2}\) ≰  \(\frac{1}{2^2}\) ⇒ \(\frac{1}{2}\) ≰  \(\frac{1}{4}\) ∈ R

∴ R is Not Reflexive

Symmetric: Let a, b ∈ R such that if (a, b) ∈ R ⇒ a ≤ b² ⇒ b≰  a²

For example. 2≰ 5² ⇒(2,5) ∈ R but 5≰ 2²

⇒ (5,2) ∉ R

∴ R is not symmetric

Transitive: Let a,b,csR such that if (a, b) ∈ R an nd (b. c) ∈ R

⇒ a ≤ b² and b ≤ c² ⇒ a ≰ c²

For Example: 5 ≤ 3²  and 3 ≤ 2² but 5 ≰ 2²

(5, 3) e R and (3, 2) ∈ R but (5, 2) ∉R

∴ R is not transitive

∴ Hence, R is neither reflexive nor symmetric nor transitive.

Question 3. Check whether the relation R defined in the set {1, 2, 3, 4, 5, 6} as R = {(a, b): b = a + 1} is reflexive, symmetric, or transitive.

Solution:

Reflexive: Given A = {1, 2, 3, 4, 5, 6}.

R = {(a, b): b = a + 1} ⇒ R = {(1, 2), (2, 3), (3, 4), (4, 5), (5, 6)} v (1,1) ∉ R, where 1 ∈ A

∴ R is not reflexive.

Symmetric: (1, 2) 6 R but (2, 1) ∉ R, where 1, 2 ∉ A.

∴ So R is not symmetric

Transitive: (1, 2) ∈ R and (2, 3) ∈ R but (1, 3) ∉ R, where 1,2,3 ∈ A.

R is not transitive Hence R is neither reflexive, symmetric, nor transitive.

Question 4. Show that the relation R in R defined as R = {(a, b): a < b}, is reflexive and transitive but not symmetric.

Solution:

Reflexive: Given R = {(a, b): a < b}

Let a ∈ R such that if (a, a) ∈ R ⇒ a < a, which is true, ∀a∈R

∴ R is reflexive

Symmetric: Let a, b e R such that if (a, b) e R ⇒ a < b ⇒ b > a ⇒ (b, a) ∉ R

For example: 1 < 2 ⇒ (1, 2) e R But 2 ∉1 ⇒ (2,1)

∴ R is not symmetric

Transitive: Let a, b, c ∈ R such that if (a, b) ∈ R and (b, c) ∈ R ⇒ a < b and b<c ⇒ a<c ⇒ (a, c) ∈ R

∴ R is transitive

Flence R is reflexive and transitive but not symmetric.

Question 5. Check whether the relation R in R defined as R = {(a, b): a < b³} is reflexive, symmetric, or transitive.

Solution:

Reflexive: Given R = {(a, b): a < b³}

Let a∈ R such that if (a, a) ∈ R ⇒ a < a’

Not true for all a ∈ R ⇒ (a, ) R, ∀ a ∈ R 1

Let a = \(\frac{1}{2} \in \mathrm{R}\)

⇒ \(\frac{1}{2} \notin \frac{1}{2^3} \Rightarrow \frac{1}{2} \notin \frac{1}{8} \Rightarrow\left(\frac{1}{2}, \frac{1}{2}\right) \notin \mathrm{R}\)

∴ R is not Reflexive.

Symmetric: Let a, b ∈ R such that if (a, b) ∈ R ⇒ a < b³ ⇒ b∉ a³

For example: 2 < 9J but ∈ ∉ 21 => (2, 9)∈R but (9, 2) ∉ R

∴ R is not symmetric

Transitiv e: Let a, b, c ∈ R such that if (a, b) ∈ R and (b, c) ∈ R

⇒ a < b³ and b < c³ ⇒ a ∉ c³

For example: 9 < 33 and 3 < 2 but 9

⇒ (9,3) ∈ R and (3,2) ∈ R but (9,2) ∉ R

∴ R is not transitive

∴ Hence R is neither reflexive nor symmetric nor transitive.

Question 6. Show that the relation R in the set {1,2, 3} given by R = {(1, 2), (2, 1)} is symmetric but neither reflexive nor transitive.

Solution:

Reflexive: Let A = {1, 2, 3}.

A relation R on A is defined as R = {(1, 2), (2,1)}.

v (1,1) ∉ R, where 1 ∈ A R is not reflexive

Symmetric: (1,2) ∈ R and (2,1) ∈ R

∴ R is symmetric

Transitive: (1,2) ∈ R and (2,1) ∈ R but (1,1) ∉ R

∴ R is not transitive

Hence, R is symmetric but neither reflexive nor transitive.

Question 7. Show that the relation R in the set A of all the books in a library of a college, given by R = {(x, y): x and y have same number of pages} is an equivalence relation.

Solution:

Reflexive: Set A is the set of all books in the library of a college. R = {(x, y): x and y have the same number of pages}

Let x ∉ A such that if (x, x) ∈ R ⇒ x and x have the same number of pages.

Which is the true, V x ∈ A

∴R is reflexive

Symmetric: Let x, y ∈ A such that if (x, y) ∈ R ⇒ x and y have the same number of pages.

⇒ y and x have the same number of pages. ⇒ (y, x)∈ R

∴ R is symmetric.

Transitive: Let x, y, z ∈ A such that if (x, y) ⇒ R and (y, z)∈ R.

⇒ x and y have the same number of pages and y and z have the same number of pages.

⇒ x and z have the same number of pages ⇒ (x, z) ∈ R

∴ R is transitive.

Since, R is reflexive, symmetric, and transitive on A

∴ Hence, R is an equivalence relation to A.

Question 8. Show that the relation R in the set A = {1, 2, 3, 4, 5} given by R = {(a, b): |a – b| is even}, is an equivalence relation. Show that all the elements of {1,3, 5} are related to each other and all the elements of {2, 4} are related to each other. But no element of {1, 3, 5} is related to any element of {2,4}.

Solution:

Reflexive: A = {1,2,3,4, 5}, R = [{a, b);| a — b| is even}

Let a ∈ A such that if (a, a) ∈ R ⇒ |a – a| = 0, is even

which is true, ∀ a ∈ A

∴ R is reflexive.

Symmetric: Let a, b ∈ A such that if (a, b) ∈ R ⇒ |a- b| is even |-(b – a)| = |b – a| is also even

⇒ (b, a) ∈ R

∴ R is symmetric.

Transitive: Let a, b, c ∈ R such that if (a, b) ∉ R and (b, c) ∈ It

⇒ |a — b| and |b – c| both are even

⇒ |a-b| = 2k, and | b — c| = 2k2, (k1,k2 ∈ Z)

⇒  (a-b) = ±2k,(1) and (b-c) = ± 2 k2  (2)

On adding equations (1) & (2), we get:

(a-c) = ± 2(k1+ k2) ⇒ |a-c| = 2(k1 + k2)

⇒ |a – c| is even ⇒ (a, c) ∈ R (v k1 + k2 ∈ Z)

∴ R is transitive

Since R is reflexive, symmetric, and transitive on A.

Hence, R is an equivalence relation to A.

Since the difference of two even (or odd) is always even.

So, every element of set {1, 3, 5} is related to each other and also every element of set {2,4} is related to each other but no element of {1,3, 5} is related to any element of {2,4}.

Question 9. Show that each of the relation R in the set A = {x∈Z : 0 < x < 12), given by

1. R = {(a, b): |a — b| is a multiple of 4}

2. R = {(a, b): a = b} is an equivalence relation. Find the set of all elements related to 1 in each case.

Solution:

Given that: A = (x ∈ Z : 0 ^x <. 12} = {0, 1,2, 3,4, 5, 6,7, 8,9,10,11,12}

1. R = {(a, b): |a – b|} is a multiple of 4}

Reflexive: Let a ∈ A such that if (a, a) ∈ R ⇒ |a- a| = 0 is a multiple of 4 which is true,∀ a ∈ A

∴ R is reflexive.

Symmetric: Let a, b ∈ A such that if (a, b) ∉ R ⇒ |a – b| is a multiple of 4.

⇒ |b – a| is also a multiple of 4,

(b, a) ∈ R

∴ R is symmetric.

Transitive: Let a, b, c ∉ A such that if (a, b) ∈ R and (b, c) ∈ R

⇒ |a — b| and |b – c| both are multiple of 4

⇒ |a-b| = 4k1 and |b-c| = 4k2 (k1 k2 ∈ Z)

⇒ a-b = ±4k1 (1) and b-c = ±4k2 → (2)

On adding equations (1) & (2), we get:

a – c = ± 4 (k1 + k2) ⇒ |a-c| = 4(k1 + k2)

|a – c| is multiple of 4 (∴  k1 + k2 ∈ Z)

(a, c) ∈ R

∴ R is transitive

Since R is reflexive, symmetric, and transitive on A

Hence, R is an equivalence relation on A,

Further, let x ∈ A such that (x, 1) ∈ R

|x-1| is multiple of 4 ⇒ |x- 11 = 0,4, 8,

x-1 =0,4,8 ⇒ x- 1,5,9

Hence, required set ={1,5,9}

2. R= {(a, b): a = b}

Reflexive: Let a ∈A, such that (a, a) ∉ R, ⇒ a = a. which is true, ∀ a ∈ A.

∴ So, R is reflexive.

Symmetric: Let a, b ∉ A such that if (a, b) ∈ R.

⇒ a = b ⇒ b~a ⇒ (b, a) ∈ R

∴ R is symmetric.

Transitive: Let a, b, c ∈ A such that if (a, b) ∈ R and (b, c) ∈ R.

⇒ a = b and b = c a = c (a, c) ∈ R

∴  R is transitive.

Since R is reflexive, symmetric, and transitive on A

Hence, R is an equivalence relation on A

Let x ∈ A such that (x, 1) ∈ R ⇒ x  – 1

Hence, required set = {1}

Question 10. Give an example of a relation. Which is

  1.  Symmetric but neither reflexive nor transitive.
  2.  Transitive but neither reflexive nor symmetric,
  3.  Reflexive and symmetric but not transitive.
  4.  Reflexive and transitive but not symmetric.
  5. Symmetric and transitive but not reflexive.

Solution:

(1) Let A = {5,6, 7}.

Reflexive: Define a relation R on A as R = {(5, 6), (6, 5)}.

Since (5,5) ∉ R, where 5 ∈ A.

∴ So, R is not reflexive

Symmetric: Since (5, 6) ∈ R and (6, 5) ∈ R. So, R is symmetric

Transitive: Since (5, 6) ∈ R and (6, 5) ∈ R but (5, 5) ∉ R.

So, R is not transitive Hence, relation R is symmetric but neither reflexive nor transitive.

2. Consider a relation R in N defined as:

R = {(a, b): a < b)

Reflexive: Let a ∈ N such that if (a, a) ∈ R ⇒ a < a, which is not true for any a ∈ N

∴  R is not reflexive

Symmetric: Let a, b e N such that if (a, b) ∈ R ⇒ a < b:⇒ b > a ⇒ (b, a) ∉ R

∴  R is not symmetric

Transitive: Let a, b, c ∈ N such that if (a, b) ∈ R and (b, c) ∈ R

a < b and b < c ⇒ a < c

(a, c) ∈ R.

∴ So, R is transitive

Hence, relation R is transitive but neither reflexive nor symmetric.

3. Let A = {4, 6, 8}.

Define a relation R on A as R = {(4, 4), (6, 6), (8, 8), (4, 6), (6, 4), (6, 8), (8, 6)}

Reflexive: Since, (a, a) ∈ R, ∀ a ∈ A.

∴ So, R is reflexive

Symmetric: Since, (a, b) ∉ R (b, a) ∈ R, for all a, b ∈ A.

∴ So, R is symmetric.

Transitive: Since, (4, 6) ∈ R and (6, 8) ∈ R but (4, 8) R.

∴ So, R is not transitive.

Hence, R is reflexive symmetric but not transitive.

4. Define a relation R, in R as: R, = {(a, b): a > b}

Reflexive: Let a ∈ R1 such that if (a, a) ∉ R1 ⇒ a³ > a, which is true ∀ a ∈ R

∴ R1 is reflexive.

Symmetric: Let a, b e R1 such that if (a, b) ∈ R1 a³ ≥ b³ => b³≤ a³

For example: 2³≥ 1³ but 1³ ≤ 2³

(2, 1) ∈ R, but (1,2) ∉ R1

∴ R1 is not symmetric.

Transitive: Let a, b, c ∈ R such that if (a, b) ∉ R1 and (b, c) ∈ R1

⇒ a³≥ b³ and b³ ≥ c³ ⇒ a³≥ c³ ⇒ (a, c) ∈ R1

∴ R is transitive.

Hence, relation R1 is reflexive and transitive but not symmetric,

5. Let A = {1,2, 3}.

Define a relation R on A as: R = {(1, 1) (2, 2)}

Reflexive: Since (3, 3) ∉R, where 3 ∈ A. So, R is not reflexive

Symmetric: If (a, b) ∈ R

Here, a = b ⇒ b = a ⇒ (b, a)∈R

∴ R is symmetric

Transitive: If (a, b)∈R and (b,c)∈R Here a = b and b = c ⇒ a = c

(a, c) ∈ R

∴  R is transitive

Hence, relation R is symmetric and transitive but not reflexive.

Question 11. Show that the relation R in the set A of points in a plane given by R = {(P, Q): distance of the point P from the origin is same as the distance of the point Q from the origin}, is an equivalence relation. Further, shows that the set of all points related to a point P * (0, 0) is the circle passing through P with the origin as the center.

Solution:

Given the set A of points in a plane and a relation R on A selection as

R = {(P, Q): distance of point P from the origin is the same as the distance of point Q from the origin}

Reflexive: Let P ∈ A such that if (P, P) ∈ R ⇒ OP = OP, which is true, ∀P ∈ A

∴ R is reflexive.

Symmetric: Let P, Q ∈ A such that if (P, Q) ∈ R ⇒ OP = OQ ⇒ OQ = OP ⇒ (Q, P) ∈ R

∴ R is symmetric.

Transitive: Let P, Q, S ∈ A such that if (P, Q) ∈ R and (Q, S) ∈ R ⇒ OP = OQ and OQ = OS ⇒ OP = OS (P, S) ∈ R

∴ R is transitive.

Since R is reflexive symmetric and transitive.

Hence, R is an equivalence relation.

The set of all points related to P ≠ (0,0) will be those points whose distance from the origin is the same as the distance of point P from the origin.

In other words, If O (0, 0) is the origin and OP – k, then the set of all points related to P is at a distance of k from the origin.

Hence, this set of points forms a circle with the center as the origin and this circle passes through point P.

Question 12. Show that the relation R defined in the set A of all triangles as R = {(T1 T2): T1 is similar to T2}, is an equivalence relation. Consider three right angle triangles T1 with sides 3, 4, 5, T2 with sides 5,12,13, and T3 with sides 6,8,10. Which triangles among T1 T2 and T3 are related?

Solution:

Given, set A of all triangles and a relation R on A defined by R = {(T1 T2): T1 is similar to T2 Reflexive: Since every triangle is similar to itself,

∴ R is reflexive

Symmetric: Let T1 T2 ∈ A such that if (T1 T2) ∉ R ⇒ T, is similar to T2 ⇒ T2 is similar to T1

⇒ (T2T1) ∈ R

∴ R is symmetric.

Transitive: Let T1 T2, T3 ∈ A such that if (T1 T2), (T2 T3) ∈ R.

⇒ T1 is similar to T2 and T2 is similar to T3.

⇒ T1 is similar to T3. ⇒ (T1 T3) ∈ R

∴ R is transitive.

Since, R is reflexive, symmetric, and transitive.

Hence, R is an equivalence relation.

Given three right angle triangles T1 with sides 3,4, 5, T2 with sides 5,12,13 and T3 with sides 6,8,10

\(\frac{3}{6}=\frac{4}{8}=\frac{5}{10}=\frac{1}{2}\)

Since the corresponding sides of triangles T1 and T3 are in the same ratio.

Then, triangle T1 is similar to triangle T3

Hence, T1 is related to T3

Question 13. Show that the relation R defined in the set A of all polygons as R = {(P1 P2): P1 and P2 have same number of sides}, is an equivalence relation. What is the set of all elements in A related to the right-angled triangle T with sides 3, 4, and 5?

Solution:

Given a relation R defined in the set A of all polygons as :

R = {(P1 P2): P1 and P2 have the same number of sides}

Reflexive: Let P, ∈ A such that if (P1 P2) ∈ R as the same polygon has the same number of sides with itself.

∴ R is reflexive.

Symmetric: Let P1 P2 ∈ A such that if (P1 P2) ∈ R ⇒ P1 and P2 have same number of sides

⇒ P3 and P1 have same number of sides ⇒ (P2, P1) ∈ R

∴ R is symmetric.

Transitive: Let P1 P2, P3 ∈ A such that

If(P1P2), (P2, P3)∈R.

P1 and P3 have the same number of sides, and P2 and P3 have the same number of sides.

⇒ P1 and P3 have the same number of sides.

⇒ (P1 P3) ∈ R R is transitive.

Since R is reflexive, symmetric, and transitive Hence, R is an equivalence relation.

The elements in A related to the right-angled triangle (T) with sides 3, 4, and 5 are those polygons that have 3 sides (since T is a polygon with 3 sides).

Hence, the required set is the set of all triangles of A which are related to T.

Question 14. Let L be the set of all lines in XY plane and R be the relation in L defined as R = {(L1 L2): L1 is parallel to L2}. Show that R is an equivalence relation. Find the set of all lines related to the line y = 2x + 4.

Solution:

Given, L is the set of all lines in XY plane and relation R on set L defined as R = {(L1 L2): L1 is parallel to L2}

Reflexive: Let L1 ∈ L such that if (L1 L2) ∈ R L1 ||

L1 which is true, ∀ L1 ∈ L

∴ R is reflexive

Symmetric: Let L1 L2 ∈ L such that if (L1 L2) ∈ R.

⇒ L1 is parallel to L2 ⇒ L2 is parallel to L1 ⇒  (L2, L1) ∈ R

∴ R is symmetric.

Transitive: Let L1 L2 L3 ∈ L such that if (L1 L2), (L2, L3) ∈ R.

L1 is parallel to L2 and L2 is parallel to L3

⇒ L1 is parallel to L3

∴ R is transitive.

Since R is reflexive, symmetric, and transitive

Hence, R is an equivalence relation.

The set of all lines related to the line y = 2x + 4 is the set of all lines that are parallel to the line y = 2x + 4.

The slope of line y = 2x + 4 is m = 2.

It is known that parallel lines have the same slope.

The line parallel to the given line is of the form y = 2x + ∈, where c ∈ R.

Hence, the set of all lines related to the given line Is given by y = 2x + c, where c ∈ R.

Question 15. Let R be the relation in the set {1, 2, 3, 4} given by R = {(1, 2), (2. 2), (1, 1), (4, 4), (1, 3), (3, 3), (3, 2)}.

Choose the correct answer.

  1. R is reflexive and symmetric but not transitive.
  2. R is reflexive and transitive but not symmetric.
  3. R is symmetric and transitive but not reflexive.
  4. R is an equivalence relation,

Solution:

(B) Let A = {1,2, 3, 4}, given a relation R on A defined as R = {(1,2), (2, 2), (1, 1)( (4, 4), (1, 3), (3, 3), (3, 2)}

Reflexive: Since (a, a) ∈ R, ∀ a ∈ A.

∴ So, R is reflexive.

Symmetric: Since, (1,2) ∈ R, but (2, 1) ∉ R.

∴  So, R is not symmetric.

Transitive: Since at least three elements a, b, c ∈ A do not exist Such that (a, b)∈ R and (b, ∈) ∉ R ⇒ (a, c) ∉ R

∴ R is transitive.

Hence, R is reflexive and transitive but not symmetric.

Question 16. Let R be the relation in the set N given by R = {(a, b): a = b – 2, b > 6}. Choose the correct answer.

  1. (2, 4) ∈ R
  2. (3, 8) ∈ R
  3. (6, 8) ∈ R
  4.  (8, 7) ∈ R

Solution:  3.

R = {(a, b): a = b – 2, b > 6}

∴ b > 6 and a = b – 2

R = {(5, 7), (6, 8), (7, 9) }

Here (6, 8) ∈ R

Relations And Functions Exercise 1.2

Question 1. Show that the function f: \(\mathrm{R}_* \rightarrow \mathrm{R}_*\) defined by f(x) = \(\frac{1}{x}\) is one-one and onto, where \(\mathrm{R}_*\) is the set of all non-zero real numbers. Is the result true, if the domain \(\mathrm{R}_*\) is replaced by N with the co-domain being the same as \(\mathrm{R}_*\)?

Solution:

Given that f : \(\mathrm{R}_* \rightarrow \mathrm{R}_*\) is defined by f(x) = \(\frac{1}{x}\)

Let x1 x2 ∉ \(\mathrm{R}_*\) (domain) such that if f\(f\left(x_2\right) \Rightarrow \frac{1}{x_1}=\frac{1}{x_2} \Rightarrow x_1=x_2\)

∴ f is one-one.

y ∈ \(\mathrm{R}_*\) (codomain) and f(x) = y

⇒ \(\frac{1}{x}=y \Rightarrow x=\frac{1}{y} \forall y \in R, \text { there exists } x=\frac{1}{y} \in R \text { such that } f(x)=\frac{1}{\left(\frac{1}{y}\right)}\)=y

∴ f is onto.

Thus, the given function f is one-one and onto.

Now, consider function g : N →\(\mathrm{R}_*\) defined by g(x) = \(\frac{1}{x}\)

Let x1 x2 ∈ N

Let y∈\(\mathrm{R}_*\), and if g(x,) = \(\frac{1}{x_1}=\frac{1}{x_2} \Rightarrow x_1=x_2\)

∴ g is one-one.

Let y=g(x)=\(\frac{1}{x} \Rightarrow x=\frac{1}{y} \notin N, \forall y \in \mathrm{R}_*\)

For example: y=\(\frac{2}{3} \in \mathrm{R}_*\) then x=\(\frac{3}{2} \notin N\)

∴ g is not onto

Function g is one-one but not onto.

Hence the result is not true.

Question 2. Check the injectivity and surjectivity of the following functions:

  1. f: N → N given by f(x) = x²
  2. f: Z → Z given by f(x) – x²
  3. f: R → R given by f(x) = x²
  4. f: N → N given by f(x) = x³
  5. f: Z → Z given by f(x) = x³

Solution:

1.f: N → N is given by, f(x) = x²

Let x1 x2 ∈ N, such that if f(x2) = f(x²) x1² =x2²

⇒ (x1 – X2) (x1 + x2) = 0 ⇒ x1 – x2 = 0 ⇒ x1 = x2 (∴ x1 + x2 ≠0)

∴ f is injective.

Let y ∈ N (codomain)

Put y = f(x) = x² ⇒ x = √y <∉ N, ∀y ∈N

Every element of the codomain does not have pre-images in the domain.

Range 54 Co-domain

For eg. range of f = {1,4, 9…,} ≠ co-domain of f

∴ f is not surjective.

Hence, function f is injective but not surjective.

2. f: Z → Z is given by, f(x) = x2 Let x1 x2 ∉ Z, such that

If f(X1) = f(x2) ⇒ x²1 = X²2

⇒ (X1 – X2) (x1 + x2) = 0

⇒ x1 + x2 = 0 or x1 = x2 ⇒ x1 + x2 = 0 or x1 = – x2

∴ The function does not have a unique solution

f is not injective Let y ∈ Z (co-domain)

Put y = f(x) = x² ⇒ x = √y ∉ Z, ∀ y ∈ Z

Every element of the co-domain does not have pre-images in the domain.

Range ≠ Co-domain

For example: f(-1) = f(1) = I, but – 1 ≠1,

f is not injective.

Since the range of f = {0, 1, 4, 9 } ≠ co-domain of f.

∴ f is not surjective.

Hence, function f is neither injective nor surjective.

3. f: R → R is given by f(x) = x²

Let x1 x2 ∈ R, such that if f(x,) = f(x2) ⇒ x²1 = x²2 ⇒ (x1 – x2) (x1 + x2) = 0

x1 – x2 = 0 and x1 = x2

x1 + x2 = 0 and x1 = – x2

The function does not have a unique solution

∴ f is not injective.

Let y ∉ R (co-domain)

Put y = f(x) = x”⇒ x = ,√y<∉ R, ∀ y ∈ R

Every element of the co-domain does not have an image in the domain

Range ≠ co-domain

For example ; f(-1) – f(l) = 1, but – 1 ≠ 1,

∴ f Is not injective.

Since, the range of f ≠ co-domain of f

∴ f is not surjective.

Hence, function f is neither injective nor surjective.

4. f: N → N is given by, f(x) = x³

Let x1 x2∈N (domain) such that if f(x1) = f(x2)

⇒ \( x_1^3=x_2^3 \Rightarrow\left(x_1-x_2\right)\left(x_1^2+x_1 x_2+x_2^2\right)\)=0

⇒ \( x_1-x_2=0 \Rightarrow x_1=x_2 ( x_1^2+x_1 x_2+x_2^2 \neq 0)\)

∴ f is injective.

Let y ∈ N (co-domain)

Put y = f(x) = x³ = (y)1/3 ∉ N, ∀y ∈ N

Every element of the co-domain does not have pre-images in the domain.

Range ≠ co-domain

Range of f = {1, 8, 27 } ≠ co-domain of f

∴ f is not surjective.

Hence, function f is injective but not surjective.

5. f: Z → Z is given by, f(x) = x³

Let x1 x2 ∈ Z (domain) such that

if  \(f\left(x_1\right)=f\left(x_2\right) \Rightarrow x_1^3=x_2^3 \)

⇒ \(\left(x_1-x_2\right)\left(x_1^2+x_1 x_2+x_2^2\right)=0 \Rightarrow\left(x_1-x_2\right)\left[\left(x_1+\frac{x_2}{2}\right)^2+\frac{3}{4} x_2^2\right]\)=0

⇒ \(x_1=x_2\left(\left(x_1+\frac{x_2}{2}\right)^2+\frac{3}{4} x_2^2 \neq 0\right)\)

∴ f is injective.

Let y ∈ Z (co-domain)

Put y = f(x) =x² ⇒ x=(y)1/3 ∉ Z, ∀y ∉ Z

Every element of the co-domain does not have an image in the domain

Range ≠ co-domain

Range of f = (0, ± 1, ±8, ± 27,…} ≠ co-domain of f.

∴ f is not surjective.

Hence, the function is injective but not surjective.

Question 3. Prove that the Greatest Integer Function f: R → R given by f(x) = [x], is neither one-one nor onto, where [x] denotes the greatest integer less than or equal to x.

Solution:

f: R → R is given by, f(x) = [x]

It is seen that f(1.2) = [1.2] = 1, f(1.9) = [1.9] = 1.

f(1.2)-f(1.9), but 1.2 ≠ 1.9.

∴ f is not one-one.

Range of f = I

⇒ Range of f ≠ co-domain of f.

f is not onto.

Hence, the greatest integer function is neither one-one nor onto.

Question 4. Show that the Modulus Function f: R → R given by f(x) — |x|, is neither one-one nor onto, where ]xj is x if x is positive or 0 and j x | is – x, if x is negative,

Solution:

f: R → R is given by, f(x) = | x | = { x,  if  \(x \geq 0\)  -x,  if  x<0

It is seen that f(-1) = |-1| = 1, f(1) = | 1 | = 1.

f(-1) = f(1), but -1 ≠ 1

∴ f is not one-one.

Range of f = R+ ∪ {O} ⇒ range of f ≠ co-domain of f,

∴ f is not onto.

Hence, the modulus function is neither one-one nor onto.

Question 5. Show that the Signum Function f: R → R is given by, \(\mathrm{f}(\mathrm{x})= \begin{cases}1, & \text { if } \mathrm{x}>0 \\ 0, & \text { if } x=0 \\ -1 & \text { if } x<0\end{cases}\), is neither one-one nor onto.

Solution:

f: R → R is given by, \(\mathrm{f}(\mathrm{x})= \begin{cases}1, & \text { if } \mathrm{x}>0 \\ 0, & \text { if } x=0 \\ -1 & \text { if } x<0\end{cases}\)

∴ Y f(x) – 1, ∀ x ∈ (0, ∞) and f(x) = – 1, ∀ x ∈(-∞, 0)

∴ f is not one-one.

Range of f= {0, ± 1}

Range of f ≠ co-domain off.

So, f is not onto.

Hence, the signum function is neither one-on-one nor onto,

Question 6. Let A = (1, 2, 3), B = {4, 5, 6, 7} and let f = ((1, 4), (2, 5), (3, 6)} be a function from A to B. Show that f is one-one.

Solution:

It is given that A = {1, 2, 3}, B = (4, 5, 6, 7}.

f: A → B is defined as f = {(1,4), (2, 5), (3, 6)}.

f(1) = 4, f(2) = 5, f(3) = 6

Since different elements of A have different images in B

Hence, function f is one-one.

Question 7. In each of the following cases, state whether the function is one-one, onto, or bijective. Justify your answer.

  1. f: R → R defined by f(x) = 3 – 4x
  2. f: R→ R defined by f(x) = 1 + x2

Solution:

1. f: R →  R defined by f(x) = 3 – 4x.

Let x1 x2 ∈ R such that f(x,)= f(x²)

⇒ 3 – 4x1 = 3 – 4x2

⇒ – 4x1 = – 4x2 ⇒ x1 = x2

∴ f is one-one.

Let y ∈ R (co-domain) put y = f(x) ⇒ y = 3 – 4x

x=\(\frac{3-y}{4} \in\) R (domain),  \(f\left(\frac{3-y}{4}\right)=3-4\left(\frac{3-y}{4}\right)=y, \forall y \in\) R (co-domain)

∴ f is onto.

Since f is one-one and onto.

Hence, f is bijective.

2. f:R→ R is defined as, f(x) = 1 + x².

Let x1 x2 ∈ R such that f(x1) = f(x2)

1 + x²1 – 1 + x²2 ⇒ x²1 = X²2 ⇒ x1 – ± x2

f(1) = 2, f(-1) = 2

⇒ f(l) = f(-l) but 1 ≠ – 1

∴ f is not one-one.

Here range of f = [1, ∞)

∴  Range of f ≠ co-domain of f.

∴ f is not onto.

Hence, f is neither one-one nor onto.

Question 8. Let A and B be sets. Show that f: AxB → B x A such that f(a, b) = (b, a) is a bijective function.

Solution:

f: AxB → BxA is defined as f(a, b) = (b, a).

Let (a1b1), (a2 b2) ∈ A x B such that if f(a,b,) = f(a2 b2)

⇒ (b1 a1) = (b2 a2) ⇒ b1= b2 and a1 = a2 ⇒ (a1 b1) = (a1 b2).

So, f is one-one.

Now, let (b, a) ∈ B x A ⇒ (a, b) ∈ A x B

i.e. ∀ (b, a) ∈ B x A, ∃ (a, b) ∈ A x B such that f(a, b) = (b, a). (Definition of f)

So, f is onto, Hence, f is bijective.

Question 9. Let f: N \(\rightarrow N \text { be defined by } f(n)=\left\{\begin{array}{ll} \frac{n+1}{2}, & \text { if } n \text { is odd } \\ \frac{n}{2}, & \text { if } n \text { is even } \end{array} \text {, for all } n \in N\right.\)
State whether the function f is objective. Justify your answer

Solution:

f(1)=\(\frac{1+1}{2}=1, f(2)=\frac{2}{2}=1 \Rightarrow f(1)=f(2) \text { but } 1 \neq 2\)

f is not one-on-one.

∴ Hence, f is not bijective.

Question 10. Let \(\mathrm{A}=\mathrm{R}-\{3\}\) and \(\mathrm{B}=\mathrm{R}-\{1\}\). Consider the function f: \(\mathrm{A} \rightarrow \mathrm{B}\) defined by \(\mathrm{f}(\mathrm{x})=\left(\frac{\mathrm{x}-2}{\mathrm{x}-3}\right)\) Is f one-one and onto? Justify your answer.

Solution:

Let \(x_1, x_2 \in\) A such that \(f\left(x_1\right)=f\left(x_2\right)\)

⇒ \(\frac{x_1-2}{x_1-3}=\frac{x_2-2}{x_2-3} \Rightarrow\left(x_1-2\right)\left(x_2-3\right)=\left(x_2-2\right)\left(x_1-3\right) \)

⇒  \(x_1 x_2-3 x_1-2 x_2+6=x_1 x_2-3 x_2-2 x_1+6\)

⇒  \(-3 x_1-2 x_2=-3 x_2-2 x_1 \)

⇒  \(3 x_1-2 x_1=3 x_2-2 x_2 \)

⇒  \(x_1=x_2\)

∴ f is one-one.

\(\text { Let } y=f(x) \forall y \in\) B

y=\(\frac{x-2}{x-3} \Rightarrow \)x y-3 y=x-2

x y-x=3 y-2

x(y-1)=3 y-2 \(\Rightarrow x=\frac{3 y-2}{y-1}\)

every element of B has pre-images in A.

Range = co-domain

∴  f is onto.

Hence, function f is one-one and onto.

Question 11. Let f: R → R be defined as f(x) = x4. Choose the correct answer.

  1.  f is one – one onto
  2.  f is many-one onto
  3.  f is one-one but not onto
  4.  f is neither one-on-one nor onto

Solution: 4.

f: R → R is defined as f(x) = x4

f( 1) = 1, f(-1) = 1 => f( 1) = f(-1) but 1 1,

So. f is not one-one

Again, Range of f = [0, <=o)

range of f c co-domain of f.

∴ So, f is not onto.

Hence, function f is neither one-one nor onto.

Question 12. Let f: R→ R be defined as fix) = 3x. Choose the correct answer.

  1.  f is one-one onto
  2.  f is many-one onto
  3. f is one-one but not onto
  4. f is neither one-on-one nor onto

Solution: 1.

Let \(\mathrm{x}_1 \mathrm{x}_2 \in \mathrm{R}\) such that \(\mathrm{f}\left(\mathrm{x}_1\right)=\mathrm{f}\left(\mathrm{x}_2\right) \Rightarrow 3 \mathrm{x}_1=3 \mathrm{x}_2 \Rightarrow \mathrm{x}_1=\mathrm{x}_2\).

So f is one – one.

Let y=f(x) \(\forall y \in\) R

y=3 x \(\Rightarrow x=\frac{y}{3} \in\) R

Range ≠ Co-domain.

So f is onto Hence, function f is one-one and onto.

Relations And Functions Miscellaneous Exercise

Question 1. Show that function f: \(\{\mathrm{x} \in \mathrm{R}:-1<\mathrm{x}<1\}\) defined by \(\mathrm{f}(\mathrm{x})=\frac{\mathrm{x}}{1+|\mathrm{x}|}, \mathrm{x} \in \mathrm{R}\) Is one-one and onto function.

Solution:

For one-one function :

Case-1: when \(\mathrm{x}_1, \mathrm{x}_2>0\)

Let f\(\left(x_1\right)=f\left(x_2\right) \Rightarrow \frac{x_1}{1+x_1}=\frac{x_2}{1+x_2} \Rightarrow x_1+x_1 x_2=x_2+x_1 x_2 \Rightarrow x_1=x_2\)

Case-2: when \(\mathrm{x}_1, \mathrm{x}_2<0\)

Let f\(\left(x_1\right)=f\left(x_2\right) \Rightarrow \frac{x_1}{1-x_1}=\frac{x_2}{1-x_2} \Rightarrow x_1-x_1 x_2=x_2-x_1 x_1 \Rightarrow x_1=x_2\)

Case-3: when \(x_1>0, x_2<0\)

Let f\(\left(x_1\right)=f\left(x_2\right) \Rightarrow \frac{x_1}{1+x_1}=\frac{x_2}{1-x_2} \Rightarrow x_1-x_1 x_2=x_2+x_1 x_2 = 0\)

which is not possible \((\mathrm{x}_1>0, \mathrm{x}_2<0)\)

⇒ \(\mathrm{x}_1 \neq \mathrm{x}_2 \Rightarrow \mathrm{f}\left(\mathrm{x}_1\right) \neq \mathrm{f}\left(\mathrm{x}_2\right)\)

∴ f is one-one

For onto function :

Case-I

When x \(\geq\) 0

Let y=f(x)

y=\(\frac{x}{1+x}\)

y+x y=x

or y=x(1-y)

x=\(\frac{y}{1-y}\)

or x=-\(\frac{y}{(y-1)} \geq 0\)

⇒ \(\frac{y}{y-1} \leq 0\)

Relations And Functions Onto Function Case 1

y \(\in\)[0,1)

Case-2

When \(\mathrm{x}<0\)

Let y=f(x)

y=\(\frac{x}{1-x}\)

y-x y=x

or y=x+x y

x=\(\frac{y}{1+y} \)

x=\(\frac{y}{1+y}<0\)

Relations And Functions Onto Function Case 2

y \(\in\)(-1,0)

From Case 1 and Case 2, y ∈  [0,1) ∪ (-1,0)

Range = (-1,1) = Co-domain

∴ So, f is onto.

Hence, f is one-one and onto function.

Question 2. Show that the function f: R ⇒ R given by f(x) = x³ is injective.

Solution:

f: \(\rightarrow \)R is given as f(x)=\(x^3\).

Let \(x_1, x_2 \in\) R such that if f\(\left(x_1\right)=f\left(x_2\right)\)

⇒ \(x_1^3=x_2^3 \Rightarrow\left(x_1-x_2\right)\left(x_1^2+x_2^2+x_1 x_2\right)\)=0

⇒ \(\left(x_1-x_2\right)\left[\left(x_1+\frac{x_2}{2}\right)^2+\frac{3}{4} x_2^2\right]\)=0

⇒ \(x_1-x_2=0\left[\left(x_1+\frac{x_2}{2}\right)^2+\frac{3}{4} x_2^2 \neq 0\right]\)

⇒ \(x_1=x_2 \)

∴ f is injective

Question 3. Given a nonempty set X, consider P(X) which is the set of all subsets of X. Define the relation R in P(X) as follows:

For subsets A, and B in P(X), ARB if and only if Ac B. Is R an equivalence relation on P(X)? Justify your answer:

Solution:

Since every set is a subset of itself, ARA, ∀ A ∈ P(X).

∴ R is reflexive.

Let ARB ⇒ A ⊂ B. ≠ B ⊄ A.

For Example: if A = {1,2} and B = {1, 2, 3}, then A ⊂ B but B ⊄ A

∴ R is not symmetric.

Since R is not symmetric.

Hence, R is not an equivalence relation on P(X).

Question 4. Find the number of all onto functions from the set {1, 2, 3,…, n) to itself.

Solution:

Since f is onto function, so all elements of set {1, 2,…n} have unique.

pre-image in set (1,2,…n}

So. the total number of onto functions = n x (n-1) x {n — 2)x …x 2 x 1= n!

Question 5. Let \(\mathrm{A}=\{-1,0,1,2\}, \mathrm{B}=\{-4,-2,0,2\}\) and \(\mathrm{f}, \mathrm{g}: \mathrm{A} \rightarrow \mathrm{B}\) be functions defined by \(f(x)=x^2-x, x \in A\) and \(g(x)=2\left|x-\frac{1}{2}\right|-1, x \in\) A. Are f and g equal? Justify your answer.

Solution:

Given A={-1,0,1,2}, B={-4,-2,0,2}.

Also, f: A \(\rightarrow B, f(x)=x^2-x, \forall x \in A\)

⇒ \(\mathrm{g}: \mathrm{A} \rightarrow \mathrm{B}, \mathrm{g}(\mathrm{x})=2\left|\mathrm{x}-\frac{1}{2}\right|-1, \forall \mathrm{x} \in \mathrm{A}\)

It is observed that :

f(-1)=(-1)^2-(-1)=1+1=2

⇒ \(g(-1)=2\left|(-1)-\frac{1}{2}\right|-1=2\left(\frac{3}{2}\right)-1=3-1=2 \Rightarrow f(-1)=g(-1)\)

⇒ \(f(0)=(0)^2-0=0 ; g(0)=2\left|0-\frac{1}{2}\right|-1=2\left(\frac{1}{2}\right)-1=1-1=0 \Rightarrow f(0)=g(0)\)

⇒ \(f(1)=(1)^2-1=1-1=0 ; g(1)=2\left|1-\frac{1}{2}\right|-1=2\left(\frac{1}{2}\right)-1=0 \Rightarrow f(1)=g(1) \)

⇒ \(f(2)=2^2-2=4-2=2 ; g(2)=2\left|2-\frac{1}{2}\right|-1=2\left(\frac{3}{2}\right)-1=3-1=2 \Rightarrow f(2)=g(2)\)

Thus \(\forall \mathrm{a} \in \mathrm{A}, \mathrm{f}(\mathrm{a})=\mathrm{g}(\mathrm{a})\)

f and g are equal functions.

Question 6. Let A = {1, 2, 3). The number of relations containing (1,2) and (1,3) which are reflexive and symmetric but not transitive is

  1. 1
  2. 2
  3. 3
  4. 4

Solution:

1. The given set is A = {1,2, 3},

The smallest relation containing (1, 2) and (1, 3) which is reflexive and symmetric, but not transitive is given by:

R = {(1, 1), (2, 2), (3, 3), (1,2), (1,3), (2, 1), (3, 1)}

Since (1, 1), (2, 2), (3, 3) ∈ R.

∴ So R is reflexive

Since (1, 2), (2. 1) ∉ R and (1, 3), (3, 1) ∉ R.

So R is symmetric Since (3, 1), (1, 2) ∈ R, but (3, 2) ∉ R.

∴ So R is not transitive

Now, if we add ordered pairs (3, 2) and (2, 3) to relation R, then relation R will become transitive.

Hence, the total number of desired relations is one.

Question 7. Let A = {1, 2, 3). The number of equivalence relations containing (1,2) is

  1. 1
  2. 2
  3.  3
  4.  4

Solution: 2.

It is given that A = -(1,2,3}.

The smallest equivalence relation containing (1, 2) is given by,

R = {(I,1), (2, 2), (3, 3), (1,2), (2,1)}

Now, we are left with only four pairs i,e., (2, 3), (3, 2), (1, 3), and (3, 1).

If we add any one pair [say (2, 3 ) ] to R, then for symmetry we must add (3, 2).

Also, for transitivity, we are required to add (1,3) and (3, 1).

Hence, another equivalence relation R, = {(1, 1), (2, 2), (3, 3), (1,2), (2, 1), (2, 3), (3, 2), (1, 3), (3, 1)}

This shows that the total number of equivalence relations containing (1,2) is two i.e. R1 and R2